Why greenhouse gas warming doesn’t break the second law of thermodynamics

This is generating many comments, see below for an update!

Behind the scenes some skeptics are suggesting that CO2 can’t warm us because the atmosphere is colder than the planet, and  it would break the 2nd Law of Thermodynamics (see Postma*, for example, p 6 – 7). I disagree. The 2nd Law of Thermodynamics applies to net flows of heat, not to each individual photon, and it does not prevent some heat flowing from a cooler body to a warm one.

Imagine three blocks of metal side by side. They are 11°C, 10°C, and 9°C. Think about what happens to the photons coming off the atoms in the middle of the medium temperature block between the other two. If heat never flows from cooler blocks to warmer blocks, all those photons have to go “right“, and not ever go “left”, because they “know” that way is towards a cooler block? (How would they?!)

The photons go both ways (actually every way, in 3D). There are more coming from the 11°C block to the 10°C block, sure, but the the 10°C block is sending ’em back to the 11°C block too. So heat is flowing from cold to hot. It happens all the time. Net heat is flowing always hot to cold. But some heat is going the other way, every day, everywhere, bar possibly a black hole.

People are being caught by semantics. Technically, strictly, greenhouse gases don’t “warm” the planet (as in, they don’t supply additional heat energy), but they slow the cooling, which for all pragmatic purposes leaves the planet warmer that it would have been without them. It’s a bit like saying a blanket doesn’t warm you in bed. Sure, it’s got no internal heat source, and it won’t add any heat energy that you didn’t already have, but you sure feel cold without one. —  Jo

————————————–

Guest Post by Michael Hammer

CAN A COLD OBJECT PASS HEAT TO A WARMER OBJECT?

I have lost count of the number of people claiming that global warming is impossible because the atmosphere is colder than the surface and thus cannot return heat to the surface since that would contravene the second law of thermodynamics. This is wrong and is based on an incorrect interpretation of the second law. The second law does not say a cold object cannot pass heat to a warmer object, it states that NET heat flow is always from warmer to colder.

As stated in the previous section, any object above absolute zero radiates energy. This energy is radiated in all directions. If such radiated energy strikes another object some or all of it is absorbed depending on the absorptivity of the object struck. The absorption does not depend on whether the object struck is warmer or colder than the object that emitted the energy, it only depends on the absorptivity of the struck object. However that object also emits energy some of which will radiate back to the first object and again be absorbed. Because the warmer object emits more energy there will be more traveling from warmer to cooler than vice versa and hence the NET heat flow will be from warmer to cooler.

Net heat flows from warmer to a cooler body, but some heat flows from a cooler body to a warm one.

Net heat flows from warmer to a cooler body, but some heat still flows from a cooler body to a warm one.

BUT THEN HOW CAN A COOLER ATMOSPHERE HEAT THE WARMER SURFACE?

This apparent paradox is again based on a misunderstanding. Imagine you are standing outside on a cold winters night. It’s really cold and you are soon chilled to the bone so you step inside. Inside it’s a pleasant 20°C and almost immediately you feel warmer. But you are at 37°C and the room is 17°C cooler at 20°C how can it warm you, the second law of thermodynamics forbids it! No it doesn’t. When you were outside, you body was radiating energy to space but because the environment was so cold there was very little radiating back to you so the net loss was substantial. When you step inside your body is still radiating exactly the same amount of energy (remember the amount radiated depends only on the temperature and emissivity) however now the warmer walls of the room radiate more energy back to you than did the cold outside. Since the walls are colder than you are you still radiate more energy than you receive (heat flow is still from you to the room) but the difference between what you radiate and what you receive is less. You lose less net energy when inside than when outside so you feel warmer inside the room and it is easy to feel the room is warming you. In fact it is more accurate to say the room cools you less than did the outside.

Exactly the same situation exists with respect to Earth’s surface. Without the green house gases in the atmosphere the surface would be radiating directly to outer space which is extremely cold (-269°C). The green house gases prevent some of that radiation to space and thus keep the surface warmer than it would otherwise be. They do not do this by reducing the amount of energy the surface emits – doing that would entail changing the surface emissivity. Instead they radiate energy back onto the surface so that the net energy loss is reduced.

————-

Michael Hammer is an electrical engineer who has spent over 30 years conducting research for a major international spectroscopy company.  In the course of this work he generated around 20 patents which have been registered in multiple countries.  Patents are rarer and more rigorous than peer reviewed papers, only available for economically valuable work, and costing thousands of dollars to process and maintain. Spectroscopy deals with the interaction between electromagnetic energy (light) and matter and it is this interaction which forms the basis of the so called “green house effect” in the atmosphere.

———————

PS from Jo: Just so we’re clear here, I think CO2 molecules absorb Infra Red and have some warming effect, but I think feedbacks from clouds or humidity keep those effects so small that the total effect of adding more CO2 is minor, and not worth taking any action over. (There’s more info on feedbacks in these posts).

*The Postma example correctly shows that two ice cubes at 0°C will not heat one cube above zero, but it’s all about context. If the surrounding air is even colder, being next to an ice cube will keep you warmer longer. Even ice can be a “blanket”.
Being wrapped in ice  would slow heat loss if you happen to be on a rock in outer space.  Eskimoes stay warmer in an igloo. (Yes there are lots of reasons why, but the point remains… if you reduce your heat loss you stay warmer.)

ADDENDUM #1

Joseph Postma himself has replied — see comment #97 (it was caught in the spam filter and delayed)

From Michael Hammer #144
Let me try to put it another way. If you lie in bed without a blanket you lose a lot of energy and feel cold. If you now cover yourself with a blanket, the blanket reduces your energy loss. With a reduced energy loss you get less cold than you otherwise would have. Whether you consider “get less cold” to be semantically equivalent to saying “you will be warmer than you otherwise would have been” is up to you but I point out that general usage would say the blanket warms you. This is despite the fact that the blanket is colder than you are. No the blanket being colder does not transfer NET heat from itself to you, it merely reduces the energy loss allowing your internal heat generation to raise your temperature more. If you put a blanket over a piece of cold steel it does not make the steel warmer.

An exactly analogous situation exists with respect to Earth. There is an external energy input notably the sun. An opaque atmosphere reduces the energy loss from the surface to space which allows the energy input from the sun to raise the temperature slightly. The effect of more CO2 is to very slightly increase the range of wavelengths around 15 microns at which the atmosphere is opaque.

The mechanism by which this energy loss is reduced cannot be by reducing the heat radiated by the surface because the atmosphere cannot influence the emissivity of the surface. Rather it acts by returning some of the energy radiated back to the surface. This is the back radiation.

If you want the analogy with a blanket to be more accurate consider the survival blankets which are simply a silvered sheet of thin plastic. Clearly the thin plastic has negligible impact on conduction. It could act by reducing convection but then again it does not need to be silvered to do that. A transparent sheet of plastic would do that just as well yet a transparent sheet of plastic does not work anywhere near as well as a silvered sheet. The silvered sheet works so well because the shiny surface has very low absorptivity and emissivity so it loses very little energy by radiation. There is still a difference in that the silvered surface reflects the energy back onto your body rather than via an absorption and then emission process but the overall impact is very similar.

—————————————————————————————————–

ADDENDUM #2

From Jo
Here’s why talk about whether its convection or conduction vs radiative cooling is irrelevant
The blanket analogy is perfect because we are discussing whether it’s possible for a cooler item to induce (somehow) an increase in temperature of a warmer item. NOTE: The cooler item has no internal heat source, but the warmer item (Earth or body) does have energy added in. The method of heat transfer is irrelevant. (Talk of two ice cubes misses the point unless one cube is heated by the sun, or burns fat. )

The point is that YES, obviously in the real world, blankets keep us warm. Pink batts “lift the temperature of your home in cold weather”. They don’t do it by supplying energy, they do it by blocking energy loss. The cooler item is not supplying a single new joule of energy, but there another mechanism of increasing an objects temperature. It’s called insulation. It’s a reality we all know and use every single day. Why deny it?

Can commenters move on from repeating the truism that a colder object can’t make a warmer one even warmer without supplying energy? We all know that, but it applies to a closed theoretical system with no extra source of energy. In the systems with blankets/people and the sun/earth there IS an extra source of energy at least until you’re dead or the sun burns out.

 

Comments have overflowed. The first 500 are on the next page.

Comments that refer to other comments will still have the original numbers (up to comment #300)

ie “comment #767” now means comment #267

7.7 out of 10 based on 22 ratings

519 comments to Why greenhouse gas warming doesn’t break the second law of thermodynamics

  • #
    KR

    Michael Hammer – Good post, very clear explanation.

    Those who disagree – I would suggest looking at Roy Spencers Yes, Virginia, Cooler Objects Can Make Warmer Objects Even Warmer Still (very well written), or for a more in depth discussion, following the Science of Doom series on the Imaginary 2nd Law of Thermodynamics.

    The energy exchanges of the climate consist of energy coming in from the sun and energy going out to space. By reducing the cooling to space, the greenhouse effect makes energy accumulate until the Earth is warm enough to radiate an equal amount of energy to space.

    Warm some soup on a stove, in an open pot, in a cold room – the soup will reach a certain temperature where the heat leaving matches the heat arriving from the burner. Put the lid on, wrap it in a heavy towel, reducing the rate of heat loss/cooling – the soup will get a lot warmer, to the point where it’s again radiating as much heat through the towel as it’s gaining from the burner, even though you haven’t turned up the stove.

    42

  • #
    KR

    BLouis79 @ 250

    As an aside, I spent quite a bit of time going through Gerlich and Tscheuschner’s paper. They violate conservation of energy quite clearly in figure 32 of their paper (arxiv version, sorry, I don’t have a direct link to the final paper). I will never recover those lost hours, which I could have spent much more profitably with a microbrew.

    It’s quite simply a ridiculous argument – whether you think the radiative greenhouse effect is of consequence or not, it does not violate thermodynamics! Take it from Jo and from Michael Hammer…

    31

  • #

    Re: Jo #215.

    “In fact, almost no solar radiation makes it to the surface.”

    Most visible light and near infrared makes it to the ground. But a lot is also absorbed at other wavelengths. Diagram here from here, which states:

    Fortunately for us, all of the high energy X-rays and most UV is filtered out long before it reaches the ground. Much of the infrared radiation is also absorbed by our atmosphere far above our heads.

    Anyway, is is not possible to “slow down” heat flow, resulting in a temperature increase of a body, unless you apply work.

    Take your red and blue blocks in the diagram above, called:

    Net heat flows from warmer to a cooler body, but some heat still flows from a cooler body to a warm one.

    If the blue object really were heating the red one then the red one would go to a higher temperature. The red object would then emit more electromagnetic radiation than it did in the first place and so heat the blue object up more than it was. The blue object, now warmer, heats the red one up more, which then, in turn, feeds back…until…..until finally the Universe explodes in infinite heat death. Sorry, but such is the murky world of belief in the greenhouse effect.

    You say:

    Technically, strictly, greenhouse gases don’t “warm” the planet (as in, they don’t supply additional heat energy), but they slow the cooling, which for all pragmatic purposes leaves the planet warmer that it would have been without them.

    That’s a contradiction. The greenhouse effect says that extra heat energy is added. It is therefore in violation of energy conservation.

    40

  • #

    I have just posted a new thread on this topic — sorry if it doesn’t flow exactly on from where your comments are at right now — but clearly there is a need for more discussion.

    http://joannenova.com.au/2011/05/so-what-is-the-second-darn-law/

    10

  • #
    RJ

    500th post

    I wonder what the record is?

    10

  • #
  • #
    RW

    Paul (RE: 493)

    “So how do CO2 molecules that are spaced at intervals of one for every several thousand molecules of nitrogen or oxygen act as a barrier to the flow of heat through the atmosphere?

    If you say that they do so by radiating some of the long-wave radiation back to the surface and that slows the rate at which it can cool, then I think that your explanation ignores the fact that this will simply increase the radiation from the surface by a similar amount, so that the net radiation from the surface will be restored. The formula for the radiation of a blackbody at a given temperature does not allow for the flow of heat to be reduced by the presence of another body in close proximity, as far as I am aware. It simply expresses the rate at which the body will radiate heat.”

    I’m not sure I understand what you mean here.

    “Besides, if you consider the mass of the material that is in thermal conjunction, the earth’s mass is so much greater than that of the entire atmosphere that the amount of heat that is being radiated in each direction is proportionately greater from the earth to the atmosphere and space than anything being radiated from the atmosphere to the earth.

    Imagine, by way of simplification, that you run a hot bath with the doors and windows of the room closed. You will soon notice a decided increase in the temperature of the air and its moisture burden. On the other hand, if you run a cold bath and turn on a fan heater to blow hot air into the room and wait for the bath to warm up, you will be likely to have a problem with hygiene before you get your hot bath. So, to imagine that the atmosphere contains sufficient heat to radiate back to earth and warm the earth by 33 degrees is a bit lacking in credibility, don’t you think?”

    I don’t see how this is analogous. The energy in the system is not sourced from the atmosphere – the atmosphere is more or less just a ‘filter’ between the surface and space. The SW from the Sun is mostly transparent through the filter and the LW emitted by the surface is largely opaque through the filter, causing energy to ‘back up’ at the surface and accumulate until the surface is emitting enough energy ‘push through’ the same amount of energy that’s arriving from the Sun (about 239 W/m^2 in and out).

    “I can almost imagine your response to that. You will say, no doubt, that the atmosphere doesn’t heat the earth it just slows the rate at which the earth loses heat, raising the temperature on the surface by those magical 33 degrees. Well even a blanket doesn’t do that without itself warming in the process. So far, the evidence from the tropics is that the atmosphere has not warmed at all, especially also it has not increased in its moisture content as was postulated. Hence, without that warming, the whole conjecture falls flat. No warming of the upper atmosphere in the tropics, no Global Warming, no Climate Change, no nothing, just the normal behaviour one would expect to find with a thermostatically controlled climate.

    And, by the way, the place where the radiation is escaping from the atmosphere is about 6 km above the surface of the earth where the temperature is close to that of a black-body exposed to the heat from the sun at that distance. Is that a co-incidence?

    How did the heat energy arrive at that height? By means of air circulation carrying warmer and moisture-laden air to the top of the troposphere [hence its name]. The moisture condenses into droplets, forming clouds and releasing the latent heat of vaporisation, and then the heat is radiated out to space.

    We know that there is negligible water vapour at that height. In fact there is negligible atmosphere at that height. Most of the mass of the atmosphere exists below that height, so the amount of atmospheric carbon dioxide that is above that height must be negligible also.

    One has to seriously wonder how most of the educated world has been convinced that atmospheric carbon dioxide is so powerfully bad!”

    Well, I’m not convinced either. In fact, I think the effects of anthropogenic CO2 are – if not infinitesimal, benignly small. However, I do think the physics supports a likelihood of some effect, though I don’t believe this to be definite – just seemingly probable.

    The research of ‘co2isnotevil’ (aka George White) has convinced me there is no physical or logical reason why the effects of CO2 ‘forcing’ would be any larger than solar ‘forcing’. If anything, I’d bet they are probably even less.

    “With regard to your illustration of a car gathering heat in the sun, I do not think that that has any connection with what happens in the open with a real atmosphere. True the sun’s rays penetrate the glass of the car and warm the interior. But take the glass away altogether and see if you do not get the same, if reduced, effect? It is not the one-way penetration of short-wave radiation through the glass coupled with the glass stopping the outgoing long-wave radiation that is heating the car. Rather it is the prevention of movement in the air that is trapping the heat, nothing else.”

    Well yes, most of the prevention of heat loss is by convection rather radiation, but I don’t see how that’s fundamentally any different.

    10

  • #
    RW

    I wrote:

    Well, I’m not convinced either. In fact, I think the effects of anthropogenic CO2 are – if not infinitesimal, benignly small. However, I do think the physics supports a likelihood of some effect, though I don’t believe this to be definite – just seemingly probable.

    By this I mean that the physics supports that the increased CO2 will perturb the climate system. Whether that will result in net increase in temperature is what I think is not definite – just seemingly probable.

    10

  • #

    An Interesting Approach here:

    http://www.biocab.org/Density-of-Energy-in-Atmosphere.html

    Which contains some graphs and calculations which Leads the Author to this conclusion:

    “The warmhouse effect, or “greenhouse” effect, is not caused by the gases composing the atmosphere, but by the inversion of the decline of sensible heat flux with respect to the decline of the thermal energy density in the atmosphere. As the sensible heat flux increases as the energy density diminishes, the warmhouse effect happens. Conversely, as the sensible heat flux decreases simultaneously with a decrease of the energy density, the warmhouse effect terminates. Therefore, any increase of the sensible heat flux in the atmosphere follows from an increase in the incident solar energy on the Earth’s surface; consequently, any increase of the incident solar radiation on the Earth’s surface implies an increase of the energy density of any mass of air. Otherwise, the warmhouse effect, or “greenhouse” effect, would be impossible since energy is neither created nor destroyed, but only transformed.”

    Obviously you have to have an atmosphere for this to happen. The guy’s first language is not English so the wording of the conclusion is a bit fuzzy and ambiguous. Just excuse this and read the article before commenting on its substance.

    Love to see some well reasoned discussion on this one Jo.

    10

  • #
    Roy Hogue

    RJ @500,

    500th post

    I wonder what the record is?

    I’ve never seen a thread this long.

    If I’m even close to a competent observer then it shows me that there’s very little actually (may I say nothing?) nailed down and accepted as “proved” about AGW in accordance with the scientific method of investigation.

    It is tragic that this debate is not happening in the halls of academia and government.

    10

  • #

    Roy Hogue (#508):

    I’d make your point in a slightly different way, for under the scientific method of investigation one cannot prove a theory right but can possibly prove it wrong. The scientific method replaces the idea of “proof” by the idea of “statistical validation.” A theory is “statistically validated” if its predictions are checked against observed events without being falsified by the evidence.

    A widely misunderstood aspect of modern climatology is that the IPCC’s climate models make no predictions. They make “projections” which climatologists confuse with predictions. Unlike a prediction, a projection does not state a falsifiable claim. It follows that none of these models can have been statistically validated and thus that a scientific investigation of the possibility of anthropogenic global warming has not yet taken place. Academia and government have presided over a fiasco which they have represented to be a scientific investigation but which hasn’t been one. The debate that should be taking place but isn’t is about how to recover from this fiasco.

    10

  • #
    Roy Hogue

    I must correct myself about the longest thread. Mark D. calls out 757 comments @502.

    10

  • #
    Roy Hogue

    Terry Oldberg @509,

    I’ll go with that!

    10

  • #
    Mark D.

    Terry Oldberg Re 495, Thanks for the link. I am digesting the material there. In the meantime, my apologies for mistaking your intentions at my first comment to you. Obviously the distinction between “prediction” and “projection” is important and valid but in a practical view on the political front, there is no difference. The average voter wouldn’t be able to argue any difference. It is my opinion that the IPCC very purposely set out to keep the scientists happy AND still be able to push (or cause to push) a political agenda.

    10

  • #
    Paul

    Mark D.:
    May 20th, 2011 at 11:08 pm

    RJ @ 500 the answer is 757 here:

    http://joannenova.com.au/2009/05/shock-global-temperatures-driven-by-us-postal-charges/

    Thanks for that link. It illustrates the point that there is no hard evidence supporting the CAGW conjecture, only circumstantial evidence.

    A clever lawyer can persuade the jury that an innocent defendant is guilty, especially when the prosecutor has suppressed any contrary evidence, the reputation of the police is on the line and they will manufacture evidence rather than be shown to be incompetent, etc, when there is some circumstantial evidence connecting the defendant to the crime. In the same way clever and authoritative protagonists in the case for CAGW plead for belief in the hypothesis and have convinced a large proportion of the ‘jury’ despite the lack of any hard evidence.

    In taking a quick look at the the thread which holds the record for the most comments I notice one comment which is very relevant to this thread, that at comment 22 : —

    JLKrueger:
    May 4th, 2009 at 2:37 pm

    BTW, the two US Air Force weather guys I work with (one of whom has a PhD in atmospheric physics) are still laughing about Boris’ clinging to MODTRAN radiative codes as proof of anything.

    I’ve seen quite a bit of reliance on that piece of software earlier in this thread, though I do not find a software ‘proof’ in any way convincing or a substitute for real experimental evidence myself.

    To my mind, the crux of the issue that is being debated on this thread is whether or not it is permissible to take the results of an experiment in a laboratory, where strict conditions must be maintained in order to get an accurate result, and to extrapolate that to the earth’s atmosphere where all the conditions of the laboratory experiment have been broken.

    No amount of sophistical argument, no amount of modelling, nothing but an experiment in the wider atmosphere that produces verifiable, quantifiable data can answer that question. And that is the sticking point that the earlier thread highlighted: the lack of such experimental data to support the CAGW conjecture persists to this day and always will.

    Paul

    10

  • #
    Paul

    By way of referencing the Global Warming hypothesis on the scale of credibility, I note this quote from Bad Science

    There is good science, there is pathological science, and then there is pseudoscience (which is, simply, a theory, methodology, or practice that is considered to be without scientific foundation). The several lists below were culled from different sources and are presented as tools for discernment. Though most of the rules apply to experimental sciences, physics in particular, the basic tenets can be generalized without much difficulty to wider applications, including archaeology.

    The term “pathological science” was coined by Nobel-laureate chemist Irving Langmuir in a lecture he gave at General Electric’s Knolls Atomic Power Laboratory in 1953. Langmuir offered several examples of pathological science, and concluded that:

    These are cases where there is no dishonesty involved but where people are tricked into false results by a lack of understanding about what human beings can do to themselves in the way of being led astray by subjective effects, wishful thinking or threshold interactions. These are examples of pathological science. These are things that attracted a great deal of attention. Usually hundreds of papers have been published on them. Sometimes they have lasted for 15 or 20 years and then gradually have died away. Now here are the characteristic rules:

    [1] The maximum effect that is observed is produced by a causative agent of barely detectable intensity. For example, you might think that if one onion root would affect another due to ultraviolet light then by putting on an ultraviolet source of light you could get it to work better. Oh no! Oh no! It had to be just the amount of intensity that’s given off by an onion root. Ten onion roots wouldn’t do any better than one and it didn’t make any difference about the distance of the source. It didn’t follow any inverse square law or anything as simple as that. And so on. In other words, the effect is independent of the intensity of the cause. That was true in the mitogenetic rays and it was true in the N rays. Ten bricks didn’t have any more effect than one. It had to be of low intensity. We know why it had to be of low intensity: so that you could fool yourself so easily. Otherwise, it wouldn’t work. Davis-Barnes worked just as well when the filament was turned off. They counted scintillations.

    [2] Another characteristic thing about them all is that these observations are near the threshold of visibility of the eyes. Any other sense, I suppose, would work as well. Or many measurements are necessary many measurements because of the very low statistical significance of the results. With the mitogenetic rays particularly, [people] started out by seeing something that was bent. Later on, they would take a hundred onion roots and expose them to something, and they would get the average position of all of them to see whether the average had been affected a little bit … Statistical measurements of a very small … were thought to be significant if you took large numbers. Now the trouble with that is this. [Most people have a habit, when taking] measurements of low significance, [of finding] a means of rejecting data. They are right at the threshold value and there are many reasons why [they] can discard data. Davis and Barnes were doing that right along. If things were doubtful at all, why, they would discard them or not discard them depending on whether or not they fit the theory. They didn’t know that, but that’s the way it worked out.

    [3] There are claims of great accuracy. Barnes was going to get the Rydberg constant more accurately than the spectroscopists could. Great sensitivity or great specificity we’ll come across that particularly in the Allison effect.

    [4] Fantastic theories contrary to experience. In the Bohr theory, the whole idea of an electron being captured by an alpha particle when the alpha particles aren’t there, just because the waves are there, [isn’t] a very sensible theory.

    [5] Criticisms are met by ad hoc excuses thought up on the spur of the moment. They always had an answer always.

    [6] The ratio of the supporters to the critics rises up somewhere near 50% and then falls gradually to oblivion. The critics couldn’t reproduce the effects. Only the supporters could do that. In the end, nothing was salvaged. Why should there be? There isn’t anything there. There never was. That’s characteristic of the effect.

    (From I. Langmuir, “Pathological Science: scientific studies based on non-existent phenomena,” Physics Today, 1989 (Oct), 36, 47. Transcribed and edited by R. N. Hall.)

    On the curve of credibility this theory must have peaked some months before the conference at Copenhagen and before the decline phase which began in earnest after the release of the Climate-gate data between the maximum and the conference. At what point on the curve are we at the present?

    Paul

    10

  • #
    Roy Hogue

    Paul,

    I think we’re at the point where the Emperor of CAGW has no cloths on.

    10

  • #
    Paul

    Roy Hogue:
    May 22nd, 2011 at 11:04 am

    Paul,

    I think we’re at the point where the Emperor of CAGW has no cloths on.

    I’ve thought that for a long time. But the ‘Emperor’ is still strutting his stuff as if his illusions were the real McCoy.

    But have you seen any recent Opinion Polls showing the percentage of respondents who are convinced that CAGW is true?

    How long do you give them before they are compelled to concede that their game is up?

    Paul

    10

  • #
    RW

    I agree the Emperor of CAGW has no clothes, but it’s not because the GHE violates the 2nd law.

    10

  • #
    Roy Hogue

    Paul,

    They are sinking of course. But a whole lot of money, power and prestige have been bet on this thing so they’ll fight tooth and nail. We have a long way to go.

    RW,

    Agree! The second law is not violated. The real problem is that the wrong question is being asked. And when you do that you always get the wrong answer. The right question is this: Is there any evidence that the world is getting warmer from increasing CO2 in the atmosphere? The answer is a resounding no! And better than that, there’s plenty of evidence that it isn’t. But the direct no answer is the one that counts.

    10

  • #
    Paul

    To answer my own question re the Opinion Polls, it is interesting to observe that belief in human-caused global warming correlates rather well with other somewhat-unrelated variables, such as the country one lives in, the political party one supports, the occupation one works in, etc.

    Another interesting point is, that even those who support the CAGW conjecture refer to this as a belief. Do you believe in the force of gravity, the heat of fire, the power of an electrical current? Do we not usually refer to those things in terms of either knowledge or ignorance? But it is not so with the all-important matter of CAGW which is universally referred to in terms of belief, as far as I can tell.

    World opinion on global warming: not so hot
    Posted on April 27, 2011 by Anthony Watts

    Results of the latest Gallup poll:

    Worldwide, Blame for Climate Change Falls on Humans
    Americans among least likely to attribute to human causes
    by Julie Ray and Anita Pugliese
    WASHINGTON, D.C. — World residents are more likely to blame human activities than nature for the rise in temperatures associated with climate change. Thirty-five percent of adults in 111 countries in 2010 say global warming results from human activities, while less than half as many (14%) blame nature. Thirteen percent fault both.

    http://wattsupwiththat.files.wordpress.com/2011/04/gallup_world.png
    [That image link does not seem to be working – not sure if I’ve done the link right, but the URL is OK]

    It is also interesting to note that the battle for the hearts and minds of the people has extended itself to the sphere of Opinion Polls itself, since even there the view of which way the polls are moving diverges depending on which side of the divide the commentator stands.

    Global Warming and the Pollsters: Who’s Right?

    And scientists, according to the participants in the Gallup poll, aren’t too sure themselves.

    But that’s contrary to one major study at the University of Illinois at Chicago. That study surveyed 3,146 earth scientists around the world and found overwhelming agreement that the world has been warming over the past 200 years (90 percent) and human activities are a significant factor (82 percent.) Significantly, this survey involved scientists who are closest to the issue. Climate scientists were almost universal (97 percent) in their belief that humans are largely to blame. Meteorologists, who deal primarily with near-term weather conditions, were less convinced (64 percent.) Climate scientists focus on long-term changes, like global climate change, leading Peter Doran, associate professor of earth and environmental sciences at Illinois, to conclude:

    “The take-home message is, the more you know about the field of climate science, the more you’re likely to believe in global warming and humankind’s contribution to it.”

    Emphasis added.

    NB the use of the words “belief” and “believe in”! Speaks volumes to me.

    Paul

    10

  • #
    BLouis79

    @RW #517 “I agree the Emperor of CAGW has no clothes, but it’s not because the GHE violates the 2nd law.”

    Semantically I could agree. Nothing much can break the laws of thermodynamics in reality. So yes the GHE cannot possibly violate the second law. The radiative GHE does not exist. Backradiation photons are measurable but result in zero net heat transfer up a thermal gradient. Other atmospheric gases heated by CO2 molecules rapidly convect to altitude in accordance with the lapse rate (just ask any bird or glider pilot how important thermals are).

    Still waiting for a prediction on how much absorption/scattering/backradiation one could measure from an incident IR laser passed through any percentage of CO2 gas (from 100% to 0.3%) as predicted by the laws of physics.

    If one wants to do some unscientific experiments. An IR thermometer pointed at a gas burner 1m away laterally detects 270degC. My hand detects no heat at 1m from presumably radiation detectable and measurable by the IR thermometer. My hand detects uncomfortable heat laterally at about 3cm from the flame. My hand detects uncomfortable heat 20cm over the bioling water in the pot. I get the distinct impression that as a heat energy transport mechanism, convection >> conduction >> radiation.

    10

  • #
    Smoking Frog

    Paul:

    [Smoking Frog] Even without the greenhouse effect, I don’t think the average temperature would be -18 C. That’s because the earth is rotating, and power is not proportional to temperature. However, this doesn’t contradict the attribution of 33 C. to the greenhouse effect, …

    [Paul] That is totally illogical. You can’t attribute a false figure to anything without the attribution being false also. So why try to defend the indefensible?

    Sorry, I was unclear. L.J. Ryan contends that if the sun supplies 240 w/m^2, the temperature at the surface can’t be greater than -18 C. or whatever the 240 w/m^2 dictates, but he misunderstands the 240 w/m^2. The sun supplies 1366 w/m^2 at top of atmosphere, not 240 w/m^2. The 240 w/m^2 is what’s absorbed at top of atmosphere (or something like that). The rest of it goes deeper. So I meant to say that the temperature at the surface of the earth is necessarily greater than the temperature calculated from 240 w/m^2, and this is not contradicted by whatever low temperature would result from 240 w/m^2 at the surface. It so happens that I don’t see that it would be -18 C. – I think it would be some other low temperature – but that’s beside the point.

    10

  • #
    Smoking Frog

    L.J. Ryan

    [Smoking Frog] Even without the greenhouse effect, I don’t think the average temperature would be -18 C. That’s because the earth is rotating, and power is not proportional to temperature. However, this doesn’t contradict the attribution of 33 C. to the greenhouse effect, because the 240 w/m^2 is at top of atmosphere, while the 15 C. and the -18 C are at the surface.

    [L.J. Ryan] I’m not sure what your saying…but 240 W/m^2 is radiation incident of earths surface.

    Your confusion is because I failed to state explicitly that that you misunderstand the 240 w/m^2. The sun delivers 1366 w/m^2 at top of atmosphere and roughly 1000 w/m^2 at the earth’s surface (when the sun is directly overhead). The 240 w/m^2 is what’s absorbed at top of atmosphere (or something like that). The rest of it goes deeper. Even if the Greenhouse Effect were fictitious, the temperature at the earth’s surface would be greater than whatever 240 w/m^2 would get you.

    [Smoking Frog] Don’t you agree that the exterior of the thermos bottle radiates less per cm^2 than the (all around) surface of the hot liquid radiates per cm^2?

    [L.J. Ryan] Yes the outer surface of a thermos does radiate less then the contents therein. That, however, does not mean the contents increase in temperature the longer enclosed in the thermos. GHG physics will have you believe the contents heat itself. Try this Smoking Frog, place an illuminate chrome flashlight in your thermos, at what point do you have a megawatt therein? Don’t dismiss analogy…GHG physics would have you believe light increases it’s energy by re-radiation and reflection.

    I don’t claim that it means that, and neither does “GHG physics.” You are missing the claim that the temperature in the absence of the Greenhouse Effect would be lower than whatever you calculate for X watts per square meter. In other words, you’re treating this as if it started out at the maximum possible, and “GHG physics” said it could go higher than that. It doesn’t say that.

    10

  • #

    A free-flowing liquid or gas cannot produce an insulating/”greenhouse” effect. The truth, as we all know, is the exact opposite. Free-flowing liquids and gases produce a net cooling effect.

    Insulation is the prevention of conduction, convection and/or radiation. CO2 does not prevent any of these three modes of energy flow. Therefore CO2 cannot produce an insulating/”greenhouse” effect.

    That in a nut shell, is the end of the matter. Or at least should be and indeed would be, if not for those who continue to repeat the meme that goes, “there is no doubt that CO2 is a greenhouse gas and that it causes warming.”

    It is becoming more and more obvious that those who continue to repeat this mantra are the “self appointed” AGW sceptics who assume the mantle of spokes-person for those of us who are still in possession of reasoned logical thought.

    These key figures in the “sceptic” camp are notable for one thing above all else. The more prominent they are, the louder their proclamation of the existence of the “greenhouse effect” fallacy. Lord Chris Monckton being the prime example here for the purpose of demonstration.

    Such people are the true enablers of AGW fraud. It is they who have kept this debate going when it should have been well and truly laid to rest by now. These people are the gatekeepers of AGW fraud.

    Their last line of defence is an appeal to the authority of John Tyndall. From Tyndall’s “Contribution to Molecular Physics in the Domain of Radiant Heat” we find a statement quoted almost verbatim by such self appointed “sceptics” on this thread.

    Quote:

    As a dam built across a river causes local deepening of the stream, so our atmosphere, thrown as a barrier across the terrestrial rays, produces a local heightening of the temperature at the earth’s surface. This, of course, does not imply indefinite accumulation, any more than the river dam does, the quantity lost by terrestrial radiation being, equal to the quantity received from the sun. The chief intercepting substance is the aqueous vapour of the atmosphere,* the oxygen and nitrogen of which the great mass of the atmosphere is composed being sensibly transparent to the calorific rays. Were the atmosphere cleansed of its vapour, the temperature of space would directly upon us; and could we under present circumstances reach an elevation where the amount of that vapour is insensible, we might determine the temperature of space by direct experiment.

    To anyone who has bothered to actually research and understand Tyndall’s work regarding “Radiant Heat” it is obvious that his experiments did not and could not distinguish between absorption and scattering/reflection.

    At no stage in any of the experiments now referenced by such appeals to authority as, “150 year old established science” and “no one doubts that CO2 is a greenhouse gas”, does Tyndall ever attempt to establish let alone even measure an increase in temperature in any of the particular gasses under examination. He simply claims that the energy missing between the source and the pile must have been absorbed by the gas in question and therefore assumes/implies, this will cause the gas, through which the energy is being directed, to increase in temperature.

    Since it is now well understood in physics that photon absorption and re-emission by molecules does not necessarily equate to an increase in temperature, as in the case of reflection for example, perhaps finally it might be a prudent move to revisit John Tyndall’s “Contribution to Molecular Physics in the Domain of Radiant Heat” to ascertain exactly why, though the “greenhouse effect is well established 150 year old physics, no one has ever been able to physically demonstrated the warming effect of CO2.

    30

  • #

    Will Pratt @ 523,

    The reason the belief still exists in spite of a total failure to demonstrate the so called heating effect of CO2 is similar to the reason for the continued belief in god or gods in spite of a million plus years of total failure to demonstrate their existence.

    They believe “it” (an unidentified, undefined, and unexamined it) has to be “something” (an unidentified, undefined, and unexamined something) and the only thing they can think of that could cause “it” is CO2, god, or other substance with totally magical and mystical properties. After which, they expect all discussion or disagreement to stop – as in “the science is settled” or “god said it, I believe it, and that settles it.” The bottom line is they believe BECAUSE they don’t know and don’t want anyone else to discover that fact. Ignorance is the real thing they worship above all else.

    Some of us, on the other hand, are more than willing to admit we don’t know. We are willing to look at and for evidence to correct our ignorance. We are not satisfied with anything less than actually knowing and being able to provide an unambiguous demonstration of what we do know.

    Ignorance is not something to be worshiped. It is something to be fought as if our lives depend upon it BECAUSE it does. Then, if after a long, earnest effort we find no evidence, we drop the pretense, refuse to believe, clearly state “it” doesn’t exist. We then move on to things far more relevant to living life on earth.

    10

  • #
    Paul

    I must confess that I have been holding onto an erroneous understanding, in part, of the way that a mixture of gasses, including CO2, behaves in the presence of long-wave EM radiation. I was under the impression that some of that energy would be converted into heat.

    Last night I read a paper that puts it into a different perspective, contrary to that which I had been led to believe, showing that none of that radiation is converted into heat in the process. Instead, whenever a molecule of CO2 is energised by absorption of a photon another molecule of CO2 will be energised to emit a photon, keeping the energy state of the pocket of air equivalent.

    If this is the case then the ability of the atmosphere to absorb heat from this process is not even negligible, it is nil. And the passage of energy through the atmosphere by way of radiation cannot be impeded.

    CO2 heats the atmosphere…a counter view

    Again in simple words , it means that if the process (2) happens then the time symmetrical process , namely CO2 + N2⁺ → CO2* + N2 , happens too . Indeed this time reversed process where fast (e.g hot) N2 molecules slow down and excite vibrationally CO2 molecules is what makes an N2/CO2 laser work. Therefore the right way to write the process (2) is the following .

    CO2* + N2 ↔ CO2 + N2⁺ (3)

    Where the use of the double arrow ↔ instad of the simple arrow → is telling us that this process goes in both directions . Now the most important question is “What are the rates of the → and the ← processes ?”

    The LTE conditions with the energy equipartition law give immediately the answer : “These rates are exactly equal .” This means that for every collision where a vibrationally excited CO2* transfers energy to N2 , there is a collision where N2⁺ transfers the same energy to CO2 and excites it vibrationally . There is no net energy transfer from CO2 to N2 through the vibration-translation interaction .

    The fact that it has taken me years of research on this topic to find out such fundamental facts is illustrative of the fact that we have been exposed to an enormous amount of propaganda with an information content of zero!

    It also illustrates why the scientific method is based on actual experiments, not just thought-experiments or models which we have made to approximate reality, due to the propensity we all have to deceive ourselves.

    Paul

    10

  • #
    Paul

    Smoking Frog: @521
    May 22nd, 2011 at 9:22 pm

    Sorry, I was unclear. L.J. Ryan contends that if the sun supplies 240 w/m^2, the temperature at the surface can’t be greater than -18 C. or whatever the 240 w/m^2 dictates, but he misunderstands the 240 w/m^2. The sun supplies 1366 w/m^2 at top of atmosphere, not 240 w/m^2. The 240 w/m^2 is what’s absorbed at top of atmosphere (or something like that). The rest of it goes deeper. So I meant to say that the temperature at the surface of the earth is necessarily greater than the temperature calculated from 240 w/m^2, and this is not contradicted by whatever low temperature would result from 240 w/m^2 at the surface. It so happens that I don’t see that it would be -18 C. – I think it would be some other low temperature – but that’

    It would help if you could make your facts/assumptions clear.

    My understanding of the figure of 240 w/m^2 is that it is calculated from the ‘average’ insolation received on the surface of the exposed hemisphere of planet-earth at the ‘average’ distance of earth from the sun which is radiating at an ‘average’ surface temperature of 5,500 C. This total ‘average’ insolation is then reduced by the ‘average’ albedo of the earth-atmosphere system to arrive at a calculated ‘average’ of insolation received, and not reflected, at the surface of the earth.

    The calculated figure of theoretical, surface insolation is then divided by four to represent a flat disk, with a surface area the equivalent of the surface of the globe, being bathed in an equivalent, constant energy flow. This completely ignores the daily rotation of the globe and the effect that this would clearly have on the surface temperatures.

    In other words, there are a slew of assumptions included in that nice round, simple figure of ‘240’, enough to require more than simple acceptance of that figure as a ‘given’, in my humble opinion.

    On the other hand, talking about the temperature of a flat disk, so exposed to a quarter of the received energy of the sun per unit of area, calculating the blackbody temperature of this hypothetical entity, comparing this theoretical temperature with the calculated ‘average’ temperature of the near-earth atmosphere and then declaring that the difference represents the warming effect of atmospheric carbon dioxide, amplified by the presence of water vapour [which is somehow controlled by the CO2] and concluding that “we are doomed” because humans are adding CO2 to the atmosphere by the use of coal and oil, that I consider to be a nonsense of the highest order.

    Paul

    10

  • #

    Paul,

    …whenever a molecule of CO2 is energised by absorption of a photon another molecule of CO2 will be energised to emit a photon, keeping the energy state of the pocket of air equivalent.

    This is untrue, and the post you linked to is grossly flawed.

    The equipartition of energy is not an ironclad law, but rather a statistical truism. Given a large enough volume of molecules in an isolated system, the net interactions will tend towards an equilibrium state where energy is distributed evenly.

    This is not a law of quantum mechanics which invokes some sort of “action at a distance,” which when you think about it is rather fanciful and magical. Every time one CO2 molecule absorbs, another emits? How do the molecules communicate to coordinate this? How do they decide which one must emit? What happens when there’s only one CO2 molecule in the mass, or the very first which doesn’t have a partner with anything to emit? And of course, too, such a law would have to apply to everything, including emissions from O2 and N2. So how does anything ever heat, if every interaction is always perfectly balanced by a perfectly equal and opposite magical action somewhere else?

    No, the equipartiion of energy is a statistical truism which diverges from the expected when the volume of molecules becomes small enough (as happens with all statistics).

    10

  • #
    BLouis79

    Gas absorption of IR in a tall enough enclosure to permit development of a thermal gradient should be measurable in the laboratory. Surely someone somewhere has done this before?????

    Simply, one would think that if IR passed through an apparently IR transparent gas, then no heating would arise. Passing IR through an IR absorbing gas should be hypothesized to create a thermal gradient – warmer near the IR source and cooler further away. Using a laser should eliminate the confounding inverse square law.

    10

  • #
    Paul

    No, the equipartiion of energy is a statistical truism which diverges from the expected when the volume of molecules becomes small enough (as happens with all statistics).

    And I suppose that when you have two or three molecules in an enclosed space there is a finite but very small probability that they may momentarily end up all in close proximity. However by the time you have added sufficient molecules to represent a measurable quantity of gas the law of large numbers will have kicked in and the probability of all the molecules gravitating to one side of the enclosure is so small as to be discarded. We are, after all, talking not about the behaviour of a few molecules in a vacuum, but about their behaviour in our atmosphere.

    As I mentioned earlier, if you toss a coin once, you may have either a head or a tail, each of which has a probability of 0.5. If you toss it twice you can have either HH, HT, TH or TT, each of which has a probability of 0.25. Hence the probability of two heads or two tails are at the extreme with probability of 0.25 each, with HT and TH being equivalent and having a combined probability of 0.5. By the time the number of throws becomes a large number, the distribution of results conforms to a normal distribution, with values approaching equal numbers of heads and tails predominating, with skewed results becoming the tails, on either side, with probabilities quickly approaching zero.

    Hence, in the macro-world of the earth’s climate, it is not logical to talk about events, with statistical probabilities as near to zero as can be imagined, occurring. No, the actual state of any body of molecules in a gas will approach to the statistically highest point in the probability distribution and the deviations will get smaller as the size of the body increases. At the global extent the deviation will be vanishingly small and may be discarded. Hence these ‘laws’ hold true for any measurable quantity and can only deviate slightly even for small, unmeasurable numbers of molecules. Arguing from the event of one photon and one collision to a macro-result is not permissible.

    Paul

    10

  • #
    Smoking Frog

    Paul

    It would help if you could make your facts/assumptions clear.

    Yes. It would also help if you’d remember what you yourself previously quoted from me. I said that the earth rotates, but now you purport to teach this to me.

    In the same breath, I said that power is not proportional to temperature. My point with the rotation and the non-proportional relationship is that, contrary to what L.J. Ryan seems to suggest, average temperature cannot be derived from average watts per square meter.

    You say it’s “nonsense” to do some calculations and conclude that we’re doomed. I agree! The fact that I accept the Greenhouse Effect and reject L.J. Ryan’s argument does not mean that I believe in catastrophic global warming or even harmful global warming. I consider it to be unlikely but possible. That’s the best I can do without knowing a lot more science. I could argue that it’s extremely unlikely and almost impossible, but I’d be slinging BS.

    10

  • #
    L.J. Ryan

    Smoking Frog:

    Your confusion is because I failed to state explicitly that that you misunderstand the 240 w/m^2. The sun delivers 1366 w/m^2

    No 1366 W/m^2 (1370) is the solar constant.

    Even if the Greenhouse Effect were fictitious, the temperature at the earth’s surface would be greater than whatever 240 w/m^2 would get you.

    No way. GHG absorbing IR terrestrial radiation only assist in cooling. If CO2 absorbs IR,then non-trace gasses can also transport(conduction-convection-radiation) energy away.

    I contend atmospheric pressure better explains the delta T between 255K (via solar input) and 288K actual temps.

    10

  • #
    BLouis79

    Postma has already explained that the mean temperature of earth from space is -18 degC as predicted. Climate scientists seem to like misapplying the StefanBoltzman law. Incoming radiation hits the earth from outside the clouds with albedo of 0.3, so 30% energy is reflected and doesn’t warm the earth. Outgoing radiation comes from the surface with emissivity of 1.0 near perfect blackbody. If clouds are good at blocking outgoing radiation as incoming, then outgoing radiation should be 0.7 times blackbody emission. Measurements of emissivity from satellites appear to suggest emissivity as low as 0.62 (accuracy and reliability unclear).

    Seeing that land and water absorb heat energy in the sun, and that clouds block both incoming and outgoing radiation, it is perfectly feasible that the “warmer” surface could be created with ZERO greenhouse gases. Clouds are constantly moving and the warmed land and water does not emit spontaneously as it absorbs heat energy. So would anyone dare run a GCM allowing for this effect and a ZERO GHG effect (not counting clouds as gases).

    For a more detailed consideration, see http://jennifermarohasy.com/blog/2011/03/total-emissivity-of-the-earth-and-atmospheric-carbon-dioxide/

    10

  • #
    L.J. Ryan

    BLouis79:

    Measurements of emissivity from satellites appear to suggest emissivity as low as 0.62 (accuracy and reliability unclear).

    Emissivity of what? And how is it “measured”? Not measured, but calculated and calculated incorrectly by GHG physics enthusiasts…the point Nasif S. Nahle explaining.

    AGWers use effective emissivity as to obfuscate the physics. Simply rearrange

    P=εσT^4 to solve for ε.

    ε=P/(σT^4)

    P = 240 W/m^2 TOA outgoing
    T= 288K surface temp

    run the numbers…ε=~.62

    So the “science” then contorts itself trying to find measurements justifying the calculations…thus satellites accuracy and reliability are unclear.

    10

  • #
    KR

    L.J. Ryan @ 533

    How does one measure effective emissivity of the Earth’s surface to space?

    * Start with the measured temperature of the Earth’s surface – lots of data for that, an average of ~14C.

    * Calculate the Stefan-Bolzman black-body radiation for 14C, which is ~390 W/m^2 or so.

    * Measure the spectra of IR radiation from the Earth’s surface, integrate to find the power emitted, which is ~240 W/m^2.

    * 240 / 390 = ~0.61 to 0.62.

    Alternatively, you can model the atmosphere using known distributions of CO2, water vapor, observed cloud levels, etc., using line-by-line modelling, and once again get a value of 0.62. The fact that these match is an excellent support for the atmospheric models.

    It’s really not hard, L.J., and the accusations of obfuscation and data distortion (aka Ad hominem fallacies) do not present a convincing case on your part.

    10

  • #
    BLouis79

    @KR #534… is exactly the kind of apples to oranges problem I have noticed. Postma says we measure Stefan Boltzman radiation from space, which gives the effective emission temperature, of -18 degrees (as predicted). Climate scientists agree with the method and formula and calculation, using albedo of 0.3 and solar irradiance. No problem so far. Except that the absorption is the “blackbody spectrum at 30% reflectance”.

    Then when climate scientists want to figure out radiation emission, they use the blackbody surface inside the clouds, not atmosphere as seen from space, which from an emission perspective should be a greybody as all that infrared is claimed to be absorbed by the CO2.

    Now given the effective Stefan Boltzman radiator in equilibrium with the sun is the atmosphere visible from space (or perhaps the atmosphere visible from space with an IR camera if one assumes zero visible light emission), then it follows that the surface of earth must be warm enough to emit more radiation that is blocked by clouds on the way out but the outgoing radiation as measured must still be correct. An AGW anti-G&T physicist agreed that this was the “greenhouse effect”. That doesn’t sound anything like a greenhouse and doesn’t even need greenhouse gases to operate as described, only clouds to block outgoing radiation from patches of earth/water warmed by the sun as earth rotates and clouds move about.

    10

  • #
    KR

    BLouis79 @ 535

    The statement “blackbody spectrum at 30% reflectance” doesn’t make much sense. Blackbody (and graybody) emission is just that – emission, not reflectance.

    The effective emission temperature from space is not -18C; it is a very different, notched, emission spectra (much less efficient than a blackbody) with a power equivalent to a blackbody at -18C. Please look at my post @348, where I note the temperature required to emit 240 W/m^2 for various emissivities.

    Glass greenhouses reduce heat loss by blocking convection and reducing conduction. The radiative greenhouse effect of the atmosphere reduces radiation to space. Not just clouds, mind you (although they do have an effect) – water vapor and CO2 alone reduce radiation to space. In both cases, given a relatively fixed input energy and reduced efficiency of cooling, energy accumulates until the area inside the ‘greenhouse’ is hot enough to dump an energy equal to the incoming.

    10

  • #
    BLouis79

    @KR #536 “blackbody spectrum at 30% reflectance” doesn’t make much sense

    Blackbody spectrum is blackbody spectrum. Greybody spectrum is something else. Earth is absorbing blackbody radiation from the sun with 30% reflected and 70% absorbed. If the emission spectrum of earth from space is a greybody spectrum with a power equivalent to a blackbody at -18deg – either way we are comparing apple to apples – absorption by earth’s atmosphere interface in thermal radiative equilibrium with emission by earth’s atmosphere interface.

    One can’t say equilibrium occurs at two different places – absorption as seen from space and emission as seen from under the clouds.

    There is a case for earth (land and water) acting as a heat sink when in the sun (fine daytime) and a heat source when not in the sun (cloudy or night) and clouds blocking both absorption in the sun and emission at any time clouds are overhead. There is no case for a radiative “greenhouse” by “insulation” or “backradiation”.

    11

  • #
    L.J. Ryan

    KR 534

    * Measure the spectra of IR radiation from the Earth’s surface, integrate to find the power emitted, which is ~240 W/m^2.

    1)Does the atmosphere absorb IR?

    2)Does the atmosphere radiated IR isotropically?

    20

  • #
    BobC

    KR:
    May 24th, 2011 at 6:23 am

    How does one measure effective emissivity of the Earth’s surface to space?

    The problem with calculating things like climate sensitivity from radiation calculations, estimates of the Earth’s emissivity, assumptions about clouds, water vapor etc. is that there are too many places that one can leave something out, put something wrong in, or just get the physics skewed a little.

    If you want to know what the Earth’s surface response is to increased energy input (whether from the Sun, or the radiative greenhouse effect, there is no better way to do it than by actual measurements. Sherwood Idso has done just that.

    His results indicate that the Climate sensitivity to a doubling of CO2 should be about 0.5 deg C. That this is a measurement, not a calculation, means that it includes all the known and unknown effects that are operating. As such, it is, to my mind, much more believable than any amount of theoretic calculations. I’m sympathetic with Paul here.

    10

  • #
    KR

    BobC @ 539

    0.5°C?

    Craig Idso has done some interesting things – I don’t know that I would call many of them “science”. And yes, I’ve read a number of his works.

    Here’s a link discussing multiple measures of climate sensitivity. These come from about nine (9) different approaches – paleo evidence, some climate models , responses to recent volcanic eruptions, responses to the 11 year solar cycle, etc. The result is a range of 2-4.5°C for a doubling of CO2, most likely around ~3°C. About half the methods are model based, the other half are based on observations.

    Lindzen calculated (in a number of later debunked papers) that the sensitivity was ~1°C. That’s one of the lowest sensitivities that has been taken seriously, and multiple faults were found in his methodology. A sensitivity of 0.5°C really doesn’t match the evidence.

    10

  • #
    KR

    BLouis79 @ 537

    One can’t say equilibrium occurs at two different places – absorption as seen from space and emission as seen from under the clouds.

    Bullpucky. If the border conditions (the amount of energy entering and leaving) the surface and the top of the atmosphere sum to zero, you can certainly have different temperatures – shaped by the differing conditions in the two locations. All that is required to satisfy thermodynamics is that input = output at equilibrium. And if both locations aren’t at equilibrium, the entire system is off equilibrium, and will change until that condition is met.

    Different locations, different conditions, different temperatures – equal amounts of energy entering and leaving each location. See Trenberth 2009, and add up the inputs and outputs at every level (atmosphere, ground, sun, space). They all add up to zero – equilibrium.

    11

  • #
    KR

    L.J. Ryan @ 538

    Please see post @280 on the “Second Law of Thermodynamics” thread.

    BLouis79 – Likewise.

    10

  • #
    L.J. Ryan

    KR 541

    All that is required to satisfy thermodynamics is that input = output at equilibrium. And if both locations aren’t at equilibrium, the entire system is off equilibrium, and will change until that condition is met.

    Equilibrium is achieved when the surface is radiating 240 W/m^2…entropy increasing. Forcing energy to 390 W/m^2…entropy decreases.

    10

  • #
    L.J. Ryan

    KR 542

    Your post 280 does not answer my questions:

    1)Does the atmosphere absorb IR?

    2)Does the atmosphere radiated IR isotropically?

    I assume yes for 1 and 2.

    3)What is the flux radiated by the atmosphere?

    10

  • #
    co2isnotevil

    Some may be confused by thinking there’s an energy balance difference between an emissivity quantifying uniform attenuation across an emitted spectrum and the planet’s system of warm surface power selectively replaced with cold cloud power, all spectrally filtered by the atmosphere. The math and physics tells us that the later has an equivalence with the former and that given a complex system like the planet, there exists many equivalent systems conforming to the same physical laws, many of which may be simpler. An equivalent system is one whose behavior at the inputs and outputs is indistinguishable from the other. For example, an electrical circuit comprised of 10 strings of ten 10 ohm resistors in parallel has an equivalent representation as a single 10 ohm resistor. This works for both electrical and climate circuits because of conservation laws and that the behavior of both can be quantified by energy fluxes.

    For the planets energy circuit, the simplest equivalent is an emissivity and equivalent temperature, which when assuming the equivalent temperature is that of the surface, you get an emissivity of about 0.62. This includes atmospheric absorption and the effects of the clouds covering 2/3 of the planet.

    A more precise equation is Po = Ae*(Sp*(1-p) + Cp*p), where Po is the power emitted by the planet, Sp and Cp are the power emitted by the surface and clouds, as calculated with SB applied to surface and cloud top temperatures, p is the fraction of the surface covered by clouds and Ae is the fraction of emitted power that escapes the planet after accounting for net atmospheric absorption.

    The ISCCP satellite data tells us that p is 0.66, Sp is about 388 W/m^2 (287.7K) and Cp is about 267 W/m^2 @ 262K. Given that Po must be 240 W/m^2 @ 255K per the solar input and average albedo, we can calculate Ae, which is the component of the emissivity dependent exclusively on GHG absorption, and whose value is 0.78. Warmists don’t like to quote this value because it implies that only 22% of the emitted surface power is retained by all GHG’s, including water vapor. Even the simplified emissivity of 0.62 implies that only 38% of the power that leaves the surface returns to heat the planet, which is still less than CAGW requires. In fact, the 38% value is correct and can be cross checked by adding 38% of the power emitted by the surface to the post albedo 240 W/m^2 arriving and it should be equal to the power leaving the surface.

    10

  • #
    L.J. Ryan

    co2isnotevil: 545

    For example, an electrical circuit comprised of 10 strings of ten 10 ohm resistors in parallel has an equivalent representation as a single 10 ohm resistor.

    Check your math, parallel resistance is added reciprocally 1/Rt = 1/R1 + 1/R2…1/R10= 10(.1)=1

    Rt=1

    Parallel voltage is as you described.

    10

  • #
    Matt b

    LJ Ryan… ten strings in parallel, with each string consisting of ten x 10 ohm resistors (in series). So that’s 10 x 100 ohm resistors in Parallel = 10 ohm.

    You’ve calculated for ten x 10 ohm resistors in parralel.

    10

  • #
    Smoking Frog

    L.J. Ryan.

    [Smoking Frog] Your confusion is because I failed to state explicitly that that you misunderstand the 240 w/m^2. The sun delivers 1366 w/m^2

    [L.J. Ryan] No 1366 W/m^2 (1370) is the solar constant.

    Sure. It’s what the sun delivers at the top of the atmosphere. You’re discounting for albedo and averaging over the entire sphere of the earth, so you get 240 w/m^2, but I’m saying that the -18 C. that we get from applying Stefan-Boltzmann to it is incorrect, because the night side temperature is not absolute zero, and the 4th root of an average is not the average of 4th roots. I know they say it would be -18 C. without the GHE, so probably I’m wrong, but I don’t see how.

    10

  • #
    co2isnotevil

    LJ,

    See how easy it is to misinterpret a problem when there’s only a little complexity? What do you think happens with the ability to parse a problem when the explanation of the system is more complicated than 10 strings of ten 10 ohm resistors in parallel? Don’t feel too bad since I choose the wording of that sentence carefully so that it was unambiguously precise, yet subject to misinterpretation if you’re not paying attention. You should be used to this as CAGW biased climate scientists must do this all the time when they try to explain the science without blatantly destroying their pet hypothesis in the process.

    George

    10

  • #
    Mark D.

    LJ Ryan @ 246, the full quote of George @ 545 is:

    For example, an electrical circuit comprised of 10 strings of ten 10 ohm resistors in parallel has an equivalent representation as a single 10 ohm resistor.

    Ten strings of 10 ohm (series 100ohms ) resistors in parallel In other words ten strings of ten in series, then connected in parallel.

    I’m rather certain that George knows Ohms law to this degree…..

    10

  • #
    L.J. Ryan

    co2isnotevil: 549

    Got it George. Sorry for misread and my subsequent correction error of post 546.

    10

  • #
    BobC

    KR (@540):

    Here’s a link discussing multiple measures of climate sensitivity. These come from about nine (9) different approaches – paleo evidence, some climate models , responses to recent volcanic eruptions, responses to the 11 year solar cycle, etc. The result is a range of 2-4.5°C for a doubling of CO2, most likely around ~3°C. About half the methods are model based, the other half are based on observations.

    The problem with sensitivity calculations that are supposedly “based on observations” is that many of them make unjustified assumptions. For example, Hansen’s calculations based on observations take the paleo-temperatures, compare the paleo records of GHGs and assume that the GHG concentrations are responsible for the temperatures (rather than the other way around, for example). This gives him the high value he wants, but is not very believable.

    While some of Idso’s calculations rely on assumptions, others are as simple as comparing the change of solar energy in vs. the surface temperature response. The arguments that you see in, for instance, RealClimate that these straight-forward measurements don’t actually give you the system response are themselves based on assumptions (many hidden) that are specific to the CAGW faith.

    The whole AGW-government loop is based on ignoring measurements (which can be contrary) and depending instead on easily molded models of reality.

    Susan Soloman’s oft-quoted paper of a few years ago “proving” that anthropogenic CO2 will remain in the atmosphere for “1000s of years” is a case in point: Soloman exclusively referenced other CO2 cycle model papers, and completely ignored the dozens of actual measurements of CO2 lifetimes which have gotten lifetimes from 5 – 15 years.

    10

  • #
    Hans

    This is another thread on the imaginary ‘Second Law of net heat flow’. In attempts to prove that the Second Law doesn’t contradict with GHG theory, the Second Law itself gets amputated by insisting that it must apply to ‘net heat’. Strangely Michael and Jo think this altering of the Second Law can be justified to fit GHG theory. 150 years long scientist were stupid not to see that the Second Law only applies to a net flow.

    At reply #33
    michael hammer:
    May 14th, 2011 at 9:39 am

    “comment 20 GorkMork you say

    Your third line implies that radiation does not occur and is rubbish. An object in a vacuum can lose heat very rapidly through radiation.”

    This is most crucial in this whole debate, the misinterpretation of heat.
    An object in a vacuum cannot lose heat, it loses energy through radiation. Heat is energy in transfer and never stored, heat itself is not a variable of the object. Heat is energy in transfer between objects.

    As for Jo’s statement “The photons go both ways (actually every way, in 3D). There are more coming from the 11°C block to the 10°C block, sure, but the the 10°C block is sending ‘em back to the 11°C block too. So heat is flowing from cold to hot. It happens all the time.”

    Heat is flowing? No, only radiation might be flowing and this doesn’t make the object gain energy to raise it’s temperature because the gradient is negative (in delta T).

    Definitions of heat:

    – The energy transferred from a high-temperature system to a lower-temperature system is called heat.
    – Any spontaneous flow of energy from one system to another caused by a difference in temperature between the systems is called heat.

    Michael and Jo, you both see radiant energy being the same as heat while it is not. This is the reason the Second Law does not fit in your reasoning and the imaginary ‘Second Law of net energy’ emerges.

    It is of course absurd that after 150 years of knowledge and usage of the Second Law in thermodynamics, to say now that it has always been interpreted and used incorrectly and it only applies to “net flows of heat”?
    The net flow is the heat, and the rest is just radiation.

    Now put one and one together and you’ll see that the real Second Law is correct as stated with the use of real heat (forget radiation) as it is meant. In a system both photons and molecules move in all directions back and forth all the time but there is only one direction of heat transfer. So indeed on average no heat (just radiation) flows from the atmosphere to the surface, because the gradient is negative and the surface temperature will not rise.

    10

  • #
    BLouis79

    (sorry mucked up the formatting – Jo could you delete the last one)

    Here’s a link to a paper by Shaviv showing why climate sensitivity is probably low CO2 doubling sensitivity of 1.3+/-0.4 degK and seems quite historically reliable after accounting for cosmic ray flux. http://www.sciencebits.com/OnClimateSensitivity However, even good statistical correlation doesn’t demonstrate causation.

    Please, surely someone has somewhere done an experiment demonstrating that CO2 *can* cause quantifiable warming and present a sound theoretical justification for the observation. I suspect we have more evidence that warming liberates CO2 from the oceans. Anyone who drinks fizzy drinks knows the phenomenon.

    10

  • #
    KR

    BobC @ 552

    The problem with sensitivity calculations that are supposedly “based on observations” is that many of them make unjustified assumptions. For example, Hansen’s calculations based on observations take the paleo-temperatures, compare the paleo records of GHGs and assume that the GHG concentrations are responsible for the temperatures (rather than the other way around, for example). This gives him the high value he wants, but is not very believable.

    Actually, Hansen’s calculations in that paper are based upon known variations in insolation due to Milankovitch cycles (the “forcing”), and the climate sensitivity is calculated based upon the total change in temperature caused by that forcing. As paleo evidence, this includes short term and long term feedback, and is a pretty strong result.

    The GHG records (where CO2 clearly acts as a feedback, lagging initial temperature changes by ~800 years in the rising part of the cycle) are definitely part of the story, as the temperature changes for just the Milankovitch forcings alone would only be a degree or two, not the 5-6C change seen in the records.

    As a feedback, CO2 (and H2O) is a response to the temperature and partially responsible for amplifying the changes. Along with albedo changes, vegetation, etc. – all the feedbacks.

    Now, of course, we’re changing CO2 independently of temperature, making it a forcing, rather than a feedback.

    The Ad hominem remark about “the high value he wants” is, IMO, neither justifiable nor adds anything to the discussion.

    10

  • #
    RW

    KR (RE: 555),

    Actually, Hansen’s calculations in that paper are based upon known variations in insolation due to Milankovitch cycles (the “forcing”), and the climate sensitivity is calculated based upon the total change in temperature caused by that forcing. As paleo evidence, this includes short term and long term feedback, and is a pretty strong result.

    Is this the one where they calculate that an increase in insolation of about 7 W/m^2 turns into about a 5 C rise (from glacial to interglacial), which is proportional to about a 3 C rise from the 3.7 W/m^2 from 2xCO2?

    I think I’ve seen this before. It’s highly misleading and flawed because it ignores the large change in the distribution of the insolation and tries to equate the positive feedback effect from melting ice from maximum ice to that of minimum ice, where the climate is now.

    10

  • #
    RW

    co2isnotevil (RE: 281)

    The law that’s broken by ‘consensus’ climate science is Conservation of Energy, not the second Law. This breaks because the 3.7 W/m^2 of incremental forcing is turned into 16 W/m^2 of incrementally emitted surface power (corresponding to a 3C rise). The ‘positive feedback’ explanation for this is what breaks Conservation. There are only 2 ways that extra energy (the 12.3 W/m^2 in excess of 3.7 W/m^2) can be emitted by the surface. First is if the Sun emits 12.3/.7 more power (.7 is 1 minus the average albedo), second is if the albedo decreases by enough to let 12.7 W/m^2 more power enter the system. On average, the planet reflects about 102 W/m^2. For it to reflect 12.7 W/m^2 less, the albedo must be reduced by 12.5%.

    The 2 mechanisms warmists cite as the source of this energy is either decreased ice causing decreased reflection or water vapor ‘feedback’. First, even if all the ice on the planet melted, the reflectivity would not decrease by enough for this much effect and second, to achieve this much incremental H2O absorption would require increasing the average water content by more than a factor of 2. Neither of these is consistent with observations or theory.”

    Also, it seems even by the warmers’ own numbers there’s been no decrease in the albedo over the satellite record and the ISCCP surface albedo shows no trend over the 1983-2008 data set.

    I see what you mean. It certainly does beg the simple question of where the energy is coming from.

    10

  • #
    co2isnotevil

    KR,

    Hansen also incorrectly treats the change in surface reflectivity owning to glacial ice. During ice ages, the average reflectivity is much higher owing to the ice extending well into the lower latitudes, even during the summer. This further reduces the power received by the planet and amplifies cooling. Instead, he calls this effect ‘feedback’, which he also assumes is only a feedback relative to warming caused by increased CO2 and not warming caused by increased solar power or changing orbital characteristics. As RW points out, he also neglects the relative asymmetry between the N hemisphere and the S hemisphere which amplifies the differences between perihelion and aphelion and increases the effective change in solar insolation. Hansen also ignores the change in the system’s net response that occurs as the Earth’s axis tilts. When the tilt is greatest, a larger portion of the poles is subject to wider insolation variability, seasonal differences are much larger and perihelion differences have a larger effect. There’s a good reason why the ice core temperature reconstructions shows a very strong correlation to the change in the Earth’s axis with a period of about 45K years. Look at this plot of ice core temperatures along with Earth axis and orbital variability.

    http://www.palisad.com/co2/domec/orbit.png

    It’s a necessary, but not sufficient, condition that the Earth’s axis to be near its minimum for an ice age to occur and for an interglacial to occur, the Earth’s axis must be near it’s maximum. This signal is so strong it should be used to adjust the time line, as the temporal accuracy of DomeC and Vostok gets much worse the further back in time we go. Note that the temperature data has been smoothed to 22K years to cancel out the perihelion/aphelion signal, which is also quite strong and acts to amplify or attenuate the axial tilt signal, where the coldest glacial and warmist interglacial periods are highly correlated to the relative phase of these 2 signals.

    http://www.palisad.com/co2/domec/orbit1.png

    In this plot, it’s clear we’ve always been in a 45K year glaciation cycle, except that the last 2 were suppressed by the relative phase between the precession of perihelion and the axial tilt. This pattern continues through the entire DomeC record as well, going back about 800K years.

    10

  • #
    BLouis79

    Help!

    I’m still looking for experimental verification of the “greenhouse” effect. Then I found this http://fgservices1947.wordpress.com/2011/05/11/there-is-no-such-thing-as-a-greenhouse-gas/ quoting Professor Gert Venter:

    My experiments show that INCREASING ATMOSPHERIC CO2 IS CORRELLATED WITH A DECREASE IN ATMOSPHERIC TEMPERATURE in my agricultural environments.

    10

  • #
    Baa Humbug

    BLouis79: #559
    May 26th, 2011 at 6:57 pm

    Professor Gert Venter: My experiments show that INCREASING ATMOSPHERIC CO2 IS CORRELLATED WITH A DECREASE IN ATMOSPHERIC TEMPERATURE in my agricultural environments

    That’s exactly what THE NET EFFECT of GHGs are, cooling.

    10

  • #
    BobC

    KR (@555):
    May 26th, 2011 at 2:10 am

    Actually, Hansen’s calculations in that paper are based upon known variations in insolation due to Milankovitch cycles (the “forcing”), and the climate sensitivity is calculated based upon the total change in temperature caused by that forcing. As paleo evidence, this includes short term and long term feedback, and is a pretty strong result.

    Assuming that Milankovitch cycles are the sole forcing is exactly the kind of unprovable assumptions I’m talking about.

    The GHG records (where CO2 clearly acts as a feedback, lagging initial temperature changes by ~800 years in the rising part of the cycle) are definitely part of the story, as the temperature changes for just the Milankovitch forcings alone would only be a degree or two, not the 5-6C change seen in the records.

    Again, a tissue of assumptions, justified only by the desire to indict CO2. Many people might look at the 800 year lag and conclude that CO2 is a consequence of higher temperatures, not construct an elaborate model that makes them the cause. How is this positive feedback supposed to be constrained to avoid thermal runaway?

    As a feedback, CO2 (and H2O) is a response to the temperature and partially responsible for amplifying the changes. Along with albedo changes, vegetation, etc. – all the feedbacks.

    Exactly — all the unknown feedbacks, which empirical evidence (as well as theoretical considerations) show have a net negative effect.

    Now, of course, we’re changing CO2 independently of temperature, making it a forcing, rather than a feedback.

    Agreed. The paleo evidence, however, suggests that it is a rather wimpy forcing (compared to the unknown ones), as temperatures have plummeted while CO2 levels remained high.

    The Ad hominem remark about “the high value he wants” is, IMO, neither justifiable nor adds anything to the discussion.

    I’ll give you that. I’ll try to keep it factual. (It is, however, impossible to observe Hansen’s behavior for long and avoid concluding that he is willing to go beyond logical argument himself to promote his cause.)

    10

  • #
    BobC

    KR:
    If you are going to chide me for snarky comments, you should try to tone them down yourself.

    KR (@540):
    May 24th, 2011 at 12:38 pm

    Craig Idso has done some interesting things – I don’t know that I would call many of them “science”.

    10

  • #
    KR

    BobC @ 561

    KR (@555):
    May 26th, 2011 at 2:10 am

    Actually, Hansen’s calculations in that paper are based upon known variations in insolation due to Milankovitch cycles (the “forcing”), and the climate sensitivity is calculated based upon the total change in temperature caused by that forcing. As paleo evidence, this includes short term and long term feedback, and is a pretty strong result.

    Assuming that Milankovitch cycles are the sole forcing is exactly the kind of unprovable assumptions I’m talking about.

    You have to work with the evidence available, BobC. We know that there are long term orbital variations, evidence from ice cores indicates that the glacial cycle isn’t driven by insolation changes – the Milankovitch cycles meet the timing and forcings needed to cause the glacial cycle. So – we have a suspect with method and opportunity; do we dismiss that and hunt for another suspect?

    The GHG records (where CO2 clearly acts as a feedback, lagging initial temperature changes by ~800 years in the rising part of the cycle) are definitely part of the story, as the temperature changes for just the Milankovitch forcings alone would only be a degree or two, not the 5-6C change seen in the records.

    Again, a tissue of assumptions, justified only by the desire to indict CO2. Many people might look at the 800 year lag and conclude that CO2 is a consequence of higher temperatures, not construct an elaborate model that makes them the cause. How is this positive feedback supposed to be constrained to avoid thermal runaway?

    Thermal runaway? Please, that cannot happen with a gain < 1.0; diminishing returns sets an upper limit to any amplification. I don't want to side-track on that, see this link for an in-depth discussion on the topic.

    And yes, historically, CO2 increases (and decreases) were a result of temperature change – upon warming, the oceans release CO2 (less soluble at higher temperatures), reverse on cooling.

    BUT – increased CO2 increases the greenhouse effect, warming things further. Warmer air holds more water at the same relative humidity (increased specific humidity), which warms things more. Melting ice lowers the albedo of the poles, increasing sunlight retained. These are feedbacks – if you’ve been doing the electronics you’ve described, you know that feedback amplification cannot be ignored.

    The final temperature changes in the glacial cycles due to Milankovitch forcings give one (of many!) measure of climate sensitivity to forcings. Perhaps that conclusion is inaccurate due to mis-measures or poor assumptions somewhere along the way.

    But please don’t confuse potential uncertainties (stated in Hansen’s paper, I’ll note) with a complete lack of knowledge. That’s the Argument from Uncertainty fallacy, and it is unjustified.

    10

  • #
    co2isnotevil

    KR,

    But please don’t confuse potential uncertainties (stated in Hansen’s paper, I’ll note) with a complete lack of knowledge.

    Please don’t confuse conclusions based on speculative assumptions with those based on facts and supported with logic. The uncertainties in Hansen’s paper (listed and ignored) are so large, that any meaningful conclusion about the forward effect of CO2 on the climate are precluded. The fact that he thinks his work shows that CAGW is important is absurd.

    10

  • #
    KR

    co2isnotevil @ 564

    Hansen listed his uncertainties – that’s how you do science. But claiming that any uncertainty invalidates everything is pure fallacy.

    As I noted earlier, it’s one of about nine different ways that sensitivity has been computed. The majority come in around 3°C temperature change for a doubling of CO2, and if multiple methods reach the same conclusion, it’s a pretty strong conclusion.

    Lindzen’s number (~1°C) is an outlier, and his methods have been widely noted as faulty. Yours (~0.5°C?) is an extreme outlier, and you have been pointed at issues in your derivation before.

    20

  • #
    Brian

    KR “if multiple methods reach the same conclusion, it’s a pretty strong conclusion” You really do science on this basis? Don’t you think it is a political way of thinking?

    10

  • #
    Paul

    Roy Hogue:
    May 22nd, 2011 at 12:10 am

    If I’m even close to a competent observer then it shows me that there’s very little actually (may I say nothing?) nailed down and accepted as “proved” about AGW in accordance with the scientific method of investigation.

    It is tragic that this debate is not happening in the halls of academia and government.

    I have just read a paper that was linked on this site, I think, where the process of the IPCC is demonstrated to be both unscientific and illogical. Which is what you would expect if you begin with the conclusion that you wish to prove and then spend all your effort in trying to convince others of what you believe but can’t really prove about the real world.

    The Principles of Reasoning. Part III: Logic and climatology

    Need for disambiguation

    In reality, the task is not so simple. Ambiguity of reference by terms in the language of climatology to the associated ideas may obscure the truth. Thus, there may be the necessity for disambiguation of this language before the truth is exposed.

    Most of the claims of the IPCC are couched in language that is deliberately chosen so as to lead to one meaning being taken by the policy-makers [and public] while seeming to adhere to the normal requirements of scientific research but not in fact doing so. This deliberate ambiguity lies at the heart of the controversy that is still raging over this issue.

    After systematically removing all the deliberate verbal confusion the article then goes on to examine the IPCC report in each criterion demolishing the IPCC claim to be scientific. Unfortunately I cannot reproduce the table here so you’ll have to go to the URL to see for yourself.

    _______model____________________________________________________modèle

    _______predictive inference_______________________________________NOT predictive inference

    _______predictions_______________________________________________projections

    _______statistical population______________________________________statistical ensemble

    _______statistical validation_______________________________________statistical evaluation

    _______falsifiable claims___________________________________________non-falsifiable claims

    _______satisfies Daubert__________________________________________does not satisfy Daubert

    A remarkable feature of the two lists is that each member of the left-hand list is associated with making a predictive inference while each member of the right-hand list is associated with NOT making a predictive inference. That predictive inferences are made is a necessary (but not sufficient) condition for the probabilistic logic to come to bear on an inquiry. Thus, I conclude that elements of the left-hand list are traits of an inquiry under a logical methodology while the elements of the right-hand list are traits of an inquiry under an illogical methodology.

    The IPCC report is entirely under the right-hand column, representing an illogical methodology.

    For the benefit of those who mock common sense, it is of note that the IPCC and the grand band of brave ‘scientists’ that support it, are shown to be using ‘heuristics’ to produce ‘modèles’. So these ‘scientists’ are using their ‘common sense’ ideas of how they think things work and basing their ‘modèles’ thereon. But I suppose that because it is scientists that are doing this that makes it legitimate, but for ordinary folks it is not legitimate to apply common sense when evaluating the extravagant claims of the IPCC and their coterie of scientists? “Good for the goose, good for the gander”, I say.

    Paul

    10

  • #
    RW

    KR (RE 565),

    “As I noted earlier, it’s one of about nine different ways that sensitivity has been computed. The majority come in around 3°C temperature change for a doubling of CO2, and if multiple methods reach the same conclusion, it’s a pretty strong conclusion.

    Not if they’re more or less just doing the same things wrong.

    Lindzen’s number (~1°C) is an outlier, and his methods have been widely noted as faulty. Yours (~0.5°C?) is an extreme outlier, and you have been pointed at issues in your derivation before.

    I think co2isnotevil’s method is the most logically sound and probably the most accurate. This is primarily because he makes the fewest assumptions and has kept any heuristics to an absolute bare minimum. It is directly based on the measured response of the climate system to changes in radiative forcing – changes much greater than the measly 3.7 W/m^2 of ‘forcing’ from 2xCO2.

    Other than the halving of the 3.7 W/m^2, what is your primary objection to his method? I fail to see how the ‘gain’ not being a constant value is an issue, or that the additional 3.7 W/m^2 absorbed by the atmosphere will increase the gain slightly. In fact, that the gain changes as the forcing changes is one of the most crucial aspects of the analysis, because the gain changes in the opposite direction of the temperature changes. BTW, if you didn’t happen to notice, this is the same thing that the results of Lindzen and Choi are showing, yet it’s derived from global data. The biggest criticism of L&C is they were only using the tropics, and since they have corrected all other criticisms in the 2010 revision and still got the same result, what’s the problem?

    I have also noticed that ever since L&C did the revision, it’s gotten very little mention in the pro-AGW press and various pro-AGW sites like Skeptical Science, etc. To me, this is very telling in and of itself.

    10

  • #
    co2isnotevil

    RW,

    Mine certainly is based on global data. The satellite data I use is an aggregation of at least 4 geosynchronous satellites and at least 1 polar orbiter and more often at least 5 geosynchronous and 2 polar orbiters. This results in virtually 100% surface coverage with 3 hour samples going back to 1983. The few times where the coverage is less than this are transient and mostly confined to the poles. I’m not surprised that L&C get the same results for the tropical data they used, since if I constrain my data to the tropics, I see mostly the same results as well. If you look at the hemispheres independently, how the system responds across a wide range of forcing becomes self evident.

    This brings up another second law issue, which is that many warmists think the hemispheres are tightly coupled and respond as a unit to seasonal changes in insolation, rather than independently. The reason they think this is because of the observed, large seasonal changes in the average temperatures of the hemispheres, even including ocean temperatures. If the hemispheres aren’t tightly coupled, the planets thermal mass must respond very quickly to changes in forcing, which is counter to the requirements of CAGW.

    However; the thermal mass must be capable of rapidly adapting to changes in forcing, because very little energy crosses the equator, especially considering how much cross equatorial flux would be required to support the slow response times necessary for CAGW. In both the atmosphere and the oceans, the general circulation patterns run parallel to each other at the equator. This is a consequence of the second law where energy can only flow from hot to cold and the hottest part of the planet circumnavigates the equator.

    What they have wrong is the model for how the ocean stores energy, It’s not the 1 degree C per gram of water in all the oceans. The ocean stores energy as a temperature difference between the deep ocean cold which is thermally connected to the cold at the poles and warmer, equatorial surface waters. The thermocline effectively acts as the dielectric in a thermal capacitor which stores thermal energy in a manner that can be quickly ‘charged’ and ‘discharged’. This effectively isolates all the water below the bottom of the thermocline (most of the oceans) from the thermal mass of the planet.

    10

  • #
    BobC

    KR:
    May 27th, 2011 at 12:00 am

    But please don’t confuse potential uncertainties (stated in Hansen’s paper, I’ll note) with a complete lack of knowledge. That’s the Argument from Uncertainty fallacy, and it is unjustified.

    OK, I’ll agree to avoid that fallacy.

    Now, you agree to avoid the Argument from Ignorance fallacy:

    You have to work with the evidence available, BobC. We know that there are long term orbital variations, evidence from ice cores indicates that the glacial cycle isn’t driven by insolation changes – the Milankovitch cycles meet the timing and forcings needed to cause the glacial cycle. So – we have a suspect with method and opportunity; do we dismiss that and hunt for another suspect?

    Well, you might keep in mind that just because you have a suspect, you haven’t necessarily solved the case.

    For example: Here’s a forcing (albedo changes) for which we have no historical records beyond 25 years ago (and no paleo proxies at all). Since we have been making measurements, the albedo forcing has changed nearly 3 times as much (7 W/m^2) as the CO2 forcing change from 1900 to 2000. The associated temperature changes are far more consistent with a 0.5 deg sensitivity than a 3.7 deg one.

    There is NO justification for assuming such large forcing changes didn’t occur in the past, which is what Hansen does to get his result. Since we have no idea what the state of albedo forcing was before 1985, no historical or paleo method can validly determine sensitivity.

    The idea that you can total up ALL the climate forcings 100,000s of years in the past is absurdly arrogant, anyway. Given our demonstrated ignorance of the significant climate forcings, the only valid (and scientific) way to proceed is to empirically measure the Earth’s temperature response to measured changes in energy input — e.g., Idso’s method.

    10

  • #

    These are all very interesting and quite relevant comments on the “green house” effect. It will be good to see Postma’s response. His paper is very similar conceptually to the much more difficult paper (made harder by the author’schoice of symbols) by Ference Miskolczi. Miskolczi, an ex NASA climate scientist whose approach to the problem of green house gas effects differed from that of many of his colleagues, uses the virial theorem from classical mechanics to demonstrate the same result which Postma has arrived at through thermodynamics. The points made in both these papers are I believe correct in strictly following the laws of physics. However, there may be other aspects of the process of warming which are not included, but from a direct analysis of the atmosphere, it is difficult to see immediately what that might be.

    The basic concept in both, which is remarked upon by Postma but not expicitly stated by Misklsczi, is that the heating of the earth above 255 K and the heating of Venus, arises because of a sort of heat pump in which the energy is converted to a higher temperature state by the thermal movement providing compression and decompression as the air circulates between high and low altitudes. The heating at the ground follows from the temperature lapse rate requirement from “de-compression mechanics” to a lower temperature at which insufficient radiation takes place to cool the earth in balance with the sun, unless the mid-height temperature is sufficiently high. The higher round temperature is maintained by compression of the air parcels as they fall from the higher levels near the tropopause to the ground. The main concept in Arrhenius’ green house is that of the back radiation. There are some misconcetions over this factor which need to be clarified.

    First and simplest is that the idea of an increase in temperature by returning more heat to the ground by additional radiation from increased carbon dioxide radiation is not physically correct. The reason can be shown in terms of the absorption by the green house gas at any frequency within the range of its spectrum including the far wings of any line or band of lines. The amount of absorbed radiation at any point, height “h” in the vertical column, is proportional to the energy density of the infra red field at theat point. This is proportional to the exponentential function exp(-k*h) where k is the absorption cefficient and proportional to Nco2, the density of CO2, or any other green house gas. The energy from this absorbed field being reradiated to the ground also decays by a factor exp(-k*h) so what reaches the ground from height h is dependent on exp(-2*k*h). If we now consider each element dh which absorbs energy proportional to Q* k*dh, where Q is a constant representing the other factors pertaining to the transfer of energy of radiation to the sample of air at h and the re-excitation of molecules which radiate, and sum (integrate) over h from 0 to infinity (or at least a great height H), all the contribution towards the total radiaton W returned to the ground, we find on integration the result for this total power (W) being 1/(2*k)*Q*k*exp(-2*k*h) Int 0 to H = (Q/2)*(1-exp(-2*k*H)) which is independent of k or the density of the green house gas in the atmosphere. This is in contrast to the claims by the IPCC that the return radiation warms the earth.

    Secondly, the IPCC makes the point that the 33 K at which the earth temperature exceeds the radiative balance temperature of 255 K is because of green house gases (100%) blocking the radiation ofrom the surface over water or over land. The fact is that over water, the temperature rise is far too low to allow the water to radiate at the equilibrium rate which effect is maintained ny evaporation which warms the air and produces a secondary field of green house gas with no assistance from carbon dioxide. Over land the air is heated by contact with slightly cooler air and the land is cooled. About half of the heating of the air over land is probably by absorption because of green house gases, mainlty water vapour. The three forms of heating lead to a common result – the warming of the warms the air above it and this warm air is carried aloft by convection. In the upper troposphere, the lapse rate in temperature means that the water vapour condenses, causing warming of the drier air which continues to rise. At heights where little or no water vapour exists, the radiation form the remaining vapour which is never actually zero, allows this air to cool. Carbon dioxide continues to entrap the radiation energy corresponding to its absorption bands until it raches a height above which there is too little CO2 to ratain the radiation which again returns to space cooling the air. Some radiation from this height ofcourse goes side ways and downwards, but for the downwards field it is again absorbed before reaching the earth – see the first point made here. We thus see that the grren house gases in the upper atmosphere radiate heat and cool the air which was originally warmed by (1) Evaporation (2) contact between the land and wind and (3) at heights further above the earth but lower in general than about 100 m, by radiation.

    Let’s now look at an atmosphere with out any green house gases other than water vapour. Radiation from the ground at the frequencies of the green house bands will be able to radiate to space from the earth. The air will still be warmed by contact and water vapour and convection will take this energy to the upper parts of the troposphere as before. The water vapour will radiate as before after most, but not all has condensed, at about 5 km height while at higher and colder regions, there will be little or no radiation from H2O. The other main components of the atmosphere, nitrogen and oxygen, cannot be significantly excited by collisions because their lowest excited states are of too high energy to be reached from energetic collisions at 300 K. Thus no further radiation can take place from here up and the air retains its energy while cooling by gravitational transfer to potential enrgy and by adiabatic expansion. It travels across the top of the troposphere to other higher latitudes witha higher energy contact thanit would had there been some small amout of carbon dioxide in place to allow it to radiate its energy. At the higher, cooler jatitudes it does, as now, fall to ground under its own weighr, warming as it does by the transfer of gravitational energy and adiabatic compression. The mid latitudes are thus made warmer than they would be if carbon dioxide had been present to form a radiation field which could escape to space.

    These are all very interesting and quite relevant comments on the “green house” effect. It will be good to see Postma’s response. His paper is very similar conceptually to the much more difficult paper (made harder by the author’s choice of symbols) by Ferenc Miskolczi. Miskolczi, an ex NASA climate scientist whose approach to the problem of green house gas effects differed from that of many of his colleagues, uses the virial theorem from classical mechanics to demonstrate the same result which Postma has arrived at through thermodynamics. The points made in both these papers are, I believe, correct, in strictly following the laws of physics in the points they make. There may be other aspects of the green house effect which should have been included in their analysis but it is not immediately apparent what this may be.

    The basic concept in both papers, which is remarked upon by Postma but not explicitly stated by Miskolczi, is that the heating of the earth above 255 K and the heating of Venus, arises because of a heat pump mechanism in which the energy is converted to a higher temperature state by the thermal movement providing compression and decompression as the air circulates between high and low altitudes. The heating at the ground follows from the temperature lapse rate requirement from “de-compression mechanics” to a lower temperature at which insufficient radiation takes place to cool the earth in balance with the sun, unless the mid-height temperature is sufficiently high. The higher round temperature is maintained by compression of the air parcels as they fall from the higher levels near the tropopause to the ground. The main concept in Arrhenius’ green house is that of the back radiation. There are some misconcetions over this factor which need to be clarified.

    First and simplest is the idea of an increase in temperature by returning more heat to the ground by additional radiation from increased carbon dioxide diation is not physically correct. The reason can be shown in terms of the absorption by the green house gas at any frequency within the range of its spectrum including the far wings of any line or band of lines. The amount of absorbed radiation at any point, height “h” in the vertical column, is proportional to the energy density of the infra red field at that point. This is proportional to the exponential function exp(-k*h) where k is the absorption coefficient and proportional to Nco2, the density of CO2, or any other green house gas. This will be the case, whether or not the functional dependence of the intensity strictly follows de Beere’s law – which it doesn’t unless one corrects for the changes in density and temperature which is the usual case and is easily achieved in numerical representation.

    The energy from this absorbed field being reradiated to the ground also decays by a factor exp(-k*h), so what reaches the ground from height h is dependent on exp(-2*k*h). We now consider each elemental slab of thickness dh, which absorbs energy proportional to Q* k*dh, where Q is a constant representing the other factors pertaining to the transfer of energy of radiation to the sample of air at h and the re-excitation of molecules which radiate. Summing (integrating) over h from 0 to infinity (or at least a great height H), all the contribution towards the total radiation W returned to the ground, we find on integration the result for this total power (W) being 1/(2*k)*Q*k*exp(-2*k*h) Int 0 to H = (Q/2)*(1-exp(-2*k*H)) which is independent of k or the density of the green house gas in the atmosphere. The last quantity exp(-2*k*H) is of course very small. This result is in contrast to the claims by the IPCC that the returning radiation increases with higher density of CO2 and warms the earth.

    Secondly, the IPCC makes the point that the 33 K at which the earth temperature exceeds the radiative balance temperature of 255 K is because of green house gases (100%) blocking the radiation from the surface over water or over land. The fact is that over water, the temperature rise is far too low to allow the water to radiate excess heat at the equilibrium rate, an effect which is maintained by evaporation, which warms the air and produces a secondary field of green house gas (water vapour) with no assistance from carbon dioxide. About half of the heating of the air over land is probably by absorption in green house gases, mainly water vapour, while it is also warmed by the slightly cooler air being in contact with the heated land surface. Meteorological estimates place the warming of the air at 20% by Green house gases, 20% by wind contact over land, 60% by evaporative cooling of water. The three forms of heating lead to a common result – the warming of the air above the earth and this warm air being carried aloft by convection.

    In the upper troposphere, the lapse rate in temperature means that the water vapour condenses, causing warming of the drier air which continues to rise. At heights where little or no water vapour exists, the radiation from the remaining vapour which is never actually zero, allows this air to cool. Carbon dioxide continues to entrap the radiation energy corresponding to its absorption bands, until it reaches a height above which there is too little CO2 to retain the radiation, which then escapes to space, cooling the air in that vicinity. Some radiation from this height of course goes side ways and downwards, but for the downwards field it is again absorbed before reaching the earth – see the first point made here. We thus see that the green house gases in the upper atmosphere radiate heat and cool the air which was originally warmed by (1) Evaporation (2) contact between the land and wind and (3) at heights further above the earth but lower in general than about 100 m, by the absorption of radiation.

    Let’s now look at an atmosphere without any green house gases other than water vapour. Radiation from the ground at the frequencies of the green house bands will be able to radiate to space from the earth. The approximately 4, 10 and 15 micron bands of CO2 will no longer play any part! The air will still be warmed by contact and water vapour and convection will take this energy to the upper parts of the troposphere as before. The water vapour will radiate as before after most, but not all of it, has condensed, at a height of about 5 km, while at higher, drier and colder regions, there will be little or no radiation from H2O. The other main components of the atmosphere, nitrogen and oxygen, cannot be significantly excited by collisions because their lowest excited states are of too high energy to be reached from energetic collisions at 300 K. Thus no further radiation can take place from here up and the air retains its energy while cooling by gravitational transfer to potential energy and by adiabatic expansion. It then travels across the top of the troposphere to other higher latitudes, driven by the circulation initiated and pumped by warming of the sun in the tropics, but with a higher energy than it would have, if there had been some small amount of carbon dioxide in place to allow it to radiate its energy.

    At the higher, cooler latitudes it does, as now, fall to ground under its own weight, warming as it does by the transfer of gravitational energy and adiabatic compression. The mid latitudes are thus made warmer than they would be if carbon dioxide had been present to form a radiation field escaping to space.

    It should be noted that the absence of any need for a green house gas to cause the full 33 K of global warming above 255 K, is that the basic and most significant effect arises through the redistribution of the energy from the tropics to higher, cooler, latitudes. The circulation and redistribution process induced by dry, “green house free” air, depending only on contact for its heating, is demonstrated every time a vacuum flask loses its valuable vacuum. A large flask of liquid nitrogen, which could be held by a good quality flask for about 16 days to half full, will lose its contests by violent boiling in about 16 minutes or less if the outer wall is punctured – bin there, done that! The air, let into the previously evacuated cavity, and in contact with the cold inner wall, is cooled and falls, producing convection and circulation in the opposite mode from the heating and rising of air in the tropics but following exactly the same principles. Eventually the whole chamber reaches the same temperature as the room, simply because of this circulation. The same thing would happen in a green house free atmosphere and the average temperature of the earth will still rise for a green house free, perfectly dry atmosphere.

    I look forward to others comments and criticisms. John Nicol

    10

  • #
    KR

    BobC @ 570

    Palle 2004 is a very interesting piece of work – the idea of using reflected Earthshine as a measure of albedo is quite nice. I understand that Leonardo Da Vinci first proposed that.

    Note, however, that this is an initial piece of work, and that there is evidence that Palle is greatly overestimating changes, such as in Wielicki et al 2005 – checking Palle’s data against satellite measurements of Earth brightness. Also see Wild 2009; there does appear to be some internal variability in albedo on a decadal level. But there isn’t any evidence for multiple century timescale albedo variability, which would be required for this to act as a long term (10’s of thousands of years) climate forcing such as in the glacial cycles.

    Given the reviews of this particular matter, I expect that the albedo changes are closely tied to indirect aerosol effects. We do have a good paleo record of those – particulates in the ice cores. And they don’t correspond to the glacial cycles.

    There is NO justification for assuming such large forcing changes didn’t occur in the past, which is what Hansen does to get his result. Since we have no idea what the state of albedo forcing was before 1985, no historical or paleo method can validly determine sensitivity.

    You still seem to be claiming “we don’t know enough, so it can’t possibly be the best estimates of what’s happening based upon the physics and data”. I would have to disagree.

    Now, as to reviewing the Earth response to measured changes in energy input, i.e. within the instrumental record period, I would recommend Tung 2007 (statistical analysis on 20th century temperature response to the solar cycle to calculate a range 2.3 to 4.1°C), or perhaps Bender 2010 (looks at the climate response to the 1991 Mount Pinatubo eruption to constrain climate sensitivity to 1.7 to 4.1°C).

    10

  • #
    BobC

    KR: (@572)
    May 28th, 2011 at 12:24 am

    BobC @ 570

    Palle 2004 is a very interesting piece of work – the idea of using reflected Earthshine as a measure of albedo is quite nice. I understand that Leonardo Da Vinci first proposed that.

    Note, however, that this is an initial piece of work, and that there is evidence that Palle is greatly overestimating changes, such as in Wielicki et al 2005 – checking Palle’s data against satellite measurements of Earth brightness. Also see Wild 2009; there does appear to be some internal variability in albedo on a decadal level. But there isn’t any evidence for multiple century timescale albedo variability, which would be required for this to act as a long term (10′s of thousands of years) climate forcing such as in the glacial cycles.

    You are pulling the “Argument from Ignorance” fallacy again. Because we don’t have evidence of albedo changes over a “multiple century timescale” the proper scientific path is to assume they don’t exist and continue to propose massive changes to the world economic system on admitedly incomplete data.

    Here’s a tip from an engineer: Data you are not aware of does not always work out in your favor. Some caution is advised — however, I see no caution in Hansen who is gung-ho on remaking the civilization based on his unsupported assumptions.

    But actually, we DO have evidence of multiple century albedo changes — studies of paintings from the LIA have shown a dearth of depicted sunshine and commonly show overcast skys. Of course, that’s not “scientific” evidence, so you’re free to claim the scientific high ground while proceeding full speed ahead with blinders on.

    (And actually, Palle did compare his measurements with satellite date — check page 10 in his Univ of Colo talk I referenced.)

    Given the reviews of this particular matter, I expect that the albedo changes are closely tied to indirect aerosol effects. We do have a good paleo record of those – particulates in the ice cores. And they don’t correspond to the glacial cycles.

    Don’t you think that we need more than just your “expectation” that this assumption is right before declaring what is fact? Does this “easy standard” of proof also apply to critics, or only to AGW proponents like Hansen (and you)? Is it not possible that albedo is a feedback that responds to climate changes? (The anecdotal evidence from the LIA certainly suggests that.)

    You still seem to be claiming “we don’t know enough, so it can’t possibly be the best estimates of what’s happening based upon the physics and data”. I would have to disagree.

    This is a misstatement of my position: I don’t deny that Hansen may have made a “best estimate” given his assumptions — just that, because of the large uncertainties in what he assumes, his conclusion is basically of no value in determining what happened in the real world. You’re bordering on “Argument from Ignorance” again: ‘Because this is the best we can do, it must be right.’

    Now, as to reviewing the Earth response to measured changes in energy input, i.e. within the instrumental record period, I would recommend Tung 2007 (statistical analysis on 20th century temperature response to the solar cycle to calculate a range 2.3 to 4.1°C)

    And naturally (seems to be the standard position in AGW) they ignore the developing evidence of a strong connection between solar activity and clouds, and just make the assumption that the only effect the Sun can have on climate is due to TSI. Since TSI only changes little during the typical solar cycle, this again gives a satisfactorily high sensitivity.

    I would be willing to consider these people scientists, if they were interested in what effects Svensmark’s work would have on their estimates. What I hear from the pro AGW blogs (and what I expect to hear from you), however, is an attack on this work and attempts to justify ignoring it in favor of the assumptions that result in supporting the alarmist position.

    This is the behavior of political acvocates, not scientists.

    10

  • #
    KR

    BobC @ 573

    I’m quite aware that variations you don’t know about can bite you. But why do you assume that all the “unknowns” would make the climate more stable? If there are hidden relationships or measurement uncertainties, they could just as well be on the nasty side. The climate sensitivity estimates I’ve read about are very solid on the low end – about 1.5°C is a strong limit, anything less contradicts a great deal of the data. But on the high end the limitations are pretty soft – it could be 4°C, it might be > 6°C per doubling of CO2!!

    Is it reasonable to expect everything to go our way? Especially when the costs of mitigation are actually not all that bad? Renewable energy supplies won’t kill us, they’ll take a while to implement, and we can always trim those back if 15-20 years out we find that the warming estimates are overblown. I think it’s only reasonable to act in our best long term interests, with the best knowledge we currently have.

    I’ve been following Svensmark’s work – so far, though, the correlations he’s found are very weak. And quite notably, the timescale for cosmic ray levels (as estimated by various proxies) does not match the temp record anywhere nearly as well as CO2 does (Royer 2004). But I will continue to look at ongoing work on this issue.

    Tung looked at TSI – Bender looked at volcanic aerosols. As to cloud changes, of course there are cloud changes with temperature! Clouds are water, and are expected to respond to changing temperatures and water vapor levels. The measures of climate sensitivity are looking at changes in forcing (initial imbalance) versus temperature, including the feedbacks. That’s what the climate sensitivity numbers are all about.

    10

  • #
    Mark D.

    KR, Spencer has a column on cosmic influence perhaps you haven’t seen yet: http://www.drroyspencer.com/2011/05/indirect-solar-forcing-of-climate-by-galactic-cosmic-rays-an-observational-estimate/

    Quote: “While I have been skeptical of Svensmark’s cosmic ray theory up until now, it looks like the evidence is becoming too strong for me to ignore.”

    10

  • #
    co2isnotevil

    KR,

    Weather satellite data tells us that the albedo exhibits hemispheric specific seasonal change in excess of 20% of it’s nominal value. This is a consequence of the ebb and flow of surface ice/snow and seasonally variable cloud cover, both of which show similar variability. Why do you think that albedo will be constant as forcing changes, when the data indicates otherwise?

    Moreover, clouds coverage increase as the temperature increases, which except at the poles when the surface and clouds have about the same reflectivity, increasing clouds reflects more power away from the planet than they trap at the surface, causing net cooling. Can you say negative feedback? In the following plot, the term called ‘flux’ is the sensible heat in and out of the planets thermal mass calculated as the difference between the measured power arriving and the measured power leaving the planet. Based on the measured temperature variability, the exact size of the planets thermal mass can be determined.

    http://www.palisad.com/co2/plots/wbg/nh/temp.png
    http://www.palisad.com/co2/plots/wbg/nh/refl.png
    http://www.palisad.com/co2/plots/wbg/nh/gain.png

    Note that the albedo is that measured by reflected solar power, as observed from weather satellites and then adjusted to fit a radiative transfer model. It seems a little low, but I can track that to the an error made in the original ISCCP analysis which was inappropriately assuming AU normalized solar power and the use of a primitive atmospheric radiative transfer model for the ISCCP reconstructions. You should also notice that the gain increases when it’s cold and decreases when it’s warm. This quantification of gain is equal 1/e, where e is the average emissivity of the plane and is yet another indication of net negative feedback. If positive feedback was in effect, the net gain would be increasing as the temperature increases.

    The southern hemisphere shows similar results, but it’s easy to be confused about how the planet behaves if you only look at the global data, all of this and more is available from this link.

    http://www.palisad.com/co2/plots/wbg/plots.html

    George

    10

  • #
    KR

    Mark D @ 575

    Thanks, I have seen that Spencer posting. I’ve also seen the rather more critical Real Climate take on the data.

    It’s noteworthy that the solar and galactic cosmic ray (GCR) data (see the RC link, figure 5) show no trends over the last 50-70 years, while we’ve had considerable temperature change over that period (which correlates quite well with CO2 changes). I think GCR and insolation variations have an influence (limited in the case of GCR’s), but don’t appear to be the dominant forcing right now.

    But as I said, I’m watching the papers and data to see what comes out.

    10

  • #
    KR

    co2isnotevil @ 576

    George, you’re still focusing on seasonal effects. None of the data you show extends more than a year – short term variation, not enough time for ocean temps (the big flywheel) to change significantly in any direction.

    Climate changes take decades.

    10

  • #
    co2isnotevil

    KR,

    Of course RC would be critical of a paper by a skeptic, it’s their purpose for existing. You have to be careful with Schmidt. He has the ability to mix real science with speculative conclusions in a way that obscures the unsupportable nature of his diatribe. I suspect this is why Hansen gave him the job of RC censor. I don’t think Hansen is stupid, I just think he’s an ideologue with an agenda, or in other words, an idiot who will say and do anything to promote his idiocracy.

    10

  • #
    BobC

    KR (@574):
    As to cloud changes, of course there are cloud changes with temperature! Clouds are water, and are expected to respond to changing temperatures and water vapor levels. The measures of climate sensitivity are looking at changes in forcing (initial imbalance) versus temperature, including the feedbacks. That’s what the climate sensitivity numbers are all about.

    I don’t think you’re getting my point. In the estimates of climate sensitivity you are talking about the feedbacks are assumed. The GCMs that AGW is based upon do not handle clouds or water vapor by calculation from first principles (which we don’t know how to do) but by parameterizations that are adjusted to get a fit with the last 100 years. (Still don’t do too well at postdicting the MWP or LIA, though — that’s why M. Mann’s “hockey stick” is so well accepted — it eliminates both through bogus statistical manipulation of tree ring data. If you think “bogus” is too strong, what do you call a method that gets the same answer if fed random “data”?)

    The only way to include the feedbacks which are poorly understood or even unknown, is to measure the input and output — e.g., Idso’s method again. Like it or not, it is the only believable method of estimating sensitivity.

    Is it reasonable to expect everything to go our way? Especially when the costs of mitigation are actually not all that bad? Renewable energy supplies won’t kill us, they’ll take a while to implement, and we can always trim those back if 15-20 years out we find that the warming estimates are overblown. I think it’s only reasonable to act in our best long term interests, with the best knowledge we currently have.

    This paragraph is so full of unjustified assumptions that it would take an essay to address them. As to “not killing us” — maybe not, but it surely will cause a great many unnecessary deaths in the Third World.

    But the statement that we use “the best knowledge we currently have” — My complaint with the AGW lobby is that they try to suppress anything that would upset the current poitical “consensus”. Not people I want making these kind of life and death decisions.

    10

  • #
    KR

    BobC @ 580

    Well, I’m going to just agree to disagree with you on the costs and risks. Just keep in mind – uncertainty cuts both ways, and climate change may just as well be worse than predicted.

    As to the climate sensitivity measurements, measuring in/out is exactly what Hansen, Tung, Bender, and others have done.

    that’s why M. Mann’s “hockey stick” is so well accepted — it eliminates both through bogus statistical manipulation of tree ring data. If you think “bogus” is too strong, what do you call a method that gets the same answer if fed random “data”?

    M&M’s analysis of Mann’s work has serious problems, not the least of which involves running thousands of random data sets and hand selecting the dozen or so most like the ‘hockey stack’ as examples (including some that they inverted). And Mann’s work isn’t the only one with those results – see Mann 2008 for a listing and figure showing half a dozen independent reconstructions that all agree.

    Idso is chairman of the Center for the Study of Carbon Dioxide and Global Change, which does not disclose it’s funding – I’ve read through their materials, and see major reiterations of skeptic myths, disinformation, data manipulation, etc. I’ve also read refutations on Idso’s papers wrt. ocean acidification, aerosol effects, the MWP, etc. – it’s entirely too easy to find the errors in his works, they’re not solid.

    To be quite honest I don’t trust Idso to give a point of view unbiased by ideology. His work reads more like advocacy papers than science.

    So – trust the people running advocacy groups? Or grant funded scientists? People whose poor methodology has been repeatedly (and rightly) criticized? Or people whose works have held up to peer review and ongoing discussions in the scientific literature?

    Take your pick – I’ll take mine.

    At this point we’re miles off topic (2nd law, radiative greenhouse effect). I have limited time for a while, so I’ll just say bye for now.

    10

  • #
    co2isnotevil

    KR,

    The plots are the averages of monthly averages going back to 1983, representing nearly 3 decades of measurements with 100% surface coverage at 3 hour samples. Everything in this plots is what you don’t see when your perspective is limited to anomaly plots, which report differences relative to these longer term monthly averages.

    George

    10

  • #
    BobC

    KR: Just a few quick comments, in case you check back in:

    As to the climate sensitivity measurements, measuring in/out is exactly what Hansen, Tung, Bender, and others have done.

    No, they have assumed the in and measured the out. Their results are only as good as their assumptions. You know my opinion of their assumptions — incomplete and deliberately ignoring current research on solar and albedo effects.

    M&M’s analysis of Mann’s work has serious problems, not the least of which involves running thousands of random data sets and hand selecting the dozen or so most like the ‘hockey stack’ as examples

    The point KR, is that is exactly what Mann does — he selects tree ring histories that match the 20th century (up to ~1980) from many sets of records. The claimed purpose is to find trees which are mostly sensitive to temperature (rather than wind, insolation, exposure, disease, etc.). However, the fact that these tree histories diverge both before (the very large error limits) and after (“hide the decline”) the selection period shows that he wasn’t successful, and might as well been selecting from random sequences. M&M showed that, indeed, if you simply used Mann’s algorithm (selection for 20th century temperature match in the first few principle components), then averaged the selected sequences together, you got the same “hockey stick” graph — complete to the “divergence problem”.

    And Mann’s work isn’t the only one with those results – see Mann 2008 for a listing and figure showing half a dozen independent reconstructions that all agree.

    “Independently” using Mann’s algorithm doesn’t count. And these half-dozen reconstructions may agree with each other, but are in complete disagreement with the hundreds of ice cores, ocean cores, other reconstructions, and historical records that show that the MWP and the LIA did exist, and were global in scope.

    So – trust the people running advocacy groups? Or grant funded scientists? People whose poor methodology has been repeatedly (and rightly) criticized? Or people whose works have held up to peer review and ongoing discussions in the scientific literature?

    Please — I am a “grant funded scientist”; I have participated in peer review (from both ends): I wouldn’t trust some of those people to feed my cat. The main agenda of a “grant funded scientist” is to continue to receive grants — if you fail, you are no longer a member of that class. You are selecting for people who will do whatever it takes to get funding. (This was obvious from the Climategate emails — and more importantly, from the faked data the released computer codes inserted into the record.)

    If you want to see what has happened here, you should read Eisenhower’s Farewell Address:

    Akin to, and largely responsible for the sweeping changes in our industrial-military posture, has been the technological revolution during recent decades.

    In this revolution, research has become central, it also becomes more formalized, complex, and costly. A steadily increasing share is conducted for, by, or at the direction of, the Federal government.

    Today, the solitary inventor, tinkering in his shop, has been overshadowed by task forces of scientists in laboratories and testing fields. In the same fashion, the free university, historically the fountainhead of free ideas and scientific discovery, has experienced a revolution in the conduct of research. Partly because of the huge costs involved, a government contract becomes virtually a substitute for intellectual curiosity. For every old blackboard there are now hundreds of new electronic computers.

    The prospect of domination of the nation’s scholars by Federal employment, project allocations, and the power of money is ever present – and is gravely to be regarded.

    10

  • #
    RW

    KR (RE: 572),

    “As to cloud changes, of course there are cloud changes with temperature! Clouds are water, and are expected to respond to changing temperatures and water vapor levels. The measures of climate sensitivity are looking at changes in forcing (initial imbalance) versus temperature, including the feedbacks.”

    Most of the 3 C of warming predicted by the models comes from positive cloud feedback. If you read the fine print in the IPCC 2007 report, they state that if the cloud feedback is neutral, the average sensitivity would come down to 1.9 C. What they don’t say is how low it would be with even a moderately negative cloud feedback (probably less than 1 C). Yet the evidence for net positive cloud feedback is sketchy a best and conflicts with the basic physics of water vapor and clouds.

    I brought this up over at Skeptical Science, and no one seemed to be able to explain how if water vapor is the primary amplifier of warming, what then is controlling the energy balance of the system if not clouds through their ability to reflect sunlight and precipitate out the water vapor from the atmosphere?

    Studies like Dessler 2010 are primarily just looking at TOA net fluxes and temperatures – he’s made little (if any) attempt to carefully discern cause and effect or come up with any physical reasons or mechanisms behind his interpretation of the data. He admits in the paper at the beginning that the net effect of clouds at the current operating point in the climate is to cool by about 20 W/m^2, yet doesn’t seem to ascribe much significance to this or any curiosity as to why this is the case.

    Furthermore, I notice that in Dessler 2010 the SW component is also positive, which seems to be consistent with decreasing clouds causing the warming rather than warming causing decreasing clouds. Even if the claim is warming causes decreasing clouds for positive feedback, how is this consistent with increased water vapor from warming? Does increasing water vapor from warming cause decreasing clouds? That doesn’t make any sense since water vapor concentration drives cloud formation.

    It seems to me that unless Dessler can explain all of this and corroborate it with all the other data and system behavior, he really doesn’t have a case.

    10

  • #
    Blimey

    LOL @ Lionell Griffith 16 !!

    10

  • #
    John Nicol

    “Voila… whatever heat transfer goes from greenhouse gases to the Earth is more than countered by the heat moving from the Sun to Earth and on to space.”
    I don’t understand the statement that the heat from GHGs to earth “is more than countered” by heat moving from the Sun to Earth and on to space”?? Could you please explain further? I may not very quick.

    “Greenhouse gases can heat the Earth as long as the entropy of the whole system increases.” I can’t see a situation in an open system such as this where the entropy as a whole would not increase.
    Any natural transfer of heat will increase the entropy and it puts no constraint on the effect of such heating. The total entropy of a closed system cannot be reduced (Maxwell’s Demon) only part of a sytem which is for instance the cooler end of a heat pump – air conditioning, refrigerator, but the entropy of ALL of the system – hot air section plus cold air section is still higher than at the beginning. The energy of the pump goes into the total mix as well.

    Back to the transfer of heat, the confusion arises about ‘Hot to cold but not Cold to Hot’ because of the omission of the word “net” when referring to heat moving between bodies. For radiation between bodies, each will radiate power from its surface at a rate according to Stefan’s Law, Power/area = Sigma*T^4. The hotter body will absorb the power from the colder body and the colder body will absorb power from the hotter. The NET flow then is from the hotter to the colder – always. But photons or if you prefer, EM radiation, go both ways but at different rates or at the same rates but at different energies for photons. Both conduction and radiation function in a somewhat similar way for this analysis. Conduction causes transfer of heat by very close collisions between particles of different enrgy and their total energy tends to divide evenly between them, if you like between molecules, or crystal segments perhaps, of the solid. In each part of the solid there will be a narrow but finite distribution of energy between all these small particles all of which will be microscopically vibrating. The energy of vibration we call heat (and can be represented by a phonon instead of a photon!). At absolute zero, (0 K)the vibration will be reduced almost to zero – except for a bit we will ignore here. All these little moving bits will not in general have exactly the same energy (~kT) or equivalent temperature T – on a macroscopic scale they will, but not on a microscopic examination. Some energy, from more highly energetic particles in the cooler parts may well transfer to the hotter part, just as for the radiation, but on average much more energy will be transferred from the hotter, more energetic particles to those which are vibrating more slowly or with smaller amplitudes.

    10

  • #
    Alistair

    RW: 584. Another thing with clouds is that they may well getter CO2 through dissolution in the water [v. high de facto Henry’a law coefficient].

    10

  • #
    Paul

    John Nicol: @586
    May 28th, 2011 at 10:41 pm
    “Voila… whatever heat transfer goes from greenhouse gases to the Earth is more than countered by the heat moving from the Sun to Earth and on to space.”
    I don’t understand the statement that the heat from GHGs to earth “is more than countered” by heat moving from the Sun to Earth and on to space”?? Could you please explain further? I may not very quick.

    Thank you, John, for a readable explanation that adds something to our understanding of the subject under discussion. Your calm tone and logical presentation are a welcome contrast to the heat rather than light that seems to pertain more often than not in comments here.

    I think that much resistance to the concept of ‘back radiation’ comes about because of the hidden assumption that back-radiation causes of a 33 degrees Celsius increase in temperature at the surface. This is then attributed entirely to the back-radiation of Greenhouse gasses in the atmosphere. The level of Greenhouse gases is then said to be controlled by the level of atmospheric CO2 as the driver with atmospheric H2O merely a dependent variable amplifying the effect of CO2. To me that chain of reasoning is a total falsity and nonsense.

    Firstly I object to the value of 33 degrees Celsius. The difference between the earth’s average surface temperature, regarded as a graybody without an atmosphere, and its estimated temperature with our actual atmosphere, is in fact about 8 degrees Celsius. Nor can it be attributed entirely to the presence of Tyndall gasses in the atmosphere, ignoring conduction, convection, advection and latent heat of vaporisation as if they add nothing to atmospheric temperatures.

    Then I object to the use of verbal reasoning to ‘prove’ a qualitative effect, the dimension of which is completely unknown, and then to attribute to that effect the difference between a blackbody and the earth’s known surface temperature.

    The argument is made that Tyndall gases act as an insulating blanket, slowing the rate at which the earth can lose, by long-wave radiation, the heat gained from the sun. Then an increase in surface temperature is argued to occur in order to overcome that resistance to heat-loss. To quantify that resistance and therefore to quantify the necessary temperature increase to overcome it, from first principles, seems to be necessary.

    While pursuing a better understanding of the physics of radiation of energy I have noticed that, where there is no pressure to conform to a consensus of understanding on the contentious issue of Anthropogenic Global Warming, there is a ready acknowledgement that there is a two-way exchange of energy between two bodies that are in a radiative relationship together.

    To take as an analogy the waves and tides in the ocean, although individual waves do exchange energy back and forth, the direction of the tide does not change except under a changed force of gravity. The behaviour at the micro-level, of individual waves, says nothing about the macro-level of the tidal movements. In the same way, my understanding of the exchange of radiation energy is that, while there are two-way interactions occurring at the micro level of particles, the flow of energy is always in the direction of the more energetic to the less energetic body, as measured by relative temperatures.

    There is one thing that needs to be quantified, to my way of thinking, and that is the measure to which the atmosphere is able to absorb increased radiant energy as the proportion of atmospheric CO2 increases. That will increase the number of potential states of particles within the atmosphere, and, since energy will be equipartitioned within those states, it will have a small effect on the total energy ‘contained’ in the atmosphere, as internal vibrational energy, without increasing its temperature significantly. That increase in retained energy would increase the backward pressure of radiation from the atmosphere that is acting against radiation emitted from the surface and would produce a quantifiable effect on the surface temperature.

    Any feed-back in the system would either dampen or amplify any direct change in temperature. Most likely there is a negative feed-back that would offset any increase in temperature, leading to the expectation of a negligibly small change in surface temperatures.

    So my conclusion is that while the back-radiation from atmospheric CO2 does not break the second law of thermodynamics, because it is a trace gas in the atmosphere, it’s radiative effect simply needs to be quantified in order to see that it is of no real significance and would be benign at best and probably imperceptible at worst.

    Paul

    10

  • #
    RW

    co2isnotevil (RE: 576),

    Have you ever plotted the post albedo with the gain plots? I do understand that you’re using the total incident solar power in those graphs because the albedo (particularly the cloud portion) is a big part of the control mechanism. However, it has occurred to me that some warmists might try to argue that the post albedo incident power is more equivalent to GHG ‘forcing’ than total incident solar power.

    20

  • #
    RW

    co2isnotevil (RE: 576),

    Oh, I do see that in the “monthly flux” graphs, the post albedo or ‘power in’ tightly tracks the total solar power in and is still 180 degrees out of phase with the ‘gain’.

    10

  • #
  • #
    Graeme Bird

    “The 2nd Law of Thermodynamics applies to net flows of heat, not to each individual photon, and it does not prevent some heat flowing from a cooler body to a warm one.”

    This is very hard to assess since we have no coherent mainstream view of what light is. There are no photons. But generally speaking most forms of energy are forms of compression. Waves contain areas of compression and de-compression. But in this case we don’t know what the medium is. If it was only to do with one wavelength we would have an easier way to think about it.

    But its all just cold referred light. Not anything to be paying attention to. And this all came about with the astronomers and Carl Sagan, tendentiously trying to prove that Venus wasn’t a new planet.

    Thats the real history to this. The extended back-history isn’t that important. The extended back-history is just a revamped dead end in science.

    10

  • #
    Roy Hogue

    BLouis79 @591,

    Possibly of interest in your discussion: the Air Force has been concerned, among other things with how hard or easy it is for aircraft to be detected by their IR signature…those jets get real hot. How far away can they be seen by IR?

    A lot of work was done and is still ongoing concerning the absorption spectra of gasses. The HITRAN project began investigating this (don’t know when it started) and is still active with a large database available (apparently) to anyone who presents passable credentials.

    My point is that back a couple of years someone tried to use the HITRAN data about CO2 absorption of IR to justify the AGW myth. He was all exercised about it, afraid for his children, etc. I was interested in where he was getting his “proof” but I wasn’t buying anything.

    Anyway, it’s been well established that CO2 does in fact absorb IR. How well laboratory measurements translate to the real world is, of course, another question. And as you ask

    The question remains what proportion of IR emitted by earth is actually absorbed by CO2 (of any concentration from 100% to 0.3%) and to what extent does that cause temperature to rise in the atmosphere. The basic heat absorption can be easily done in the laboratory, so where is the data or is (climate) science so one-eyed that there is none?????

    It seems that there is none.

    CO2 as I understand it is ten times less abundant than 0.3%, something like 0.02x%, whatever x is. I see various numbers but it’s a very small component of the atmosphere. It’s ironic that such a small thing can cause so much trouble.

    10

  • #
    Thomas T S Watson

    I have skipped through these points of interest and have come to the conclusion that your have all missed the basic point of how our climate has been initiated.

    Our climate is totally controlled by the Earth’s orbital position within the Sun’s Heliosphere. It is orbiting within this current position because of the reactions between the Earth’s Magnetosphere to the Sun’s Positive Heliosphere and is controlling our seasons.

    If you doubt it, then ask yourself. What keeps our Earth in its present oscillating orbital position within our Solar system? and secondly, What creates the tilt of the Earth? When you become aware that it is not Gravity but the magnetic reactions between these two volumes of magnetic powers, namely the Earth’s Magnetosphere and the Sun’s Heliosphere. You may begin to realize that Carbon Dioxide has a very small part to play in this process of changing our Earth’s orbital phasing of the Earth’s seasonal changes.

    Have a wonderful day find the truth about Climate Change.

    I have proven this concept and while awaiting to be acknowledged by journals that will not accept my thesis, I am confident to show any one out there that our poles emit their magnetic fields from each pole and they interact to each other near their relative equalized position near to their respective equators. My book: Climate Change-Explained by Magnetism? ISBN9780646477220 is available in your public library. End of commercial.

    10

  • #
    Thomas T S Watson

    Roy Hogue: Australia’s contribution to the world’s CO2 is 0.000871% and I do not totally agree with the claimed statements that CO2 is found in the upper layers of our Atmosphere.

    If CO2 is there, then they are on their way to ground level because CO2 is, as you are aware, a heavy gas (44). That is why the Fire Brigades use it to put out fires. It pushes away the Oxygen and Nitrogen from burning carbon.

    This also is why it is within our atmosphere, CO2 is applied and accepted by all plant life, and applies the Synthesis process. It absorbs the Carbon molecule; to build up it wood texture, and throws out the Oxygen molecules

    10

  • #
    David Wood

    I’m an old PhD chemical engineer and follow the arguments in this blog with some difficulty since I haven’t been active in the field for many years.
    I have read Joe Postma’s papers and found them extremely understandable and plausible. I was particularly impressed by his analysis of the ‘standard’ GHE treatment which seems to assume that the earth is similar to a photosphere both in respect of energy input (which it clearly isn’t – the sun only shines on one hemisphere) and in respect of output ( which it is but in a very dynamic way during the daily cycle from dawn to suceeding dawn). It seems to me that the choice of this simplistic model may have something to do with the obvious difficulty of handling the more realistic model ( insolation of one hemisphere during the day and comple emission during both day and night).
    I’ve seen some comment suggesting that Postma’s analysis contains “simple errors’, although the supposed errors (as usual) were not elaborated. This seems just like ad hominem to me.
    What I haven’t seen is any evidence that the models beloved of the IPCC and warmists generally, take into account the terrestial realities of single hemisphere insolation and night and day heat emission. Perhaps someone who thinks the models accurately reflect terrestial conditions can enlighten me, and not just with a reference to some authority!! I really would like to see some evidence that the models don’t treat the earth as receiving insolation at around 340 W/m2 all around the surface, rather than around 1360W/m2 at the zenith of one hemisphere.

    10

  • #
    David Wood

    Saw that site before and was distinctly unimpressed. The usual sort of argument along the lines He’s wrong (but no evidence given of where) and we who know we are right, cause everybody says so. Also it doesn’t answer the question I posed about the reality , or lack of it of the warmist models which assume a weak sun surrounding the earth!! I think Joe Postma is spot on, the daylight sun obviously causes heat transfer which raises the temperature at the surface, in the tropics at least, to well above 30 degrees most of the time during the ‘heat of the day’. The gases in the atmosphere act to cool the surface air below what it would reach if there was no atmosphere. At night the atmosphere, and particularly its water vapour content, act to slow the rate of heat transfer to less than it would have been without an atmosphere.
    A few illustrative examples; On a summers day when there is little wind, dry sand at the beach gets so hot it’s hard to walk on with bare feet, on a sunny day with little wind in summer the air inside a closed car becomes suffocatingly hot, and the dry air in a desert location becomes extremely cold when the sun goes down at night. These things are real, the mathematically contrived models of the IPCC are not, at least there seems to be no evidence that they are.

    10

    • #
      Tristan

      David, if you have issue with their Postma rebuttal, feel free to post there. Ask questions that they can’t answer and prove to everyone that they’ve no real argument.

      10

  • #
    Paul

    Spot on, David. I struggled all the way through that reference [http://www.skepticalscience.com/postma-disproved-the-greenhouse-effect.htm] and found it all smoke and mirrors.

    They are just trying to keep enough people bamboozled long enough to get their political agenda in place. Once the noose of these carbon regulations is around the necks of the people it will be very hard to get out of it. Observe how the current minority government in Australia is intent on making it impossible for a future government to reverse their current ‘carbon tax’, as evidence. And the bureaucracy that this will put in place will assist the totalitarian objectives of some in high places.

    Some simple, every day observations provide a complete refutation of the absurd ideas of CAGW. I have been recording the temperatures around the house frequently throughout the day for over two years and see a pattern that repeats constantly. As the sun is about to rise the temperature starts to rise, but once this minor increase occurs the Troposphere begins its daily overturning, keeping the near-earth air temperatures from rising quickly. The temperature of the air in the attic rises more quickly but that also levels off as radiation and convection step in to cool the roof. Inside the house insulation slows the rise of temperature even further.

    From my data I can tell with reasonable accuracy whether the day was clear or cloudy. On a heavily overcast day the temperatures remain fairly static all day whereas on a clear day the sunshine continues to heat the atmosphere until a peak in the late afternoon.

    Overnight the rate of cooling is again mostly dependent on the amount of cloud cover, with clear nights losing heat faster, and the loss tapers off to a minimum as the bulk of the atmosphere cools leaving a thin layer of warmer air near the surface. The effect of clouds has to be one of the major factors in determining the daily temperatures, far outweighing any postulated effect of any increase in atmospheric carbon dioxide, and, as the scientists agree, this is one area that is almost a complete mystery to them.

    Since these observations will be true of most of the earth’s surface, outside the Arctic circles, these simple observations are a complete refutation of the simplistic idea that the near-surface air temperature is controlled by the amount of carbon dioxide in the atmosphere. It simply never is. And if it never is in any place at any time, then there is simply no way that it can be so ‘on average’.

    In fact, the concept of an average global temperature is totally non-physical and without any scientific value. It is by the postulation of such non-physical ‘values’ that the smoke and mirrors of this conjecture maintains its tenuous credibility in the minds of some who fear to think otherwise, even though common sense should kick in to tell them that they are being spooked by a phantom.

    Paul

    10

    • #
      Tristan

      And the bureaucracy that this will put in place will assist the totalitarian objectives of some in high places

      Spooked by phantoms indeed.

      10

      • #
        Paul

        No, the bureaucracy is real and will become all-pervasive with localised bureaucracies in each individual country and centralised control at the UN. The inital flow of money, instead of helping with reducing emissions, will be funnelled into creating all these bureaucracies. How many billions do they want?

        If you think that that does not produce opportunities for centralised control over other areas than ‘climate control’ then I think that you are not realistic in your thinking.

        The stupid part, though, is that even on their own admission, there will be no measurable change to the climate for a millennium.

        Paul

        10

    • #
      Jose_X

      >> found [criticism of Postma paper by skepticalscience to be] all smoke and mirrors

      I’ll read the Postma paper later. Note the article states that Postma made some simple errors (that cancel out) and criticized, not real climate modelling, but very simple models that are not used in practice. Note also the observation that Postma’s results do not fit measured atmosphere temperatures profiles.

      >> They are just trying to keep enough people bamboozled long enough to get their political agenda in place.

      I disagree with the view that skepticalscience is trying to lie to people.

      I hope you can separate science from politics. I’m not saying you can’t. I’m only bringing this point up since I thought we were talking about science. You seem fairly sure of skepticalscience intentions.

      >> Some simple, every day observations provide a complete refutation of the absurd ideas of CAGW.

      I did not read a single thing from your observations that disproves anything.

      >> Since these observations will be true of most of the earth’s surface, outside the Arctic circles, these simple observations are a complete refutation of the simplistic idea that the near-surface air temperature is controlled by the amount of carbon dioxide in the atmosphere.

      Who told you that climate scientists attribute our temperature to CO2 and to nothing else? That would make no sense. The scientists have said that a very small amount of change in temp (a few degrees Celsius) will arise in a 100 year period (or so is their best safe guess at this point in time).

      Are you measuring your temperatures for 100 years and then comparing the measurements in the last year to those made in the first to see if there is a noticeable difference? If you aren’t doing that, then you aren’t grabbing the sort of evidence that can possibly help disprove the CO2 effect theories.

      Roy Spencer has been grabbing satellite data (I hear without revealing the details of the software he uses to process the raw data.. or the raw data itself.. but I could be wrong). He is a skeptic and probably wants to show that temperatures are not rising much, yet since satellites have been grabbing data, we have seen almost a one way rise in temps, even as the solar irradiance has been neutral to down. But he is doing the right thing by recording that data over all of these years. Lot’s of people are grabbing data and recording it in order to disprove/verify scientists.

      >> In fact, the concept of an average global temperature is totally non-physical and without any scientific value.

      You have never used average values to say something meaningful about a quantity? Almost every measurement in the world is based on average values. Your grade in school is an average value. Your compensation in work is frequently a near steady value based on average performance. Even bonuses and uneven income/profits are based on various averages. Pricing (eg, based on futures contract hedging and competition pressures) is based on averages to a large degree (despite immediate supply demand concerns). Average is a meaningful way to make a statement about a lot of data points. If the average amount of water you drink every day is 2 cups vs 8 cups, we are making a very real and useful statement of an average with real consequences.

      Please cite a source that says that average climate temp values are not meaningful. [I’m sure there have been some abuses of this average value, of course.]

      You get into problems when you average *too much* and use that to derive results. Basic mathematics can show that is the case (see below). But using average values to summarize results or after careful analysis is more than just acceptable, it is sometimes the only way to avoid the impractical. Further, Postma and many others that don’t like mainstream climate science appear to be the ones using oversimplified models in their work. I have seen various examples — all using averages and simplified models more than mainstream climates scientists (who actually look at time derivative evolution instead of trying to assume future values will remain constant or must be simple lines and curves with very little variability).

      Average done wrong:
      A) The area of a rectangle is side1 * side2. It’s very useful to look at area if we have crops and want to have an idea of how much we’ll be able to sell at the market. If we know we added 100 meters of fencing around our small production, we can’t just take the average value 100/4=25 meters per side and throw away the other information. Knowing the average side length can lead to very wrong estimates of how much crop we have. A square of side 25 has 2500 square meters of area (of crops). This is a problem because if our plot was 40 long by 10 wide (40 + 10 + 40 + 10 = 100), then we have crops on only 400 square meters. And 400 is much less than 2500.

      B) If we have to pay monthly bills, we can’t just focus on earnings/month averages. If most of our income comes from the end of year holiday season, that will prop up the average monthly value and lead to problems when the bills come in January, February, etc.. unless we plan and take out loans or dip into accumulated savings. Cash flows must be managed more carefully than assuming we make average value every month.

      Average done right:
      A) On the other hand, if we know we have a square area (or close to it) for crops, then we can just worry about the average side length and toss out the other information.

      B) If we are going to take out a loan or decide how much of past year earnings to retain, then we can rely on average monthly earnings.

      We must perform analysis to see if averages make sense under the context in which they are used.

      As stated, the simple models are being used by Spencer, Postma (apparently), and a number of other individuals that disclaim climate science results. If you look at actual climate scientists, you will frequently find a lot of detail in their work and will rely on models that have complex details to handle many eventualities.

      They also try to see how a real atmosphere evolves over time. In fact, “time evolution physics” is something significantly missing from most people I see disagreeing with climate science. They use today’s measured values, make a number of unstated assumptions about future values by using pure algebraic equations to derive future results. You frequently won’t find a single time-varying equation in their work. And time-varying differential equations forms the foundation of every modern scientific or engineering problem solving effort. We don’t get modern cars or airplanes or bridges without sophisticated computer processing of differential equations. And the climate is complex relative to these other disciplines.

      10

  • #
    Lars P.

    Jo, what I am missing in the greenhouse graphs is the “other side” of the cool object. As the atmosphere radiates both ways – down and up – and the radiation is not directional, the same w/m2 should flow up from the cool object as it is going down. It is not a ceiling with one way direction.
    When one adds the down+up flow of the cool object as shown on most greenhouse graphs one gets to numbers above the total inflow from warm to cold object. This does not make sense for me. How do you work this out?

    10

    • #
      Jose_X

      I think you should provide an example.

      When you put the details on paper, then we can analyze what you mean.

      10

      • #
        KinkyKeith

        Hi Jose X,

        You comment “I disagree with the view that skepticalscience is trying to lie to people”.

        Really?

        They use the KISS principle based on the fact that most visitors to their site have no real scientific education and will fall for anything that “sounds” good.

        Keep It Simple Stupid.

        If they don’t analyze the science and they refuse to correct the science they put out are they being deliberately nasty or just extremely self interested or perhaps delusional or perhaps in a semi religious fervor?

        10

  • #
    Paul

    “97% climate scientists”

    This figure of “97%” is often quoted in support of the claim that human activities are endangering the climate of our planet. However it represents merely the wishful thinking of one researcher, that he could influence public opinion, and was a foregone conclusion of his study. Read why at climatequotes.com. Study claiming ’97% of climate scientists agree’ is flawed

    Anyone who uses such quotations to support the CAGW ‘consensus’ is using rhetoric that is of zero scientific value.

    To put the whole issue into context, while there has been an increase in the ‘world-wide temperature anomaly’ over the past two centuries, this increase is not in itself sufficient to cause concern. Any concern as to the direction and magnitude of further temperature change is entirely dependent on a postulated multiplier effect which, to date, has been more conspicuous by its absence than by anything else.

    The concept of a ‘temperature anomaly’ is itself non-physical and as such a poor index of whether or not the globe is warming or cooling and by how much. The amount of energy resident in the atmosphere is a tiny fraction of the energy resident in the oceans and the land surface, as simple physics tells us. Which is why we have such concepts as ‘maritime’ and ‘continental’ climates. Leaving aside the question as to whether the ‘temperature anomaly’ has been biased by ‘the heat island effect’ or deliberately manipulated by skewed ‘adjustments’, the choice to use this metric rather than the more useful one of ocean temperature seems to be dependent on matters other than scientific ones.

    Then there is the issue of causation. To link that small ‘anomaly’ to an index of atmospheric carbon dioxide and then postulate that the former is caused by the latter, ignores so many other factors, all of which are potentially many orders of magnitude greater in their effect, as to be simply an empty speculation. This is why, after two decades of research and the spending of billions of dollars, there remains no direct observational data to support the speculation. Reliance on computer models as an alternative to hard data is sufficient proof of this failure. Yet the models cannot predict even the current anomaly, let alone the current temperature, accurately. How much less can they predict the future!

    Nor is it possible to equate the observed increase in atmospheric carbon dioxide with the use of fossil fuels. Other factors, such as changes in land-use and the out-gassing of carbon dioxide from the equatorial oceans accompanying their warming, have first to be quantified and allowed for. Only then can the residual increase be related to the use of fossil fuels and any relationship be examined.

    Since atmospheric carbon dioxide is a small part of the carbon cycle, one needs to look at both the natural sources and sinks of carbon dioxide and what changes may be occurring therein. Clearly, even in the official data of the IPCC, the natural flows of carbon dioxide into and out of the atmosphere are orders of magnitude greater than those caused by the use of fossil fuels. It is astonishing how so much fever of concern has been stirred up by ignoring all the naturally occurring flows and concentrating all the attention on the smallest flow of them all, then attributing to that smallest flow the power to change the global climate – and that, not in a marginal but in a catastrophic way. Some very great powers of persuasion have been in operation here!

    Getting back to the main topic of this thread, the role that atmospheric carbon dioxide plays in conserving the surface temperature on earth and how this relates to the second law of thermodynamics, it seems to me that there is one great miss-match being made. It is claimed that carbon dioxide, and other ‘greenhouse gasses’ such as water vapour, trap energy in the atmosphere and hence keep the surface temperatures warmer than in their absence. The miss-match that I see is to apply the results of experiments in the laboratory, which measured the radiative properties of still air after carefully eliminating all conduction and convection, to the chaotic and turbulent conditions in the troposphere where the laboratory conditions, essential to reproducing the experimental results, are entirely absent and where the radiative relationship therefor does not apply. Then, to support this ‘bait and switch’ argument the claim is made that anyone who doesn’t agree is ‘unscientific’, denies verifiable science and is a ‘denier’!

    Paul

    10

    • #
      KinkyKeith

      Hi Paul

      Your comment about keeping the “surface temperatures warmer” is a comment on the mysterious return energy that warmers use.

      Most of us think the temporarily “trapped” energy is in the atmosphere which is cooler than the surface and therefore cannot heat the surface.

      This energy eventually escapes to that big ole heat sink in the sky.

      Hopefully we wont lose too much or we’ll freeze.

      🙂

      10

      • #
        Jose_X

        The sun warms the earth more when the atmosphere makes it harder to dissipate the sun’s continuous raining of heat.

        Put enough layers around you between your body and the outside (including on your face and hands and even inside a room with great insulation, etc), and you will eventually heat up to get a high fever and die (at which point your internal radiation level will cease and you will cool down). This process will be sped up if you first remove your sweat glands (with profuse sweating, you might possibly die of dehydration and mineral loss before you die of a fever).

        10

        • #
          Truthseeker

          Jose_X, this is not a good example because a person generates their own heat. Put more matter in contact with a heat source and more heat is stored. You are not proving anything relating to GHG here …

          10

          • #
            Jose_X

            The analogy matches the sun’s radiation upon the earth as your inside body’s radiation upon your skin.

            [I support these two distinctly located energy sources since theory and evidence seems to support that a lot of the sun’s radiation is not blocked on the way down but the earth’s IR radiation is blocked on the way up.]

            10

        • #
          Paul

          Jose_X
          January 21, 2012 at 3:37 pm · Reply

          The sun warms the earth more when the atmosphere makes it harder to dissipate the sun’s continuous raining of heat.

          Put enough layers around you between your body and the outside (including on your face and hands and even inside a room with great insulation, etc), and you will eventually heat up to get a high fever and die (at which point your internal radiation level will cease and you will cool down). This process will be sped up if you first remove your sweat glands (with profuse sweating, you might possibly die of dehydration and mineral loss before you die of a fever).

          You imply that an increase in atmospheric carbon dioxide “makes it harder to dissipate the sun’s continuous raining of heat.” And how does it do that?

          Where the sun is shining the troposphere is also busy ‘overturning’. Incoming radiation is converted to heat at the surface. This surface heat then either warms the air by conduction or evaporates surface water. The warmed, moist air rises and, with height cools and the water vapour re-condenses into water droplets releasing heat and forming clouds which precipitate rain or snow that further cool the surface. In the process heat is transported, quite apart from radiation, towards the top of the troposphere, somewhere about 14000 to 18000 M above the surface where the temperature is about -55° C and water vapour is entirely absent.

          In the presence of the sun’s radiation, temperatures within the troposphere thus result from the interplay of incoming radiation, conduction, latent heat of vaporisation and convection. Radiation has a negligible part in this movement of heat and is certainly not the controlling factor by day.

          With heat being transported away from the surface, uninhibited in any way by the radiative properties of still air, in what way is the atmosphere acting as an insulation blanket?

          Paul

          10

  • #
    Paul

    See my post at #600 above.

    Why was it given that number? Arrh!

    Paul

    10

  • #
    Jose_X

    Paul, I was paraphrasing Harry Huffman. He used that figure as you can see below.

    I agree that the 97% figure is likely abused. Part of the problem with that number is that different “skeptic/deniers” have different views. For some people, there are 97% (or maybe higher) of “climate scientists” who would disagree with their theory or with some important part of it. For other people, maybe less than 20% would comfortably disagree.

    My guess is that it’s probably a fair bet that “97% of climate scientists” accept the main points of the greenhouse effect. A very large percentage also probably accept that CO2 is an important greenhouse gas worth tracking.

    >> Being a competent physicist rather than an incompetent climate scientist (which 97% of them demonstrably are), I was able recently to post an answer on yahoo.com to a question about the greenhouse effect on Venus, an update to which I give here:

    >> Surprisingly to most, there is no greenhouse effect at all, and you can prove it for yourself.

    10

    • #
      Paul

      My guess is that it’s probably a fair bet that “97% of climate scientists” accept the main points of the greenhouse effect. A very large percentage also probably accept that CO2 is an important greenhouse gas worth tracking.

      And I would suggest that it is entirely irrelevant to the discussion whether any percentage of climate scientists accept anything or not!

      Can we agree that a greenhouse is not warmed because of the glass blocking radiation but because the glass [or plastic or rock-salt] prevents the normal circulation of the air inside the greenhouse?

      In which case, I would like to know, at which point in the atmosphere does carbon dioxide prevent air circulation? And if it does not prevent the circulation of air in just what way is it a ‘greenhouse’ gas?

      Just wondering.

      Paul

      10

      • #
        MattB

        Paul – greenhouses make air warmer. Greenhouse gases make the planet warmer. It is a popular term that is not particularly based in the actual science mechanisms of the warming. It will not be the first and it will not be the last. What does that have to do with anything anyway?

        10

        • #
          Dave

          Hey MattyB,

          A Greenhouse description is as follows: at Wiki

          A greenhouse is a structure with different types of covering materials, such as a glass or plastic roof and frequently glass or plastic walls; it heats up because incoming visible solar radiation (for which the glass is transparent) from the sun is absorbed by plants, soil, and other things inside the building. Air warmed by the heat from hot interior surfaces is retained in the building by the roof and wall. In addition, the warmed structures and plants inside the greenhouse re-radiate some of their thermal energy in the infrared spectrum, to which glass is partly opaque, so some of this energy is also trapped inside the glasshouse. However, this latter process is a minor player compared with the former (convective) process. Thus, the primary heating mechanism of a greenhouse is convection. This can be demonstrated by opening a small window near the roof of a greenhouse: the temperature drops considerably. This principle is the basis of the autovent automatic cooling system. Thus, the glass used for a greenhouse works as a barrier to air flow, and its effect is to trap energy within the greenhouse. The air that is warmed near the ground is prevented from rising indefinitely and flowing away.

          Amazing for a legend like yourself with knowledge of solar radiation etc. Where is the glass, polycarbonate, plastic etc that makes our planet earth similar to your “Greenhouse, Glasshouse or MATTYHAUS” that implies that CO2 is the driving force of more wet weather, more dry weather, more of everything.

          Is your electoral material stating your glasshouse effect on the world?

          10

        • #
          Paul

          Greenhouse gases make the planet warmer.

          That claim, I suggest, has been disproved.

          What difference does it make? Well people people know that greenhouses are warmer. They are told that it is because they ‘trap heat’. They are told that ‘greenhouse gases’ ‘trap heat’. They are encouraged to believe that both processes are the same. That, to me, is straight deception.

          Paul

          10

          • #
            Jose_X

            It hasn’t been “disproved”. Would you care to explain when it was disproved and who accepted that?

            Perhaps you will contact one of the top physics universities in the world because they didn’t get the memo: http://acmg.seas.harvard.edu/people/faculty/djj/book/bookchap7.html

            > We examine in this chapter the role played by atmospheric gases in controlling the temperature of the Earth. The main source of heat to the Earth is solar energy, which is transmitted from the Sun to the Earth by radiation and is converted to heat at the Earth’s surface. To balance this input of solar radiation, the Earth itself emits radiation to space. Some of this terrestrial radiation is trapped by greenhouse gases and radiated back to the Earth, resulting in the warming of the surface known as the greenhouse effect. As we will see, trapping of terrestrial radiation by naturally occurring greenhouse gases is essential for maintaining the Earth’s surface temperature above the freezing point.

            And so you don’t think that is just a crazy rogue professor, look at this: http://green.harvard.edu/greenhousegas

            10

          • #
            Paul

            Jose_X
            January 21, 2012 at 5:22 pm

            It hasn’t been “disproved”. Would you care to explain when it was disproved and who accepted that?

            Do I have to tell you that science is not validated by whether someone in a position of authority accepts it or not?

            Just because activists occupy positions of authority and propaganise that this theory is ‘scientific’ does not make it so. That you should so quote them as if it did shows that you are not arguing from scientific evidence but from belief in a proposition.

            Thank you for clarifying where you stand.

            Paul

            10

      • #
        Jose_X

        >> whether any percentage of climate scientists accept anything or not

        This sounds to me like you don’t have much preference between having a florist help you learn chemistry vs getting advice on chemistry from a chemist.

        OR

        You really have a low opinion of this one particular group of experts (climate scientists).

        >> greenhouse is not warmed because of the glass blocking radiation but because [of convection]

        I agree that shutting off convection is what keeps a greenhouse warm, but what gets it warm in the first place (if convection is kept at bay) is the transparency to higher frequency sunlight. I haven’t carried out the experiment formally with thermometer, but I am of the belief that it gets hotter in a greenhouse with plants than in a large closed dark room with plants.

        The glass allows heat to flow inside while walls block such radiation heat.

        >> carbon dioxide prevent air circulation?

        1a) A greenhouse doesn’t prevent air circulation within itself.
        1b) CO2 doesn’t prevent air circulation within “itself”.

        2a) The convection denied a greenhouse is with the cooler environment outside the greenhouse.
        2b) The earth is also automatically denied convection currents flowing outside its CO2-mixed-in atmosphere into outer space.

        Greenhouses and greenhouse gases work similarly with respect to convection in that the air inside can move but can’t leave to be replaced with cooler air from outside (eg, allowing the earth’s air to be replaced with cooler Martian air).

        10

        • #
          Paul

          Jose_X
          January 21, 2012 at 12:29 pm · Reply

          >> whether any percentage of climate scientists accept anything or not

          This sounds to me like you don’t have much preference between having a florist help you learn chemistry vs getting advice on chemistry from a chemist.

          On the contrary, it means that I accept scientific explanations on their merit not on who offers them. I expect those explanations to be supported by verifiable data. If a chemist is trying to sell me a brand of toothpaste I don’t just accept his assertions as ‘scientific’ because a scientist is saying the words.

          And, true, it does indicate that I have a very low regard for most climate scientists. Shortly “climate science” will be used as a reproach in the same way as we use “tobacco science” today.

          It does not help your cause when you revert to the fallacy of “argument from authority”. Give facts, explain processes, convince by illuminating the issue and leave out who else agrees with you if you wish to enter a scientific argument.

          10

          • #
            Jose_X

            >> On the contrary, it means that I accept scientific explanations on their merit not on who offers them. I expect those explanations to be supported by verifiable data.

            I haven’t seen any evidence from you that finds inconsistencies in greenhouse effect theory. You seem to be against it but without evidence. That sounds to me like faith/bias.

            I do see your claims going against evidence. For example, you appear to say that the ground doesn’t radiate very much, yet measurements indicate otherwise.

            You appear to be picking your side based on something other than evidence.

            When I appeal to “authority”, I am actually appealing to people who rely on evidence.

            Does your theory explain why where ozone lies in the atmosphere the temperature is higher than further below?

            Do you have calculations to suggest why the earth’s average global surface temperatures are around their current figure rather than lower as dictated by simple Stefan Boltzmann derivation? If you don’t believe in average global temperatures, can you give a reason. We use averages all the time in meaningful ways, so please explain why you are against global average temp.

            Do you agree that CO2 absorbs and emits in certain frequency ranges? If not, what explanation do you have for its line spectra, and I am not just talking about physicists, line spectra are used in engineering, forensic science, chemistry, and many other areas to great success. What about other gases like N2 whose line spectra is noticeably missing in the IR range? And what about satellite reception of large quantities of radiation in the spectrum where CO2 has been repeatedly seen and recognized to absorb by scientist after scientist? What about quantum theory that explains such phenomenon, is that theory incorrect?

            Can you explain why some of the top physics students in high school go to Harvard year after year? These are kids who aren’t into climate science but simply understand lower level physics much better than most people ever will. Do you honestly think these university programs are a joke? It’s not just Harvard. Can you find a few programs that deny greenhouse effect? Of course, feel free to ignore this last paragraph, but it would be nice if you had a reply for the other ones.

            I sound a little agitated, I know. Don’t respond if you don’t want, but from my vantage point, I see someone claiming he is for evidence yet rejecting the work of people who do rely on evidence and have fairly consistent explanations for many things.

            10

          • #
            BenAW

            Do you have calculations to suggest why the earth’s average global surface temperatures are around their current figure rather than lower as dictated by simple Stefan Boltzmann derivation?

            That’s a simple one. SB simple gives 255K for averaged radiation spread around the globe.
            70% of he earths area is ocean, kilometers deep, with a temperature around 275K below the thermocline.
            This is already 20K above SB simple.
            Now add solar incoming and all your backradiation etc. to reach our pleasant 288K.
            Just 13K difference.
            Effect solar vs GHE ~100%/~0% in my world.
            Being generous make that 99/1, GHE is 1% of 13K, and most of that is by watervapour.
            Scary.

            10

          • #
            Jose_X

            >> 70% of he earths area is ocean, kilometers deep, with a temperature around 275K below the thermocline. This is already 20K above SB simple.

            The 275 exists in part because of “back radiation”/ghg effect, so it goes above and beyond the simple SB scenario.

            If you don’t want to accept ghg reasoning (which is what I wondered), then what explanation do you have for why 275 would exist if the sun is presumably (via SB) only responsible for about 255 on the surface? In other words, from whence cometh that extra temp if not from ghg?

            10

          • #
            BenAW

            My reply got lost, re-try:

            “The 275 exists in part because of “back radiation”/ghg effect, so it goes above and beyond the simple SB scenario.”

            I specifically stated “below the thermocline”, where solar heating has no influence anymore.
            I assume you’re not suggesting that backradiation can heat the oceans where solar radiation can’t?

            The oceans temperature is just there, probably left over from cooling since earths creation, or the last major meteorite impact.
            Remember earth is in radiative balance with solar radiation at TOA, so no change in total heat content. The oceans are probably in balance with the hot core below, being isolated by the crust, although there is a geothermal gradient of ~20K/km in the continents, ocean beds much higher since the crust there is much thinner.

            Ben Wouters

            10

    • #
      KinkyKeith

      Can you clarify “main points of the greenhouse effect” since people are starting to ascribe new meanings to the greenhouse effect to create confusion.

      10

  • #
    KinkyKeith

    Hi blouis79

    Good analysis.

    One of the problems with using the term “greenhouse effect” is that it has different meanings to different people.

    For warmers it may include the ridiculous second round of radiation from the atmosphere.

    For myself, the meaning I attach to it is this: the energy from ground origin IR is delayed in its journey to deep space and the accumulation of energy in the atmosphere will lead to either increased temperature or increased pressure or both.

    The fact that there are no “walls’ on our greenhouse means that high pressure air pockets will expand and rise, or if overhead clouds exist, simply push air sideways to achieve equilibrium.

    CO2 and especially man made CO2 plays a very minor roll in “my greenhouse” effect.
    It absorbs IR and transmits the energy immediately to other gases unable to absorb at the same frequency.

    Transmission mechanisms (physics – long time no see – be patient) will be either KE via impact of excited CO2 mols with other gases, or radiation (?).

    I notice in the last paragraph that “Bo Nordell has quantified the effect”.

    Without any real assessment I would guess that the sum total of all heat produced by human activity would be insignificant.

    Our Planet is surrounded by a gigantic limitless heat sink, a fact which all “Climate Scientists” either don’t know or want to keep hidden.

    We are not in danger of overheating, heat is easily dissipated, we are in great danger of FREEZING.

    10

    • #
      Jose_X

      >> For warmers it may include the ridiculous second round of radiation from the atmosphere.

      Are you aware that back radiation (downward longwave radiation, DLR) is very real and measured?

      And it is more than just a second round.

      >> the energy from ground origin IR is delayed in its journey to deep space

      But isotropic radiation means that we don’t just have radiation going up and sideways but also down. High up, the earth’s roundness means most radiation won’t hit the earth, but, near the ground, about 50% of the radiation will hit an object on the ground (actually very near the ground more tha 50% of “re-radiation” will hit unless you are talking about a super “flat” ocean or land terrain).

      With mirrors (“mirrors” aren’t a perfect analogy, naturally), you can raise the temperature in a space on earth by a lot. The radiation that would have left to outer space is being focused into an area.

      >> CO2 and especially man made CO2 plays a very minor roll in “my greenhouse” effect. It absorbs IR and transmits the energy immediately to other gases unable to absorb at the same frequency.

      Yes, CO2 does that, but that won’t be the only effect. A gas with a significant amount of vibrational energy that can radiate will do so to match its non zero Kelvin temperature.

      Back radiation is real and very significant in size near the ground.

      What happens, btw, if two already “hotter than average” molecules collide?

      And note that CO2 isn’t the main effect near the ground. H2O radiates more, and there is more H2O in the atmosphere from the extra bit of temperature raise that CO2 would cause alone (so increasing CO2 alone value gets multiplied a fair amount near the ground). [H2O condenses out of the air, so H2O does not play a significant role hither up.]

      >> Our Planet is surrounded by a gigantic limitless heat sink, a fact which all “Climate Scientists” either don’t know or want to keep hidden.

      Are you crazy? Can you point to a single published paper of the many thousands that have been published in climate related journals, or point to a single model used by the IPCC, that ignores space as a sink?

      If models ignored space, they would predict 100000K within a relatively very short time frame.

      If you are talking about the oceans, then climate models certainly consider the oceans. Go to skeptical science to see references to over 95% of the extra absorbed heat taking place in the oceans.

      >> heat is easily dissipated

      You know you can burn up inside an oven that is turned on, even if the oven is left outside in subarctic weather, right? The earth keeps adding energy to the planet. The planet radiates it away or it would have melted a long time ago, but the fact it radiates into a sink doesn’t mean the surface can’t get hot.

      Scientists and engineers use mathematics and models based on evidence just because they want to know if in one example the sink is powerful and leads to cooling but in another example the sink is not as effective and leads to warming.

      A semiconductor is simple in relation to the climate. We engineer devices to simple models. We take great pains to manufacture high quality so we can get quality electronics that meet engineered and useful high demanding specs. But we can’t engineer and mass produce our planet the same way to our liking. We take the complexity given and try to do our best. [BTW, “our” refers to humankind.]

      10

      • #
        KinkyKeith

        Obfuscation,

        Your latest :

        “Yes, CO2 does that, but that won’t be the only effect. A gas with a significant amount of vibrational energy that can radiate will do so to match its non zero Kelvin temperature.

        Back radiation is real and very significant in size near the ground.

        What happens, btw, if two already “hotter than average” molecules collide?”

        You don’t know what you are talking about.

        There is only ONE parcel of energy to assess.

        You cannot add more to create a magical “Back Radiation” effect.

        10

        • #
          Jose_X

          >> There is only ONE parcel of energy to assess.
          >> You cannot add more to create a magical “Back Radiation” effect.

          There is no magic. Instruments measure “back radiation”.. which is just radiation like any other.

          What “one parcel of energy” are you talking about?

          Are you saying that a mirror cannot fire back a photon fired at it? How do you see yourself on a mirror?

          Why is it so hard to believe that a CO2 molecule might fire back (or in some direction) a photon fired at it which it happens to absorb?

          10

      • #
        KinkyKeith

        From Jose

        ‘And note that CO2 isn’t the main effect near the ground. H2O radiates more, and there is more H2O in the atmosphere from the extra bit of temperature raise that CO2 would cause alone (so increasing CO2 alone value gets multiplied a fair amount near the ground). [H2O condenses out of the air, so H2O does not play a significant role hither up.]

        HERE WE HAVE JOSE DESCRIBING THE FAMOUS AMPLIFYING EFFECT OF CO2 ON WATER IN THE AIR.

        Pst .. ask somebody who knows some science what is meant by relative humidity and saturation point of water in air. CO2 amplifies water … ha ha ha .

        Ha Ha Ha.

        Nice try Jose, you lasted what was it, two days here or was it 3 ?

        10

        • #
          Jose_X

          [I’m resubmitting this from yesterday since it didn’t post.]

          >> ask somebody who knows some science what is meant by relative humidity and saturation point of water in air. CO2 amplifies water … ha ha ha

          Higher temperatures allow more H2O molecules to dissolve in the air. In other words, warmer air can hold more water vapor.

          I stated this, that the rise in temperature from CO2 would be multiplied as more water would be able to exist in the air.

          See wikipedia: Relative_humidity#Other_important_facts

          10

          • #
            John Brookes

            Hey Jose, you are right.

            10

          • #
            Jose_X

            @BobC, I appreciate his measured discussion, concluding:

            > I am not saying that’s what I necessarily believe. I will admit to having waffled on this issue over the years, but that’s because there is evidence on both sides of the debate.
            > At a minimum, I believe the water vapor feedback issue is more complicated than most mainstream researchers think it is.

            Of course, keep in mind this is one person only, and one whose views are in the minority of climate scientists who study these topics. Barry Bickmore lists some examples of Spencer going a little over the top http://bbickmore.wordpress.com/roy-spencer/ .

            10

          • #
            Jose_X

            @BobC, btw, I generally don’t like to post criticisms of other works people have done as if that were argument against the current writing under the spotlight, but Roy (Spencer) has a number of related theories he is pursuing relating to water/clouds and positive/negative degree of “feedback”.

            Not only did he not take a position in that essay against CO2 warming being multiplied by water (he was rather cautious and mostly wanted to express his doubt on the matter), but at other times, when he has done something similar (eg, arguing for a much lower climate sensitivity, in part because of these issues), he has not been too successful arguing the point to others with know-how.

            Feel free to follow some of the links Barry has posted on that topic since he is one who has shown problems in one or more of Roy’s works.

            10

          • #
            BobC

            Hi Jose_X,

            Here are some experimental results on water vapor in the upper troposphere over a number of decades.

            Here’s the situation:

            1) Radiosondes report (by direct measurement) that WV has been decreasing since 1973.

            2) Some satelite measurements agree with the radiosonde data, some do not.

            3) Climate scientists have been doing “reanalyses” — begining with the assumption that the radiosonde data can’t be trusted, since they are “inconsistent with climate-model calculations” and that “there exists no theoretical support for having a positive short-term water vapor feedback and a negative long-term one” (Refering to the WV following the 1998 El Nino temperature bump, but being decorrelated since then, as shown in this graph ).

            Some of these reanalyses show that maybe water vapor has been rising. (Can we say “Confirmation Bias”?)

            Apparently, engineers are no better at making humidity sensors than they are at making thermometers. (I sense a government grant to develop such sensors that never produce data violating politically-correct theories. I think it could be done, if all such sensors are continuously connected to the Internet and have available the latest politically-correct “data”.)

            As to Spencer’s work, apparently NOAA agrees with him that, even if we knew for sure that WV in the upper toposphere was increasing or decreasing, we still wouldn’t know whether the feedback (from more WV) was positive or negative.

            Bob C also wrote this comment which was later lost:

            Jose_X
            January 23, 2012 at 12:06 am ·

            I stated this, that the rise in temperature from CO2 would be multiplied as more water would be able to exist in the air.

            Now, if only the world would fall in line and comply.

            (Another beautiful theory destroyed by cruel reality.)

            10

          • #
            Jose_X

            @BobC,

            We can probably agree on these three points:

            1: Higher temps allow for more WV in the air

            2: The net effect of more WV (assuming there is more of it in our atmosphere) is something over which there probably is significant amount of uncertainty. NOAA stated in that link that there is much uncertainty. Spencer stated this in that essay. [Note that uncertainty doesn’t mean the hypothesis is disproven.] I have heard many state that we really aren’t very sure.

            3: I’m guessing from your comment and from a 2010 Dessler paper that most published research analyzing radiosonde data appear to say that humidity is not going down. [You claim this is confirmation bias. I recognize that it might be, but so can anything whenever you have a few people gathered together believing something.]

            Also, I’d like to say some more things

            4: As concerns the graph you linked covering some 17 years where CO2 is going up fast, temp is going up a little, and humidity is going down a little, I don’t agree that that disproves AGW as that chart states. There are so many variables present and climate science has been clear that the long term direction of the climate (on the 30+ year range), eg, for global average temperature, can contradict the shorter term many times. To use an economics analogy, we can have inflation going up fast even while the price of wheat is going down a little and the amount of currency in circulation is going up a little. The reason is that numerous factors and product prices define inflation (especially in a world economy of imports/exports). Similarly, we might have a DJII component going down and some others going up a little even while the general Dow goes up a fair amount.

            5a: I replied in another blog posting to a thermometer fix example by the BEST project by stating that those fixes for the mid 20th century were focused sort of around the time satellites came into the picture, and they resulted in low error bars from that point forward. The older thermometer values that didn’t get the fixes still maintained their higher level of uncertainty.That transition helps match older data with satellite data. [I am assuming the satellite data was a motivator for making that connection, and I am not saying there aren’t potential statistical traps.]

            5b: I haven’t researched this too much, but another motivator, perhaps the main one, might have been to deal with temperature standards. If new thermometers have readings based on a different definition of mercury movement (or whatever), that could be reason to calibrate past values with current definitions. I did find the following after a quick google search: From http://en.wikipedia.org/wiki/International_System_of_Units
            > The history of the metric system has seen a number of variations, the use of which has spread around the world, to replace many traditional measurement systems. At the end of World War II, a number of different systems of measurement were still in use throughout the world. Some of these systems were metric-system variations, whereas others were based on customary systems. It was recognised that additional steps were needed to promote a worldwide measurement system. As a result, the 9th General Conference on Weights and Measures (CGPM), in 1948, asked the International Committee for Weights and Measures (CIPM) to conduct an international study of the measurement needs of the scientific, technical, and educational communities.
            > Based on the findings of this study, the 10th CGPM in 1954 decided that an international system should be derived from six base units to provide for the measurement of temperature and optical radiation in addition to mechanical and electromagnetic quantities. The six base units that were recommended are the metre, kilogram, second, ampere, degree Kelvin (later renamed kelvin), and candela.

            5c: The fixes might be to account for common past materials and techniques used in manufacturing relative to current techniques/materials. This is actually related to 5a and 5b.

            I don’t know the reasons or details of those changes, but I am suspect of a theory stating that those changes would be done to promote global warming. “Suspect” means I place the burden of proof on such a theory.

            6: I don’t want to downplay your comment about radiosondes. I would like to research more on that or maybe glance at one of the “reanalysis” papers.

            10

          • #
            BobC

            OK Jose_X;

            You seem to be open to investigating the whole “thermometer adjustment” thing, but are skeptical that it could represent fraud.

            Here, then, is an example of “reanalysis” of arctic and world temperature data. Notice that temperature readings over 120 years old are being “adjusted” downwards — just “happening” to show that warming during the last century was significantly larger than had been thought (even by the IPCC, in 1992).

            In case you think there might be something even remotely valid about this, this site demonstrates (by quoting local news articles from the time) that the “new, improved” arctic temperture “history” is inconsistent with reports of ice melt and advance made by people living there.

            The website, http://www.real-science.com archives the flaky manipulations that the AGW crowd of climate “scientists” are doing to the historical record. It’s remarkable that every “adjustment” serves to increase the perception of current warming and erase the evidence of past warming (e.g., during the 1930’s and 40’s), even to the extent of completely contradicting observational evidence from the period. What are the chances of that being accidental and/or done in good faith?

            It is either fraud or massive incompetence.

            10

          • #
            Jose_X

            BobC, anyone who cares about science takes data manipulation (at least when done by others) very seriously.

            Of the many many people with access to this data (I presume many have access if such a website has access), anyone who doubts the integrity and appropriateness of the methods now has to step up and “write a paper” arguing this.

            Without measurement context who can tell if the changes were justified? I can’t judge that from what little I read on that page and what you told me.

            It’s time to step up to the bat if “you” think there is a problem or have significant suspicions. If “you” succeed, others will be less likely to rely on the modified data (or modification algorithms) and instead rely on the older (or existing raw) data.

            10

      • #
        BobC

        Jose_X
        January 24, 2012 at 4:31 pm
        @BobC,

        4: As concerns the graph you linked covering some 17 years where CO2 is going up fast, temp is going up a little, and humidity is going down a little, I don’t agree that that disproves AGW as that chart states.

        I linked the chart to demonstrate the fact that WV followed the 1998 El Nino temperature bump, but has been uncorrelated with temperature since then. Because climate scientists don’t have a theoretical explanation for this, they are using it as evidence that the WV measurements are in error. This is a classic example of confirmation bias.

        10

  • #
    BenAW

    “The 275 exists in part because of “back radiation”/ghg effect, so it goes above and beyond the simple SB scenario.”

    I specifically stated “below the thermocline”, where solar heating has no influence anymore.
    I assume you’re not suggesting that backradiation can heat the oceans where solar radiation can’t?

    The oceans temperature is just there, probably left over from cooling since earths creation, or the last major meteorite impact.
    Remember earth is in radiative balance with solar radiation at TOA, so no change in total heat content. The oceans are probably in balance with the hot core below, being isolated by the crust, although there is a geothermal gradient of ~20K/km in the continents, ocean beds much higher since the crust there is much thinner.

    Ben Wouters

    10

    • #
      KinkyKeith

      Hi BenA

      I was looking for a place to comment on another question by Jose and found you hhad already made my comment.

      Here goes anyway.

      Jose asks: in relation to ocean temperature: “words, from whence cometh that extra temp if not from ghg?”

      Well, as I’ve said before on many occasions the Earths Atmospheric heat balance is very complex. Deep down isde ourselves we are all vaguely aware that the Earth is cooling but don’t factor this cooling into heat balances. For example, I’ve never seen it mentioned in any warmer balance because all they want to do is implicate CO2 as an atmospheric accelerant and stuff the science.

      The problem for warmers is that there is a constant heat flux from ground to air that must be accounted for. I wonder if Jose can quantify this loss. Perhaps use Watts/ m2.

      Perhaps some of this leaking heat can get into the water supply??

      We lose too much heat and we are going to freeze.

      10

      • #
        Jose_X

        >> We lose too much heat and we are going to freeze.

        [Ben,] it’s nice to hear something new (I started looking into global warming last year I think). I hadn’t looked at oceans much (because it hardly comes up), and it’s about time.

        KinkyKeith, the temperature profile suggests to me the oceans are being heated net from the top to the bottom. So the more time passes, the warmer the oceans should get. http://www.onr.navy.mil/focus/ocean/water/temp3.htm [I haven’t really studied water and pressure/temp variations.]

        You appear to be saying that the center of the earth (or other phenomenon) would keep the bottom of the ocean at a few degrees above freezing point of water (even at those depths) with the help of ghg atmosphere, but you don’t want to credit ghg effect too much.

        I’ll probably post some more in a while.

        10

        • #
          KinkyKeith

          Jose

          Ever been down a vertical 1300 metre mine shaft.

          20 deg C on the surface and 45 deg C at the bottom.

          Any good scientist can tell you how the ocean picks up heat energy and to what depth – just ask one.

          10

          • #
            Jose_X

            >> Any good scientist can tell you how the ocean picks up heat energy and to what depth

            Yes, but how much does it pick up and from where?

            Did you see the chart of ocean temperatures how it gets colder as you go down? Explain how that means that the earth is releasing more heat than the atmosphere provides? It doesn’t.

            The release of heat from the earth appears to be very slow.

            From http://en.wikipedia.org/wiki/Geothermal_gradient#Heat_flow
            > Total heat loss from the earth is 44.2 TW … This is approximately 1/10 watt/square meter on average, (about 1/10,000 of solar irradiation,) but is much more concentrated in areas where thermal energy is transported toward the crust by convection such as along mid-ocean ridges and mantle plumes.[13] The Earth’s crust effectively acts as a thick insulating blanket which must be pierced by fluid conduits (of magma, water or other) in order to release the heat underneath.

            Look at this graph http://upload.wikimedia.org/wikipedia/commons/0/0b/300px-Geothermgradients.png . It suggests the temperature profile changes very slowly near the surface and hardly at all further down.

            So earth is hot inside, but this moves to the surface very slowly. This is consistent with ocean depth being cold.

            Good scientists appear to have concluded based on measurements that the earth cannot keep us warm. The temperature is defined by the sun’s irradiance and how that heat eventually makes its way into cold space.

            10

          • #
            KinkyKeith

            Hi Jose

            “So earth is hot inside, but this moves to the surface very slowly. This is consistent with ocean depth being cold.

            Good scientists appear to have concluded based on measurements that the earth cannot keep us warm. The temperature is defined by the sun’s irradiance and how that heat eventually makes its way into cold space.”

            “This is approximately 1/10 watt/square meter on average”

            Good.

            10

    • #
      Jose_X

      @BenaW

      The temperature of the planet, if there were no atmosphere or white ice (so low albedo), might be closer to 0 C.

      With more reflective surfaces, we end up blocking some sunlight (which promotes colder environment) but ghg bring the temperature up to the 15 C global average range.

      This might be why the ocean depths are near 0C — that this would be the natural temp without atmosphere.

      However, the extra heat absorption off the planet’s IR that ghg give us is countered by the clouds/ice rise in albedo, yet ghg dominate. The difference is the 33C or so people talk about.

      If we use Venus as a model, we might even hypothesize that as CO2 increases many many orders of magnitude, total back radiation increases significantly (maybe 20-fold) to double or triple the temperature (in Kelvin, since fourth root of power multiplier) while the albedo roughly doubles from .3 to .6.

      This description makes sense.

      It’s also not too far from the idea that the deepest parts of the oceans would be around 0C as you guys suggest.

      Now, what I don’t understand is, if we start at this 0 point (why not?) and consider albedo increases that will reduce sun irradiance, how can we conclude that ghg has negligible effect? That doesn’t follow unless you describe an alternate mechanism to come up with the missing heat that leads us to 15 C or so when otherwise the surface temperature (top of water, land, and bottom of air) would be near -15C (because of .3 albedo).

      In other words, it doesn’t matter what we theorize about the ocean depth because at the surface we can see that we would have a driver towards -15C or so but instead experience 15C or so. If ghg doesn’t do that, what does?

      10

      • #
        KinkyKeith

        Totally scientifically incomprehensible.

        Please do carry on. It’s amusing, or if you feel more comfortable go back to skeptical science.

        I have no issue with people who are genuinely searching, but Jose you are not what you claim to be ie. someone who is qualified to tell Harry dale Huffman about science.

        10

        • #
          Jose_X

          KinkyKeith, I left some questions for Huffman that you may want to answer for him.

          If you didn’t read what I wrote or if you don’t understand it, I don’t think you are in a position to judge who is being scientific or not.

          10

      • #
        BenAW

        You’re talking total nonsense now.

        Oceans are 275K (+2C) and assume no heat transfer from hot core to oceans.
        Incoming solar in the tropics can reach ~1000W/m^2 AFTER correcting for 30%
        albedo. This heats the oceans top layer. Google thermocline for more info.
        Warm ocean heats atmosphere from below by conduction and convection.
        GHE is non-existent.

        Ben Wouters

        10

        • #
          KinkyKeith

          Hi BenAW

          I said before, in post to Jose, that : “Any good scientist can tell you how the ocean picks up heat energy and to what depth”.

          I actually didn’t know much about it but wanted to give him a bone to work with: The possibility that core heat did something to the ocean.

          I think it was you who said the oceans are probably the temp they are because of residual heat from an earlier time.

          The basic convection mechanism is that cold water sinks in the polar regions.

          This spreads and pushes up other areas at the equator which are warmer and moves that water towards the poles. Very general.

          10

          • #
            BenAW

            In the polar regions the thermocline “surfaces” so the surface temp is about the same as the deep temps.
            Seasons make the thermocline layer move north and south, following the sun with 1-2 months delay, allowing freezing of the polar sea where the sun isn’t shining that season. Probably the mechanism for iceages imo.
            Reduced incoming solar (multiple or large vulcano eruptions blocking the sun for quite some time. Thermocline layer “retreats” towards the equator allowing freezing over at both poles. Albedo increases rapidly, a tipping point (finally a real one 😉

            10

        • #
          Jose_X

          >> Incoming solar in the tropics can reach ~1000W/m^2 AFTER correcting for 30% albedo.

          Yes, you can see graphs of a location in California that approaches the 1000 W/m^2 for downward shortwave radiation during the daylight hours. This comment http://bbickmore.wordpress.com/2011/07/26/just-put-the-model-down-roy/#comment-8132 covers that and related graphs.

          >> Warm ocean heats atmosphere from below by conduction and convection.
          GHE is non-existent.

          Graphs showing upward/downward longwave/shortwave radiation can be found from that link just given. The radiation level numbers add up similarly to the popular Trenberth diagrams. There is significant radiation near the ground both upward facing and downward facing.

          Considering water has an emissivity http://en.wikipedia.org/wiki/Emissivity very near to 1 http://www.infrared-thermography.com/material-1.htm , I think I will place my best with the view that radiation is escaping into the atmosphere in significant quantities.

          CO2 and H2O together can absorb significant amounts in that IR range emitted by the earth+oceans. [ http://en.wikipedia.org/wiki/File:Atmospheric_Transmission.png ] That absorbed energy goes somewhere. And the complementary “re-radiation” is isotropic, meaning that near the ground about half is aimed at it. The ground doesn’t discriminate between photons that hit is from the sun or from somewhere else.

          I think I am making the point fairly, but I’ll ask for a little bit of help. Here are lecture notes on greenhouse effect by a Caltech professor http://bicep0.caltech.edu/~ebierman/Teaching/Spring2011_raw/Sukumoto/ranking%20task/GreenhouseEffect.doc . Like with the Harvard professor I linked to, I want to add evidence that the Caltech lecture isn’t controversial as seen by the school (or by the world). Caltech, arguably the top graduate physics institution in the country if not the world, describes their commitment to trying to keep a check on the greenhouse effect. http://sustainability.caltech.edu/climate . From http://en.wikipedia.org/wiki/California_Institute_of_Technology “Caltech was ranked as the best university in the world in two categories: Engineering & Technology and Physical Sciences.” Caltech, like Harvard, teaches a lot more than simply “climate scientists”. If you mock their well accepted teachings, you are mocking physicists and scientists in general. Sure, what do those folks know? But, please, feel free to completely ignore this paragraph and instead focus on the prior ones providing the argument and associated evidence.

          10

          • #
            BenAW

            Graphs showing upward/downward longwave/shortwave radiation can be found from that link just given. The radiation level numbers add up similarly to the popular Trenberth diagrams. There is significant radiation near the ground both upward facing and downward facing.

            I’m not a radiative expert by any stretc of the imagination, so I have to select which theory feels right to me.
            I like to go with radiative engineers who actually work with the stuff.
            They seem to agree that radiation from a colder to a hotter body is possible, but doesn’t change the temp. of the hotter body, because the colder radiation doesn’t have the energy to bring the hotter atoms at a higher vibrational level.

            Another point is all the “radiation” measurements. All these devices seem to measure a TEMPERATURE, and then calculate the amount of radiation using SB.
            So which temperature are you mesauring when looking upward?

            One more thing, not all temperatures are made equal 😉
            compare a sauna at 90C and a hot bath at 90 at 90C.
            Sauna no problem for me, hot bad? Ask a lobster.

            10

          • #
            KinkyKeith

            Hi BenAW

            You’ve got it.

            ALL objects above minus 273.16 deg C radiate energy but do so in the context of moving towards equilibrium with surroundings.

            For example: Two large cubes of ice (temp 0 deg C) with sides 1 metre length are place near each other in a freezer at minus 5 deg C.

            The two cubes will radiate from all 5 exposed faces (ignore the bottom one) and no doubt some of the radiation will be picked up by the neighbouring ice cube.

            The freezer air will be heated by the warmer surface of the cube, small convection cells will be set up and aid heat transfer to the freezer air which is maintained mechanically at – 5 deg.

            Eventually equilibrium will be reached and the ice will be – 5 deg C all the way through.

            At this point we now face our question: do these two blocks now radiate towards each other. maybe but from an engineering point of view it is irrelevant on a macroscopic scale that we live in.

            In the same way discussion of the special “Back-Radiation” used by Climate Scientists is irrelevant at the macroscopic level.

            It is only the net radiation that is important.

            Jose: appeals to authority are POINTLESS in a scientific discussion.

            10

          • #
            Jose_X

            @BenAW
            >> I like to go with radiative engineers who actually work with the stuff.

            For the record, the scientists who work at Harvard and Caltech and many other universities, as well as those from many companies and who believe in the traditional theories that support greenhouse effect, have plenty of experience. [I am countering an existing appeal to authority in likewise manner.]

            >> They seem to agree that radiation from a colder to a hotter body is possible, but doesn’t change the temp. of the hotter body, because the colder radiation doesn’t have the energy to bring the hotter atoms at a higher vibrational level.

            Let me ask this. Isn’t the sun at much higher temperature than the earth’s surface? Although I don’t understand the details of the theory you are talking about, can you clarify how having a very very hot sun (at much over 1000C) providing the initial source of energy to the earth somehow wouldn’t allow the earth to get hotter, say “merely” to 90C, if we were to place a sufficiently powerful ghg blanket over the planet? In other words, doesn’t the sun have sufficient energy for the 90C vibrational level?

            I’d also like to know (in the theory you are talking about) if a mirror somehow dilutes the ability of that radiation to later raise matter to the same vibrational level (temp) as it could do before?

            And can you explain how we can create temperatures in a laboratory or manufacturing plant that are much much much hotter than the earth’s temperature.. or even hotter then the temperature of the surface of the sun? How can it be that engineers can use low temperature items yet create the super high components (photons?) that would allow some substance to get into a super high “vibrational level”?

            >> Another point is all the “radiation” measurements. All these devices seem to measure a TEMPERATURE, and then calculate the amount of radiation using SB. So which temperature are you mesauring when looking upward?

            Why are you assuming the devices are measuring temperature?

            Why is it that at the same exact height, turning the instrument up to face the sky will produce a smaller value (even much smaller, if we go up in altitude enough) than when the instrument is facing down towards the earth? A thermometer will produce the same value no matter how we rotate it because that spot (and all spots nearby) are at the same temperature, but a pyrgeometer and a pyranometer will produce different values.

            >> compare a sauna at 90C and a hot bath at 90 at 90C.
            Sauna no problem for me, hot bad? Ask a lobster.

            Yes, part of the reason has to do with there being more energy packed into the denser water than into the air. Temperature is a measure of averages. [I didn’t catch how this helped disprove either the ghg effect or what the article at the top stated.]

            10

          • #
            Jose_X

            @KinkyKeith
            >> but do so in the context of moving towards equilibrium with surroundings.

            The atmosphere and earth have a statistical tendency to move towards equilibrium. One of the main argument here is about what that equilibrium value would be and what can help set it.

            >> the special “Back-Radiation” used by Climate Scientists is irrelevant at the macroscopic level

            I didn’t see how the -5C ice cube example disproves anything about greenhouse effect.

            >> It is only the net radiation that is important.

            Greenhouse gases increase the net radiation level hitting the earth’s surface.

            Also, the atmosphere acquires greater net radiation because the ghg are there to absorb energy that otherwise would quickly make its way towards space.

            As with the oven examples, after equilibrium, we have same energy going in as going out, but this happens at a higher temperature. Also, eventually all heat dissipates when we turn off the oven. There was no creation or destruction of energy.. it merely resided in different forms and in different locations at different times.

            How is it that we can turn on cold gases in a cold laser shell so that it can fire a beam of radiation that would be able to melt steel? How does your theory explain this?

            >> Jose: appeals to authority are POINTLESS in a scientific discussion.

            I argue the case, but every now and then I reply to someone who already has “appealed to authority” or I wonder what the person thinks of all the experiments and writings that have been done in support of what I am talking about. I haven’t seen anyone here give a rebuttal of current theories or explain “gotchas” with the alternative theories. For example, I pose __numerous__ questions that go __unanswered__. You aren’t going to convince me to follow a theory if you can’t defend it against challenges or if you can’t argue that our current accepted theories used to great success by many for many years would be wrong.

            10

          • #
            BenAW

            Jose_X
            January 24, 2012 at 1:53 pm

            I only stated that imo radiation from a COLDER body can’t warm a WARMER body.
            Of course the opposite is happening. pse read carefully

            “Why are you assuming the devices are measuring temperature?”
            Google pyrgeometer, sensor is a thermocline, google thermocline, find answer.

            How is colder air, with a lower “energy density” than the ground going to warm it?
            Only situation this is possible imo is when a low level inversion exists, and even then the “energy density” difference will make this a minor event.

            For a good read on measuring radiation:
            http://principia-scientific.org/publications/New_Concise_Experiment_on_Backradiation.pdf

            10

          • #
            Jose_X

            BenAW,
            >> pyrgeometer, sensor is a thermocline

            I found that little remark extremely distracting.

            🙂

            The word is “thermopile”, btw. For some reason I continue to forget it and have to keep looking for its spelling. Go figure.

            This comment comes in 5 sections.

            1. Laser Cooling: Yes, a warm object can further cool a cold object via radiation
            2. Temperature, Thermodynamics, Radiation, Chemical Storage, and the Sun
            3. Mirrors: Helping to achieve power
            4. Pyrgeometer: Does back radiation exist?
            5. The Greenhouse Effect

            Any theory which aims to displace certain properties of our radiation/photon model needs ultimately to be able to explain a whole lot of “modern” technology.

            1. Laser Cooling: Yes, a warm object can further cool a cold object via radiation

            First let’s read a little news, take your pick
            a) http://www.sciencedaily.com/releases/2007/04/070406171036.htm
            b) http://www.theregister.co.uk/2012/01/23/laser_cooled_semiconductor/

            Next, let’s look at how laser cooling works via a short online learning demo
            http://www.ph.unimelb.edu.au/~scholten/opticshome/lasercool/lasercool.html

            What we see is that there are ways to adjust the frequency of laser light radiation and aim it to slow down some molecules more than others are sped up, eventually leading to an extremely cold surface.

            The moral of the story is that humans can engineer materials and power sources to apparently defy thermodynamics if we don’t look at thermodynamics carefully. Also, we need many theories beyond thermodynamics to understand many details in nature. Arguably the most successful family of theories are quantum mechanics. Radiative transfer and many of the fundamental aspects behind the greenhouse effect are clearly within quantum mechanical predictions. While thermodynamics gives a rough guide to go buy and sets up some bounds, quantum mechanics deals with many low level details.

            2. Temperature, Thermodynamics, Radiation, Chemical Storage, and the Sun

            Quickly, let’s see how the first and second laws of thermodynamics are likely not violated with laser cooling. These laws have roots going back to before we understood radiation well, but the laws still hold.

            From http://en.wikipedia.org/wiki/Laws_of_thermodynamics , the first law:
            > Increase in internal energy of a system = heat supplied to the system – work done by the system.

            This says nothing about warm objects not being able to cool cold objects. As long as we (aka, the environment) keep adding more heat or doing more work to the system than it releases or does, the temperature will keep increasing.

            And what about thermodynamic equilibrium? or the second law?

            The link above also explains the second law a bit.

            An important idea to understanding laser cooling is that we create an interchange media (aka, the boundary between the target and the environment) that slow down conduction and convection to almost negligible rates in comparison to the rates of radiation energy transfer. Radiation is more easy to guide precisely than conduction or convection and allows us to avoid strong conductive/convective effects since it can work very fast “from a distance”. Also, radiation can interact with matter through reflection and transmittance in addition to absorption, allowing us to potentially guide energy towards places without losing much along the way.

            So is the second law violated? Probably not (if we could measure it accurately to know). The second law allows one system (or one part of the system) to get cold while another gets warm. What it doesn’t allow is for there to be less “disorder” (entropy) after that interaction. This basically means that we can possibly get something to be much hotter or colder than the surroundings, but we must sacrifice something in the process. For example, we must scatter extra energy around to the environment (“waste heat”) as we remove heat from the target. Air conditioners and laser cooling each do this.

            A key ingredient to humans engineering “defying” acts of temperature manipulation has been our mastery of various properties of electromagnetic radiation. Principally among these has been our mastery of generators/motors and of other energy transfer tools (like radiation-emitting lasers). So electromagnetic radiation has played a very important part in conquering heat flows. Naturally, we create disorder in the process, but without radiation we could not achieve such control.

            Note that at the core of all our abilities is the sun’s energy (and eons old potential energy stored in the nucleus of atoms, energy which arguably also originated from the sun). Chemical storage is how “plants” have tapped the sun and allows us to have energy from food and to power machines. The energy of molecules can certainly be transferred around as electromagnetic radiation. Via radiation, the molecular and electronic potential energy changes to provide doses of energy to far away objects. One object cools while another later gains heat. And the radiation doesn’t require that the source be warmer than the destination. We can, as laser cooling shows, have a warmer object, such as the gas heated within the laser via a cooler energy source and mirrors, provide lots of radiation yet manage to cool down the target. As long as we have energy to waste and keep increasing the entropy of the universe, we can heat or cool almost anything to almost any temperature.

            Mother nature too can perform these feats since the same physical laws that guide our products also guide the rest of nature.

            3. Mirrors: Helping to achieve power

            Mirrors allow us to divert radiation from some source to a desired destination. This reflective property is key to enabling various forms of heat concentration. We see a simple example here, http://en.wikipedia.org/wiki/Concentrated_solar_power .

            We can also control transmission and use it to guide photons to a destination of choice.

            Reflection and transmission are in fact two powerful ways to keep the radiation on a controlled path instead of being absorbed and later dissipated in random directions (eg, as waste heat). To say that mirrors or lenses cannot be used to redirect radiation energy so that it concentrates and potentially heats a source further is to deny the existence of the magnifying glass that enables us to start fires, the telescope allowing us to see otherwise weakly faint objects, superior cooking techniques, and the laser, to name just a few inescapable tools in our society. Throughout our uses of these tools, entropy is increasing. We increase the rate of heat flow to the heated object to overcome enough of the cooling that would otherwise result. And the source of this energy need not be as hot as the destination. The mirrors surface may not be warm enough to cook a hotdog or boil water, but the radiation focused by the mirror certainly can.

            Let’s wrap up mirrors with the example of the laser.
            http://en.wikipedia.org/wiki/Laser#Design
            http://en.wikipedia.org/wiki/Laser_pumping#Optical_pumping
            http://www.colorado.edu/physics/PhysicsInitiative/Physics2000/lasers/lasers2.html This one has animation on page 4.

            These links basically show that we can heat up a core heating element (eg, with electricity) so that it releases isotropically as a “blackbody” a fair amount of radiation to its surroundings. Mirrors surrounding this heated element (for example, leveraging our mathematical understanding of the properties of ellipses and various surfaces) reflect much of this radiation to, for example, a glass tube containing CO2. The radiation not absorbed by the gas (because the gas absorbs limited spectra of radiation) is absorbed by the tube itself and “re-radiated” as a blackbody again, providing another dose of the right frequency acceptable to the gas. The electrically excited gas molecule can lose that energy in various ways, including by bumping around, but when enough of the gas is excited from enough doses of radiation it absorbed at the characteristic frequencies, new radiation at those frequencies hitting the excited molecules again leads to two photons leaving alongside each other and a relaxed molecule ready to be excited once again. In essence, rather than having the excited states decay ordinarily and without fanfare, the high frequency of radiation bombardment leads to magnification of characteristic frequency(ies). Eventually, the bouncing photons once aligned and aimed properly can leave through a narrow exit, contributing to the laser beam we see leaving the laser cavity. The laser beam radiation can be designed to be extremely energized and focused to weld, cut, superheat, etc, steel and many other materials. All of this from a normal power source and thanks to mirrors.

            Yes, mirrors can help really heat up a target surface by helping to aim sufficient radiation.

            4. Pyrgeometer: Does back radiation exist?

            Our eyes sense visible light. Our skin reacts to a lot of IR. There are a number of precision instruments which can be used to measure radiation of various frequencies. A quick googling for “spectroscopy instruments” yielded this selection from some firm http://www.newport.com/Spectroscopy-Instruments/995901/1033/content.aspx .

            Let’s focus on a pyrgeometer.

            At its heart we have a device called a thermopile, which consists of numerous thermocouples. A thermocouple is basically two wires each made out of a different metal alloy and joined together to form a circuit. One end of one wire is fused to one end of the other wire and the remaining two ends likewise are joined to each other. This simple instrument will develop a small potential difference between each of these two joints whenever the joints are at different temperatures. Thus, a thermocoupler, and a thermopile by extension, can be used to identify a difference in temperature. Note that a thermocoupler does not identify a temperature, only a difference in temperature between its “hot” joint and its “cold” joint. For standardized pairs of alloys, formulas are provided for converting the voltage values into degrees.

            To understand how we jump from a temperature gradient sensor to a radiation sensor, we can ask, what is temperature, what is radiation, and how do they differ? Intuitively, we know that things tend to move towards the same temperature when they are near each other. We know hot, and we know cold. Without going into precise experiments, we also understand that radiation is something that likely exists and can transmit energy across distances without involving molecules in between to do the work. We are familiar with lasers, televisions, x ray machines, and the sun, to name a few sources of radiation.

            The focus next will be on briefly describing an experiment using a thermocoupler so as to find a measurable relationship between a temperature gradient, which we already know the thermocoupler can identify and quantify, and radiation, which is what we would like to be able to measure. In fact, the experiment will serve as a weak test of the Stefan Boltzmann relationship between temperature and radiation. The motivation for the experiment is the exercise described at the bottom of this page of a thermodynamics course http://mit.edu/16.unified/www/FALL/thermodynamics/notes/node136.html

            Thermocoupler Experiment

            The “cold” end of a thermocoupler is kept at a reference temperature. The “hot” end is inserted into a metal cylinder that has its ends missing so that air can circulate. One thermometer measures the temperature of the hot joint. Another thermometer measures the temperature of the air inside. A third thermometer measures the temperature of the cylinder. We place a heating element around the cylindrical shell to heat up the shell significantly. Finally, we press one end of a volt meter to the hot joint and the other against the cold joint to measure the voltage drop once steady state is reached. Then we repeat the measurements (recording the steady state value) after we have added a protective narrower cylindrical radiation shield barrier ( http://en.wikipedia.org/wiki/Cold_shield ?) around the hot joint so that it eliminates direct line of path from the hot joint to every part of the hot outside cylinder. Also consider taking measurements at various temperatures of the outside shell if want to test the fourth power relationship between flux and temperature.

            Although this experiment is not tightly controlled, it does show a very interesting effect. [BTW, I have not carried it out.] Without the shield, we should find that the temperature of the air inside the cylinder will be close to room temperature but certainly less than the temperature of the hot joint. The hot joint will be less than the temperature of the outside cylinder. When we put the shield on, we should find that convection cools the hot joint to the same temperature as the inside air.

            Does anyone want to propose an explanation besides the following: direct radiation in sufficient quantities heats up the hot metal joint beyond what the air would otherwise do by itself through convection? In other words, accepted theories stipulate that adding “sufficient extra” radiation can raise the temperature of the target beyond ambient temp, even when convection is working to re-establish equilibrium.

            “Sure, perhaps the radiation from the hot outside cylinder added extra heat to the inside metal joint — yes, I accept that radiation exists and can transfer heat,” you might be thinking, “but we knew the outside cylinder was very hot so it makes sense its radiation can heat things.”

            Pyrgeometer (and Pyranometer)

            Well, the point of the experiment was simply to suggest that a pyrgeometer can in fact measure radiation levels if it knows the ambient temperature and yet senses a temperature gradient when the “hot joint” is exposed to the atmosphere. If this happens, we might have to accept there is “back radiation”.

            So, under the belief (supported by this experiment above and implied by Stefan Boltzmann) that extra net radiation will register as higher temperature, we now want to isolate the hot joint in a way that will be able to register that higher temperature if any such back radiation exists. Let’s add a filter (eg, glass or other covering) around the “hot joint” of the thermopile in order to block out shortwave radiation. This transparent barrier would be subject to being cooled by convection, so it would have the “same” temperature as the outside. The barrier would be separated a little from the thermopile hot joint so as not to cool (or heat it) by conduction. Additionally, the barrier is a full seal so prevents convection from the outside from affecting the any extra temperature that might be registered on the hot joint. We can even test the pyrgeometer inside under a low IR radiation environment (eg, covered with a full infrared shield) to verify that no matter the temperature of the room, the device will register almost no “back radiation”. Of course, we have to add electronics so that we can record the voltage values on the thermopile. We also need a thermometer inside the pyrgeometer next to the “cold joint”. This temperature reading of ambient temperature is needed calculate the proper flux (radiation) values from Stefan Boltsmann (using an emissivity value probably near 1 but which can be measured). Later we can process all of those values via the conversion formulas and S-B to derive W/m^2.

            AND we can construct a similar device but aimed instead at picking up shortwave radiation only. Such a device is called a pyranometer.

            We take our pyrgeometer and pyranometer outside above the level of buildings and see what it records.

            We expect the pyrgeometer will register (in a place like Point Reyes, California) radiation values in the 300 to 400 range both day and night in the Summer and lower values during Winter. We expect the pyranometer to register values approaching 1000 during the middle of the day in Summer, dropping off to zero for the night, and to have lower values in Winter. Clouds and heavy fog will have an impact on the measurements. We might also turn the devices around to record upward radiation levels and compare those as well to theory. [In other comments, I posted links to some graphs.] And we might also record at different altitudes.

            5. Greenhouse effect

            Nature doesn’t use mirrors in our atmosphere, but it does leverage the sun’s constant supply of power, the energy of molecules, and the ability of molecules to transfer energy to each other via radiation. It leverages these things to divert some radiation that would otherwise be on its way into deep outer space back towards the planet. As could be seen in earlier examples, any radiation that is sent to a target, regardless of the source, has the potential to heat the target (although interestingly, it can be used to cool the target under very precise conditions, eg, through laser cooling). With instruments measuring hundreds of W/m^2, we can see that back radiation is a real “source” of heat. That is, the capture and recycling of radiation on its way into space provides a mechanism for the earth to remain at an elevated temperature.

            Well, that is the theory anyway. With confidence that back radiation exists, it is much easier to support such a theory and to reject theories that don’t allow for downward IR radiation or which cannot derive a temperature for the earth’s surface that is consistent with this level of radiation.

            10

          • #
            blouis79

            #604.2.2.2.6@Jose_X

            The demo http://www.ph.unimelb.edu.au/~scholten/opticshome/lasercool/lasercool.html clearly shows what happens to light at the wavelength of atomic resonance – absorption, and emission within 30ns typically in random directions – as I had mentioned in another post #619. So how this causes greenhouse gases to warm is a complete scientific mystery or fallacy. The experiment shows GHGs could cause cooling.

            10

          • #
            Jose_X

            blouis79, you have to hit “next” button numerous times to see the whole segment and explanation.

            The beginning shows what happens when the frequency is tuned exactly to “resonance”: the atom (likely) acquires electrical energy and some motion away from the laser, and then it releases a randomly directly photon by spontaneous emission (if it doesn’t lose the energy first through some other means). While the kicks it gets from the spontaneous emissions “cancel out” to 0 over time, there is a net growth in velocity in the direction of the laser. In this way the atom acquires energy.

            If we detune the laser below resonance, then the photons getting the greatest kick are the ones moving a little towards the laser. The kicks thus slow the atom down and causes them to start moving away from the laser, but as they slow down and change direction, the kick gets weaker and weaker. By adding an opposite directed laser also detuned below resonance (and then adding more sets to cover y and z dimensions), you create a situation where atoms experienc eincreasing force against motion.

            One analogy is that you are surrounded by springs and when you move in any direction, the springs slow you down towards where you came from. You are boxed in. Remember, this happens because the laser the pushes the strongest against you (of the two opposite aligned pairs) is always the one towards which you are moving.

            A second analogy (to explain resonance concept) is seen when pushing a kid on the swings in the park. If you drive the kid at resonance, as you normally do, you give them a little boost each cycle in the direction of their motion when they are most likely to react favorably to that boost (right after they crest and change directions). What laser cooling does instead is to give them a net little push in the opposite direction.

            ..so when you finish reading the frame, hit “next” again to see the next part of the story.

            10

          • #
            Jose_X

            > While the kicks it gets from the spontaneous emissions “cancel out” to 0 over time, there is a net growth in velocity

            Ie, while the kicks the atom gets from every phase of *spontaneous emission* cancel out to 0, the kicks it gets from the phase of *laser photon acquisition* give the atom a net growing velocity away from the laser.

            BTW, if you want, start a new thread and reference the main comment from there.

            10

        • #
          Jose_X

          [The real comment has links and is in moderation. This version is missing the links since maybe that is why it got trapped.. and I added a bit at the end.]

          >> Incoming solar in the tropics can reach ~1000W/m^2 AFTER correcting for 30% albedo.

          Yes, you can see graphs of a location in California that approaches the 1000 W/m^2 for downward shortwave radiation during the daylight hours. This comment -link1- covers that and related graphs.

          >> Warm ocean heats atmosphere from below by conduction and convection.
          GHE is non-existent.

          Graphs showing upward/downward longwave/shortwave radiation can be found from that link just given. The radiation level numbers add up similarly to the popular Trenberth diagrams. There is significant radiation near the ground both upward facing and downward facing.

          Considering water has an emissivity -link2- very near to 1 -link3- , I think I will place my best with the view that radiation is escaping into the atmosphere in significant quantities.

          CO2 and H2O together can absorb significant amounts in that IR range emitted by the earth+oceans. [ -link4- ] That absorbed energy goes somewhere. And the complementary “re-radiation” is isotropic, meaning that near the ground about half is aimed at it. The ground doesn’t discriminate between photons that hit is from the sun or from somewhere else.

          I think I am making the point fairly, but I’ll ask for a little bit of help. Here are lecture notes on greenhouse effect by a Caltech professor -link5- . Like with the Harvard professor I linked to, I want to add evidence that the Caltech lecture isn’t controversial as seen by the school (or by the world). Caltech, arguably the top graduate physics institution in the country if not the world, describes their commitment to trying to keep a check on the greenhouse effect. -link6- . From -link7- “Caltech was ranked as the best university in the world in two categories: Engineering & Technology and Physical Sciences.” Caltech, like Harvard, teaches a lot more than simply “climate scientists”. If you mock their well accepted teachings, you are mocking physicists and scientists in general. Sure, what do those folks know? But, please, feel free to completely ignore this paragraph and instead focus on the prior ones providing the argument and associated evidence.

          Really, though, the greenhouse effect has genuine, bona fide, evidence-supported physics behind it. Challenging it is not a high probability shot.

          10

          • #
            KinkyKeith

            Jose

            I agree when you talk about “our current accepted theories used to great success by many for many years”

            They have been very successful and have raked in a a lot of MONEY.

            10

  • #
    KinkyKeith

    Hi Jose

    Checked out your link.

    “:http://green.harvard.edu/greenhousegas

    That’s not science, it’s advocacy. Green Nutter stuff – like religious fanatics wearing a hair shirt in the old days – pointless.

    Sure maybe it sounds good because it comes from Harvard but I don’t think Harvard Lawyers

    have much to contribute to the engineering problem of Global CO2 Warming.

    10

    • #
      Jose_X

      Right. I gave an example of physics lecture notes, and then I showed Harvard’s official position agreeing with it.

      Feel free to provide Harvard researcher views that disagree.

      10

  • #
    blouis79

    For those who think IR radiation is a dominant mode of heat transfer, try this in the kitchen:
    Turn on a stove burner. See how close you can put your hand over the top of it. See how close you can put your hand to the side of it. Put a pot of water on top. See if it makes a difference to how close you can get to the burner top and sides.

    An IR thermometer will measure the burner temperature by IR radiation from some distance away, but the perception of heat will not be the same.

    The surface of earth is warm because it has an atmosphere, but that has nothing to do with a “greenhouse”. According to Postma, the temperature of earth is as it should be without any need for a “greenhouse” effect.

    10

    • #
      Jose_X

      >> Turn on a stove burner. See how close you can put your hand over the top of it

      Energy is conserved, so I’m not sure what is the point beyond risking burning your hand. [There are losses of heat at each boundary.. and radiation is also more isotropic (escaping to the surroundings more easily) than the steam rising (and cooling before it gets to your hand.]

      10

      • #
        blouis79

        The point is simply to demonstrate in the real world the modes of heat transfer – conduction, convection, radiation. Basically, convection dominates over the others, since conduction and radiation are omnidirectional.

        10

        • #
          KinkyKeith

          Well said Blouis79.

          You’ve just got to look at te trees being blown about to know intuitively that convection is the major mechanism of moving heat around the Earth.

          Nobody has ever broken down a rough estimate of energy involved in the various atmospheric processes using up energy ie. radiation vs LHV water (2 metres off oceans pa) vs heat taken to upper atmosphere vertically vs friction (energy/ turbulence) losses of lateral winds.

          10

          • #
            Jose_X

            @blouis79
            >> The point is simply to demonstrate in the real world the modes of heat transfer – conduction, convection, radiation. Basically, convection dominates

            The dominant form between the burner and the pot is radiation if the pot is held off the burner (otherwise conduction perhaps); however, the water in the pot will not acquire more (net) energy than the burner gave the pot.

            If your hand felt otherwise, then perhaps you are “starting” to understand why science is not done with people’s organs for measuring purposes (ie, hands being unreliable was one point I tried to make). You would also have to make sure you don’t let any significant amount of radiation or steam escape.. meaning you would burn yourself trying to perform a careful experiment.

            >> convection dominates over the other, since conduction and radiation are omnidirectional.

            Convection reshuffles particles around. Radiation is the transfer of energy across space so that what was hotter net loses energy and what was colder net gains. They are different phenomena. Sometimes one applies, sometimes the other, or both.

            So I am not sure what you mean by “dominates”.

            Note that radiation can help increase the temperature in a lot of places “at once” while convection generally is associated with higher temperatures in some areas and colder temperatures in others.

            Maybe what you are asking is answered below:

            @KinkyKeith
            >> Nobody has ever broken down a rough estimate of energy involved in the various atmospheric processes using up energy

            Did you intend for that to be a question, or are you really saying you have researched this extensively and found nothing?

            Would you consider the Trenberth diagrams to be an example of such atmosphere accounting? The paper is based on research other scientists have published. Eg, http://www.google.com/url?sa=t&rct=j&q=&esrc=s&source=web&cd=1&sqi=2&ved=0CCYQFjAA&url=http%3A%2F%2Fwww.cgd.ucar.edu%2Fcas%2Fpapers%2FKiehlTrenbBAMS97.pdf&ei=VTIeT7KCMISEtge35NQl&usg=AFQjCNGdRdbgWevsR8NZD0Ov7M7Gsfah9w
            see diagram on page 10.

            >> You’ve just got to look at te trees being blown about to know intuitively that convection is the major mechanism of moving heat around the Earth.

            I think you are confusing concentration of energy vs total energy across a large area.

            Intuitively, the sun doesn’t feel like it is doing very much at all in comparison to hurricanes, tornados, etc, yet it is the source of where the wind events get their energy.

            Goes to show you that science and engineering are not built upon intuition because intuition is frequently biased and incorrect.

            For example, you don’t want to glue airplane wings onto the plane simply using intuition of what will work. Intuition offers you a start or a last resort when you have to guess, but it should not form your primary tool for decision-making in important matters.

            Eg, casinos use math to come out ahead. Gamblers frequently use intuition and frequently lose. And the smart methodical gamblers are kicked out of casinos! Moral of this example: intuition is great for spending dollars for entertainment purposes but not for making money on a reliable basis. The big money makers are studious of their “art”.

            10

  • #
    Jose_X

    [I am resubmitting this comment since it wasn’t posted yesterday (maybe I only thought I submitted it). It was in reply to someone (can’t remember) saying radiation plays small role.]

    Your claim that radiation plays a small role appears to go against the evidence of radiation measurements taken pointing up as well as pointing down. Warm bodies radiate.

    Advection might play an important role in redistributing heat that is in the air already, but conduction in general is not an efficient way of transferring from the ground to the air molecules. Evaporation is more significant but not as much as radiation.

    Again, radiation levels are very significant as instrument measurements suggest. Remember that we have a huge atmosphere. Radiation is being absorbed and also radiates from many places above, even if each such location is far apart from the others.

    Radiation raining down from ghg is extra photons bombarding the earth in addition to what the sun does. Like the sun transfers heat via radiation, the atmosphere can transfer heat to the earth through radiation. Photon energy that would be flying through outer space if there were no ghg instead spend some extra time near the planet because of absorption and “re-emission”. The total energy near the earth is higher. For a given emissivity, Stefan Boltzmann associates temperature with incident radiation. That is, the earth’s surface temperature correlates strongly to the radiation it gets from the “back radiation” from ghg in addition to what it receives from the sun directly.

    [ghg absorb/emit in a limited range of frequencies.. “shortwave” coming from the sun in large quantities largely pass through the atmosphere (except the higher ranges like uv, which are absorbed by ozone), while a fair quantity of the lower frequency “longwave” radiation originating from the earth can be absorbed by the ghg]

    Photons of a diverse (if partly limited) range of energies have a probability of passing near enough to a CO2 or other ghg molecule and being absorbed. In particular, IR frequencies include the vibration quanta levels of these molecules. So instead of going into outer space, they can energize a ghg molecule. This essentially adds to the average temp of the atmosphere via collisions with other molecules ( nonradiative relaxation ) in a way that is in addition to whatever evaporation or any other heat transfer method would achieve.

    But, just like warm solids radiate in IR range, so too, in more limited ways, do gases with spectral lines in the IR range. http://en.wikipedia.org/wiki/Spontaneous_emission .

    Note that at a given temperature, a gas would have molecules that are more energized and molecules that are less energized than the average temp dictates ( see wikipedia Maxwell Boltzmann distribution ). I think those in the upper range of energies are the ones most likely to emit a photon spontaneously. You can even have a photon radiate away as two molecules are “colliding”.

    10

    • #
      KinkyKeith

      Hi Jose

      Radiation only ??

      Go and stand near the ocean late on a summers afternoon in my home town and you will be blown away by the winds coming up as the “southerly buster” from Sydney and parts further south.

      Now try and assess how many giant wind turbines would be needed to create that blow. Impossible – millions. How much energy ??

      On a cloudy day, look up at the clouds, thousand of tonnes of WATER just hanging in the sky 1000 metres up. How much energy was used to lift at that mass to that height??

      The energy for these natural occurences comes from incoming UV and visible light.

      It has been used up and cannot be re radiated back to space, and yet “climatologists” tell us the incoming and outgoing radiation are in balance??

      What about a total energy audit, long overdue.

      ps. Take no notice of T, you are out of your depth.

      10

      • #
        KinkyKeith

        In all my previous posts I have allowed that the basic greenhouse effect (my definition – no stupid rebound radiation) would allow CO2 to absorb ground IR which could not be absorbed by other gas molecules and pass this on instantly as radiation or KE though impact.

        Not having done any physics for many years I allowed that this might be possible for one reason, to explore the quantification issue of man made CO2 in that scheme.

        I showed that ALL CO2 had pretty much no possibility of increasing world temperature, because water vapour did near all of the work, but that Human Origin CO2 would NEVER be an issue.

        Now after reading G & T I find that even the basic premise I allowed for arguments sake, is not a real mechanism, there being other factors at work.

        The collapse of the AGW scheme is complete.

        10

        • #
          Jose_X

          >> no stupid rebound radiation

          You mean spontaneous isotropic emission, which would include “back radiation” some of the time? Sure, go ahead and ignore the evidence. I’m sure you will come up with a wonderful model of reality. Alice in Wonderland wonderful.

          >> I showed that ALL CO2 had pretty much no possibility of increasing world temperature, because water vapour did near all of the work

          If you have a link to your paper, I would appreciate it, you know, so I could go over the math and proof for myself.

          >> after reading G & T I find that even the basic premise .. is not a real mechanism

          Will try to get to the paper as soon as I find time, but I’m not holding my breath it proves such a thing. BTW, why don’t you use the arguments from that paper here?

          10

      • #
        Jose_X

        I don’t know what you mean by “radiation only” since I said nothing of the sort.

        It’s great that you mention all of this massive power, but don’t forget the following two points.

        1) The sun packs a punch into every square meter on the planet every second.

        2) Energy is conserved. What you might call “waste heat” is just that, heat that warms the environment and adds to what goes into the atmosphere. How much energy is “consumed” by a massive pendulum swinging over and over? Well, a little is surely lost to heat with each cycle (and warms something), but most of it is simply converted from one form of energy potential into kinetic energy and back. Once we tap into that pendulum, we get work out of it, essentially changing the energy into some other form and/or becoming heat right away. There is no escaping this if you want to stick with the laws of physics people have been testing and leveraging for many years.

        10

  • #
    Tristan

    Jose, I admire your calm, thorough approach.

    10

  • #
    Paul

    The point at issue, as far as I am concerned, is not whether the accepted scientific theories of thermal dynamics are ‘true’ or ‘not true’ but whether they apply to our atmosphere in the way that they are being applied within the framework of the accepted ‘greenhouse’ gas theory.

    As an analogy, consider the movement of passengers on a train or bus. Does the movement of the passengers within the vehicle have any appreciable impact on the rate of transfer between stations?

    In just the same way, the properties of still air have very little to say about the transport of heat from the surface of the earth to the top of the troposphere, where the main vehicle for transporting the heat is the convection currents that begin early each day and persist until early evening.

    Overnight the air does become still and the insulating properties of still air keep the side of the earth that is shielded from the solar radiation from cooling more rapidly than it would in the absence of an atmosphere.

    The main fallacies implicit in the accepted theory, are therefor, firstly, ignoring day and night, averaging the solar radiation received as though it were continuously received, and secondly, ignoring that the troposphere acts as a giant air-conditioner by day.

    Postma has done a very good job of showing that, apart from these two fallacies, there is no mystery in the average near-earth atmospheric temperatures and showing that the back-radiation of atmospheric Tindall gasses is a hypothesis posed to answer a non-existent problem.

    All discussion as to whether or not Tindall gasses, that are cooler than the surface can warm the surface, as is alleged, and supporting this contention with word-plays that allude to recognized properties of carbon dioxide molecules, etc, etc, are just semantics. The experiments which established those properties had to eliminate, as far as possible, all other transport of energy, such as conduction and convection, in order to measure the one remaining property, radiation through the gas. Those laboratory conditions would require that the surface of the earth be isolated from the atmosphere by a vacuum gap, to prevent conduction and be kept still to prevent convection! Not very likely.

    Paul

    10

    • #
      Jose_X

      >> As an analogy, consider the movement of passengers on a train or bus. Does the movement of the passengers within the vehicle have any appreciable impact on the rate of transfer between stations?

      I would not be surprised if there is a correlation with what people are doing. For example, there is an increased chance that the person about to leave is moving towards the door or getting up. A person not leaving has a greater chance of moving away from the door or of taking up the seat of someone who has just gotten up.

      >> In just the same way, the properties of still air have very little to say about the transport of heat from the surface of the earth to the top of the troposphere, where the main vehicle for transporting the heat is the convection currents that begin early each day and persist until early evening.

      Radiation is a property of molecules and photons, and all of them are moving all the time regardless.

      Radiation is also closely associated with temperature, and these molecules essentially all have temperature whether there is a tiny net drift in their localized movements (eg, air mass moving up or down) or not.

      Movement of molecules increases the range of photons they have a chance to absorb (eg, doppler shift to widen the absorption/emission band).

      The rules are there to cover many cases.

      Anyway, measurements have to be accounted for. A theory that says radiation levels are small relative to the sun flux will have a problem.

      A theory that says that you can’t “get hot” from your own radiation (eg, as in mirror reflection) will face a lot of potential contradictions. For example, the theory might resolve (a) to tagging photons around, claiming some photons will have the ability to impart or not impart their energy based on where they came from and not just based on their frequency/energy or (b) to creating an artificial distinction between “heat” and “light” or something like that. Any of these views already puts them at odds with quantum mechanics (and QED). And, more importantly, this is fundamental physics being challenged, not just “climate science”.

      You state you are not fighting accepted principles and basic physics theories, just their application, but if you agree with the referenced Postma paper, then you are attacking basic physics.

      If you don’t mind attacking fundamental physics used in numerous disciplines, then you should be clear about that and realize you are going to have to explain a whole lot of things currently explained by the views being dismissed.

      >> The main fallacies implicit in the accepted theory, are therefor, firstly, ignoring day and night.. and secondly, ignoring that the troposphere acts as a giant air-conditioner by day.

      Do you have numbers to show what sort of difference would exist if the current climate models used a different assumption (I’ll assume they do make this assumption of focusing on average radiation from sun).

      I think I read (as I would guess) that some model runs can be more precise but require a huge amount of running time to go far into the future, but the less accurate runs have produced decent results when compared with the more accurately modeled runs. From some sort of analysis they might use the less detailed modeling and try to add error bars.

      The Trenberth diagram has a nonnegligible spot for evaporation and for convection, so these aren’t being ignored. Perhaps you think it is being under-represented. If so, consider providing a link to mathematics+models or “measurements” that produce different numbers. Just thinking something is fishy is not enough. Sometimes approximations are very legitimate (everything is based off some approximation or other) and sometimes they are deceivingly not so. Gut feelings don’t really convince the wider scientific community.

      >> Postma has done a very good job of showing

      I read/skim through large portions of that paper but did skip a few pages in the middle where some calculations are done. I’ll review that.

      However, Postma has stated properties of photons (eg, the mirror issue) that defy Quantum Mechanics, energy conservation (as believed applies to radiation intensity), etc, and I believe contradict lots of measurements (although he may want to provide a reinterpretation of experiments as he is challenged).

      >> Those laboratory conditions would require that the surface of the earth be isolated from the atmosphere …

      That’s generally how theory and experimentation works. There is always an approximation carried out.

      I suspect the problem here is us not knowing exactly what has or hasn’t been tested in a laboratory. I am sure you have never fallen off the 100th floor of building without a parachute or something to gently break your fall, yet you probably are rather convinced that falling off the 100th floor will hurt you and cause you to follow a certain path. You might even come to this belief based on smaller model laboratory experiments (maybe a 4th floor drop). Generalization and extrapolation is a part of being a human and doing science, and we aren’t always right. But I suspect the problem here is that we simply aren’t aware of the details of experiments that have convinced scientists of certain physical rules.

      Do you want to list realistic laboratory requirements that if met might very well convince you the greenhouse effect exists?

      10

    • #
      Jose_X

      I just went back to reading the Postma paper. On page 20 it says:

      > Let’s consider a vertical column of gas, or air, in the atmosphere. In the long-term average, this column of gas will be in thermal equilibrium with the solar input energy, and so will contain a
      finite and constant total energy.

      So right away it seems Postma is violating what you say climate scientists are doing wrong: Postma is also assuming various average conditions, including night+day approximations that discard natural movement of air and temperature variations through every such air column he is modelling.

      So did you realize that Postma’s proof made these assumptions you were so critical of?

      Do you now see this (oooo horrific, evil, inaccurate) assumption as evidence that Postma hasn’t disproved the greenhouse effect? He didn’t even offer an analysis of the assumption/approximation!!!!

      Note also that he used the average day/night approach (and for good reason) to arrive at standard power radiation levels earlier in the paper. He stated right before the above quote:

      >The really nice thing about thermodynamics is that you can use the simplest equations to predict the outcome of complex systems, such as we have seen for the thermodynamic equilibrium temperature of Earth’s
      ground + atmosphere system. The calculation perfectly matched the observation. We can do the exact same thing for the distribution of temperature of a gas in a gravitational field.

      So to criticize “climate scientists” but not Postma weakens your case.

      10

    • #
      Jose_X

      OK, I sort of read pg 21 and I get the feeling this paper makes the same sort of mistake as do 2 other papers I read recently. It gets its values by measurements made today, yet tries to infer those results would apply tomorrow.

      That’s not how it works. You can make measurement today, but then you have to have a way to evolve that system as it might then exist tomorrow, unless you “prove” that the time variation can be assumed to be constant.

      Ignoring other possible flaws with the assumptions made, the lapse rate (eg, because of the specific heat value it depends on) vary with the composition of the atmosphere. Don’t take my word for it, he quoted two values, one for atmospheres with more or less water (“dry” vs “moist”). And the composition of the atmosphere (including its specific heat capacity and its adiabatic lapse rate) vary with time if CO2 (H2O, etc) levels change it and if those CO2 levels were to vary with time (and they are currently varying and are also assumed to vary in calculating 2xCO2).

      An actually useful analysis (for predicting future effects) would model these values as functions of time (or as functions of CO2 levels and/or other things that cause these values to change in the future).

      Note that, mathematically, the assumption of constancy through time was made when solving the simple differential equation of {14} to arrive at {15}. In other words, it was assumed g and Cp were constants for all time or otherwise {15} would have been different (and would have had to depend on CO2, etc).

      BTW, that simplified assumption he made might be fine if those values don’t change much with CO2 level changes, etc. But one has to analyze that.

      Also note that g varies with height. For a few kilometers it’s probably not an issue, but keep in mind we’d have to analyze this as well. Again, such a dependency would lead to a different {15}.

      Anyway, to go from those (likely bad) assumptions to “proving” that the greenhouse effect doesn’t exist is beyond me. I mean, I can make a measurement of your height X and then state that the height of people with your name and age and place of birth and favorite color is always X. It’s always easy to make a model that agrees with a limited set of data, but you can’t use that to argue that eating doesn’t help people grow just because you found an equation that works for the selected case and you got that equation without taking into account “eating”.

      Disproving greenhouse effect means seeing what the greenhouse model predicts and then showing significant deviation from what it predicts. However, finding an alternative path (whether such path is silly or insightful) doesn’t disprove some other path doesn’t exist.

      And if two paths exist, I’ll take the one that was designed to make future predictions (to the extent it is correct, of course).

      10

      • #
        KinkyKeith

        Who Cares.

        More Mumbo Jumbo.

        10

        • #
          Jose_X

          >> Who Cares.
          >> More Mumbo Jumbo.

          The gist of the problem I see with the paper.

          It assumes you can’t direct photons back at you in a way to give you a net gain in temperature. I disagree this is impossible or that it violates the second law of thermodynamics. The paper makes the assumption without much explanation.

          The paper then derives a formula for finding the temperature up and down the atmosphere. Due to this formula and the assumption just stated, it deduces greenhouse effect is not necessary for explaining temperature and is actually incorrect.

          The 3 problems with the formula are that:
          (a) it is oversimplified and doesn’t actually deduce the temperature accurately today (but this is fine if we just want an approximation);
          (b) it doesn’t explain why sometimes going up the atmosphere leads to temperature increases (while perhaps related to radiative transfers, some of these changes aren’t related to the “greenhouse effect” so we can give it a partial pass on this point); and
          (c) the formula does not apply years from now when the atmosphere content changes appreciably.

          (c) is why we need greenhouse theory, since it allows us to make a reasonable guess at what the formula would look like in the future. The paper deduces the formula only for today and only by making measurements. We can’t obviously measurements the atmosphere of tomorrow today because it hasn’t yet changed today.

          We want a predictive formula, a formula that tells you want happens as time evolves, so we can plan for tomorrow today (and maybe avoid that tomorrow if the tomorrow would be bad enough).

          Note that the paper solves an equation that takes time evolution into account, but it is insufficient for the purposes of analyzing CO2 since it basically assume outright that no changes would occur.

          If you are out to find the truth of what is going on, then details such as I mention here are important. We don’t build a plane that stays in the sky by carrying out an analysis that assumes the plane will stay up in the sky. You have to be darn confident in something to make that assumption ahead of time.

          10

          • #
            Jose_X

            Two quick clarifications:

            > it deduces greenhouse effect is not necessary for explaining temperature and [it deduces the greenhouse effect] is actually incorrect

            > but it is insufficient for the purposes of analyzing CO2 since it basically assume outright that no changes [related to CO2] would occur [in the equation].

            10

          • #
            Jose_X

            > The paper deduces the formula only for today and only by making measurements. We can’t obviously measurements the atmosphere of tomorrow today because it hasn’t yet changed today.

            The paper swept the greenhouse effect dependency under the rug in those equations.

            If we look at formula {15} on page 21, we will see one unknown value that must be measured, h_0.

            [T_0 can be picked arbitrarily because h_0 and T_0 are dependent variables.. never mind if this statement doesn’t make sense.]

            The problem with this derivation is that it assumes h_0 doesn’t depend on CO2. It can pretend that is true because, at the end of the day, to derive h_0, someone is going to go outside with a thermometer to measure some temperatures. The thermometer factors out the greenhouse effect. It doesn’t care if the temperature came from an asteroid collision or from a collapsing sun. It hides that information.

            When we measure today in order to derive this equation only as it stands today, we are able to ignore the causes that lead to those temperatures.

            The problem with this approach, of course, is that if the greenhouse effect does exist, we may have some ugly temperatures in the future. Those who assume there is no effect and don’t model it in the equations, will never figure out tomorrow’s temperature except by measuring it when “tomorrow” arrives (but it might be too late, as they say).

            Let’s see. How do we measure h_0 today if we know T today?

            Well, let’s say the T at ground level today (where h = 0) is about 288 K. This gives us:

            288 = -6.5 * (0 – h_0) + 255

            h_0 = 5.08 which is approximately 5

            The paper stated h_0 was about 5 km, just as we derived. [The paper should have been more forthright and made it clear how we derive h_0.]

            So, let’s review what we just did. We derived this equation that allegedly doesn’t depend on the greenhouse effect by taking measurements outside today. But if the greenhouse effect is real (let’s assume, for argument’s sake), then T is dependent on the greenhouse effect (and on CO2 specifically). When we calculate h_0 in order to get our final complete equation, we had to take a measurement today that depended on the greenhouse effect; thus, our final equation {16}: T = -6.5 (h – 5) – 18 is only accurate for the present moment when we could measure outside in order to derive these values.

            The real equation is

            T = -6.5 (h – GE(t)) – 18,

            where GE(t) is the greenhouse effect’s change over time t. It’s value is 5 today, GE(t=0) = 5, but it will likely be something different “tomorrow”. [Gosh, I wish we had modeled it a bit better.]

            Now, there is an important happenstance that makes it easy to sweep the greenhouse effect under the carpet for a number of years. What does the greenhouse effect modelling used by the IPCC predict 50? years from now at 2xCO2? It predicts a “modest” rise in temp of 3 C. That may or may not be too great news for the planet, but the fact is that its a very small number that fits within the error bars of this equation. Remember the discussion. The lapse rate could be 9.8 but it can also be much lower .. like 6.5 (who knows.. it’s a mystery how much it can vary, they say). There were also a number of assumptions about the air column. There were assumptions of how high to go and what the gravitational constant or the specific heat would be in those cases. Dry air that only rises adiabatically? What happened to significant convection and weather?

            My point is that it’s easy for people to fudge these numbers tomorrow to claim the formula still holds.

            Ex, if we change the 6.5 to 7.1 (hey, we measured it that way in 2060 .. remember, it could be “any” value somewhat less than 9.8) and the average global temperature at ground level does rise by 3 to 291, we get 291 = -7.1 * (0 – h_0) + 255, giving h_0 = … drum roll … 5.07!!!!!

            So, there is enough imprecision in how these numbers are derived so that we can continue to bury our heads in the sand if we really want to. Hey, that’s what we measured outside!!! There is no greenhouse effect because the thermometer and hygrometer and barometer didn’t say anything about no greenhouse effect.

            [Keith, I haven’t forgotten you. The executive summary to the re-explanation of the first executive summary is coming right up!!]

            10

          • #
            Jose_X

            Executive Summarizational Report No. 2

            The “there’s no stinking greenhouse effect” paper sweeps the greenhouse effect under the rug because thermometers are oblivious to the greenhouse effect and the lapse rate can be made up on the spot.

            10

  • #
    CHIP

    I agree, to some extent, with what Jose X says in comment 607. I have always thought that back-radiation from atmospheric greenhouse gases increasing the surface temperature was a given and didn’t really need to be debated. For proof one only has to compare the night time temperature in the tropics to the night time temperature in the Sahara Desert – the one major difference between these two environments is water vapour and without the vast amounts of water vapour in the tropics temperatures would plummet like in the Sahara Desert. Of course one could argue that the day time temperature in the Sahara is much more and they cancel each other out, although the surface temperature at night is still higher due to atmospheric greenhouses. Hence a colder atmosphere can increase the surface temperature, not by creating energy, but by simply inhibiting the flow of out-going radiation.

    In Postma’s paper, entitled ‘Understanding the Thermodynamic Atmosphere Effect’, he offers an analogy of an ice-cube to show how the concept of back-radiation from atmospheric greenhouse gases cannot increase the overall planetary temperature. What he fails to take into consideration though, is that the ice-cube does not have a constant, almost inexhaustible supply of energy. Obviously, the Earth does, because it is continuously receiving fresh power in the form of solar radiation from the Sun. Imagine for a second, hypothetically, if the ice-cube has a continuous supply of fresh energy. The consequences would be different. Increasing the temperature of the surroundings would slow the rate at which the ice-cube cooled. Analogously the Earth emits radiation which is re-emitted by atmospheric greenhouses, including CO2 (in the 15 micrometer range) which inhibits the flow of radiation out to space. Since all bodies above -273C emit radiation and they emit radiation multidirectionally (up and down equally) then it follows logically that some of the re-emitted radiation should be re-directed back to the Earth’s surface. The absorptivity/emissivity of the Earth’s surface is 1, and therefore the downward radiation should be absorbed too, and all radiation, to some degress, possesses some energy according to Lambert-Beer law. So in theory, as I see it, there can be no doubt that atmospheric greenhouse back-radiation increases the Earth’s surface temperature by a finite amount. Maybe not as much as CAGW-proponents say, but by some amount nevertheless.

    One moe thing: According to Wikipedia’s page on “black-body” radiation the net radiative-power of a body is the difference between the power emitted and the power absorbed. Hence: Pnet =Pemit – Pabsorb. If the temperature of a body is 288K (the Earth’s average surface temperature) and if the surroundings are 255K (the Earth’s average atmospheric temperature) then according to the Stefan-Boltzmann law, the net radiative heat-loss amounts to: σε(288^4 – 255^4) = 150.35W/sq.m. If the temperature of the surroundings were to increase from 255K to 260K then the net radiative heat-loss would change accordingly and decrease to: σε(288^4 – 260^4) = 131W/sq.m. We can see then, that increasing the temperature of the surroundings diminishes the body’s overall heat-loss. In other words, the Stefan-Boltzmann law explicitly states that a warmer body absorbs energy from its cooler surroundings. I hope I have interpreted this correctly.

    Perhaps a more appropriate analogy to the one offered by Postma would be that of a saucepan sitting on a heated-stove. The heated-stove represents the Sun and the water within the saucepan represents the Earth. What happens if someone decides to place a lid partly over the saucepan? The temperature of the water increases. Right? No additional energy is required – you have merely slowed the rate at which energy leaves the saucepan. When considering the 2nd law of thermodynamics, which states that heat spontaneously flows from warmer bodies to colder bodies, never the reverse, one should perhaps also take care in understanding the difference between “heat” and “radiation”. Technically a photon is not heat. It is just an undifferentiated blob of electromagnetic radiation. It only becomes heat when it is absorbed by a body where the energy is converted into kinetic energy – heat. The transfer of heat by convection and conduction is not the same as radiation. Convection and conduction requires the movement of particles, radiative heat-transfer does not. And the energy transfer by atmospheric greenhouses back to the Earth’s surface is mainly by means of radiation rather than conduction or convection, so there really is no paradox here, as far as I can see.

    As Joanne has pointed out too, the 2nd law applies to only the net flow of energy, not to individual photons.

    As far as I can tell, the 2nd law remains intact.

    10

  • #
    Jose_X

    AHA! How about *this* support for the greenhouse effect:

    Minutes ago, the President of the UN gave his backing to the idea that …..

    [I’m kidding.]

    Seriously, how about this analogy:

    Throw one 100kg body into deep outer space and what will happen to its temperature? It will cool rather quickly.

    Now, throw 100,000,000 similar 100kg bodies into outer space so that they remain near each other, and what will happen to their temperatures? Most will stay hot for a while while the outer edge of the pack will be the ones losing heat the fastest (although still slower than if those outside bodies where each by themselves).

    This is just what a ghg atmosphere is! It’s a bunch of radiating bodies together so that only the outer edge lose heat significantly. [although note that the outer edge of the exosphere has molecules that are not ghg and these molecules have the equivalent of very very high temperatures]

    On top of that, now spin the group around exposing them to the radiation of a “nearby” sun 24×7!

    Without ghg in the atmosphere, the earth is naked towards outer space. With ghg, it has a layer of many teeny radiating warm bodies protecting it from the direct super cold.

    10

    • #
      Truthseeker

      Jose, that is a poor analogy and shows that you have no understanding of the physics involved. No one is disputing that atmosphere has mass and therefore retains energy including heat, but that does not mean or support the idea that any particular gas is doing more than any other in any significant as far as overall heat is concerned. The fact that the measured ambient temperature at various pressures (more mass = more ability to retain heat) for two planets with completely different atmospheric compositions is in the precise ratio of the distance from the Sun (as far as energy received is concerned) shows that it is the fact that having an atmosphere allows heat to be retained, not the composition of it.

      10

      • #
        KinkyKeith

        Hi truthseeker

        I have given Jose a hard time, in the past, but I kind of liked his analogy – even though, in using radiation as the heat transfer mechanism it only resembles what’s happening in the very upper extremes of our atmosphere, and not where we live.

        Somebody put up a link to a really interesting machine translation of a Russian paper by Sokotin? which basically says at near 1 atm pressure the only things that count are heat capacity of the air (add up all the components)and variations in pressure. Which is what you said I think.

        It makes no difference if you double CO2.

        20

        • #
          Jose_X

          FWIW, I’ve been focusing on describing/debating the greenhouse effect and radiative transfers of heat.

          I agree that the situation as concerns water vapor is trickier. That doesn’t mean I lean towards a low climate sensitivity, think there is a net “negative feedback” for water vapor related processes beyond the albedo changes from some clouds, or have significant doubts against the Trenberth diagram values (eg, for thermals and evaporation), but I do agree there appears to be a significant amount of doubt in this area and I have not thought about it too much personally.

          10

      • #
        Jose_X

        >> poor analogy

        Might as well mention that the sun isn’t shinning at the edge of the bodies but nearer to the center. I’ll cover this later.

        >> does not mean or support the idea that any particular gas is doing more than any other in any significant as far as overall heat is concerned

        I had to run and didn’t keep working with the analogy, but one item that is clearly different and I didn’t clarify is that solid bodies radiate a much wider range of radiation than do gases.

        Evidence says you are wrong. N2 does not radiate like CO2 which itself is not like your hand or almost any other solid.

        What does your theory say about line spectra of atoms and molecules?

        This is old physics you are disputing not “climate science”.

        If you can’t provide an explanation to that or explain how most lasers work or ….

        Really, to say any gas radiates like any other is a slap in the face of physics, and you are going to have an awful lot of explaining to do of a lot of accumulated evidence and engineering.

        We can start with the line spectra: why do different gases block out different part of the “rainbow” that a prism produces?

        Simply, different gases absorb and emit different ranges of radiation from each other. CO2 covers parts of the IR range, eg, because these larger wavelength “fit” the molecule size to enable vibrations of various types, while N2 and O2 do not, covering instead the smaller wavelengths which generally correspond to about the size of a single atom or smaller and tend to involve electron excitation. So while CO2 has lots of radiation it can absorb (IR range) from what the planet radiates, N2 and O2 do not.

        Also, there are other ghg gases besides CO2, but CO2 is the one that humans are releasing in huge amounts beyond what the planet naturally had maintained for at least hundreds of thousands of years.. and we it is still rising quickly. If we were releasing methane in such large quantities, we’d be noticing the effects much more quickly.

        10

  • #
    Lars P.

    The main problem is in equalising “back-radiation” with solar radiation and comparing these two. This does not make sense.

    Is 1 Watt per square meter from greenhouse back-radiation equal with 1 Watt per square meter from solar insulation? Can these 2 be compared as such?

    Once upon a time there was a small rocky planet inhabited by a race called the avians.
    The avians reached a relative advanced civilisation, they learned radiation physics and understood their planet’s radiation budget and were happy to realise the greenhouse gas was warming the planet just at their best confort. Their wise scientists told them to take care and reduce their greenhouse gas production as this will overheat the planet in a runaway greenhouse process, but many citizens ignored them out of simple indolence, greed or ignorance.
    A group of concerned citizens, not happy how their co-planetary citizens were endangering all life on their planet out of sheer greed and stupidity decided to make a planetwide action to raise awareness of what greenhouse means.
    So in one evening using the coming dark they glued a foil over the whole surface of their rocky planet. (It is a very small planet indeed) The foil would not let the planet radiate away any heat. From above, the atmosphere was radiating a tremenduous 350 Watts per square meter downwards.

    Now the first question: how much did the ground heat under that 350 Watts/m2 downwards radiation bombardment in that night knowing that the ground was +15°C warm and the atmosphere -18°C? By how many degrees did the ground warm in that night?

    The night passed and the avians found to their surprise that there has been no warming at all. It even cooled a bit instead of warming with all those 350 Watts per square meter coming down from the atmosphere. But the night was soon over, the sun was going to shine and they were not able to remove the foil as it was glued with superglue.
    So what to do now?
    The concerned citizens gathered now with all the other citizens and they started to think what could they do? “How could you do this to us?” said the other citizens to the concerned citizens.
    We will all boil here when the sun shines if the plannet cannot cool? The concerned citizens were a litle uneasy as their experiment failed and the planet did not warm, but they said, “we are sorry, we really though it will warm and wanted to raise your awareness to it, but don’t worry, we thought at everything, in a couple of days the foil will get absorbed by the soil, it is a natural recyclig foil. Until then we have enough foil to put above everybodies head, one just needs to turn it the other way up and no heat will come down to you, so only wait until the foil decays naturaly and meanwhile we see where the problem is, either our data is wrong or the planet has a heat sink that we do not know about? “
    So everybody put the new foil above their heads and they measured the downwards radiation and saw satisfied that all the downwards radiation was blocked by it. Now at ease they expected the sun to rise trying to solve the problem where the heat leak of the planet was.
    Once the sun rose they saw that a litle bit of light came through the foil. The concern citizens measured with their devices and saw nothing, then finally one of them said – “maybe it is beyond the measurement error of our devices which is about 1%, there is maybe 1% of radiation that passes through”. Now they were eased. If only 1 % of radiation passed through the foil the planet will certainly not warm-up as in the night with 350 Watts radiation it did not warm. (1 % from 1350 W/m2 than must be about 13 Watt passing, so who cares about 13 Watt thought they). Maybe it will be really cold if the heat sink in the night absorbed more then 350 Watt said the concerned scientists so maybe they need to lift the upper foil from time to time to let some light shine on them.
    And yet the citizens started to feel a litle warmer under their foil. Was this only an illusion?
    Slowly it was warmer and warmer, and sweating they asked themselves what went wrong?
    Was their science wrong? How can it be that 13 Watt per square meter warms and 350 Watts per square meters does not? What is the difference between 13 Watt per square meter from the sun and 350 Watt per square meter from the atmosphere?
    Is this an ilusion? Or do they need to rethink their science? No there must be some other explanation, the heat sink by night becomes heat generator by day – said the concerned citizens.

    Can you answer why do the concerned citizens of that planet sweat under their tin foil? Is this story right or has its physics wrong?

    It is not my intention to say here “there is no greenhouse”.
    I want simply to put it there where it belongs: heat transfer through radiation. Heat transfer goes always from warmer to cooler object and not the other way around. If the cooler object gets warmer then less heat will be transferred from the warmer but it will never make the warmer object “sweat”.
    To take one member of Prevost heat transfer and compare it with sun radiation is avian science and it does not give proper results. Radiation has a temperature signal with it and whilst lower temperature radiation “equalises” or “neutralises” or “cancel” each other, lower temperature radiation will not “equalise” or “neutralise” higher temperature values.
    “Backradiation” from -75°C object will “equalise” or “backradiate” only the -75°C radiation of the object with higher temperature, whereas signals with higher values will go from the higher temperature object to the lower one.

    10

    • #
      Jose_X

      I want to talk about avians.

      Lars P., I am not sure what properties the aluminum foil has in your story or if you considered that the sun is always shining on half the planet. Did you also want to account for convection?

      Instead of covering those questions, I want to instead describe how MagicFoil ™ affects avians.

      MagicFoil comes in many varieties. Their premium product line, for example, has 16 different types, with at least one guaranteed to solve any of the Five Generally Accepted Common Types of Potential Planetary Catastrophes.

      Here are the 16 types of MagicFoil Premium ™ foil (side 1 / side 2):

      1) reflective in / reflective out (r/r)
      2) reflective in / transparent in (r/t)
      3) reflective in / absorbent-radiant (r/ar)
      4) reflective in / absorbent-non-radiant (r/anr)
      5) transparent out / reflective out (t/r)
      6) transparent out / transparent in (t/t)
      7) transparent out / absorbent-radiant (t/ar)
      8) transparent out / absorbent-non-radiant (t/anr)
      9) absorbent-radiant / reflective out (ar/r)
      10) absorbent-radiant / transparent in (ar/t)
      11) absorbent-radiant / absorbent-radiant (ar/ar)
      12) absorbent-radiant / absorbent-non-radiant (ar/anr)
      13) absorbent-non-radiant / reflective out (anr/r)
      14) absorbent-non-radiant / transparent in (anr/t)
      15) absorbent-non-radiant / absorbent-radiant (anr/ar)
      16) absorbent-non-radiant / absorbent-non-radiant (anr/anr)

      We have 4 types for a side and 2 sides, giving the above 4×4=16 possibilities.

      Overview: Reflective means that Most’all ™ radiation coming from that side is reflected back. Transparent means that Most’all radiation coming from that side passes straight through. Absorbent-radiant means the radiation hitting from that side is absorbed and then 50% radiates back isotropically and 50% radiates isotropically to the opposite side. Absorbent-non-radiant also absorbs Most’all radiation hitting from that side, but the foil simply rises in temperature as much as necessary radiating Hardly Anything At All ™, instead losing the heat slowly via conduction or convection as applicable.

      Assembly instructions: The foil will be nearly instantaneously wrapped around the planet’s suspiciously flat surface, suspended above numerous strategically placed well-insulated poles each about 3 meters high so that avians can trot along fairly comfortably. Side 1 is the side facing the ground. Side 2 is the side facing up and out towards space.

      Special powers: I will now invoke a special privilege I have as analogy creator and turn off the sun. Let’s see what happens to the temperature of the planet’s surface and atmosphere.

      [As a further simplification, I’ll also consider the planet holds more energy than the atmosphere but not necessarily too much more (so unlike Earth since oceans hold much more than atmosphere).]

      P is for planet. A is for atmosphere (lower, middle, and upper). s1 is for side 1. s2 is for side 2.

      A has CO2 and other ghg that absorb in the IR region as provided by P.

      Experiment #1: Off Sun

      1 (r/r): P keeps its temp, losing nothing (s1) and gaining nothing (s2). A cools slowly with lower A staying warmest thanks to side 2 (note that upper, middle A still radiate downward partly through lower A and back up again allowing for extra opportunities to absorb).

      2 (r/t): P keeps energy it had (thanks to side 1) and gains lots extra energy via lower A (thanks to side 2); thus, P rises in temp and holds that level. A cools fast with lower and upper A coldest. MagicFoil #2 essentially acts as if A is surrounded above and below by cold space.

      3 (r/ar): P keeps energy it had (s1) and gains some extra (s2), so temp rises (but less than in r/t case) and holds. A cools at medium speed with lower possibly being colder than middle but not as cold as upper.

      4 (r/anr): P keeps its temp. A cools fast as if it had space above and below. Foil gains in temperature to match energy lost by lower atmosphere in downward direction.

      5 (t/r): P will lose its energy (s1) as if next to cold space (s2). In other words, P no longer feels any back radiation from either itself or from A. Lower A will be warmest and will initially gain in temp as P starts cooling fast. High heat flow from P means A will cool more slowly than the corresponding r/xxx cases above.

      6 (t/t): P can lose energy (s1) but gains “back radiation” from atmosphere (s2) so rate is moderate. Lower A is warmest part of A. This scenario is similar to there being no MagicFoil.

      7 (t/ar): Similar to t/t, but P loses a little faster and A a little slower.

      8 (t/anr): P will lose energy fast like t/r. MagicFoil heats up with back radiation from A. A gets some original energy from P but none from its downward radiation so cools medium speed.

      9 (ar/r): P cools medium speed as it gets some back radiation from self. A cools slowly but not as slowly as with t/r. Lower A is warmest part of A.

      10 (ar/t): P cools slowly as it gains some back radiation from self and back radiation from A. A cools fairly slowly with lower as warmest. Overall cooling is slower than t/t.

      11 (ar/ar): P cools medium to slow. This is almost like ar/t.

      12 (ar/anr): P cools medium speed, getting no back radiation from A. MagicFoil gets hot. A cools medium speed, getting no back radiation from self.

      13 (anr/r): P cools fast (no back radiation). MagicFoil gets rather hot. A cools medium to slow with lower A being warmest part.

      14 (anr/t): P cools medium and MagicFoil gets rather hot. A cools fast from both ends.

      15 (anr/ar): P cools medium to fast (some back radiation from A). MagicFoil hot. A cools medium to fast (some back radiation from self).

      16 (anr/anr): P and A cool fast (no back radiation from either to either). MagicFoil gets very hot, ultimately absorbing all energy from P and some from A.

      To summarize the 16 cases:

      Reflective allows that side to get its own back radiation while the other side won’t gain any of such “back radiation” being reflected on the facing side. r/xxx means (a) P will keep all its energy and keep Avians warm except as MagicFoil degrades over the years and (b) A will cool faster than if MagicFoil weren’t there (and we had simply turned off the sun) since A loses the radiation it would have gotten from P as P otherwise would have cooled through A. xxx/r means lower A will stay warmest, middle A next warmest, and upper A coolest as A loses its energy towards space.

      Transparent allows that side to lose its energy gaining no back radiation from itself. All lost is gained by the other side. t/xxx allows lower A to likely increase in temp (thanks to faster cooling P) before eventually cooling.

      Absorbent-radiative allows that side to keep some energy and lose some to the other side. This 50/50 result lies between transparent 0/100 and reflective 100/0. For A, we would likely see lower A warmer than middle A and followed by upper A as the coolest.

      Absorbent-non-radiative allows that side to lose energy without gaining back radiation from self. None also goes to the other side. MagicFoil gets hot.

      We will note that radiation photons in some sense allows energy to be accounted for in discrete amounts; however, since everything mixes (and photons get annihilated) you can’t really say some “back radiation” comes from self vs. from the other side or differentiate cleanly between “forward radiation” and “back radiation”. However, in some cases you can clearly know that radiation is from self or from the other side of MagicFoil. Conceptually, despite losing their identities, the energy represented by a photon at some point in time can be tracked.

      Special cases.
      a) t/t and t/ar approximate normal conditions as if there were no MagicFoil. A few other cases also come close (eg, ar/ar). Keep in mind in this experiment there was no sun, so “normal” just means what would happen if we could shut off the sun.
      b) r/xxx will have P retain its temp for a long time and perhaps even get a little warmer. If we turn on the sun for this hypothetical case, the temperature of the planet keeps going up as the planet captures and retains a piece of the continual stream of energy given off by the sun.
      c) xxx/xxx: In every case the atmosphere cools to the temp of space.
      d) r/t, r/anr, anr/t, anr/anr are the cases where the atmosphere cools as if it was in contact with space from above and from below. These are the cases where the temp of the middle of the atmosphere stays above the temp of lower and upper. NOTE that the very cold atmosphere would still heat up the very hot planet as long as it kept radiating! This happens because photons can go to the planet but none escape back out through the MagicFoil. It’s a sort of ideal black hole effect.
      e) anr/anr has the greatest cooling effect of all on the combination of P + A because it draws the most heat from that system. This case ties for worst in both rankings below.

      P warmness is promoted by (in order):
      1) r/t
      2) r/ar
      3) r/r, r/anr
      5) ar/t
      6) ar/ar
      7) ar/r, ar/anr
      9) t/t
      10) t/ar
      11) anr/t
      12) anr/ar
      13) t/r, t/anr, anr/r, anr/anr

      A warmness is promoted by (in approximate order):
      1) t/r, t/ar, ar/r, t/t, ar/ar, ar/t (these each have varying effects hard to rank separately)
      7) t/anr, ar/anr
      9) anr/r, r/r
      11) anr/ar, r/ar
      13) r/t, r/anr, anr/t, anr/anr

      Studying all the scenarios together helps in appreciating their effect on temperature and that energy is accounted for when we look at radiation as composed of photons.

      Experiment #2: On Sun

      Now, let’s discuss quickly what would happen if we turn on the sun.

      In the r/xxx cases, the planet will heat up and melt as it keeps acquiring more and more energy from the sun without releasing any back.

      In the r/t, r/anr, anr/t, and anr/anr cases, the atmosphere cools “completely” if we assume this atmosphere can’t absorb sunlight directly but requires the help of the planet to convert into the IR range. The planet covered in these premium MagicFoil types doesn’t help the atmosphere gain heat in these cases, so the atmosphere just loses all of its heat and gains none further despite the steady On Sun.

      In every other case, something in between happens, where the planet and the atmosphere reach some steady state temperature or other that is neither 0K nor too hot. We simply get different degrees of greenhouse effect depending on which of the remaining MagicFoil Premium foil type is used.

      [Disclaimer: Planet internal conductive heat loss is not really considered beyond the surface layer radiation (particularly the mixed ocean layer). There are other approximations made, I’m certain of it. Do not try this at home.]

      10

      • #
        Gee Aye

        Amazing that this thread is still active and all because there are people who think that an emitted photon has some idea about the location of other photon emitters. These photons, instead of heading off in the direction of emission will flee in some other direction if they are heading towards an object that is emitting more photons than the one from which they were emitted.

        They say, “Noooooo I wont add more energy to that other object! It has more than enough already and the poor object I just left did not have so much and since I just left it, it has even less”.

        Then there are those people who don’t understand that molecules floating about in the air have different properties with respect to how they interact with photons. They don’t even realise that these interactions have been intensively studies and verified in different contexts for many years, but that is another story.

        10

  • #

    The Second Law of Thermodynamics has to be valid between any two points at any time.

    The IPCC models assume processes happen 24 hours a day, so they assume that, when the surface is getting hotter on a sunny morning, that radiation from a cooler atmosphere can make it warm even faster – yes, even when net radiation is into the surface. That can only be done by adding thermal energy. So thermal energy has been transferred from a cooler source to a warmer target. That is heat transfers from cool to warm. That violates the Second Law.

    You cannot increase the rate of cooling, such as when evaporation and diffusion transfer thermal energy from the surface to the atmosphere, by sending in some radiation from the atmosphere, unless the energy in that radiation is first converted to thermal energy. But it never can be without violating the Second Law. How could radiation itself directly affect evaporation? It can’t. Only thermal energy could.

    Absorptivity measurements are usually done with much higher frequency sources in the visible spectrum. I can find none done with low frequencies such as in the radiation from the atmosphere. The reason is that absorptivity must hit zero (0) when the source temperature becomes lower than the temperature of the target. This is the only way the Second Law can work in all situations, including when there are concentrators of radiation (like a funnel) or special one-way filters.

    For more detail see the Radiation page of my site . . .
    http://climate-change-theory.com/RadiationAbsorption.html

    10

    • #
      Jose_X

      Doug, those ideas disagree with a lot of established physics.

      Let me ask, how do you define temperature?

      Do you think all molecules in a body move around with exactly the same energy?

      If yes, you contradict a lot of established science. I think we can show some paradoxes or at least violations of conservation of energy and momentum.

      If no (eg, if you believe in Maxwell Boltzmann distribution is a real description of parts of reality), then you have to agree that some CO2 molecules will be going slow enough below the average for the cutoff to be met so that they absorb radiation emitted from the planet. A similar argument means the planet will have molecules that can absorb what the atmosphere emits.

      Maybe start by answering some of the above, so that I know where to go next. BTW, I am reading some of that website link you provided, so you don’t have to repeat something if it is there (just point it out).

      10

    • #
      Jose_X

      Doug, I want to point something else out. A number of “climate skeptics” have taken to refashioning accepted physics. Their goals might be to attack global warming projections, but they are debating in the wrong forum. Climatologists are not experts in radiation physics or thermodynamics, generally. If you can’t convince the experts in those areas nor publish in those journals, why would you be surprised if a climate science forum dismisses your views as fake/false science? Interdisciplinary individuals will take from one field and migrate to others, but most in either field leverage what already exists. Mother nature weens out ideas that don’t hold up. Most scientists are comfortable using work whose details they don’t understand if it is established science. If one can’t win the argument with the experts, then others generally will not listen.

      So, keep that in mind. You really are not attacking climate science. You are attacking core basic physics and you should be debating on specialized forums (perhaps journals) if you really want to seek the truth and have your ideas be tested up and down to gain traction among large numbers of scientists and related engineers.

      Trying to overturn basic stuff is very very very likely to fail. Lots of engineers and scientists have been able to make lots of discoveries and products by leveraging long held physics, so they will keep using accepted ideas unless you make an overwhelming case.

      10

    • #
      BobC

      Doug Cotton
      February 24, 2012 at 11:50 am ·

      The Second Law of Thermodynamics has to be valid between any two points at any time.

      Laws of physics are determined by empirical data, just like all valid theories. They do not determine what is real, they are just our best guess as to what is real.

      That said, I do agree that the atmosphere most likely follows the Second Law.

      …This is the only way the Second Law can work in all situations, including when there are concentrators of radiation (like a funnel) or special one-way filters.

      The existance of the kinds of concentrators and one-way filters that you envision would indeed violate the Second Law. So far, however, no one has successfully constructed any. James Maxwell also considered these kinds of filters and believed that they would violate the laws of thermodynamics. What he concluded, however, is that they can’t be built. In this, you are in conflict with Maxwell — not a place likely to be supported by empirical data (although, many things are possible). (See Maxwell’s Demon)

      10

  • #
    blouis79

    For anyone who thinks the “greenhouse effect” is consistent with conventional physics, please read Gerlich and Tscheuschner.

    For anyone who wants to understand how scientists can make historical mistakes, please read Tyndall’s original works and find the part where he measured warming of IR “absorbing” gases (hint: not).

    For anyone who wants to understand how heat is really moved about the atmosphere, read Claes Johnson.

    For anyone who really thinks atmospheric CO2 absorbs significant heat energy raising atmospheric temperature, please show the results of a proper physics experiment demonstrating same.

    10

    • #
      Jose_X

      G&T has been criticized (scienceofdoom did an informal one and there was mention of formal papers in the pipeline. I haven’t checked). I glanced at parts of it and was ..umm.. not too impressed. I can’t say much more now I suppose since I haven’t really read it.

      You mention scientists making mistakes. Were you referring to G&T?

      You mention C. Johnson. I heard about him today after reading Doug’s comments and some of the webpage he referenced, and I have been skimming some of the the papers/books he wrote. From what I have read so far, I don’t think he tries to hide that he isn’t presenting something that is mainstream. Given he is advocating material that is not mainstream, what makes you think this guy knows how heat moves through the atmosphere while most other physicists don’t? New theories come, and we are supposed to abandon what exists because some guy says so and another guy in an online forum agrees?

      How do you pretend we test CO2 concentrations (with a control) in the full atmosphere? We only have one planet. People have already done laboratory work that supports the greenhouse effect.

      I am going to keep reading Johnson a bit because I am curious, if rather skeptical. [The online book has over 200 pages, so it’s certainly not an afternoon’s read, but maybe I can capture important points before I move on to something else.] BTW, in the “blackbodyslayer” paper (see link at website Doug listed), section 2.1 is called “Enigma” but none of the four questions are enigmas. Well and long established models of atoms answer these rather cleanly. Of course, he rejects statistical mechanics (probability interpretation of entropy/direction of time) and perhaps doesn’t like probability analysis too much, even though statistics does seem to me like a rather clean interpretation. To substitute it, he mentions finite computations (I think this means approximate computer solutions) as the way to address the ultraviolate catastrophe and perhaps other ugly situations from deficient models. I don’t know the details yet of what he suggests, but alarm bells have been ringing in my head for a while now. He also mentions judging problem solutions a posteriori rather than making predictions and seeing if they come true. More alarm bells are going off, as I fear this may mean that you try to fit your conclusions to the approximate computer math after the fact and deny the physics model if you can’t. So if you don’t like the prediction, you a posteriori say it is a nonsense prediction by picking the data you want and manipulate it how you want. Anyway, I am guessing as to what he might mean since I haven’t read the details yet. I *am* curious, but I would be lying if I told you my medula skepticum organ has not been strained.

      10

  • #
    Bryan

    Jose_X says

    “G&T has been criticized (scienceofdoom did an informal one and there was mention of formal papers in the pipeline. I haven’t checked). I glanced at parts of it and was ..umm.. not too impressed. I can’t say much more now I suppose since I haven’t really read it.”

    Jose your passage above shows an evasive approach.

    You are frightened to engage with a peer reviewed paper from a physics journal yet want to imply you have;

    ” I glanced at parts of it and was ..umm.. not too impressed”

    Well here’s another peer reviewed paper that backs up G&T

    http://www.scirp.org/journal/PaperInformation.aspx?paperID=9233

    So if you want to be part of a real traditional physics discussion, read the papers and then come back and tell us the bits you agree/disagree with.

    10

    • #
      Mark

      Bryan, you should know that trolls like Jose_X are only good for deflection and rejection of any evidence (even when it has been peer-reviewed) which doesn’t fit their politics.

      If they are asked what it would take for them to change their minds all they do is obfuscate and state something like “we need to wait”…..and in the meantime, keep wasting billions on worthless “green” rorts.

      Jose likes tongive the impression of being open-minded but always comes down on the “warmist” side. I don’t think he’s ever stated his qualifications amidst his discursive dissembling but I could be wrong there.

      Whatever, don’t encourage or engage as he is a recidivist thread-bomber.

      10

      • #
        Jose_X

        blah blah. I’m glad you know so much about me.

        And feel free to list out the evidence that the greenhouse effect is not real.

        .. let me add something here since I forgot to add it in the reply to Bryan.

        @Bryan, I mentioned I was not too impressed. That doesn’t mean I think the authors are clueless. It means the title of the paper suggests the greenhouse effect is shown to be fictitious, yet I got the impression the arguments weren’t very good.

        Let me see… here is the link to a scienceofdoom quick critique: http://scienceofdoom.com/2010/04/05/on-the-miseducation-of-the-uninformed-by-gerlich-and-scheuschner-2009/

        I’m hoping you will read that article, as it is much shorter than the G&T paper and critiques it (meaning I would likely bring up some of those points anyway and you can get a head start on that while I go try to read some of G&T).

        10

    • #
      Jose_X

      >> your passage above shows an evasive approach.

      So you are saying you want me to criticize it? praise it? …without having read it?!

      I skimmed some parts some weeks back and noted a few items I thought were incorrect, but I didn’t take written notes and intended to sit down with it.

      >> You are frightened to engage with a peer reviewed paper from a physics journal yet want to imply you have

      ??

      >> So if you want to be part of a real traditional physics discussion, read the papers and then come back and tell us the bits you agree/disagree with.

      If I sent you off now to read 100 pages of something, you’ll just go and do it and come back..

      I want to at least start reading that paper in earnest. I will consider spending some time on it today, perhaps putting aside Johnson’s paper and other things I might have looked at.

      I hope you have read it since you are asking me to read it and come back here and share my thoughts.

      10

    • #
      Jose_X

      Oh, I refreshed my mind a bit.

      I didn’t bother to read it because I thought I got enough of the paper from the scienceofdoom discussion.

      For starters, what an actual greenhouse has in common or not with the atmosphere may be an interesting point but has nothing to do with the atmosphere “greenhouse effect”. Spending so much time on that point hints that maybe the authors don’t really know what the atmosphere greenhouse effect is or aren’t too interested in getting down to the point, perhaps because they really don’t have a convincing argument.

      Second, the scienceofdoom article also stated that G&T spend a lot of time attacking simplified toy models of the greenhouse effect instead of attacking what is actually used in the computer models. It mentions that the G&T paper, if it was a serious paper, would have gone after Ramanathan and Coakley (1978) instead of after toy models.

      SoD states “It is hard to know where to start with this paper because there is no logical flow.”

      The picture in my mind was that G&T not only weren’t addressing the actual greenhouse effect physics model but were dancing around a lot hoping to distract.

      So, this is why I took a pass on this paper.

      I’ll try to read some of it today though. Hopefully, you will try to read the SoD review and consider defending G&T from it.

      10

      • #

        I’ll try to read some of it today though.

        That’s good.

        but were dancing around a lot hoping to distract.

        On second thoughts, don’t bother reading the paper. You’ve already attributed motives to the authors.

        10

    • #
      Jose_X

      2: Page 6 “However, carbon dioxide is a rare trace gas, a very small
      part of the atmosphere found in concentrations as low as 0, 03 Vol %”

      This is probably correct, but this is the first page of actual text (after the table of contents) and they are already pointing out CO2 is in tiny amounts. I have seen a number of skeptics who try to sell the idea that a trace amount of CO2 cannot possibly lead to greenhouse effect.

      So my alarm has been triggered. Are the authors going to try to use this irrelevant point? If they do, they might not be appealing to science but to emotion. Trace amounts of some chemicals can kill humans.. or save lives. Anyway, I smell weak arm waving arguments might be in the works. .. This is a minor point, of course. Nothing technically wrong at this point.

      3: Page 11 “that the relevant mechanism is the atmospheric greenhouse effect, a mechanism heavily relying on the assumption that radiative heat transfer clearly dominates over the other forms of heat transfer such as thermal conductivity, convection, condensation et cetera [23–30].”

      That is probably false, depending on what the authors mean by “dominates”.

      The point of radiative absorption isn’t that this “dominates”, but that it exists and captures radiation that otherwise would go straight into outer space.

      Local thermodynamic equilibrium (LTE) interactions and convection likely dominate heat transfers in the atmosphere, depending on how you define “dominates”.

      At this point, it appears the authors might be confusing the reader, but since they aren’t using numbers or more descriptive phrases, we can’t tell. Minor point.

      4: Page 12 “In particular, from the viewpoint of theoretical physics the radiative approach, which uses physical laws such as Planck’s law and Stefan-Boltzmann’s law that only have a limited range of validity that definitely does not cover the atmospheric problem, must be highly questioned [31–35].”

      First, let me point out that, despite what Doug said, spectroscopy measurements have been taken in the infrared part of the spectrum. CO2 and many molecules absorb in this range as it overlaps with molecular vibrational states. There are huge “tables” of these values used by scientists and engineers.

      Note the wide range of lines observed

      [ http://en.wikipedia.org/wiki/Spectral_line ]> Mechanisms other than atom-photon interaction can produce spectral lines. Depending on the exact physical interaction (with molecules, single particles, etc.) the frequency of the involved photons will vary widely, and lines can be observed across the electromagnetic spectrum, from radio waves to gamma rays.

      Second, Planck’s law and Stefan-Boltzmann are not used to define how the gases absorb or radiate. These equations are used for the planet.

      The gases effect, seen as absorption and emission lines (referenced a lot when discussing astronomy, chemistry, etc), fall under quantum electrodynamics theory.

      [wikipedia]> Quantum electrodynamics (QED) ..describes how light and matter interact and is the first theory where full agreement between quantum mechanics and special relativity is achieved. QED mathematically describes all phenomena involving electrically charged particles interacting by means of exchange of photons… Richard Feynman, has called it “the jewel of physics” for its extremely accurate predictions.

      The absorption science and gas-photon interactions have been well studied. The authors might not be too aware of the relevant results. In fact:

      5: The authors continue: “It cannot be overemphasized that a microscopic theory providing the base for a derivation of macroscopic quantities like thermal or electrical transport coefficients must be a highly involved many-body theory. Of course, heat transfer is due to interatomic electromagnetic interactions mediated by the electromagnetic field. But it is misleading to visualize a photon as a simple particle or wave packet travelling from one atom to another for example. Things are pretty much more complex and cannot be understood even in a (one-)particle-wave duality or Feynman graph picture.”

      Then add “On the other hand, the macroscopic thermodynamical quantities contain a lot of information and can be measured directly and accurately in the physics lab.”

      The authors appear not to be aware of the many observations and calculations performed on atom and molecular interactions probably every day all over the world in many laboratories in order to derive important information about the substances. There are extremely accurate measurements.

      I’ll quote Feyman again, quantum electrodynamics is “the jewel of physics.”

      6: Page 12 “About 80 percent of this warming is attributed to water vapor and 20 percent to the 0.03 volume percent CO2 . If such an extreme effect existed [33 K greenhouse effect], it would show up even in a laboratory experiment involving concentrated CO2 as a thermal conductivity anomaly. It would manifest itself as a new kind of ‘superinsulation’ violating the conventional heat conduction equation. However, for CO2 such anomalous heat transport properties never have been observed.”

      The authors appear not to understand the key components of the greenhouse effect and are making statements that are likely incorrect (about the requirement to see a thermal conductivity anomaly). They are barking up the wrong tree (I think.. I’ll have a better idea later) with their overemphasis on thermal conductivity.

      They add, “Therefore, in this paper, the popular greenhouse ideas entertained by the global climatology community are reconsidered within the limits of theoretical and experimental physics.”

      I’ll take this moment to mention that emissivity values have been measured for CO2, H2O, and many other gases. These values take the spectral line evidence a step further. I believe the measurements largely agree with the Beer Lambert law.

      Hottel, Leckner, and others are well cited in the literature. This is work largely done in the 60s 70s period, I think. [I think I provided links earlier.. I’ll check] Here is one link to a 1960 thesis from caltech http://thesis.library.caltech.edu/2809/1/Lapp_m_1960.pdf . Just read the beginning to get an idea. Measurements at low temperatures are common (and not what the thesis is about). Hottel is mentioned. Lot’s of engineers use these values.

      I think it is impossible to look at greenhouse effect without looking at this sort of result, so I’ll be watching to see if G&T recognize this since they are giving the impression they think Stefan-Boltzmann is the key equation and that the scales are not realistic for a “cavity” electromagnetic radiation analysis, and that there are might be no further experimental evidence to support the greenhouse effect.

      OK, well this is enough for now. I wanted to post something today. The authors haven’t “proved” anything by this point in the paper (pg 13), but I see many trouble signs. Many of the above points suggest they don’t understand the greenhouse effect; however, the silliness of section 5.1 by itself has to really make one wonder how much experience they have with climate science. They likely have not read much of the literature or taken very many courses is the impression they give.

      10

    • #
      Jose_X

      I want to mention “quickly” a few items that have caught my attention.

      1: This first one was one of the things I had noticed before.

      In section 3.5.1 the authors appear to have confused “short wave” and “long wave” in climate science with the use of those names in the radio range of the spectrum.

      Note the difference for shortwave:
      Short wave in the radio spectrum is in the range of 10,000,000 Hz.
      wiki/Shortwave_radio

      Short wave in climate science is around visible, uv, etc, and is on the scale of 100,000,000,000,000 Hz.
      wiki/Visible_light

      Long wave radio might be 10,000 Hz.
      wiki/Long_wave

      Long wave climate might be 10,000,000,000,000 Hz.
      wiki/Infra-red
      wiki/Pyrgeometer

      Notice I am not making this up. http://en.wikipedia.org/wiki/Climate_model
      > All climate models take account of incoming energy from the sun as short wave electromagnetic radiation, chiefly visible and short-wave (near) infrared, as well as outgoing energy as long wave (far) infrared electromagnetic radiation from the earth.

      Besides that these use the same name, the confusion might also come because when we convert the climate frequencies (Hertz) above into wavelength (micrometers) we get values that look like the radio short wave. Compare 1 MHz (radio frequency) to 1 micro-m (near-infrared wavelength). or 10^6 vs 10^-6. They look similar. However, we are not taking the reciprocal only. We also have to multiply (or divide) by the speed of light and that is how 10^14 becomes around 10^-6: you get the reciprocal 10^-14 and then multiply by the speed of light, 3*10^8 m/s, to give order 10^-6.

      OK, this is an easy mistake to make, I agree, but this is one major clue that the authors don’t totally have the ideas behind greenhouse effect under control. They don’t appear to be climate scientists or to have read very much in the field.

      10

      • #
        Bryan

        Jose_X

        “In section 3.5.1 the authors appear to have confused “short wave” and “long wave” in climate science with the use of those names in the radio range of the spectrum.”

        Now who is being silly?

        G&T were writing for professional physicists, so they would use the physics definitions of infra red, microwaves and so on.

        Why climate science has adopted its own definitions of long wave and infra red and so on (that no other science use) is quite odd don’t you think?

        Perhaps its because climate science has developed cut of from a substantial input from the other sciences

        10

        • #
          Jose_X

          Hey, all right!

          After reading your comment, I went googling for that picture online since it’s so difficult to read.

          I came across a presentation of that movie http://www.youtube.com/watch?v=C_mr9EqJg18 . In that presentation, the speaker refers to “reflection” to the laymen audience.

          Now, I don’t know what the context is in the movie, but if this speaker is reproducing that context a bit, I can see how G&T were mocking that use of “reflection”. I reread 3.5.1, and this interpretation would be consistent.

          So it appears G&T were in fact referring to real short/long wave!

          I did expect that “mistake” would have been just too silly.

          OK, so point 1 does *not* apply. Scratch it.

          Also, I put 5.1 instead of 3.5.1 in the concluding section (a typo). I see that threw you off (in comment below). I was referring to that 3.5.1 and not to the summary section.

          10

  • #
    blouis79

    Bryan, thanks for the Kramm et al paper.

    I note they have some inside knowledge of the Halpern et al rebuttal of G&T.

    It should be noticed that—based on the reviews re-
    quested by the IJMPB—the manuscript of Halpern et al.
    first submitted in 2009 was rejected. Surprisingly and
    unfortunately, it was eventually published by this journal,
    but none of the authors’ big physical mistakes criticized
    by the reviewers were removed from the manuscript.

    Interesting.

    10

  • #
    blouis79

    Kramm and Dlugi do indeed critique the Ramanthan description, before concluding

    “Based on our findings, we conclude that 1) the so- called atmospheric greenhouse effect cannot be proved by the statistical description of fortuitous weather events that took place in past climate periods, 2) the description by AMS and WMO has to be discarded because of physical reasons, 3) energy-flux budgets for the Earth- atmosphere system do not provide tangible evidence that the atmospheric greenhouse effect does exist. Because of this lack of tangible evidence it is time to acknowledge that the atmospheric greenhouse effect and especially its climatic impact are based on meritless conjectures.”

    10

  • #
    Bryan

    Jose_X says

    “however, the silliness of section 5.1 by itself has to really make one wonder how much experience they have with climate science. They likely have not read much of the literature or taken very many courses is the impression they give.”

    Section 5 is the summary section.

    To say something is “silly” you should have established which earlier part of the paper is in error!

    I don’t want to be too hard on you Jose as many attempts by professional IPCC advocates have been tried to pick holes in the G&T paper without success.

    You have skated round the G&T paper without saying anything of substance.

    Pick any one item and let’s see if you can show its “silly”

    Remember G&T do not consider a lot of what goes on in climate science as Science or Physics hence the title of their paper.

    Mann’s Hockey Stick graph and the 33K greenhouse effect are but too widely discredited items.

    10

  • #
    Paul

    Bryan @ #615.5.1
    February 26, 2012 at 10:17 am ·

    Jose_X

    “In section 3.5.1 the authors appear to have confused “short wave” and “long wave” in climate science with the use of those names in the radio range of the spectrum.”

    Now who is being silly?

    G&T were writing for professional physicists, so they would use the physics definitions of infra red, microwaves and so on.

    Why climate science has adopted its own definitions of long wave and infra red and so on (that no other science use) is quite odd don’t you think?

    Perhaps its because climate science has developed cut of from a substantial input from the other sciences

    Jose_X is all blather and smoke and mirrors. He pretends to be seeking after the truth but constantly takes the side of the advocates of the AGW conjecture.

    I personally do not regard it as useful to reply to him and agree with another poster that he simply spams this thread, preventing any serious discussion of the science or for anyone new to the thread to learn anything new from it.

    His appeal to advocacy sites, such as the Science of Doom and RealClimate, indicates that he has no independent knowledge of the science and buys into the propaganda as his starting point for ‘research’.

    Paul

    10

  • #
    blouis79

    Won’t get any sense out of scienceofdoom, who is wedded to the “greenhouse theory”.

    10

  • #
    BenAW

    Would you all have a look at this post:
    http://tallbloke.wordpress.com/2012/02/23/ben-wouters-how-the-earths-surface-maintains-its-temperature/

    I’m looking for a serious discussion.
    If my hypothesis holds, AGW is dead.

    10

    • #
      KinkyKeith

      Hi Ben – what about energy lost as work on the atmosphere?

      10

    • #
      Jose_X

      @BenAW, I am still reading it.

      First point, AGW won’t be dead until a mathematical model that does at least as well a job supplants it. Without math, we could be designing/modeling a fantasy land of dragons and magic and we would not tell otherwise. So keep that in mind if you are serious.

      Second, I have never put a glass of warm water on the counter and felt that the top got hot and the bottom cold. If there is a difference it was very slight. On the other hand, I have added warm water with colder water and they mix together by themselves. This mixing effect is natural and can be seen when two different colored liquids are put into the same container (or a boundary between them is removed). While I understand that the idea that hot rises and cold falls seems like something we have always heard, this only happens a little bit to match certain physical constraints (that is my guess). My guess is that there is a natural gradient that develops based on, eg, pressure and the boundary conditions (eg, bottom of ocean land.. atmosphere.. space). [I have ideas about the physics/math but really am not sure, so I am guessing.] Eg, like if you put one end of a metal rod in ice and the other in fire, every point in between will adjust part of the way towards each end depending on how close it is to that end. Anyway, any variation from this natural gradient will lead to a reshuffling of gas/liquid. This not only seems to follow evidence, it not only makes sense with current physics (eg, brownian motion mixing), but it explains why a lot of sun heating the top of the oceans would eventually lead to some degree of mixing (some warm falls while some cold rises to replace it and approach equilibrium gradient).

      10

      • #
        Jose_X

        >> it explains why a lot of sun heating the top of the oceans would eventually lead to some degree of mixing (some warm falls while some cold rises to replace it and approach equilibrium gradient).

        If you are serious about the science of this, remember that people are measuring the ocean temperatures in many locations. http://en.wikipedia.org/wiki/Argo_%28oceanography%29 . They see cold rise and warm fall in various places at times. They see some shifts in temp throughout at times. If you want an accurate model, you should consider getting that data and studying it as you build your model (and make sure you sharpen some relevant math if you want to improve your odds).

        10

    • #
      Jose_X

      >> http://tallbloke.wordpress.com/2012/02/23/ben-wouters-how-the-earths-surface-maintains-its-temperature/
      > Reason being that incoming solar and cooling through the atmosphere cancel each other out. So we have radiative balance with the sun, and a temperature at the surface that is at least 275K in the polar regions, and higher at the equator (~300K)

      My question is, why would the balance be some value over another? Why not 1000K or 400K or 280K or 257K?

      What if there already is mathematics with accepted physical models that recognizes (theoretically and experimentally) “downward longwave radiation” to arrive at approximately the temperatures we measure?

      I haven’t done the math, but I believe that fairly good model already exists using DLR.

      So if I am a neutral third party, which model am I likely to adopt or place my trust in? The one with the fancy math, decent consistency, and fairly good answers, or the one with a lot of wand waving?

      I am trying to convey why pretending climate scientists are wrong without understanding their equations and math is an uphill battle and probably foolish (unless you have reasons that make it worthwhile — eg, a learning experience of some sort). It will take you a long time and likely you won’t be successful in reaching a competitive model without a lot of help. And what if when you get there (assuming you do) you then find out your model is about the same model as what already exists?

      So, I am not trying to discourage you from learning and being creative and enjoying yourself or whatever, but keep that in mind.

      If you want a link to a paper that I think is supposed to solve some of these issues, try http://climateknowledge.org/figures/Rood_Climate_Change_AOSS480_Documents/Ramanathan_Coakley_Radiative_Convection_RevGeophys_%201978.pdf . If you can show why that paper is bogus, you will really help your cause. If you can’t show it, you might want to consider learning from it in some way.

      BTW, I haven’t read the paper. I hope I find time enough to read it and understand it.

      10

      • #
        BenAW

        “My question is, why would the balance be some value over another? Why not 1000K or 400K or 280K or 257K?”

        Unless you can change the amount of radiation the sun emits, this is the number we have to work with. To balance incoming solar system earth has to emit the same amount, otherwise it’s cooling or warming.

        “What if there already is mathematics with accepted physical models that recognizes (theoretically and experimentally) “downward longwave radiation” to arrive at approximately the temperatures we measure?”

        What if these models are total nonsense? What is the relevance of the greybody value earth is supposed to have (255K) if the bulk of the oceans are already 20K or more above that number?
        Even if DLR was real, how do you explain the atmosphere heating the earth, if you realise that the heat capacity of the atmosphere is the same as that of 3,2 METER of ocean?

        10

        • #
          Jose_X

          If you decide to do math, the following estimate might be relevant.

          1 cubic meter of water weighs about 1000kg. 100 cubic meters of water covers a column of a square meter of water 100 meters deep and has mass of about 100 000 kg.

          If water at the top of that column is at 275 K and the air temperature is at 255 K there is a net loss of about 85 W = 85 J/s.

          Now, let’s do a very rough approximation. Water loses (or gains) over 4000 J/kg per Kelvin. For a change of 1 Kelvin, this means that the column will have to lose some 400 MJ (megajoules). At 85 J/s, that would take around 4.7 million seconds or around 55 days.

          Basically, I think this suggests (even if the numbers are wrong, but assuming the analysis makes some sense) that within a decade perhaps the oceans should approach equilibrium in a measurable way. In centuries, the ocean should probably be tracking the sun. In a million years, we probably have more than enough time for all the heat to have risen and to reach a balance.

          So, the next question is, if you think that the current temperature of the oceans match the energy received purely from the sun (since excesses would have radiated away by now as just argued). I’d expect your model to quantify that more carefully. [Maybe there is ground energy helping out, but you would have to quantify that too.. and it should make sense across millions of years.]

          Besides equations/math and physical arguments to the above, I would also want to know more about this radiation falling from the sky and why it appears that while the shortwave radiation approximates these calculated values of what the sun should send to the earth (and Planck’s energy distribution), there is probably a lot more radiation energy measured as DLR not accounted for via the Planck/S-B analysis. Keep in mind that we have satellites making measurements towards the sun as well as ground stations, so we should have a clue of what the sun sends our way and then what hits the ground.

          If you do some of the math legwork and present it, then I (for whatever my opinion counts) would consider the model more carefully, but, otherwise, I don’t see any reason to believe it more than the standard models. I would not really believe it, in fact, to whatever degree it would appear to conflict with established models.

          It does seem to me by your own numbers that perhaps you are attributing a lot of heat to the water that should have dissipated by now. Note, I am not making that claim of failure until I see from that model a more comprehensive theory that tries to model the sun’s and the ocean’s radiation (or whatever the model uses for energy transfer); I am not saying that the numbers you give aren’t justified. I am saying there appears to be some inconsistency with the average numbers given by many scientists, the standard equations, and the values you have picked, assuming no ghg effect. So I presume a more detailed explanation of the model would try to more carefully quantify these details in some way. When we have something more concrete, then we could take more steps to see if it is consistent with various observations or experiments we might perform.]

          In summary, I can’t trust the current model you described over anything standard because it lacks too many details and on the surface it appears not to follow standard equations or it fails observation. Of course, you can challenge observations or make your own (or give competing equations/theories or fix errors in the argument above, etc), but all of that has to be carefully detailed, I think, for most any scientist to give it serious consideration. Ask yourself if you would think differently if some random stranger offered you a new model that presumably disagreed with what scientists at large believed? [Perhaps you want to team up with some person/group who already has equations and theories further along and see what math results comes from it. Someone with strong math skills can help. Let me answer the following question, “why would I in general not spend too much time helping someone in your position to develop that theory further?” Because there are a zillion possible alternative models, and unless I have a strong favorite in the race (because I don’t have a zillion years of life), I tend to believe that the standard models developed by many people over years is the strongest theory, and I ask that anyone wanting to convince me to support any significant change to the standard models to first clearly point out how the models fail observations and then why a minimal change to the model would not be enough.]

          10

      • #
        Truthseeker

        Jose_X,

        This might answer the question about which average temperature to use …

        http://theendofthemystery.blogspot.com.au/2012/02/true-energy-balance-of-earthatmosphere.html

        If you have any questions, please ask them at the source.

        10

        • #
          Jose_X

          Should I assume your message was addressed at BenAW?

          Taking one part from one model and another from another model and not sure which theory or equation people have is something for BenAW and the other people to settle on. Third parties generally are not going to risk wasting lots of their time chasing unproven models.

          As I said before, you need to show what is wrong with current theories people have invested time learning, fixing, improving, etc, and then how your new cohesive theory fixes a problem or otherwise improves upon it.

          If your theory doesn’t add much except alleged simplicity, then you have to demonstrate that a significant number of results can also be derived from that theory. You have to specify what you are discarding and then how you are replacing that and results that depend on that.

          Of course, you don’t *have* to do anything, but if you want to get people’s attention, you need to do your part. People have investments, so don’t expect anyone to drop major investments unless you make it very easy and appealing. You need to relate your additions or changes to what they know.

          That said, I will consider that page a distinct model and try to find some time to relax and look at it. I’ll also keep in mind the Venus/Earth ratio discussion from before.

          [Right now I am looking at how feedback analysis is used by climate scientists and how that may differ from traditional feedback analysis… All of this takes time for me to do. I always appreciate a summary or anything that will arouse curiosity.. eg, like something it can do that other models can’t.]

          10

        • #
          Jose_X

          Truthseeker, I read the beginning of that page you just mentioned, and again it seems the person is finding patterns (the one from venus/earth page being rather intriguing to me based on my limited experiences). Anyone can find ratios or any other math to tie various numbers together. That doesn’t do much except in some cases maybe suggest that we should poke around that idea further.

          For Venus/Earth, I already suggested (but haven’t spent too much more time playing with numbers) that the similarity between the sizes and masses (and g constant) of Venus and Earth might be directly involved with why the two planets have a similar lapse rate at similar pressures.. at least to first order approximation at their “outer” edges of the atmosphere (ie, where the pressures are in the vicinity of 1 atm).

          I haven’t pursued those calculations too much because the simple Postma analysis didn’t derive the correct lapse rate, so, if I have not seen it (or done it) for Earth, I won’t get the Venus results either (the rates look very similar at the 1 atm range). I think the right answer would come from a more precise analysis that also considers radiative heat transfers.. but I have not yet played with that math. This author (Huffman) also didn’t calculate the lapse rate from basic principles. He merely noted that the ratios were almost the same at the 1 atm region, and from that observation stated a more general hypothesis.

          I have not provided a readable analysis of Huffman’s claims, but let me repeat what I said before. A new theory has to find a flaw with an existing theory. There is no evidence that this new theory gets anything right that we don’t already know how to calculate. And worse there is no reuse of more fundamental equations that people already have validated against mother nature. ..none, except the use of S-B.. which itself was used wrongly because it ignores that the sun’s radiation does bounce/reflect off the earth’s and venus’ atmospheres. We see the earth from the moon because of this reflection. Huffman doesn’t account for that and uses the entire S-B result (with 0 albedo) in his fairly simple calculations. Right there he *appears* to be violating conservation of energy (I don’t know enough of this theory to know for sure). Of course, he can try to broaden his theory to answer many questions and address the conservation of energy bit. That is what he should be doing now if he wants a fair number of people to give him serious consideration. I have not done a serious critique yet, but these are the sorts of questions I would explore. [I have been motivated by the Venus Earth example, but I expect after I have learned more things and have had time to play with a computer to solve challenging equations I would not solve otherwise, that I will confirm many of my hunches and be able to write a convincing rebuttal… So Venus/Earth is on “pending” if you want to know.. with the footnote that I think I could detail some flaws now or at least argue that the author is not solving anything we don’t already know from more basic and cohesive principles.]

          Concerning this new page, I don’t think I read (in the part I read) where the 2:5 ration comes from (2 parts atmosphere warming to 5 parts “earth” warming). Anyone can look at measured data and figure out a nice ratio that applies. That is not a theory. That doesn’t use simple core principles and equations to derive pieces. At best, it is an alternative, but the author claims climate science is all wrong. If climate science is all wrong, then his theory is probably broken as well because he relies on numbers derived from “climate science”.

          Patterns exist everywhere. This is not astrology. We need a physical explanation that integrates with our body of physics.

          I don’t yet think I can post to his website, so anyone can feel free to copy/paste this comment there or link to it. If I had a question for him, it would be his opinion on conservation of energy implications for using an albedo of 0. You can get around this by saying that S-B doesn’t describe all the energy radiated from body A, that it only covers the part that is also absorbed by body B, but this has implications that may require reinterpretations of photon energies and the speed of light limit since you can’t know (using current accepted theories of nature) from far away how much body B will absorb and reflect at the time you emit your radiation so as to emit an extra amount so that B can absorb the full amount calculated from S-B. [I think I brought this critique up recently.. not sure if it was an initial impression of Johnson or not.] .. I don’t remember all the details of the Venus/Earth, so feel free to correct me.

          10

  • #
    BenAW

    Hi Keith

    Only way system earth can loose energy is by radiating out to space. As long as incoming solar and outgoing radiation match, nothing is lost, just redistributed.

    Ben

    10

  • #
    Jose_X

    I have a question. [And I will also say a few things about what motivates me, so that people (eg, Paul) don’t think I am trying to fool anyone.]

    Let’s say you “suspect” skeptic scientists and engineers will come up with something better, and you want to wait around for those results or you want to learn about the details of the work the skeptics are doing (with little interest in that “other” science).

    This sounds reasonable to me if for some reason you don’t want to trust the scientists that have been working on this problem till now (or you don’t trust certain key points).

    Maybe you ordinarily have no reason to distrust but you don’t *like* the conclusions, so you want to make sure no rock has been left unturned in seeking alternative projections, etc.

    Again, this sounds reasonable. [little aside]..I don’t share those views in that I address my doubts without assuming “everything” is wrong. Of course (and this may explain part of the reason why), I have no problem with a degree of imposed moderation and a bit (more) large-scale discipline when it comes to exploitation and consumption of non-renewable resources and also for getting a handle on externalities being passed on to each citizen by various industries. I do see resource problems in the future, and we have way too many people on the planet (not that anyone is more deserving of living here than another.. but we do have too many people for my level of comfort.. and the number is growing at a healthy pace). It makes lots of sense to me that we make sure we are putting focus on renewable technologies and research. This just makes sense. And yes I also value nature in many ways. ..So, to wrap up this little aside, I’ll add that I fully recognize we all have biases and will disagree on methods and particulars.

    Here is my question, in the end, if the greenhouse effect exists, if people are threatening the comfort of future generations (w/ or w/o greenhouse effect), if these things ultimately bear out, wouldn’t you want to “know” that as quickly as possible if in fact this would be the case?

    The default for societies is obviously to do nothing and keep consuming. “Let those who are around when X happens, deal with it.” But I will assume that most people care to some degree about whether they might have been able to really simplify the lives of those of future generations or otherwise complicate their lives.

    So to me it makes sense to study up on what I think are the most accurate ideas, and to me that would include ideas that supposedly have strong mathematical and physical backing. If a theory is integrated into our accepted science in many ways (and there are several), it has legs to stand on and has some minimal guaranteed level of robustness. Odds are that it is right, mostly. Odds are that investigating the details of it would be worthwhile.

    Now, I know there is subjectivity that creeps in all over the place in judging. I know we all accept as near fact many things we don’t “prove” for ourselves, but, regardless of our biases and to whom we give the benefit of the doubt, I think it makes sense ultimately that we’d want to make sure the most likely best ideas were a part of the model we are investing time trying to improve and get to be as accurate as possible.

    I know that there is the alternative of temporarily preferring to bury the proverbial head in the sand at least for a while, but I hope ultimately we all want to be exposed and understand the best science (or have our proxy scientists do so on our behalf). And since we can’t create reality [some may disagree with this statement I suppose], the sooner we utilize accurate components of models of reality the more time we’ll have to deal with threats these may imply. ..If I were eating my own foot, for example, I’d want to know ASAP, even if it would initially mean possibly forgoing something I currently found entirely delicious.

    OK, so I welcome strong criticism on greenhouse effect and am interested in alternative views.. but this doesn’t mean I will be convinced easily. Right now, I feel fairly comfortable with this ghg eff.

    I am not a troll, yet I have no choice but to have biases and bad judgement calls that will surface here and there ..but oh so rarely! 😛

    [FWIW, I don’t have money in this race. I do have preferences, but I think I could pick a side and probably make acceptable money with it. If anything, more money seems to be with the skeptical side. Just add up the expenditures and the profits of our traditional energy industry. It’s huge. Although there are many interests on the other side as well, eg, almost any business that is not based on fossil fuels but competes with it.]

    [FWIW, I once upon a time did study a fair amount of physics, math, and/or various engineering of one sort or other (no degrees, however, as I bailed out). I also currently return to tech study from time to time (special thanks to Wikipedia and the Internet), but I have never dedicated a career to it and know a lot less than I would like to know.]

    10

    • #
      Jose_X

      In case anyone cares (since I already brought it up), I don’t think we *have* to act now. I don’t favor one policy over another to address CO2 emissions (I probably do but haven’t thought about it much). I do think we have other more important short-term issues. I even imagine future societies possibly wanting to release some CO2 on purpose; however, I think we should be able to control levels, so I really favor research and like the idea of CO2 sequestering. I also think it would be less painful if we begin to manage CO2 sooner than later since we can start off very easily (arguably we already have started to manage it or at least plan/research). Whereas, I don’t think CO2 action is super high priority in the next few years or maybe decade or more (and I could be misjudging this), I do think we need to work to improve our renewables in general, the sooner the better.

      10

      • #
        Truthseeker

        Jose_X,

        CO2 is irrelevant to the climate, especially compared to the elephant in the room that is water vapour. Are you suggesting that we control water now? Maybe we could sequester water in, say something like … a dam perhaps? Yes, then we could release the dammed water to drive turbines to make electricity …

        00

        • #
          Jose_X

          >> CO2 is irrelevant to the climate

          CO2 absorbs and emits radiation, and a great deal of professional research, developed theories, and observation, with key points covered in text books (as well as near consensus among climate scientists) disagree with your claims.

          >> elephant in the room that is water vapour.

          Yes, the warmer it gets, the stronger the GHE from water.

          >> Are you suggesting that we control water now?

          No, controlling CO2 would resolve the problem since keeping the temperature stable keeps the absolute humidity stable. Allowing CO2 to rise and raise the temperatures a little increases the absolute humidity and leads to 2x more warming.

          We can’t keep the water from evaporating, but we should try harder to keep the CO2 under control.

          00

          • #
            Truthseeker

            Jose_X, where is the observable evidence that CO2 affects climate? There is no data to support this assertion. Computer models are not proof of anything other the confirmation bias of the people that wrote/paid for those models. There is no consensus amongst actual scientists on this Jose, none at all.

            Water vapour acts as a negative feedback mechanism. More heat > more evaporation > more clouds > less sunlight > less heat > less evaporation > less clouds > more sunlight > more heat > etc, etc, etc. No catastrophe there ….

            00

          • #
            Jose_X

            Truthseeker, there is plenty of observational data on greenhouse gases absorbing and emitting. Many gases have their specific spectral profiles. The physics describing this is used in engineering and in other sciences to get results.

            So the best theories on the atmosphere are founded by solid physical models (and feel free to point some other atmosphere theory out) and predict the greenhouse effect.

            How would we measure this on the planet? Well, for example, we make predictions and then we see if the temperature rises as predicted. And we repeat the experiment under different stresses and many times until reasonable scientists are convinced.

            Can we pump the earth with CO2 and run many experiments? Not really. We are running one experiment at a time on earth and it takes a while to see the results. If we end up cooking our planet during this experiment, we don’t get to run another.

            Now, there is evidence from ice cores and elsewhere that allows us to have degrees of confidence that the earth behaves a certain way in the past, but those experiments are limited by what happened and by natural events in the past. There is room for doubt.

            a) Question: can we agree on what I have said so far above this point?

            This aside, the current experiment we are running on planet earth has signature effects that adhere to our greenhouse theory predictions. The earth is very complex, so that too leaves room for doubt, but given that we have only one earth, if you can’t come up with a competing theory of worth, risk management dictates we follow our paper theories (which currently are being verified within reasonable error tolerances).

            I don’t have to be able to conduct all the experiments I would like to conduct ideally in order to assess that GHE is likely enough real and we should do something about it. Remember, it is based on the best theories, but we can’t experiment fully on the planet in a large scale without essentially killing the planet and ourselves.. and we can’t even do that at this point because of the size of the experiment and costs. One experiment is being carried out at a time and slowly.

            There is strong evidence that CO2 is being pumped into the environment because of fossil fuel burning. There are many calculations we can make on this and measurements of the type (isotope) of CO2 in the air. We also have strong reasons to believe the current levels are higher than in the last few millions years (but almost certainly since man’s recent ancestors evolved).

            So what do you offer, knowing that we can’t conduct real experiment on the full planet? You offer we take our chances and risk death… even though many laboratory experiments exist. Even though current predictions about how the planet should change have found supporting observations (including steady temperature rise in the atmosphere when we factor out natural cyclical forcing events like the sun and steady and large temperature gains in the oceans.. and lots of ice melts)?

            At this point in time, the reasonable approach is caution because of the great risks that the theories suggest. As time goes on, we keep reevaluating the risks. That is sanity.

            b) What part of the above do you not agree with?

            c) What evidence are you looking for specifically to convince yourself that the risks are significant enough to warrant action?

            00

          • #
            Jose_X

            >> Water vapour acts as a negative feedback mechanism. More heat > more evaporation > more clouds > less sunlight > less heat > less evaporation > less clouds > more sunlight > more heat > etc, etc, etc. No catastrophe there ….

            More heat > more evaporation > more ghg effect by a significant amount. This is a well-supported understanding that is a positive feedback.

            Besides that strong positive feedback from having more water vapor, water evaporation I am guessing has a weak negative feedback imposed on top. [BTW, “feedback” is used to mean a different thing in climate science than is the case in ordinary controls theory. I am more aware of the ordinary meaning.]

            There are also positive feedback cloud effects (eg, more extra absorption into the atmosphere than sunlight reflection) besides the negative feedbacks you mentioned.

            The water evaporation effect is well-supported and positive. The cloud effects (and the direction of their net contributions) are not well-known. These little known effects in total appear to be less than the other known effects.

            This is my current understanding, but I there is much research and text book material I have not read. I’m not a climate scientist or I would know more.

            00

          • #
            Truthseeker

            Jose_X, there is very little about this that I agree with.

            “Truthseeker, there is plenty of observational data on greenhouse gases absorbing and emitting. Many gases have their specific spectral profiles. The physics describing this is used in engineering and in other sciences to get results.”

            This is where the GHG proponents such as yourself come unstuck. You seem to equate radiative energy with thermal energy and say what is true for one is true for the other. I am not a physicist, but even I can see this is nonsense. Radiation may be absorbed and emitted, but heat only increases if work has been done.

            As for the ice cores and elsewhere, there is a large number of completely independent proxies looking at the past in geological timeframes that show the CO2 changes after heating has occurred, not the other way around.

            As for pumping more CO2 into the atmosphere, well we have been doing this for a while now and neither the sattelites or argo buoys can detect any increasing heat trend for the past 15 years. Do not forget that 97% of the CO2 in the atmosphere is naturally occurring. Why should we be concerned about the other 3%? The only thing about CO2 that concerns me is not having enough. At 150 ppm it is all over. No plants > no animals > no humans. We are only at 390 ppm. I would be much happier (as would the biosphere of the planet) if it was up around 600 ppm.

            00

          • #
            Jose_X

            I would like to know what of the following feels wrong to you. I am confident I am not coming unstuck, but please ask (or google) if you don’t understand.

            Photon radiation (which is what we are talking about.. in contrast to radiation of alpha particles and other examples) follows the same basic electromagnetic rules of engagement regardless of the frequency. The frequency of the particular photon does impact if a specific material will absorb it or if it is reflected or transmitted, but the same general laws of motion, energy, etc, apply regardless of frequency.

            Thermal radiation, which is a form of heat transfer, gets the “thermal” adjective from the fact it can lead to temperature increases.

            Radiation at a wide range of frequencies (basically all classified as infrared, aka, IR) can excite molecules into vibrational and other forms of motion that contribute to temperature. [In fact, even radiation that affects mostly electrons has some probability of “relaxing” into an increase in kinetic energy.] Radiation in these IR ranges can be called thermal radiation.

            The technical term “heat” just means a non-zero net transfer of (non-work) energy. Heat can exist via radiation as well as from some form of contact or all together. So thermal radiation, aka, IR radiation, when it results in a net transfer of energy, would be called heat. [Ie, heat just means there was more energy going in one direction as another. We call heat when “non-work” energy is transferred.. regardless of the mechanism of transfer or, if radiation, the frequency.]

            So keep in mind these points:
            1: All radiation, no matter the frequency, would be called heat as long as there was a net transfer into or away from the body being described.
            2: Thermal radiation, aka IR radiation, is a name given to radiation in a certain range of frequencies. Many molecules absorb energy of these frequencies to increase their vibrational energy.

            This absorption and emission of IR/thermal radiation applies to gases as well as to liquids and solids, except that gases have a much more limited range of absorption/emission because (eg) solids have many molecules together and the potential energies associated with any given molecule or set of molecules at any given point varies (in most solids) almost as a continuum. Gases in ordinary pressures are individual particles basically independent of each other and so are limited to a narrow range (ie, narrow absorption band) by the interactions of the few protons and electrons that constitute that individual molecule.

            So some gases can absorb IR to increase vibrating energy. These are greenhouse gases. Most gases are greenhouse gases. A few gases are very homogenous (eg, N-N or O-O), and quantum mechanical analysis reveals that they absorb and emit in very narrow bands and very few bands.. lying essentially above the IR range of frequencies. Our atmosphere is composed largely of N2 and O2, two of the few non-ghg.. meaning that the vast majority of the radiation from the planet cannot be absorbed by the vast majority of molecules in the atmosphere.

            [If you want, I can try to explain blackbody radiation, which is radiation of an ideal “black” body based essentially only on its temperature. This is what the sun and earth release but at different ranges because they are at different temperatures.]

            Essentially having greenhouse gases is like a blanket.. subject to what I just mentioned that a solid cotton blanket absorbs over a wide range and even if “thin”. Gases are very spread out and require much more volume .. and can only absorb a range of the total radiation even if the volume was huge.

            Anyway, radiation form the earth can be called thermal radiation because most of the frequencies are in the IR range. [See blackbody] This radiation, whether absorbed by a solid, liquid, or gas, readily excites motion in the atoms. It adjusts the temperature.

            Let’s get back to the atmosphere. The ghg absorb radiation, but before an emission can occur, the energy is usually transferred to nearby much less excited gas molecules (including non-ghg like N2, O2, etc). So the atmosphere ghg absorb thermal radiation to heat up, but they usually don’t radiate right away and instead share their newly acquired energy with other gas molecules. Periodically, there is emission. The air has a temperature and has ghg that can radiate similar to how liquids and solids radiate, so they will radiate some amount dependent on their temperature. “Back radiation” comes from the same basis as does blackbody radiation, but it is limited in frequency range (depending on the particular gas doing the emission) and comes from all throughout the volume of gas rather than largely at the surface.

            OK, so let’s recap a few more key points:
            3: The earth radiates in the IR range.
            4: Most gases (but notably excluding N2 and O2) absorb in a fraction of this IR range, the particular range depending on the particular ghg.
            5: After such absorptions, the energy is usually shared (eg, with abundant N2 and O2), but every now and then, because of its temperature, a very excited ghg will emit thermal radiation in its particular frequency range.

            Satellites measure the frequency of radiation and they confirm that a very large fraction (I think near 50%) of that radiation received is in the narrow bands of CO2 (ie, in the frequencies where CO2 absorbs and emits). This is confirmation of the ghg effect of CO2 in action. Why near 50%? Well, because water vapor only lies near the ground. There is a lot of CO2 above the topmost layer of H2O. Any natural temperature emission from above this layer will not come from H2O but largely from the ghg that are way up high.. and most of this is dominated by CO2.

            Last summary point:
            6: Evidence of CO2 emission in large quantities exists and is consistent with the spectra of CO2 and our knowledge of which ghg are in the atmosphere and where we believe them to be.

            OK, I’ve said a lot. If you read this all, let me know why you think the infrared radiation from the planet can’t be absorbed by CO2 and heat it up and later have this energy be passed along to other gases bouncing against the CO2.

            00

          • #
            Jose_X

            I have to run, but let me say something about ice cores info and CO2 pumping.

            CO2 historically follows temperature rises. The total quantities of CO2 that ultimately rise afterward is related to the greenhouse effect to some extent: after CO2 increases, temp increases some more, leading to more CO2, etc, up to equilibrium.

            However, regardless of what sets CO2 to increase initially after temp increases (eg, the oceans warming), that does not disprove the greenhouse effect. We can have temp increase CO2 AND CO2 increase temp.

            The ice core data is useful to give insight and help in tuning calculations, but it does not contradict/nullify the greenhouse effect. The ice core data suggests mostly only today is CO2 leading. This is evidence that man is playing a role.

            Man’s role is further enhanced by the large amounts of CO2 we release (say 3% of yearly totals), and the steady accumulation of this amount only partially checked CO2 release, year after year after year, leading to a significant rise in CO2 after decades of release, now to levels not seen in millions of years. The current 3% (but earlier much smaller %) — only partially absorbed by oceans — has added up, and we now have CO2 levels way beyond what the human species has ever “witnessed”.

            00

          • #
            Truthseeker

            Jose_X, of course CO2 and H2O absorb incoming radiation, increase in heat and then pass that on to surrounding molecules via conduction. That is the process by which the atmosphere gets the bulk of the heat that it has. What is not happening is the warmer surface getting more heat from cooler gasses by radiation, conduction or any other method. This is the catastrophic scenario proposed by the alarmists. Also CO2 is going to reflect as much radiation out to space as it is towards the surface so it is not going to have a net warming effect. Water vapour is a better holder of heat because, when it exists, it is much denser than the surrounding gas. It is also not evenly spread. There are large areas of the planet with very little humidity and others with a great deal. This causes temperature differences, which causes air currents and other weather effects. After all every living thing on the planet is affected by the weather. Nothing living is actually affected by climate.

            Also the time lag between temperature increases and CO2 increases is in the order of 800 to 1000 years. This is far too long for any feedback effect to occur and no feedback mechanism is shown with any part of the geological record. CO2 concentration at the moment is the highest in human history but not nearly the highest when you look at the geological record.

            You have nothing useful to contribute to this discussion Jose_X. I’m done with you.

            00

          • #
            Jose_X

            >> What is not happening is the warmer surface getting more heat from cooler gasses by radiation, conduction or any other method.

            If the sun keeps adding heat and the earth dissipates it more slowly at the existing temperature because more ghg have been added so it takes longer for added quantity of thermal energy from the sun to dissipate, then some amount of thermal energy will accumulate. That sounds intuitive, right?

            Why is it so hard to imagine that the temperature would rise as a result?

            How do you explain that the earth is warmer than predicted by Stefan-Boltzmann (and if you have to appeal to Huffman’s ratio, I think you know you are not paying attention to established physics)?

            As H et al pointed out, the S/4 approximation that GT criticized demonstrated a lower bound for the GHE. GT’s more elaborate approach implied the GHE was much larger than 33 K. In any case, GT did not show a calculation or theory that would have explained the current temperature ranges. GT’s math (even ignoring albedo) had the temp lower than Stefan-Boltzmann. Even if you ignore albedo entirely (and this contradicts energy conservation and measurements from up in space) and use the “hotter” S/4 method, you still don’t get to today’s temperature (you get a slightly lower value). So how do we get to the higher temperatures we see today? It has to be something.

            Let me ask, would you be convinced back radiation is radiation like any other and adds thermal radiation (that had left earlier) if a Wood experiment conducted on a high mountain showed higher temperature when the rock salt covering is covered by glass?

            The simplest theory of photon radiation (and the one accepted by physicists) is that when a photon leaves, the molecule lost energy. When a photon is absorbed, energy is increased. So “back radiation” adds energy because it is energy that may have left the planet earlier but instead of flying through space ready to add energy to some space alien far away, that energy was redirected back towards the earth ready to add energy back to the earth.

            >> Also CO2 is going to reflect as much radiation out to space as it is towards the surface so it is not going to have a net warming effect.

            To keep the math simple (and this is an incorrect calculation, btw), that would mean that 50% more radiation is hitting the earth than it otherwise would be getting. Something is something. Every time a photon goes up and gets absorbed, it again has a 50% chance of coming back down.

            >> Also the time lag between temperature increases and CO2 increases is in the order of 800 to 1000 years. This is far too long for any feedback effect to occur and no feedback mechanism is shown with any part of the geological record.

            I haven’t done the analysis myself, so I can’t say for sure, but the models that supposedly hindcast successfully and leverage the greenhouse effect of CO2 would reach those values after “invoking” the greenhouse effect. Anyway, we can pass on this minor point. I was just trying to be a bit more precise when I brought it up.

            >> CO2 concentration at the moment is the highest in human history but not nearly the highest when you look at the geological record.

            Did you see how fast CO2 rose? What natural causes do you give for that? The point is that man has done that.. or at least that and other evidence create a very strong case that we are responsible.. like the many changes we have created to the face of the planet. Humans are very capable and keep getting more so as time passes. Heck, we have the weapons to totally decimate most advanced life on the planet. We deface huge landscapes and could do much worse if we unleashed our weapons. Yes, man has the power to change the earth significantly.

            >> You have nothing useful to contribute to this discussion

            but if I keep you entertained and excited about physics and math, I consider that a victory.

            00

        • #
          Jose_X

          >> we could release the dammed water to drive turbines to make electricity

          One day we’ll probably have large artificial lakes with microorganisms soaking up sun, water, and carbon dioxide to produce oil. In the meantime, over a hundred million years of fossil fuels are disintegrating over a period of a few hundred years. It will probably be close to a 1 million to 1 ratio (creation vs consumption time).

          We need to do better, and I don’t think we’ll ramp up oil generation (still in the early laboratory stages) anywhere near our current consumption rates for a very very long time if ever.

          If we can’t even be bothered today to trim our consumption (and pressure industry to innovate) what kind of life style will our great great grandchildren be forced to have?

          00

  • #

    Jose_X

    Were you really genuine in saying The mirrors surface may not be warm enough to cook a hotdog or boil water, but the radiation focused by the mirror certainly can. ?

    When considering the application of the Second Law (on a macro scale) what on Earth has the temperature of the mirror got to do with the price of eggs when you are reflecting, say, visible light from the Sun?

    Surely you have an understanding of what we mean when we talk about a warmer or cooler source of spontaneous blackbody emission. Surely you know we are talking about the peak frequency (the mode) which is proportional to absolute temperature of the source, as per Wien’s Displacement Law.

    What on Earth have lasers, microwave ovens or any artificially generated radiation got to do with the Second Law? Are you assuming the “temperature of the radiation” has anything to do with the temperature of the machine or mirror?

    The Second Law of Thermodynamics cannot be violated on a macro scale in natural processes when radiation passes frpm a spontaneous emitter to a target, anywhere where there is matter in the Universe. If it could be violated, you would be generating energy out of nothing.

    Radiation spontaneously emitted from a globule of air with mean temperature, say, 220K cannot transfer thermal energy to the Earth’s surface at, say, 294K.

    All it can do is set up a standing wave which does not transfer energy to the surface, but can slow the rate of radiative energy transfer from the surface to the atmosphere to a degree dependent upon the temperatures of the source and target and other factors. So it may take a few minutes longer to cool off that night.

    Carbon dioxide molecules will have no significantly greater effect than water molecules when involved in (ie terminating) such standing waves. So their effect is negligible.

    No radiation is going to slow the rate of cooling due to evaporation, chemical processes, conduction, diffusion and subsequent convection. In fact these rates must increase to compensate for the reduced radiation for reasons which are explained on my website (Explanation page.)

    00

  • #

    Jose_X

    A similar argument means the planet will have molecules that can absorb what the atmosphere emits.

    No you won’t be able to observe on a macro scale any absorption and conversion to thermal energy of any of the energy in the standing wave. To an insignificant extent on a macro scale (and only due to unusual weather events) there may be patches of air just above the surface which are warmer than the surface itself, so sure there could be warming then, but the net worldwide mean is certainly not warming of the surface by the atmosphere. And that’s what climate is all about.

    Please read the Radiation pages on my site http://climate-change-theory.com/RadiationAbsorption.html and also some of the posts by Markus Fitzhenry where he mentions the standing waves.

    00

  • #
    David Wood

    It’s often said that the simplest models are the best. Einstein’s famous equation is a case in point. However the models also need to replicate the real world as closely as possible. If they don’t they probably give outcomes which may be mathematically correct, but which are completely incorrect physically.
    Take the case of the simplistic model of the sun/earth radiation equilibrium, widely used in the pseudoscience of climatology.
    This is a model which assumes the earth is a photosphere like the sun, which it isn’t. A model which assumes the earthy is constantly bathed on all sides by a quarter of the radiation from the sun, which it isn’t. A model which takes no account of the existence of night and day, which is patently absurd. A model which ignores the existence of any other form of heat transfer except radiation A model which assumes a static equilibrium, when it is clearly a dynamic equilibrium. A model which ignores the existence of water vapour, except when convenient! This is the model beloved by the warmistas.
    If it sounds like a crock, looks like a crock, then it probably is!!!
    The warmistas have posited a very simple but physically unrealistic model and then used a mathematical construct to “prove” the earth is 33 degrees warmer than it would be without an atmosphere.
    However this mathematical model is greatly at variance with the real world and while it produces mathematically ‘correct’ results (with the help of the previously unheard of assumption of ‘backradiation’), these are likely to be (IMO certainly are) quite incorrect physically.

    00

    • #
      Jose_X

      photosphere? Where did you get that?

      The models look at the energy coming to the planet. Because of convection, the temperature is relatively stable and the S/4 approximation is presumably a decent one. Do you know of a research paper that addresses this topic and concludes there the climate models use a value that would lead it to very distinct predictions?

      Who told you the models only consider radiation?

      Who told you they assume static conditions.. or rather, can you more precisely define what you mean here? Scientists and engineers assume quasi-static conditions all the time. It depends on what you are looking for if the assumption is reasonable or not. We are not talking about weather models (climate is not weather).

      Can you define what you mean when you say that the models ignore water vapor when it is “convenient”?

      Who told you that scientists claim the earth is 33 C warmer than if it didn’t have an atmosphere? The 33 C is the difference you get if you have an atmosphere with .3 albedo but no greenhouse effect vs if you do have the greenhouse effect. No where is the contrast made to an earth with no atmosphere.

      “Backradiation” is just downward infrared radiation. This exists and is measurable and is accepted independently of climate science.

      00

  • #
    Jose_X

    Doug Cotton, I have been busy with other (related) distractions and unrelated work. I’ll try to find time to read some more of G&T.

    Your rejection of photon radiation is something you should take up with quantum physicists and others before you worry about climate science paying attention. Same goes for the theories of various authors mentioned by blouis79.

    Let me ask because I am curious. (1) What specifically determines the energy transfer among these standing waves? You need to get specific enough so that we can come up with an experiment to test it. (2) Can you use Maxwell’s equation to explain why a standing wave arises and at what point energy is transferred? (3) Can you extend that standing wave theory to cover a laser’s functioning and interaction among the internal components?

    (4) Can you explain the temperature of the planet?

    (5) Can you explain why the temperature rises when you put on a blanket? (6)How do standing waves from the blanket to your body function? (7) Why differs in the physics so that a blanket can make the temperature around you hotter but an atmosphere somehow just can’t possibly do same to the planet’s surface or else thermo laws are violated as you say? In each case, the air is essentially “stuck” from going around the insulation, so I don’t see how we can invoke convection.

    (8) Why can’t chemical energy in the atmosphere, recharged by the sun, not warm the surface of the earth just as chemical energy inside your body warms your skin? (9) How can you categorically claim that atmosphere back radiation (which is constantly under the sun… like solar cells that charge up to allow a laser to shoot off) would violate thermo laws but still believe your body can have chemical reactions that warm you hotter than the environment?

    (10) What makes you think that what man engineers, mother nature can’t engineer also?

    Just saying that there is a standing wave and energy is not transferred doesn’t cut it. If you want to turn physics upside down, you need math and precise rules, and you need to do a better job than current physics. Just because you don’t like the greenhouse effect theory doesn’t mean you can invent physics that only works a certain way for the planet and atmosphere. You won’t convert many physicists if that is what the theory requires.

    00

  • #
    Jose_X

    I pretty much finished G&T [shake head and roll eyes]. If anyone wants to point out something specific, go ahead.

    I haven’t yet read a certain major rebuttal to it (see next link), but the Abstract in that rebuttal suggests the authors focused in on important flaws in the paper. For reference, here is the link to that rebuttal: http://scienceblogs.com/stoat/upload/2010/05/halpern_etal_2010.pdf

    Let me summarize briefly (the paper did cover lots of items):

    Basically, G&T put up a strawman they could beat the crap out of (a hypothetical 1 dimensional radiative-only model). The authors in the rebuttal above called G&T on it. While G&T appear to think they were beating evil climate science down, the rebuttal (I suspect) de facto (and perhaps very unintentionally) mocks them for arguing using such a primitive construct to argue the physics of the climate.

    If you want to skip to the section that presumably “proves” that the greenhouse effect doesn’t exist, read section 3.7.5 “Non-existence of the natural greenhouse effect” and the very end of 3.7.6. Of course, reading just this may not make too much sense, but apparently that is the climax of their argument in case anyone is interested in jumping ahead. Also note that chapter 3 is the chapter where they go insane. Earlier in the chapter (3.3.x) they claim to shoot down a long list of candidate explanations (overviews) of the greenhouse effect, for example (frequently appealing to 3.7).

    [Note, I also read over one of the papers mentioned in section 3.2, “CO2 : The Greatest Scientific Scandal of Our Time”. It’s a personal cry of frustration by someone who makes many claims but offers no proofs. It’s author gives the impression, for example, that CO2 measurements have been done for many decades and show no correlation with temperature. Of course, Keeling pointed out a major reason this is the case (eg, hot day measurements of mixed CO2+atmosphere vs night measurements where CO2 given off my many organisms rests near to the ground, sometimes even moving in flows upward). Keeling did make it a point to make precise measurements in Hawaii, using “dry” laser methods of high precision vs “wet” chemical methods used to that point. Many people over the world make such measurements and the results agree. See http://www.youtube.com/watch?v=k7jvP7BqVi4 and http://www.esrl.noaa.gov/gmd/ccgg/about/co2_measurements.html . I may comment on some of this further later.]

    What did they prove? They made up a model (1D radiative-only) which is so likely to be wrong that it isn’t even the simple “shell” model (also a 1D radiative-only) people usually talk about to introduce climate radiation. G&T’s model comes about by taking a simple shell model and breaking the main assumption of that model, that convection keeps the earth’s temperature very close to some average value. They create this scarecrow in order to unleash a torrent of mathematics and provide a clear path to discrediting those who presumably use it (climate scientists at large) and its use.

    By essentially giving 2 particular examples of a generalized version of the old triangle inequality law. They claim to show that this radiative model can’t possibly give a reasonable calculation of the claimed greenhouse effect: therefore, the greenhouse effect does not exist.

    This logic doesn’t really make sense. I think section 3.7.6 at the end tries to sort of claim that not just the math but the physics has been dispelled, but it’s a complete failure. You can’t prove one model is broken and use that to claim a particular physical effect doesn’t exist. The physical effect exists (or doesn’t) independently of any model we may create to try and understand that effect. Scientists know you don’t really prove something exists. You merely “prove” that our existing models agree or disagree with observations. If you pick a bad model, expect bad results, but you can’t claim that a physical effect can’t exist or that no other model could possibly be created to describe it. We don’t know how much we don’t know. And in any case, we can’t wave our hands to dispel things. The mathematical analysis in this paper was very limited in its scope.

    G&T appear to have a phobia to the idea that some/many gases can absorb and emit thermal (IR) radiation. While G&T pay passing tribute to navier-stokes, quantum electrodynamics, and various other more sophisticated physics (eg, Chapter 4), they avoid any mention of the observations of line spectra and related experiments or uses of the absorption and emission power of gases.

    In briefly acknowledging via Chapter 4 that more advanced physics exists, they try to write all of that off by appealing to chaotic complexity (they make many references to other manuscripts, so potentially those works would back up the claims made by G&T.. I certainly have not read the 200 or so references in the paper, in part, because I don’t expect those papers would prove G&Ts broad-brush claims). In writing off climate science into the complexity trash bin, they give no indication that they understand that complexity has always been managed to give less precise but still useful results. All of physics and science is our attempt (with much success) at managing complexity. For example, I don’t care how hard it is to predict where a volume of water will end up precisely, we can with great confidence say that it won’t end up flowing in the air for miles alongside a flying bird or underground amid lava. Hurricanes aren’t going to fly off into space. The temperature is not going to naturally reach 500 K during ordinary weather events. G&T also do absolutely no analysis to what might be a particular range of time before a system’s chaotic behavior goes beyond some margin of error. They simply state that weather forecasting is good for only a few days at a time (why not only a few seconds or a few centuries.. what is special about a few days?) and that all the supercomputers in the world will not allow climate forecasting to be conquered (whatever climate forecasting might possibly mean). While the details of a chaotic system can be impossible to pin down after some time, we can and do successfully pin down the boundaries of many systems, G&T’s handwaving notwithstanding.

    This general failure to honestly analyze any part of the climate is likely why they picked such a broken model to go after. Arguably, they don’t consider convection and many others things since presumably that is how simplified they think climate science might be, but, despite the excuses that they apparently tried to find the physics explanations of the greenhouse effect, a clear theme throughout is that all hope is lost trying to analyze the weather.. I mean the climate. Abandon ship, says captain G.T.

    One could try to fill in some blanks and come out of this reading with a little insight into the math of global average temperatures (something that could have been taken further if they weren’t so obsessed with their simplified model), but even in this learning/teaching opportunity, they don’t analyze our actual planet’s temperature distribution which is what would have been useful rather than focusing on a clearly broken unphysical and extreme model.

    00

  • #
    blouis79

    Jose_X, suggest reading the Kramm and DLugi paper.
    eg via http://www.gi.alaska.edu/node/1067

    Note my comments in post #616 on Halpern’s “rebuttal” of G&T, which Kramm thinks is patently wrong.

    You may be confused by reading scienceofdoom, since SoD appears not to have a good grasp of physics.

    The entire notion of radiative energy balance in a system (atmosphere) where conduction and convection operate is a violation of the first three laws of thermodynamics.

    00

  • #
    Jose_X

    Radiative balance is something not done in climate science except in toy models.

    If you look at Trenberth diagram you will see that you don’t get balance unless you take into account latent heat and convection. There is no radiative balance. [Trenberth diagram is not a computational model, but it shows the results consistent with computational models.]

    G&T beat up a punching bag they created… or like a voodoo doll that was supposed to represent mainstream climate science.

    One specific critique of G&T: the pot with water (3.8.3). If all you have is a radiative model, then of course that example doesn’t match observations, but the correct model is that water acquires through conduction a lot of energy. The conductive heat flow dominates over the net exchange of radiation between water and the pot. And water will boil away at a temperature which is obviously less than the temperature the pot would have without water. The pot won’t get much hotter than the water (which is capped by 100 C).

    The radiative only model fails, but climate science doesn’t use that. The radiative effect of this water on the pot exists but is dwarfed by the energy flow into the water via conduction.

    If water were suspended somehow so that conduction would be negligible and with enough of an opening so that the air in between could cool through convection, then we would see that the water would not boil so easily, the pot would get very hot again, and “back radiation” would still be modest. There is no disappearing heat. Back radiation is not imaginary; it’s just much lower in effect (for such a small quantity of water) than what we see with conduction during contact.

    If water where not there at all, we still get radiation from the air, ceiling, etc added to the radiation from the burner (minus the radiation coming out of the pot).

    There is no violation of 2nd law. The “isolated” system is of the kitchen with pot and stove. The net flow of heat, likely into the kitchen away from the pot+stove, is a greater entropy gain than the loss by the pot+stove, so there is a net increase in entropy. [dS = dQ/T, where a smaller T of the kitchen produces greater entropy change (positive since into the kitchen).]

    G&T’s thermo discussion in 3.9.3 is wrong. Their critique of the climatologist’s quote makes little sense. [Please, feel free to offer a defense.] The climatologist can refer to energy since this just equals heat if no work is done. And the climatologist was considering the whole system. From what words does G&T claim the climatologist was not looking at the whole system?! G&T is seeing climate apparitions. In fact, the whole system consists of the sun, earth+atmosphere. Here we see that the sun’s loss of entropy is much less than the gain by the earth+atmosphere.

    Let’s look at the earth and atmosphere only. Trenberth’s diagram suggests the earth is giving the atmosphere more energy (the net flow of heat is to the atmosphere, ignoring the sun’s flux, remember). Again, dS=dQ/T shows what we want because the earth is hotter than the atmosphere. The entropy gain by the cooler atmosphere is greater than the entropy loss by the hotter earth. We simply have a blanket placed over a person. With the blanket, the temperature around the skin rises, even though the blanket is cooler than the person — heat flow is still into the cooler blanket where the entropy gained is greater than the entropy lost by the person.

    I agree we need conduction and convection. G&T argued as if climate scientists don’t use conduction or convection. You also claimed now that climate science insists on radiative balance with conduction and convection, but that is not supportable. Trenberth’s popular diagram quickly should dispel that belief.

    00

  • #
    blouis79

    Whenever climate scientists talk about balance of or effect of W/m2 forcing, they are assuming dominant radiation and radiative energy balance. Thus notion is rife. This notion is inconsistent with any sensible principle of physics.

    For a different perspective, try Johnson
    http://claesjohnsonmathscience.wordpress.com/2012/03/01/two-proofs-of-plancks-law-vs-backradiation/

    00

    • #
      Jose_X

      I tried to follow the logic of large parts of the Kramm/Dlugi’11 paper (it takes a while for me to absorb the material.. so this is a quick but focused warm-up run). I get the feeling the paper zooms in on the temperature averages point in order to try and knock down “greenhouse effect” as explained using various models.

      It’s not going to work.

      Finding imperfections does not tear down a model because every model is imperfect. The IPCC’s job is to try and quantify risks (likelihoods). You don’t need a perfect model. People don’t wait to buy insurance until there is certainty of a calamity. That is not how life works. We have only one earth. We can’t prove calamity just as we can’t prove safety. We merely try to quantify risks.

      What they (GT and KG) haven’t done yet is to actually try to model the distribution of temperatures we find on earth and then use that “value”. This is what the computer climate models try to do. The simpler models they are attacking are recognized by their authors as allowing for a “back-of-envolope” approach that is useful to a human trying to understand and develop ideas to later test on more detailed climate models [see Ramanathan quote at end]. Attacking a back-of-envelop calculation is not going to disprove the greenhouse effect.

      I would guess this paper also entirely avoids dealing with the physics/observations that form the foundation of the greenhouse effect: that ghg absorb IR radiation and emit it. Until greenhouse effect “deniers” present a credible explanation of how ghg are supposed to behave and react to radiation that is passing their way, they are not going to convince very many serious scientists. Once they deal with that question, they will likely find that the current models are a decent approach to try and quantify what is a very difficult problem in many ways.

      Claes Johnson is deviating from mainstream science. Whenever someone does that, they don’t get the benefit of the doubt. As concerns climate science, Johnson first needs to gain widespread traction within the community of statistical mechanics experts to show that what he offers is as good as or better than existing science in order for most scientists in other fields to invest in learning those tools (which can rationally be assumed today to be bogus or inferior). If he wants to gain traction at the application level (eg, climate science), then he can start by deriving a model that makes predictions that beat current predictions. I am curious enough that I might read a little more of what he has written, but it’s not a priority (climate science and physics generally is not a long term priority for me.. in the sense of taking up as much time as I currently spend on it) and I don’t have high expectations based on the intro I read into the methods.

      Here is a quote from Ramanathan ’78 conclusion on page 23 (a radiative-convective 1D model):

      > Here we will discuss the future use of radiative-convective models for climate studies. Such models should continue to be used for obtaining first estimates of the potential sensitivity of global surface temperature to perturbations in radiatively active gases for several reasons. First, the global surface temperature changes predicted by the models are in reasonable agreement with those obtainred from the more complex three-dimensional general circulation models (GCM). For example, for a doubling of CO2, Manabe and Wetherald [1967] obtained 2.24 K from a radiative-convective model, while they obtained 3 K from a GCM [Manabe and Wetherald, 1975]. It is important to note that both studies used the same radiation model. Second, because of its simplicity a radiative-convective model is capable of including many details of the radiative processes without overburdening the computer resources, and thereby it can give valuable information on the importance of such processes. Third, for climate change experiments, analysis of radiative-convective model results would be useful for GCM studies, since it is much more difficult to infer cause-effect relationships in a GCM model.
      > The important limitation of the model is tha the model results are mostly of academic interest, since the model does not give any information about regional and latitudinal temperature changes. Furthermore, many of the model parameters (cloud amounts, surface albedo, relative humidity, and critical lapse rate, to name a few) are prescribed on the basis of present-day conditions which may not apply for large departures from present-day conditions. For example, the study by Wetherald and Manabe [1975] indicates that for a 2% increase in solar constant the radiative-convective model results for dTs are within 20% of the GCM results, while for a 4% decrease in solar constant the two models differ by a factor of 2 in the estimated value of dTs. Clearly, radiative-convective models cannot be applied for large perturbations from present conditions.

      And here is a link for anyone curious about how GCMs contrast with weather forecasting models: http://www-das.uwyo.edu/~geerts/cwx/notes/chap12/nwp_gcm.html .

      00

      • #
        Bryan

        Jose_X says

        “What they (GT and KG) haven’t done yet is to actually try to model the distribution of temperatures we find on earth and then use that “value”. This is what the computer climate models try to do. The simpler models they are attacking are recognized by their authors as allowing for a “back-of-envolope” approach that is useful to a human trying to understand and develop ideas to later test on more detailed climate models [see Ramanathan quote at end]. Attacking a back-of-envelop calculation is not going to disprove the greenhouse effect.”

        To show that a theory like greenhouse theory is wrong you don’t have to supply a complete new theory of climate of your own.

        All you have to do is point out the fatal flaws in the greenhouse theory.

        Jose you are to be commended for taking time to read these two papers however it would be a better use of your and everybody else’s time to pick on one or two items where you think that G&T and K&D got it wrong.
        Then have an in depth discussion on that particular point.

        00

        • #
          Jose_X

          >> pick on one or two items where you think that G&T and K&D got it wrong.

          [I skimmed KD]

          For GT, let’s start with anything that is covered in the first four pages of the rebuttal http://scienceblogs.com/stoat/upload/2010/05/halpern_etal_2010.pdf . What I stated overlaps those pages a fair amount; however, that rebuttal does a good/better job of addressing some points.

          Here is one example from page 4:

          > We find that Gerlich and Tscheuschner obtain an absurd result by using a very unphysical assumption, that each part of the planet’s surface immediately cools or heats to reach an equilibrium with the locally impinging solar radiation, thereby neglecting the thermal inertia of the oceans, atmosphere and ground and all other heat transfer processes within the atmosphere and surface.

          > It is shown here that a uniform surface temperature model gives a more realistic bound on the greenhouse effect, the commonly quoted 33 K. This value is a lower bound on the magnitude of the greenhouse effect and even Gerlich and Tscheuschner’s result for their unphysical case obeys this bound.

          00

          • #
            Bryan

            Jose X on Halpern et al

            1.Check the units used for irradiance on page 1317.
            2. They proclaim that heat can spontaneously travel from hot to cold objects in the paper.
            Joel Shore one of the authors later said this was a mistake.
            3. They accuse G&T (wrongly) of saying radiation cannot pass from a colder to a hotter surface despite several diagrams and equations in the paper showing just that.
            All their other comments such as…
            “neglecting the thermal inertia of the oceans, atmosphere and ground ”

            ..come about because of Halpern et al failed to read properly the G&T paper.
            G&T make it clear that the ground flux needs to be included.

            I’m afraid that you will just have to start at the beginning and read the G&T paper properly because you should have found this out for yourself.
            Relying on flawed sources such as Halpern is not a good idea.

            00

    • #
      Jose_X

      >> Whenever climate scientists talk about balance of or effect of W/m2 forcing, they are assuming dominant radiation and radiative energy balance…This notion is inconsistent with any sensible principle of physics.

      Feel free to be more detailed. I have looked over a number of papers in the past few weeks that are rife with bold claims like that, but the papers have not followed up with convincing argumentation.

      00

      • #
        Jose_X

        “Forcing” refers to a driving signal. It’s a way to refer to various terms in equations. This can come potentially from anything (and depends on the equations/models). I think it refers to drivers that are assumed to be independent and not calculated from other values. This is why the sun and CO2 are forcings.

        00

  • #
    Mark

    Oh thank God, Jose is back. Just what I need to counteract my excessive caffeine habit.

    Ahhhhh, beautiful sleeeep…thanks so much Jose…………zzzzzzzzzzzzzzzzzzzzzzzzzzzzz.

    00

  • #
    Jose_X

    Bryan from #631.1.1.1.1,

    1. I don’t know what page that is. I have the downloadable pdf and haven’t yet read past pg 4 of H et al.
    2. Can you be more specific about this point? I haven’t read the rebuttal yet.
    3. GT says cold can give heat to hot if you add work, I agree. I haven’t read the rebuttal so don’t know what H et al have said. Can you give a specific quote from H et al? I did comment earlier (yesterday, I think) about how GT claimed some climatologist violated the 2nd law, but GT made no sense. I’ll get back to this point if you want.

    >> All their other comments such as… “neglecting the thermal inertia of the oceans, atmosphere and ground ” ..come about because of Halpern et al failed to read properly the G&T paper. ..G&T make it clear that the ground flux needs to be included.

    Finally, you get to the quote I mentioned. Here I can follow.

    GT do ignore “thermal inertia of the oceans, atmosphere and ground” in their climactic section 3.7.5 and related sections that critique the 1D radiative-only model.

    Let’s wee what GT09 says on page 63:

    > Such a calculation, though standard in global climatology, is plainly wrong. Namely, if one wants to calculate the average temperature, one has to draw the fourth root first and then determine the average, though

    That calculation they criticize (which precedes this quote above in GT09) is absolutely *not* standard in climatology research. That calculation forms a small part of some introductory texts to students but has nothing to do with the work done by climatologists.

    GT is wrong here and in all the other places in GT09 where they suggest that climatologists use models like this in practice.

    After showing this “wrong” way to calculate effective temperature for an albedo .3 earth (w/o ghe), they go on to show us the “right” way to do it by setting up and processing an integral which calculates the temperature on a sphere that is radiated by a distant sun (parallel radiation). The temperature is defined using Stefan Boltzmann based on the exact radiation hitting that part of the sphere.

    This model they use forms an interesting exercise for first (or even second) semester undergraduate calculus course, but is totally unphysical. The earth does not have the implied fatally high temperatures near the equator at noon nor 0K everywhere it is night time. This model is much worse than the S/4 average temperature model GT just finished attacking.

    GT claims the right way to calculate the temperature was through that integral. That method absolutely does not apply to the Earth with an atmosphere (which is where the -18 C S/4 average temp intro text book number comes from). That method..

    ..most definitely ignores “thermal inertia of the oceans, atmosphere and ground”, just as H et al stated.

    Let me continue. To “prove” the paper’s primary claim as framed in the paper’s title (and as just stated with the integral above), GT resorted to attacking this made-up 1D radiative-only model that is worse than what beginning students might read their first week as an introduction.

    I return to the quote I provided. H et al stated in that quote: “Gerlich and Tscheuschner obtain an absurd result by using a very unphysical assumption.”

    H et al also stated, on pg 2:
    > The authors [GT] describe “problems” that are not really problems, either being not related to the greenhouse effect, or well known and understood minor issues such as the differences between the mechanisms by which a glass greenhouse warms and that by which the greenhouse effect leads to a warmer surface.

    GT’s criticism of the 1d radiative only model is a contribution fit for the 1950s period. Some may have argued that already even earlier, but at least into the 1960s it would make sense. It doesn’t make much sense in 2012 unless it is meant as a “classroom” type of discussion. To critique climate science at large on this model is a little short-sighted on their part. They seem very out of touch in trying to engage mainstream scientists in this way.

    Let me add a bit about thermal conductivity in the atmosphere:
    H et al page 2:

    > A recurring element in GT09 is the claim that others neglect the thermal conductivity of the atmosphere, presented in the introductory (“Problem background”) section covering the first 4 pages of the paper and also Sec. 3.8, as well as elsewhere in passing. However, they fail to place this in a quantitative context. In fact the heat flows associated with conductivity are tiny, and hardly different from the value of zero they repeatedly criticize in the work of others.

    This comment is long, but let me add something else. GT make a big fuss about how a real greenhouse relies on convection. GT appears almost angry at times that climate scientists would use such an analogy. H et al state the following accurate summary of the GT09 paper. ..page 3:

    > The next 18 pages of GT09 are devoted to showing that the atmospheric green-house effect relies on different physical processes than the warming in a glass green-house. This is a well-known fact that can be found even in popular expositions of the atmospheric greenhouse effect and is mentioned on p. 115 of the 2007 IPCC report.13 The short concluding paragraph in Sec. 2.6 of GT09 would have sufficed. Concisely, greenhouses work by restricting the outward flow of thermal energy to the surrounding atmosphere by convection, while the atmospheric greenhouse effect works by restricting the outward flow of thermal energy to space by radiation. In both cases restricting outward energy flow causes warming, so the analogy is not as irrelevant as GT09 claim.

    Alright. So I provided a lot more meat around that initial quote I took from H et al. I also added a few new quotes. Hopefully, you will address to some extent some of these quotes I have given, but I am also interested in further clarification to the 3 points you listed out in #631.1.1.1.1.

    00

    • #
      Jose_X

      >> I’m afraid that you will just have to start at the beginning and read the G&T paper properly because you should have found this out for yourself.
      Relying on flawed sources such as Halpern is not a good idea.

      In addition to the prior comment, I want to add.

      None of your 3 points you gave dealt with the quote I gave or with the first 4 pages of H et al. I found the quote I gave (and generally the first 4 pages of H et al) to be accurate based on my reading of virtually all of GT09.

      If you think this is not so, then clarify.

      I would also appreciate if you clarify what part of those 4 pages (or anywhere else if you are willing to be specific) is “flawed”.

      I surely have found numerous flaws in GT09, but it is true I read all of that and haven’t read all of H et al.

      00

    • #
      Jose_X

      I said> The earth does not have the implied fatally high temperatures near the equator at noon nor 0K everywhere it is night time.

      More clearly, this is not the real earth we were considering but an earth with atmosphere but without a greenhouse effect having taken place yet (a hypothetical in order to roughly quantify what ghg add to temperatures). Such an earth will not have 0K at night or super high temps during the day because convection still exists.

      I said> GT is wrong here and in all the other places in GT09 where they suggest that climatologists use models like this in practice.

      I am referring to serious research (and the state of the science). Obviously any specific individual may or may not toy around with these primitive models now and then.

      00

    • #
      Bryan

      Jose X

      I am coming to the impression that you are a waste of my time, you say

      “1. I don’t know what page that is. I have the downloadable pdf and haven’t yet read past pg 4 of H et al.”

      You ask us to read a paper that you have not read yourself!

      You flit from point to point like a butterfly.

      Go back and read the paper YOU RECOMMENDED.

      THEN address the points I raised about your wonderful recommendation.

      00

      • #
        Jose_X

        The pdf I have has only 24 pages. I offered a link to it. Where do you get page 1317?

        Second, you asked for me to pick something to discuss and I did, yet you failed to reply and you jumped to something else when I specifically said I have not read that rebuttal yet.

        I read GT and lightly read over KD. I have been very clear about this. It’s a shame you aren’t reading what I write.

        Now, with this clear, I hope you will address at least some of the many points I have raised. I also hope that as a courtesy, when you say something about a paper and the person (who already stated they had not read it) asks for a specific citation like a page number, that you oblige.

        I am very serious about these material, but you are not answering what I propose and so far indicate you haven’t read things very carefully.

        00

    • #
      Bryan

      Jose X says that G&T

      ..most definitely ignores “thermal inertia of the oceans, atmosphere and ground”, just as H et al stated.

      Jose if you read the second paragraph on page 65 you will find that Halpern et al did not read the G&T paper properly.
      In fact G&T are saying that you cannot determine the temperature from the radiative flux on its own.
      The ground heat flux must also be included.
      Its good that you are trying to understand the thrust of the paper.
      Their first language is not English and you will need to make allowances.
      K&D deal with the more advanced version of the greenhouse effect that you refer to.

      00

  • #
    Jose_X

    Bryan>> 3. They accuse G&T (wrongly) of saying radiation cannot pass from a colder to a hotter surface despite several diagrams and equations in the paper showing just that.

    H et al section 2 (pgs 7-12) appear to be making that claim against GT.

    I would agree with you that GT did not make that claim.

    One of the things I did the past week was to use the Internet (mostly wikipedia) to dig into some basic thermodynamics (and some applications to the atmosphere.. eg, potential temperature). I am slowly getting comfortable with the terminology and some of the basic results.

    One problem I see in section 3.9.3 is a misuse of terms. GT appears to be correct in most of the first paragraph of their reply, but they aren’t attacking the greenhouse effect. Effectively, GT is just chiding someone on a misuse of terms. GT might be taking the literal interpretation and criticizing that, but then that wouldn’t be the greenhouse effect that was described literally.

    The greenhouse effect does get attacked by “skeptics” on this 2nd law issue. H et al gave an explanation to what one can easily erroneously believe GT had said since GT was attacking a climatologist talking about just this issue. If GT had attacked the essence of the quote, they would have been incorrect, but GT appears only to have attacked the literal interpretation of the quote. H et al probably jumped the gun and didn’t realize GT was effectively only doing a literal critique.

    GT, too, appears to have misunderstood and thought the target of their own critique (the climatologist) was referring to one side heat flows.

    GT quoting>> > However, the second law is not violated by the greenhouse effect, of course, since, during the radiative exchange, in both directions the net energy flows from the warmth to the cold.

    GT reply> It is inadmissible to apply the second law for the upward and downward heat separately redefining the thermodynamic system on the fly.

    The ending part of the climatologist’s quote (knowing little else about the context of that argument) might be misunderstood to refer to 2 separate systems if we judge by where the comma was placed here. Who placed that comma at that spot originally? I don’t know. In any case, the quote states “both directions” not each direction, so it is referring to one entity comprising of both pieces. GT appears to be wrong.

    If so, it seems both GT and H et al though the other party was referring to individual directions.

    .. There is another option (which I didn’t notice before writing this comment). GT might have made that last sentence as a simple statement of fact *without* alleging the climatologist was making that mistake; however, nearby context does somewhat suggest he was still criticizing the climatologist.

    I am less confused now about this section than I was before, but I still am not sure exactly what GT is claiming. And, yes, H et al section 2 (pg 7-12) appears to be addressed specifically at claims GT did not make.

    00

  • #
    Jose_X

    GT starts 3.9.3:

    > The use of a perpetuum mobile of the second kind can be found in many modern pseudo-explanations of the CO2-greenhouse effect

    I don’t see that case made with the example quotes given.

    00

  • #
    Jose_X

    Bryan, I am not forcing you to discuss GT.

    GT is flawed in its primary claim and in numerous other instances (despite the truckload of accurate if mostly irrelevant details it provides), and you should not feel pressured to defend it its flaws.

    I also think it is a bit odd you would accuse me of not reading when it has been me who has provided by far the most detail about that paper (as well as H et al), including exact quotes with page numbers and links.

    00

    • #
      Bryan

      Jose X

      Sorry that I was a bit sharp with you in the post above.
      If you look at the Harpern pdf link you gave you will find the page numbers such as page 1317.
      You appear to be more flexible in your outlook than I had given you credit for.

      00

  • #
    Jose_X

    Bryan #633.4,

    I’ll keep the language point in mind (and I certainly don’t claim mastery of English). Normally, I try to figure out what the author(s) means, but in heated and precise discussion inevitably it seems precise words end up getting used against someone.

    My impression on Ch 3 was that they were attacking a particular model and certainly not embracing it, but there are places where their position on some point or other is not always clear.

    GT appears to have some clear and important gaps in knowledge on what they were criticizing. Perhaps out of frustration, they went over the top in accusations. H et al offered a fairly calm response, but they may have not been willing to give the benefit of the doubt, also taking the opportunity to try and clear issues that are generally accepted but which GT raised or appeared to have raised.

    Bryan #636.1,

    On page 9 (1317!), it says:

    > The radiative flux leaving each surface in any defined period, which we take for convenience to be one second, can be calculated from the Stefan-Boltzmann law.

    This is why they use Joules. It’s the energy transferred in one second at the usual power flux.

    From energy (as heat), they calculate the entropy [dS = dQ/T] in order to make their point.

    You probably missed that by skimming. It happens.

    00

    • #
      Bryan

      Jose X

      I have their final draft before publication

      “in any defined period, which we take for convenience to be one second”

      This part is missing

      It looks like they corrected the text without changing the diagram.

      This is why they use Joules. It’s the energy transferred in one second at the usual power flux.

      Power flux is measured in w/m^2

      On page 9 you can find their violation of the second law with heat apparently spontaneously being transferred from colder to hotter surfaces.

      00

  • #
    Jose_X

    ..so even if GT did not claim explicitly that heat always flows from hot to cold, they did create (to a reasonable spectator aware of the issues) some confusion about whether they think the accepted views on greenhouse effect violate thermodynamic laws, eg, whether a colder body can radiate onto a warmer body.

    H et al clarified (on page 9/1317) that point with sample calculations. They use 1st law to balance energies and then the second law to show that each side does radiate onto the other (and is radiated by the other), but the next entropy change is positive.

    00

    • #
      Jose_X

      > eg, whether a colder body can radiate onto a warmer body.

      ..in other words, in the case of no work being performed.

      > next entropy change is positive

      “net” (not “next”)

      00

  • #
    Jose_X

    5 upcoming replies back-to-back-to… include:

    A: a quick review of the J/m^2 and entropy calculation from H et al,
    B: observations on GT’s view of emissivity
    C: explanation of why car gets hot (consistent with GT and GHE)
    D: Gore movie “reflection” redux
    E: infrared facts, and who is really confused, GT or climate scientists?

    00

    • #
      Jose_X

      A: No violation

      Bryan >> On page 9 you can find their violation of the second law with heat apparently spontaneously being transferred from colder to hotter surfaces.

      They draw a diagram and label it “Fig. 4. Heat and entropy exchange between two parallel, infinite plates at temperatures TA = 300 K and Ta = 260 K.”

      The diagram resembles the power flux diagrams we tend to see, but it expresses energy not power. Specifically, it expresses the energy exchanged (work=0) by two bodies during 1 second.

      What one body losses in energy, the other gains. This agrees with 1st law of thermo and is captured in almost trivial equations (1) – (3) on pg 9.

      Then it calculates the entropy lost by the body losing energy (ie, heat) and gained by the body gaining energy (ie, heat). The 2nd law states that this net system entropy change must be >= 0. Since the net heat flow was from hot to cold (analogous to the hotter earth’s surface losing to the cooler atmosphere), the entropy change ds = dQ/T by the higher temp body is negative but smaller magnitude than the positive entropy change of the lower temperature body. The net is positive, just as stipulated by 2nd law.

      We note that the net heat flow was calculated based on this: the cold body *does indeed* send radiation to the hot body but this radiation amount constitutes less thermal radiation than what the hot body sends to the cold body. Entropy changes from this interchange adheres to the 2nd law as just shown.

      00

      • #
        Bryan

        Jose X

        Have you worked through the Carnot Cycle?
        If you did you would find the idea of “net heat” is a false starting point
        There is only heat moving spontaneously from hot to cold object.

        Radiation is a two way transfer
        Energy is a two way transfer
        There is only one way heat transfer spontaneously from hot to cold object.

        00

        • #
          Jose_X

          I didn’t use the term “net heat”.

          I assume you agree with what I wrote. If not please be specific.

          00

          • #
            Jose_X

            Wait, I did use that term.

            Wikipedia says this: “According to some authorities in physics, chemistry, engineering, and thermodynamics, heat can only be energy produced or transferred from one body, region, set of components, or thermodynamic system to another in any way other than as work.”

            My understanding is that heat is the name of the net energy transfer (excluding work). So it is redundant/incorrect to use net heat as I used it near the bottom of my comment.

            In any case, short of pickiness over correct use of terms, is there anything with which you disagree? The claim is that there is no violation of 2nd law. This is the opposite of what you had been saying. I don’t expect you to change your mind that quickly.

            00

          • #
            Jose_X

            BTW, I don’t mind being corrected (“pickiness”) to the extent it helps make things clearer or more accurate. Changing the few instances of “net heat” into “heat” would fix that problem.

            00

          • #
            Bryan

            Jose X

            Its not being picky to refuse to allow misuse of sharply defined thermodynamic terms.

            Someone for instance says

            “radiation from a colder body spontaneously heats a warmer body.”

            You could say what they really mean is ……

            But what if they did really mean exactly what they say?

            They are not children!

            Especially if the statement was made in a physics journal!

            00

          • #
            Bryan

            Jose X

            Heat transfer only occurs spontaneously from a higher to a lower temperature, never the reverse.

            This is a popular way of expressing Clausius second LoT.

            00

          • #
            Jose_X

            I don’t know the context of that quote. I agree with you but not entirely. There is room for lack of precision. In the end, if you dismiss a comment someone makes (or claim to disprove a theory, etc) because of form and not function, you aren’t trying to get at the truth but are more interested in scoring points.

            Regardless, the proper response is to address GT’s argument. Adding precision to language in that case certainly helps the overall scientific conversation.

            As for heat transfer.. you aren’t contradicting anything I said above or from H et al. Heat was defined as a net change. It is fully permissible to have transfers from each side to the other as long as the net change adheres to the laws of thermo (which I pointed out it did).

            00

        • #
          Bryan

          Jose X

          You seem to have a flexible mind and are not pushing a particular agenda.
          I guess your training is in climate science but you have considerable knowledge of Physics.
          I on the other hand have never taken a climate science course and so I tend to comment only where fundamental physics is involved.
          I have a theory that climate science thermodynamic courses miss out on the Carnot Cycle.
          Perhaps they do not think it is relevant to climate studies.
          This would be a mistake as it is the natural introduction to the 2LoT and heat.
          There again maybe I’m wrong about missing out Carnot.
          G&T also say that the radiative effects of a gas are already included by the bulk thermodynamic quantities such as Cp (the heat capacity of a gas at constant pressure).
          For instance for CO2 Cp increases by 13% between 250K and 350K.
          If radiative transport calculations are used there is a chance of double counting energy if care is not taken.
          Best of luck with your further reading.

          00

          • #
            Jose_X

            >> G&T also say that the radiative effects of a gas are already included by the bulk thermodynamic quantities such as Cp (the heat capacity of a gas at constant pressure).
            For instance for CO2 Cp increases by 13% between 250K and 350K.
            If radiative transport calculations are used there is a chance of double counting energy if care is not taken.

            When people really think they are right and others who contradict are likely wrong, we can get very aggressive. This view explains some of the more aggressive moderation we see on some climate websites and explains the tone of GT, who obviously has a strong background in various areas.

            It would be interesting to see if a model of the atmosphere can be developed to help answer some questions and, in the spirit of the Cp comment, do so while bypassing the current usual radiative approach.

            I suppose GT and many others come from an environment where “back radiation” is not accounted for (eg, is hard to quantify or the effects wash out by convection or something else that is modeled or measured). I am not placing the large portion of my bet on GTs side of the table, but lots of science has complementary models and so we might find a new mostly non-radiative model, largely consistent with the greenhouse effect traditional model, to describe also why the earth has a particular temperature.

            My bets are mostly with traditional climate views (while holding out hope for an alternative/complementary view as well) for 3 reasons I can think of now.

            One, I think evolution is more likely than revolution, generally.

            Two, from what I have learned, the current view seems very reasonable to me.

            Three, I have thought about various forms of heat transfer, and in many cases it is difficult to cleanly separate the distinct effects. This might be why radiation can be ignored in a number of fields; however, ultimately the atmosphere is almost unique on earth in (a) interfacing with the vacuum of outer space and (b) in providing an example of effects that might only be seen at the very large scale of the earth climate system. It makes sense numerous scientists and particularly engineers might be used to using other tools and have a distinct intuition that might need a make-over when crossing over into the climate domain.

            00

      • #
        Jose_X

        Let me add this about the example from H et al.

        The assumption of infinite heat sink is so as to ignore that a gain or loss of heat, in a real object, would affect the temperature. It keeps the calculations easier. Carrying through the non-ideal calculations might result in several decimal places of extra math and with little change to the magnitude if the bodies are large and lose a small fraction of their total internal thermal energy. This idealization and others like it are used all the time in engineering and science. We can easily approach it arbitrarily close by looking at arbitrarily small quantities of time. Eg, in 1/100th of a second, approx 1/100th of the amount of heat would leave or enter the body as in the 1 sec case, and this would affect its temperature by 1/100th of the 1 second scenario.

        The assumption that we don’t look at the radiation towards the outside can be justified, for the case of the earth+atmosphere system, because that forms the system boundary in which we are at roughly equilibrium with the external environment. We gain from the sun (and other sources) roughly the same we lose towards outer space. I suppose we could explore this assumption more carefully. If you have an example where you think the entropy change would become negative, please post it.

        Without counterexamples or (decent) explanation of why the math/model is wrong, the conclusion is that the 2nd law is not violated.

        00

    • #
      Jose_X

      B: GT’s emissivity critique is too harsh.

      GT, in the discussion of section 2.1.5 and Fig 4, on page 21, state:

      > The constant σ appearing in the T^4 law is not a universal constant of physics. It strongly depends on the particular geometry of the problem considered.
      > The T^4 -law will no longer hold if one integrates only over a filtered spectrum, appropriate to real world situations. This is illustrated in Figure 4 .

      If we interpret GT in a generous way, we’d say that their claims can be technically correct if we define sigma always as the coefficient in front of T^4. However, in practice (and according to Wikipedia), sigma is considered to be a constant when dealing with colored bodies (ie, not perfect absorbers). When we have colored (realistic) bodies, we simply define another entity called emissivity. This is standard across physics and is the reasonable way to approach this in order that we can have a constant sigma quantity as a reference point.

      If we are not generous, we say GT is wrong and point to lots of examples where sigma is treated as a constant and emissivity is used.

      There is no perfect absorber in reality. That is an idealization that is only ever approached. [Jumping to a new page, for a minute…] GT is technically correct on page 60 when they say, referring to emissivity (aka, “general phenomenological normalization factor”):

      > Rigorously speaking, for real objects Equation (70) is invalid. Therefore all crude approximations relying on T^4 expressions need to be taken with great care. In fact, though popular in global climatology, they prove nothing!

      The goal of science is not to absolutely “prove” things (an impossibility) but to find good explanations for reality which allow us to use models to better mankind. Newton’s laws weren’t proven. They were generally assumed to be true .. until after Einstein. Such is the way of science. If we took a more aggressive approach, we could not find a single system description that would be proven beyond a shadow of a doubt. There is error in every measurement, for starters. QED is among the best theories we have, but it is not applied directly to complex systems. GT’s complexity point might suggest all of science, by its nature, is useless. Yet GT don’t make that bold claim; they aim their qualitative it’s-not-perfect guns only at climate science.

      Emissivity captures our inability to further precisely define all the dependencies in a real-world problem exactly. It is a useful model in many cases, even if a particular body (like the earth) has that emissivity value that would likely dependent in some minor way on temperature (“spoiling the T^4 law”).

      [For anyone reading GT closely, note that their T^5 relationship of section 2.3.3 is for radiation intensity value at the peak (Wien) and not for total power flux (area of whole curve). Power flux does vary with T^4 to a good first approximation.]

      [Also note that GT’s allegation of impropriety in the next section (2.3.4) gets called out in H et al on page 6 (1314).]

      To return to the 2.1.5 conclusion section:

      They then explicitly state there, “many pseudo-explanations in the context of global climatology are already falsified by these three fundamental observations of mathematical physics.”

      They offer no argument beyond naming three general points and offer no specifics of which “pseudo-explanations” they are talking about. That is a complete fail.. or at least it is a case where we can’t accept their conclusion based on the evidence provided to that point in the paper.

      I’ll note that the first of the three points (the one I did not quote) has to do with that fact that the preferred mathematical models (of radiation) have changed over time, but no specific connection is drawn from this fact to the greenhouse theory except perhaps to suggest that because greenhouse theory explanations (that GT knows about) use a simplified explanation and not the modern formulations that the greenhouse effect is non-existent (or ill-defined).

      00

      • #
        Bryan

        The critics of G&T who are strong in physics generally concede that G&T are correct here.

        “The constant σ appearing in the T^4 law is not a universal constant of physics. It strongly depends on the particular geometry of the problem considered.”

        00

        • #
          Jose_X

          I actually didn’t intend to keep that part of the quote in there (the second sentence, where “geometry” is mentioned). I wasn’t addressing that portion. I was addressing the claim that sigma is not a constant when dealing with “filtered” spectrum.

          I don’t know why GT claimed sigma was not a constant.

          See this wikipedia page http://en.wikipedia.org/wiki/Stefan%E2%80%93Boltzmann_constant

          and see this US NIST reference page http://physics.nist.gov/cgi-bin/cuu/Value?sigma . At least in the US, you don’t get much more authoritative on issues like this than NIST.

          I don’t know German very much, but the German version of the Wikipedia page also refers to the value as a constant and references the NIST page as well.

          The geometry aspect is a red herring. Solving a physics problem of heat transfer (or almost anything else) requires you consider the geometry. Of course.

          I really don’t know what GT is after or why they make such a claim.

          And if you can provide a link to “critics of G&T who are strong in physics” who “generally concede that G&T are correct here”, whatever you were referring to, then I would appreciate it. As is, I don’t really know what you mean.

          00

          • #
            Bryan

            Jose X

            G&T were saying that sigma is not a fundamental constant like the charge on an electron.

            Its is a derived constant that depends on, for instance geometry.
            Gravitational Field Strength 9.81N/Kg is an example of a derived constant.

            00

          • #
            Jose_X

            Gravitational constant depends on the body’s radius. I would agree in that case the geometry matters. Eg, it is the gravitational constant on the earth surface at some hypothetical location.

            Stefan-Boltzmann constant does not depend on the body (the assumption for calculating the constant is “blackbody”). As for the geometry, the definition says per unit area. I’ll quote GT, pg 19:

            > For a perfect black body and a unit area positioned in its proximity we can compute the intensity I …

            We then take that intensity and integrate over all frequencies (remember, we have a blackbody .. filtering is sold separately) to arrive at the unique Stefan-Boltzmann constant.

            That constant does not (and need not) change in any scenario. We use emissivity to denote deviations from that. With that same constant in the formulas, we use geometry to figure out radiation, heat flows, etc, but we still use that constant. [In contrast, see how unnatural and impractical it would be to actually define g = 9.8 m/s^2 or any other specific value.]

            00

          • #
            Bryan

            Jose X you say

            Gravitational constant depends on the body’s radius. I would agree in that case the geometry matters. Eg, it is the gravitational constant on the earth surface at some hypothetical location.

            Stefan-Boltzmann constant does not depend on the body (the assumption for calculating the constant is “blackbody”). As for the geometry, the definition says per unit area.

            Sigma does depend on geometry
            see

            http://scienceofdoom.com/2010/10/24/planck-stefan-boltzmann-kirchhoff-and-lte/

            00

          • #
            Jose_X

            Thanks for the link. Until I see an example of the terminology used on a material that is not diffuse, I will continue to believe that sigma refers to the integral result from the diffuse derivation done by Planck. If the material has preferences in direction and intensity, then Planck’s calculation may not apply (I don’t know).

            For this diffuse case:

            Every material (your hand, a pin, the sun, a car, yarn) can be seen to have “points” on its surface. At a high resolution, all surfaces are flat. So when trying to model (diffuse) blackbody radiation, we can consider each point to be on a plane and to radiate into a hemisphere.

            This is true for all shapes of surfaces. No matter the shape of the emitter and the locations of the objects and other variables, each emitting point follows this same hemisphere calculation.

            Once we have that hemisphere integrated for the generic point emitter in diffuse radiation material (ie, the steradian variable is gone), what is left are the wavelength (or frequency) and watts per meter squared. There is no more geometry. We integrate over all wavelengths to get the Stefan-Boltzmann equation which has the sigma and the units of W/m^2.

            If the material is not diffuse, then the spectral intensity calculated by Planck probably doesn’t apply and sigma probably doesn’t exist.. at least it might have a different name (and probably would be a tensor). Really, I don’t know, but I now feel a little more confident now that, for the diffuse case, geometry doesn’t matter. [NIST calls it a constant!] .. Anyway, thanks for the link, but it did make me curious about non-diffuse scenarios.

            00

      • #
        Bryan

        G&T say

        “The T^4 -law will no longer hold if one integrates only over a filtered spectrum, appropriate to real world situations. This is illustrated in Figure 4 .”

        This again is true.
        It is unbelievable that the filtered spectrum of CO2 and H2O both comprising less than 1% of atmosphere are claimed to back radiate a flux almost as large as the continuous upwelling radiation from the much denser Earth surface.

        00

        • #
          Jose_X

          Concerning the quote, yes, in real world objects, and especially gases, we deviate from blackbody. This is what emissivity is for.

          As for your remark, I don’t think it is that “unbelievable” when you consider that the earth’s surface (or any other solid’s surface body) where radiation comes from is very thin, while the “back radiation” comes from all throughout the entire volume of atmosphere.

          I’m curious, had you realized what I just pointed out?

          Do you think now it is believable, or do you still think it is unbelievable?

          00

          • #
            Jose_X

            I was sarcastic at the end there on purpose, but that wasn’t very friendly of me. It’s called frustration from reading too much into some of your replies and wondering what it will take to address doubts properly.

            00

          • #
            Bryan

            Jose X

            Its the magnitude that’s unbelievable.
            Upwelling is almost the same as downwelling despite;
            1. the radiation window
            2. random direction emission in 3D.
            3. Filtered as opposed to continuous surface radiation.
            4. radiation from a colder temperature (T^4) effect
            5. and so on……

            00

          • #
            Jose_X

            I agree that the value is not obvious. I am not sure of the details yet.

            1. Window: Keep in mind that we can include the window, but we also have evaporation, convection, and the sun’s radiation. An energy balance equation on the surface, if we assume 0 net absorption (rather than the .9 or .5 or whatever) and know/measure the upwelling IR, evaporation, and convection, produces that very DLR value that appears very large to you. That value is what it takes for energy balance. In other words, energy balance means that if the downwelling were not that high, then evaporation and convection must be significantly different (assuming we are certain about sun’s radiation hitting the surface and the radiation coming upward). However, we can measure DLR and the value is in that range used by Trenberth (it varies significantly by time of year, weather, latitude, and altitude).

            2. random direction: This still boils down to upward half plane (or hemisphere) vs downward half plane. If a photon hits you from “above” it counts whether the angle was 0.1 degrees or 90 degrees. For every photon that flies off to the side, another, from somewhere else, is flying at a similar angle to make up for it. No problem here, even if it is initially unintuitive.

            3. ??

            4. ??

            5. ?? .. I don’t understand what you mean.

            00

    • #
      Jose_X

      C: Car in the sun.

      a: Both, convection within the car and convection within the atmosphere, cool the heated surfaces (of the car and of the atmosphere, respectively); however, a hot car has much lower ratio of *convection air volume* to *sun-exposed area* than does the atmosphere, so the car’s relatively small quantity of air heats up faster.

      b: Additionally, convection in the car brings heated air to the top of the car. Because the car’s cavity surface is next to a warm environment, there is less cooling through it than in the case of the atmosphere, where the air that rises is brought much closer to outer space where it can radiate away without getting significant “back radiation” and not restricted by thermal conduction gradients.

      These above are two factors helping to explain why a car can get much hotter than the environment outside. This explanation is consistent with the greenhouse effect. BTW, I don’t think GT stated anything in 2.4 that was inconsistent with this. Their argument in that section was a criticism of the use of the common analogy between the atmosphere and a greenhouse. As already quoted, H et al on page 3 succinctly explain why an analogy is indeed proper.

      00

      • #
        Bryan

        Jose X

        “These above are two factors helping to explain why a car can get much hotter than the environment outside.”

        If you read about the historic experiment of Wood you will find no need of a greenhouse effect to explain this.

        00

        • #
          Jose_X

          Woods experiment took place on the planet’s surface, which is inundated with radiation from above. Both box interiors got radiation from above (either with a stopover at the glass boundary or straight through from the atmosphere).

          At a very high altitude, the glass would radiate back say 50% of what it absorbs from the box’ interior while the other box interior would get almost no downwelling radiation. In this case, the box with the glass top would noticeably be warmer.

          In any case, I was not contradicting anything specific GT had stated in that section, but I did have some “insight” on that car problem while I was reading GT and wanted to mention it.

          00

    • #
      Jose_X

      D: Gore movie re-reflections.

      GT was rather harsh on its description of the Gore movie “reflection” allegation (I already covered this and initially misinterpreted GT’s discussion). I finally saw the movie and did not catch any claim that “reflection” was intended by Gore. GT likely interpreted the ambiguous animation a particular way, but it isn’t the only way to interpret that animation. The essential effect depicted that animation is correct, is consistent with line spectra observations and theory.. that radiation back to the ground exists.

      00

      • #
        Bryan

        Jose X

        “that radiation back to the ground exists.”
        We are all in agreement including G&T

        00

        • #
          Jose_X

          I cannot remember a point in GT09 where significant amounts of IR radiation “raining” down on us was acknowledged.

          I would like to reread that section or anywhere else where GT may have commented on this.

          00

          • #
            Bryan

            Jose X

            Its true that G&T never used the phrase
            “radiation raining down on us”
            However they do agree with two way radiative transfer.
            Several examples in their paper.

            However have you read the G&T reply to Halpern et al?

            “Reply to ‘Comment on ‘Falsification Of the atmospheric CO2 greenhouse effects within the frame Of Physics’ by Joshua B. Halpern, Chistopher M. Colose, Chris Ho-Stuart, Joel D. Shore, Arthur P. Smith, Jorg Zimmermann” by Gerhard Gerlich and Ralf D. Tscheuschner, International Journal of Modern Physics B, Vol. 24, No. 10 (2010) pages 1333–1359.
            http://www.skyfall.fr/wp-content/gerlich-reply-to-halpern.pdf

            00

          • #
            Jose_X

            >> However they do agree with two way radiative transfer.

            My question is if they agree a significant amount of DLR reaches the earth’s surface. If so, do you have a link where I can read that. So far I have seen nothing to suggest they accept that.

            >> However have you read the G&T reply to Halpern et al?

            I just skimmed over it (thanks for the link). I am sure I will be able to learn something from it, but it seemed to touch on very little of my criticisms of them. I am curious about the entropy calculations (I think they criticized the modeling of an “infinite” boundary and they complained about a failure to consider reversible processes).

            Since I read it fast, I am not sure how much they left out and how much is reasonable. I already recognized that H et al went too far in attributing the one-side-heat analysis to GT.

            They also mentioned a few other papers I would want to look at.

            I am not sure what I will do next in terms of reading, maybe writing something up a bit more congruous or formal, and working on other unrelated material I want to get to. So much to do.. so little time.

            00

    • #
      Jose_X

      E: Infrared conspiracy or just confusion?

      GT critiques after Table 8 on page 22:

      > In any case, a larger portion of the incoming sunlight lies in the infrared range than in the visible range. In most papers discussing the supposed greenhouse effect this important fact is completely ignored.

      This insinuates that climate scientists are oblivious to important observations or else that maybe the scientists are trying to deceive. Neither of these two potential insinuations depicts the truth.

      GT ignores the concept of gas emission spectra (founded on quantum electrodynamics and heavy with observational confirmation) throughout the entire paper. If they understood it better, they would realize that the visible spectrum range is not a key defining boundary. The key between “shortwave” and “longwave” isn’t whether radiation lies below visible light but whether it lies above a certain infrared range or within it. Near-infrared and visible each falls into the shortwave category. The (middle and far) infrared falling into the longwave category constitutes a very small fraction of sunlight energy, just as climate scientists claim.

      00

      • #
        Bryan

        Jose X

        Climate science has a very odd set of definitions.
        Physics and all the other sciences use UV,IR,microwaves,visible light, gamma rays,X-rays and so on.

        Short wave?
        Shorter than what?
        Long wave likewise.

        If you said short wave to a scientist without contact with climate science they would not know what you were talking about.
        They might hazard a guess that it means shorter than visible light.
        No wonder there is such confusion.

        Why don’t they adopt the same definitions shared by the other sciences?

        00

        • #
          Jose_X

          I also had a problem with feedbacks (plural is not traditional usage). Climate science calls feedbacks the contributions to temp changes from individual factors. Traditional feedback analysis labels pos/neg the direction of influence (and not the alleged individual contribution value). It’s a bit like the difference between positive/neg displacement (climate science) vs positive/neg velocity.

          I hope they change terminology over time, but that is a slow process.

          During the past few weeks, I spent time on various papers and reading some material. Among that I wondered about the Venus temp profile (see quick discussion below). Under that same article, I posted on feedback. It may be hard to make sense of it, but the key understanding I came to can be read here http://www.skepticalscience.com/news.php?p=2&t=96&&n=1310#76402 and was already summarized in the first paragraph above.

          As to the Venus issue, I am still not satisfied, but I know the profile has been calculated to close precision by others (I could not get free access to a paper on it). Because of the radius and gravity similarities with earth, the lapse rate is similar to the earth lapse rate. Additionally, doubling CO2, if the 1 C logarithmic rule holds, would hardly lead to a noticeable increase in temp after a certain point. In other words, adding some greenhouse gases when the quantity is low creates a noticeable difference, but as the quantity increases as a percentage of the whole, the change happens much much more slowly and is overwhelmed by the fact very high pressures (with at least some greenhouse gases I think) dominates the temp increases. Anyway, these two effects (similar lapse rate and marginal ghe at high concentrations) leads to a similar temp slope on each planet. However, I have not seen mathematics yet that lines up these similarly sloped lines to coincide near the 1 atm pressure location leading to the pattern Huffman pointed out.

          00

        • #

          Byran,

          A better EE analogy is to consider Pi the total power arriving from the Sun and Po the total power leaving the planet which is the sum of reflection and emission. These are instantaneously different where the difference is the sensible heat. If E is the total solar energy stored by the Earth, the rate of change in E, or dE/dt, is the sensible heat or the difference between the power arriving and the power leaving. Expressing in equation form,

          Pi = Po + dE/dt

          Define an amount of time, tau, equal to how long it would take to deplete E at the rate Po.

          E = Po*tau

          substituting,

          Pi = E/tau + dE/dt

          You should recognize this as the differential equation describing an RC circuit which has exponential solutions for E which include exponential decays with a time constant of tau and attenuated sinusoidal output in response to sinusoidal input.

          Hemispheres can be accurately modeled independently since on average, the net power crossing the equator relative to the total power involved is small enough to ignore. The seasonal solar power entering each hemisphere is a sinusoid and the signature of this differential equation is clearly evident in the data.

          These plots show the average response extracted from the ISCCP data set supplied by GISS. This has complete surface coverage of the planet with with at least one measurement per day and in at least 1 IR and 1 VIS band, but most are sampled at 4 hour intervals and have multiple VIS and IR channels, moreover; large regions of the surface have redundant coverage from 2 or more satellites. These yearly response plots represent an average of about 3 decades of measurements.

          http://www.palisad.com/co2/plots/wbg/plots.html

          It’s not entirely obvious that the global response follows the equation until you realize the planets response is the sum of 2 LTI systems, each responding independently to its stimulus. BTW, the global sinusoidal solar signature in the plots arises as the difference between perihelion and aphelion. There is one mislabeled item which is the albedo in the reflection plots. This is not the albedo, but is the global reflectivity averaged between the surface and clouds. The actual albedo is shown in the flux plots.

          George

          00

          • #
            Bryan

            co2isnotevil thanks for the reply.

            Yes analogies can be useful as long as they are not pushed too far.

            The temperature/time record of fixed point at the Moon equator looks exactly like an RC circuit with a diode.

            Latent Heat acts like an Inductor in that often it opposes any temperature change.
            This suggests a possible LCR circuit effect where the energy circulating in ‘the system’ can be larger than the ‘make up’ energy from ‘the supply’ if supplied to an area at the right frequency.

            00

          • #
            Jose_X

            >> Define an amount of time, tau, equal to how long it would take to deplete E at the rate Po.
            >> E = Po*tau

            You could probably approximate the earth system with an RC circuit (exponential decay towards a value), but how good is that approximation? Being 5% off 385 W/m^2 is +- 19, which could error bound the IPCC’s conclusions at a high confidence level while even suggesting we might be in for frost. In short, it’s not very useful to the AGW question.

            I also want to point out that you provided no rationalization for taking tau as a constant independent of time. If it does depend significantly on time, then you could potentially have a very different solution to the equation. This is why I considered a 5% error, but the error could potentially be much greater.

            Not knowing the relationship of Po to E, you may want to linearize as you did (again for a crude approximation). You can do that, but you just have to rationalize that you are analyzing the system only while it remains in a certain range. For the purposes of AGW, we are staying in a fairly small range (a few % of T or P), but picking such a range without some data or other rationalization still doesn’t mean you can expect to have success with a linear approximation.

            I think it is important to keep these things in mind when trying to shoehorn simple equations into climate analysis and expect to get a result that can at all compete with the accuracy of physics supported nonlinear partial differential equations and lots of CPU running time as is provided by the computer model runs you wish to displace.

            00

          • #

            Byron,

            Where is the diode in your circuit? If its in series with the capacitor, the C would charge to the maximum input voltage and stay there, which is how AC is turned in to DC. If the diode was in parallel with the C, it would either short circuit the input and prevent it from charging, or do nothing. You are probably referring to the shape of Pi, which for a point on the equator of the Moon looks like half wave rectified DC. Of course, the Sun is constant and its the rotation of the Moon that causes the shape of Pi. The time constant of the Moon is very short since Po is large and E is small, so the surface temperature and Po will roughly follow the Pi waveform since the surface has time to reach equilibrium with the instantaneous incident solar power.

            Jose,

            The RC model is surprisingly good at predicting both the short term (diurnal) and long term (seasonal) average response to change and correlates very well with the satellite data whether modeling aggregated 280km square pixels or slices of latitude and up to and including complete hemispheres. Moreover, given arbitrary initial conditions and the same input, the model will always converge to the same state, which is a necessary property of a zero divergence model. A single LTI can’t describe the global response, but the sum of 2 LTI’s, one per hemisphere, does so very well. The model is useless at predicting the weather, but then again predicting the weather, which is what GCMs do, doesn’t work for longer term predictions due to accumulating error from divergence. Running the model multiple times with different initial conditions and averaging the results just isolates the average divergence, which unless the many, many knobs of a GCM have highly accurate settings, the average divergence will never be close enough to zero to discriminate the effect from increasing concentrations of a trace gas. The RC model has only one knob, although tau is actually a vector. GCM’s also require real short time tics, while the RC model remains accurate over a very wide range of sample intervals and the total divergence is proportional to the number of samples.

            Yes, tau is not necessarily constant. The clear sky has the shortest tau, clouds have a longer tau, land has an even longer tau and the oceans have the longest tau and across these cases tau varies between hours and many months. The instantaneous tau at any one point on the planet can change significantly and quickly, but the average tau of the planet changes very slowly and is dependent on the average temperature, which also varies slowly. This dependence on temperature is because Po has a T^4 dependency on the surface temperature and for constant E, when T is larger, Po is larger and tau is smaller. For all intents and purposes, E is a constant and why the average temperature varies slowly since dT/dt is linearly proportional to dE/dt (i.e. 1 cal, 1 cc, 1C).

            Po is a complicated variable comprised of albedo modulated reflection and emissions. E and dE/dt are similarly complicated representing the energy stored in oceans, land, clouds and air and the ebb and flow therein. Pi is an almost perfect sinusoid about of 80 W/m^2 p-p and known to better than 1% accuracy at each square meter of the surface. While the variables are complicated, the equation is exact. If Pi is not instantaneously equal to Po, where is the extra energy going to or coming from, if not the matter of the planets climate system which is storing solar energy, thus the difference either adds to or subtracts from E, which is the total instantaneous solar energy stored by the planet.

            Regarding linearity, the IPCC relies exclusively on a linear approximation of 0.8C per W/m^2 as the sensitivity of the planet, which the LTI formulation shows to be so far off its almost comical, although the linearity over about a +/- 5C range is mostly confirmed and accounting for T^4 extends the accurate range well beyond the extents of where the current climate operates.

            CPU time is the least of my concerns, (check the date of this article).

            George

            00

          • #
            Bryan

            co2isnotevil says Byran,

            “Where is the diode in your circuit?”

            The diode would be in series.

            If you look at an isolation/time graph for a point(on the equator say) of the Moon it closely resembles an RC circuit with a diode being connected to an AC supply.
            The rotation of the Moon relative to a fixed Sun produces a similar situation to that of rectified AC smoothed by a capacitor.

            Diagrams can be found in link below.

            What can we learn from this?

            1. If the rotation period is shorter the capacitor will not charge or discharge fully.
            This produces a smoothed temperature response such as we have on Earth.

            2. The link also makes clear the importance of the ground heat flux which IPCC science ignores.

            http://www.ilovemycarbondioxide.com/pdf/Greenhouse_Effect_on_the_Moon.pdf

            00

          • #
            Jose_X

            >> The RC model is surprisingly good at predicting both the short term (diurnal) and long term (seasonal) average response to change and correlates very well with the satellite data whether modeling aggregated 280km square pixels or slices of latitude and up to and including complete hemispheres.

            You can’t be serious, right?

            Put some numbers up. The RC model gives the simplest of solutions. These do not fit the climate “surprisingly” well unless you are surprised very easily or have a very limited view of the climate.

            If you are serious and you don’t change your mind on this, I will be in for a treat whenever you write something up formally. [Didn’t you partake in the discussion at Barry Bickmore’s website on overly simplistic curve fitting or was it a different article? Did you read the 6 trillion degree misprediction article by apsmith about roy spencer’s paper?]

            >> the average divergence will never be close enough to zero to discriminate the effect from increasing concentrations of a trace gas.

            And you worked out the math for this? rebuttaled a number of papers? or is this another hypothesis you are still working on?

            If you missed the botulinum toxin reference earlier… in usage, it’s scant a trace compound, but it’s effect is well measured and very real. I’m sure many investigators, however, wrote it off after noticing it was only found in “trace” amounts.

            If you make claims like this against accepted science, you have to put a little rigor behind it.. at least if you expect to be taken seriously.

            >> predicting the weather, which is what GCMs do, doesn’t work for longer term predictions due to accumulating error from divergence.

            You mistakenly referred to climate modelling as weather modelling, or did you do so on purpose?

            >> The RC model has only one knob, although tau is actually a vector. GCM’s also require real short time tics, while the RC model remains accurate over a very wide range of sample intervals and the total divergence is proportional to the number of samples.

            [Rolling eyes] ok, so remember I’ll be waiting for the data and actual analysis on why this RC approach beats the GCM approach.

            However, a part of me is getting the feeling you are not working towards rebuttaling the GCM results at the level of scientific scholarship (vs making unsubstantiated claims on forums) if these RC model and what not really are the tools you are relying on.

            >> The instantaneous tau at any one point on the planet can change significantly and quickly, but the average tau of the planet changes very slowly and is dependent on the average temperature, which also varies slowly.

            You need to quantify these group of “slowly” if you wish to compete with the GCMs or just show that something can be considered linear.

            y=x^10 is hardly a linear function. If 1 represents one year for x, you are certainly not going to beat the GCMs on a 10 or 100 year basis (or come up with anything short of comical), yet each varies “slowly” over a suitably narrow range. So you aren’t saying much that helps explain why you think RC might do “surprisingly” well against the GCMs.

            >> This dependence on temperature is because Po has a T^4 dependency on the surface temperature

            Did you conclude using existing physical theories that the power leaving into space after interacting with the atmosphere has a 4th power dependency on surface temp just as does the surface power? [This would mean that .62 is independent of surface temp.] What is the physical basis for this?

            OK, so going on that assumption
            Po = ko*Ts^4

            >> and for constant E,

            For constant E, Pi=Po, and you don’t have an RC circuit (except in a trivial sense)!

            You should be more clear here and fix this.

            >> For all intents and purposes, E is a constant and why the average temperature varies slowly since dT/dt is linearly proportional to dE/dt (i.e. 1 cal, 1 cc, 1C).

            Again, you are being sloppy and not making mathematical sense.

            If we assume dT/dt = k2 dE/dt (as you just said) and E = k1 (from a little earlier), then T is constant as well, meaning that no matter how Pi (the sun) changes T won’t change! And that would mean, based on the first assumption above, that Po won’t change.

            So you are saying that no matter how hot the sun gets, T and E won’t change (nonsense).. and Po won’t change either, so now Pi=Po is violated (from Pi=Po+dE/dt).

            It would help me understand why you think an RC circuit performs “surprisingly” well, if you use consistent math to convey the point. Reword the “constant E” part and redevelop the rest of the argument more carefully. If you simply use math in the presentation, you probably will be more likely to catch these slip-ups.

            Again, you can write off accepted science in forums, but if you are serious about displacing (or improving) GCMs in the scientific community, you have to do better.

            >> Pi is an almost perfect sinusoid about of 80 W/m^2 p-p

            ??

            >> the IPCC relies exclusively on a linear approximation of 0.8C per W/m^2 as the sensitivity of the planet, which the LTI formulation shows to be so far off its almost comical

            I await the paper that validates this hypothesis.

            00

          • #

            Bryan,
            Yes, and the difference between the ‘predicted’ and the measured is a consequence of the time constant being finite. If you set tau to zero, dE/dt must be zero, thus Pi = Po and this is what the ‘predicted’ temperature is based on by applying SB to Po This is just more evidence that the the climate system is an LTI described by the equations,

            Pi = Po + dE/dt
            E = Po*tau
            Pi = E/tau + dE/dt

            But there is no diode in the circuit unless you model the Sun as a sinusoidal source. The Sun is constant and a better way to model this is where the rotation of the moon modulates the incident solar power. Of course, the difference between the solar signature seen on Earth and the moon is the period, where the Earth is 1 day and the Moon is 28 days. A periodic E and subsequent T to a periodic Pi is a requirement for solutions to this differential equation.

            The Earth exhibits the same kind of delay between peak incident solar and peak surface temperature. The hottest part of the day is about 4 hours after peak solar power and seasonally, the hottest/coldest part of the year follows peak solar input by 2-3 months. The Earth also shows the same exponential decay after the Sun sets where the coldest part of the night is just before sunrise.

            The Moon is simpler since there is no atmosphere and its clear that even the instantaneous behavior is exactly as expected when modeling the system with the differential equation that describes an RC circuit, where E is the voltage across the capacitor, Pi is the stimulus and the time constant, tau equals R times C, although in the case of the Moon, one only needs to specify tau and not R and C individually. The storing and releasing of energy described in the article you referenced is the dE/dt term which is also the sensible heat.

            Mars, Venus, Earth and the Moon can be quantified with the differential equation that describes an RC circuit which is why despite Jose’s objection, I stand by the veracity of this model. Chaos in the transition between states confuses many into thinking that the end states must be chaotic as well, after all, GCM end states are chaotic relative to the initial conditions, which is why they need to run them dozens of times with different initial conditions and use an average. But, since the end state is not the same independent of initial conditions, an average only isolates the average divergence of the model.

            George

            00

          • #

            Jose,

            I presented the data earlier. Here it is again.

            http://www.palisad.com/co2/plots/wbg/plots.html

            BTW, the data these plots show is usually obscured. The ‘consensus’ only likes to talk about anomalies, which is whats left when you subtract the averages shown in these plots from the raw data. The problem with anomaly analysis is that you can’t distinguish between an anomalous trend and analytical errors, in fact, anomaly analysis is frequently used to discover analytical errors.

            You do know that o*T^4 and e*o*T^4 both have a T^4 dependence? The attenuation by the atmosphere by ‘e’ is O(N) and this doesn’t change the T^4 dependence.

            When I say E is constant, this is relative to the average tau and its dependence on T. T and E do change, but do so very slowly relative to how fast the atmosphere can change, but as the instantaneous tau varies over a wide range, the average tau varies slowly. The point was in response to your objection that tau was not constant, but for the purposes of describing a response centered around a mean, we can consider a constant average as representative of this time varying value.

            Pi has an average of is 1366 W/m^2 with an 80 W/m^2 p-p sinusoid superimposed on it as a consequence of perihelion and aphelion. But since superposition applied to an LTI, we can consider the response to the average and the response to the p-p variability around that average independently and sum them. Oddly enough, many warmists believe that the p-p seasonal variability of solar power is tiny, but this is because most of the solar data presented by the consensus is normalized to 1 AU and the 80 W p-p is basically subtracted out and only the average is considered (equivalent in principle to considering an average time constant representing a time varying one).

            And here is the data which demonstrates that the sensitivity is far less than the IPCC claims.

            http://www.palisad.com/co2/sens

            Again, all of this data came from Nasa, I’m just presenting it in an informative manner. The raw data these results came from is about 1 TB of satellite images of 30km pixels covering the entire planet. Each month of data is the average of about 500,000 individual satellite images spanning 3 decades.

            Pay specific attention to the plot labeled ‘surface temperature vs. post albedo input power in’. This shows that the response is to +/- 3.7 W/m^2 of incremental post albedo incident power results in only a .71 C change in temperature. Forcing, per the IPCC definition, is nearly identical to a change in the post albedo power input.

            No possible amount of feedback can boost this to 3C.

            Another interesting one is ‘surface power vs. water column’. This is interesting because the water column is roughly proportional to atmospheric absorption by water vapor which illustrates the net effect of changes to atmospheric absorption on surface temperature, as would occur from increasing CO2.

            Sorry that I’m not being as precise as you need. I guess I’m just assuming that you have a working understanding of basic concepts and that I don’t need to spell them out for you. But based on some of your objections, there do seem to be significant gaps in your understanding, specifically with regard to the differences between average behavior and instantaneous behavior in the context of system analysis. Although, this could just be that you are grasping at straws trying to find a way to discount what I’m saying because to understand how the climate really works will require you to give up a core belief regarding how much you think CO2 influences the climate and you are not quite ready to handle the truth.

            I’ll only be on-line sporadically until the new year skiing all this great ‘global warming’ we have going on in the Sierras.

            George

            00

          • #
            Jose_X

            In replying again to this discussion, I’m making what I hope will be one of very few exceptions. [I don’t intend ill will, btw.]

            >> You do know that o*T^4 and e*o*T^4 both have a T^4 dependence?

            Yes, that was unfair of me. My skeptical side was criticizing the essence of the formula since I anticipate rising temps would change weather, clouds, etc, sufficiently to change emissivity “constant”.

            >> The point was in response to your objection that tau was not constant, but for the purposes of describing a response centered around a mean

            One should probably try to be mathematically precise in quantifying the bounds of a model rather than just claiming without accurate mathematical analysis that the model being used at the moment for the occasion is a superior/sufficient one. As another example, I don’t see you “derive” why an LTI model is sufficient. Engineers want to engineer LTI systems where possible, but when you have to take what is given, nonlinear analysis is sometimes the only reasonable option. I don’t know if you believe that, as you have not tried to quantify error bounds on any of these. I also reject the notion that a simple first order linear circuit is superior to what GCMs use (or sufficient) for the purposes of trying to forecast climate evolution, especially since the computer programs are already written and are cheap to use for the simple approximate cases.

            >> Oddly enough, many warmists believe that the p-p seasonal variability of solar power is tiny, but this is because most of the solar data presented by the consensus is normalized to 1 AU and the 80 W p-p is basically subtracted out and only the average is considered

            I’m not sure what you are referring to here. I doubt you know enough about the software running the GCMs to make such a statement; however:

            Oddly enough, you were just arguing (without any math) that “for the purposes of describing a response centered around a mean, we can consider a constant average as representative of this time varying value”.

            So it’s OK when you do it (without mathematical analysis, might I add) but not when “warmists” allegedly do it?

            >> And here is the data which demonstrates that the sensitivity is far less than the IPCC claims. …Again, all of this data came from Nasa

            So NASA supports climate change and greehouse effect. They collect this data and even designed the instruments to make that possible. Yet they are clueless warmists who don’t even know their own data.. but you know better!!! and without mathematical analysis of your models!!! [I did not and don’t anticipate going over that data, btw.]

            >> skiing all this great ‘global warming’

            If you were serious (which I doubt you are), that statement would show ignorance of the issues and of the predictions (especially for 2012 Winter high on a mountain).

            Oh, wait, 2012 was one of the coldest years on records!!! I forgot!!!

            00

  • #

    I have belatedly moved #500 comments to an overflow thread to make this page load faster. But it means comment numbers above in discussions will be 500 too high.

    00

  • #

    Sorry, the comment flow here may be fatally wounded by me moving comments. Some comments here are orphans made recently to older comments which moved to the new overflow thread. Apologies. Jo

    00

  • #

    With 50 years of physics behind me I can tell you quite plainly that the Second Law applies to a single process, not to two-way heat flow in separate processes which can be radiation one way and then conduction another way, even at a much later time.

    A siphon works because entropy increases in a single process. But cut the hose at the top and you make it two processes. And, guess what, it doesn’t work any longer. You can’t expect water to run uphill into the town’s water tank, just because it will later flow further downhill to the houses.

    My publication linked from my site clarifies the issue and explains what really happens. It is linked from my site http://climate-change-theory.com

    00

    • #
      Jose_X

      I hope I don’t sound disrespectful. I started reading the paper you linked (and I may finish it). You mention parallel plates moving towards each other in temperature until each is at the same temperature. The paper mentions Johnson (presumably leveraging his unorthodox statistical mechanics). Why should I focus on this paper when it possibly involves me reading a bunch of other material which is not a part of accepted physics and can have any number of flaws with it simply by the nature of the beast? If I was an expert in the area, I might look forward to that, but in my position, why should I take that cost?

      I have mentioned this numerous times, you probably have to gain acceptance with these other theories/models at a large scale before people will give the material, as concerns climate science, significant attention. There is information overload and too much that would have to be carefully re-evaluated to see if the theory agrees with observations, etc. For every *revolutionary* theory that is correct there might be hundreds or more that are incorrect or of lower quality.

      Two plates at different temperatures, whether via radiation, conduction, or any other form of heat exchange (including adjusting various chemical or other potentials), have well known formulas that approximate the evolution of such heat flows in time. Each plate is drawn in the direction of the other, getting closer in temp as a function of time. There isn’t a shortage of equations. The well described the observed effect. What does the new statistical mechanics or any other theory offer that is new or improved? The paper almost appears to suggest that this problem cannot be solved at all or efficiently with accepted theories.

      As for the blackbody being at the edge of the planet+atmosphere, it’s not that simple because a lot of the energy reaches to the ground bypassing the atmosphere so the atmosphere is largely/partly warmed from the bottom up.

      As for conduction into the earth, oceans, etc, that happens during the day ..and the opposite effect happens at night (as the outermost layers keep radiating, they “draw” heat from deeper below as well as from the atmosphere. I think this explains the temp profile (including ocean skin and mixed layer change in temperature over a 24hr period) well. And there is also lateral heat flow of course.

      00

  • #

    With 50 years of physics behind me I can tell you quite plainly that the Second Law applies to a single process, not to two-way heat flow in separate processes which can be radiation one way and then conduction another way, even at a much later time.

    A siphon works because entropy increases in a single process. But cut the hose at the top and you make it two processes. And, guess what, it doesn’t work any longer. You can’t expect water to run uphill into the town’s water tank, just because it will later flow further downhill to the houses.

    My publication linked from my site clarifies the issue and explains what really happens. It is linked from my site.

    00

    • #
      Jose_X

      In what way do you think “the Second Law applies to a single process, not to two-way heat flow in separate processes” contradicts anything in climate science? Do you have specifics? [Is there a page number in your paper that covers a specific?]

      00

  • #
    Jose_X

    D Cotton, I saw the example you gave of skepticalscience deleting your post. FWIW…

    First, I do think they are very aggressive when it comes to moderation; however, I anticipate that most of the time when they remove non-profane comments it is when people do not answer the question posed to them and they keep repeating statements as if they were fact. [In my most recent case, they were questioning the fact I was questioning and invoking a reference here or there to an unorthodox webpage (for reference purposes.. I don’t see why I would not.. I wasn’t invoking the science as fact but wanted to make it easier to address a question on that website).] If you can’t rely on accepted science, you probably won’t have a lot of success posting on that website.

    With some exceptions, (eg) Einstein addressed problems that were unexplained during his time and used traditional physics to explain it. In any case, climate scientists of the time probably wouldn’t pay much attention until he had a track record. Once you have a track record or your tools are accepted, people will pay a lot more attention.. whether they agree or not.. even if you throw new curve balls frequently.. and will continue to do so as long as you bat (or pitch) for a decent average.

    00

  • #
    Jose_X

    Doug C, you stated on pg 4 of the paper, Radiated Energy and the Second Law of Thermodynamics (C12):

    > If thermal energy could transfer from cold to hot, then what happens when radiation from the atmosphere penetrates some small distance into the ocean waters? Does it warm the water which then rises to the surface by convection and causes more evaporation? Such a scenario can not be right and the only feasible explanation is that, even though there may be two-way radiated energy transfer, the radiation from the cooler body to the warmer one cannot be absorbed and converted to thermal energy when it reaches the warmer body.

    I don’t know what you mean by from cold to hot, but I’ll assume you object to radiation from a colder body having an impact on a warmer body, specifically, you object to radiation from the atmosphere imparting “energy” to the oceans. Note, I didn’t say imparting “heat”. Consistent with this flow of heat is that some radiation does go from colder to warmer.. but we don’t have to agree on this point. The quote above assumes this is what is happening, so let’s see if a contradiction is reached. [Note, a failure to find a contradiction won’t prove the statement correct; however, finding a contradiction is important if you want to show there is a problem.]

    Greater depths get less radiation. A buried element of volume would not generally move up ahead of a higher volume since that higher volume has been acquiring more radiation and is likely to be at a higher temperature already. Why is this true?

    Radiation at some delta path length will be absorbed as a fixed percentage of the quantity of radiation. This means the largest amount is absorbed at the nearest point (the ocean’s highest layer) because at every lower point there is less radiation penetrating (since some got absorbed already) and a constant fraction of a diminishing quantity is also a diminishing quantity.

    So since the answer is no, it does not rise “to the surface by convection and causes more evaporation”, the solution offered in the remainder of the paragraph is apparently replying to a problem that hasn’t been shown to exist.

    Does the rest of the paper answer a different question?

    Let me add. There are many competing paths to energy transfer. Energy has to be conserved. If something absorbs heat, some portion of that will lead to evaporation, another portion to radiation, another portion to conduction, etc. Each such process is described in time by some mathematics equation or other. A solution to all such equations would be a solution to the behavior seen. Anyway, I am being vague.

    If you find a specific case where math leveraging a certain model leads to absurd results, that is the kind of result that would help convince people to pay more attention (they will try to correct you) and possibly recognize that model has problems.

    00

    • #
      Jose_X

      I haven’t studied the ocean and heat flows too much, so what I mentioned of radiation absorption doesn’t necessarily imply that we wouldn’t have convection at the same time. The hotter radiates more and the hotter tends to be nearer the top. Meanwhile that nearer the top may get less overall radiation back into it (I’m not sure) than the lower layers if we assume the atmosphere gives the top layer less radiation from above than the top layer gives to the next layer below it radiation from above. In this case, the top would cool faster than the next layer down (or at least at some time of the day/night.. since during the hottest part of a sunny day, the sun offers a lot of radiation, more than the ocean layers do to each other).

      I haven’t modeled this and looked at all the math in order to know how the water behaves (not to mention wind and many other variables).

      00

  • #
  • #
    blouis79

    (Response to Jose_x #501 in the overflow thread http://joannenova.com.au/2011/05/overflow-thread-for-greenhouse-gases-dont-break-2nd-law/)

    You mean “greenhouse gases magically delay heat flow from earth to space at night and don’t have any impact of heat flow from sun to space during the day” or some such thing that climate scientists are into?

    Show me experimental proof of any of:
    a. thermalization of IR by IR absorbing/emitting gases
    b. magnitude of time delay of IR passing through IR absorbing/emitting gases compared to IR transparent gases (hours, minutes, seconds, msec, microseonds, nanoseconds)
    c. some other physical mechanism by which climate scientists think the greenhouse works

    Show me experimental proof that changing albedo (or material composition) has any effect at all on equilibrium temperature when exposing the material to alternating heating and cooling cycles.

    What goes up must come down. If it heats faster, it will cool faster. If it heats slower, it will cool slower.

    Hand wave all you want. I want to see a real laboratory experiment.

    The null hypothesis: changing albedo or material composition (without the addition of work) will make no difference.

    On the “snowball earth” hypothesis: if we consider the case of an earth with no sun and no ability to radiate. In such conditions, the earth would be isothermal. Isothermal earth would not be a snowball, it would be rather hotter because of the molten core. It is the ability to radiate and lose heat to space (there is no other mode of heat loss to space of note) that results in the real earth surface including a sun warming it being cooler than an isothermal radiatively insulated earth with no sun.

    Mainstream climate science operates in scientific fairyland. Their major defence is “the atmosphere is so complex we can’t possibly perform any valid experiments on parts of it”.

    Jo, you are still a believer that GHG’s don’t violate second law – do you have answers to my quetions??? What I really want to see is a real physicist with a real physics lab and some real experiments.

    00

    • #
      blouis79

      (Not sure how but my post #149 landed above KinkyKeith #150 when I thought I’d posted at the bottom.)

      Some other statements of laboratory experiments which will help the debate:
      1. Do IR absorbing/emitting gases thermalise IR radiation outside of an IR reflecting measurement chamber.
      2. Does *any* known physical property of any material change equilibrium mean temperature when subject to alternate heating and cooling effects over day-length cycles.
      3. Does presence of IR absorbing/emitting gases accelerate or retard thermal mixing of gas mixtures of different temperatures.

      00

    • #
      Jose_X

      blouis79
      >> You mean “greenhouse gases magically delay heat flow from earth to space at night and don’t have any impact of heat flow from sun to space during the day”

      Do you know what longwave and shortwave refer to in the context of greenhouse effect (which is different than in amateur radio)? Quick wikipedia lookup will give the basics. Basically, the radiation from the sun to the earth happens at a distribution of wavelengths that allow a large fraction to pass through the gaseous atmosphere with only some impediment and then into mostly the tippity top layer of the solid earth and also into the liquid ocean before being absorbed. In contrast, the distribution of wavelengths of the radiated energy originating from the earth is at a range that interacts strongly with various “greenhouse gases” in the atmosphere. So the atmosphere interferes more strongly in one direction (upward, from earth to space) than in the other (downward, from sun/space to earth). This difference forms part of the puzzle.

      An analogy might be of a two-way mirror. In one direction, you get a different effect than coming at it from the other. This analogy breaks down of course because the earth system doesn’t much have a preferred direction but the result is similar because the atmosphere is being radiated differently from above than from below.

      A quick overview of this “distribution of wavelengths” can come from googling “blackbody radiation” for example.

      >> If it heats faster, it will cool faster. If it heats slower, it will cool slower.

      Try this. Put on a heavy coat vs a light coat. You will notice the difference.

      The key reason why slowing down heat loss is important to final equilibrium temperature is that we don’t just heat up the body and then wait and wait.. until the temp comes near absolute zero. Instead, the sun keeps adding energy at a fairly constant rate. So if the cooling is slower, less energy will have dissipated before a new boost arrives. [I’m using informal descriptions]

      An analogy would be to break into discrete 10 second steps the heating of a thermometer and its cooling before another round happens. If the heating is from a 100 C hot iron and the cooling is from 0 C cold ice cube, we will get an average value on the thermometer that is lower than when we replace the ice cube with a heavy cloth “insulated” ice cube, also at 0 C, but which draws in heat from the thermometer more slowly than the direct ice cube contact would.

      Another analogy is an oven. If you add energy (electricity passing through heating coils), then the oven will reach equilibrium at a high value rather than cool down to the room temperature. How high the value depends on how good is the insulation. Eg, with open door, the equilibrium temp will be lower than with closed door.

      Yes, convection is different than radiation, but the “insulation” result exists in each.

      >> Show me experimental proof of any of:
      >> a. thermalization of IR by IR absorbing/emitting gases

      Not sure what you mean. Can you define “thermolization of IR”?

      A model used sometimes is that radiation photons (quanta of energy) at one wavelength have a different tendency to be absorbed by a particular media/matter than do photons at a different wavelength. You may want to google “line spectra”.

      I won’t address some of your other comments at this point because I’m not sure what you believe or not. We can’t describe experiments that rely on the above if you don’t believe it. We need a common basis.

      Also, I will only spend limited time on this subject because I’m busy and don’t feel any crucial need to resolve anything for myself. If you feel that need, consider taking physics courses beyond whatever you have taken. In particular, look on through Quantum Electrodynamics. The physics community has performed many experiments over the years and you would as well in a formal university course. These people get paid to spend all of that time with you. Now, if you want to send a bit of money my way, I’ll try to spare extra time even if I otherwise wouldn’t. Anyway, keep that in mind, as I don’t expect to spend as much time here as I did in the past.

      00

  • #
    Greg House

    Jo says: “I disagree. The 2nd Law of Thermodynamics applies to net flows of heat, not to each individual photon, and it does not prevent some heat flowing from a cooler body to a warm one.

    Imagine three blocks of metal side by side. They are 11°C, 10°C, and 9°C. Think about what happens to the photons coming off the atoms in the middle of the medium temperature block between the other two. If heat never flows from cooler blocks to warmer blocks, all those photons have to go “right“, and not ever go “left”, because they “know” that way is towards a cooler block? (How would they?!)

    Guest Post by Michael Hammer: “This is wrong and is based on an incorrect interpretation of the second law. The second law does not say a cold object cannot pass heat to a warmer object, it states that NET heat flow is always from warmer to colder.”
    ========================================================

    First of all, the 2nd Law does not say “NET”, it is what Michael Hammer says, but not the 2nd Law. Well, problem solved, next, please (kidding).

    Michael Hammer probably refers to the Clausius statement (you can find it in Wikipedia in the article about the 2nd Law): “Heat can never pass from a colder to a warmer body without some other change, connected therewith, occurring at the same time”. And they add: “This may be restated as “No process is possible whose sole result is the transfer of heat from a body of lower temperature to a body of higher temperature. Heat cannot spontaneously flow from cold regions to hot regions without external work being performed on the system,…”

    Jo, do you see the word “NET” in the quotes? I am afraid, you can only find this “net” in the warmists literature.

    OK, let us talk about your photons, travelling in all directions. It is very important to be strictly logical here and let us forget for a while the conclusion you have already made. There are 3 other possibilities: 1) the IR from a warmer body enters a colder body and this prevents the photons of the colder body from travelling in the opposite direction to the warmer body (it is like people streaming into your office and you can not get out at the same time, because they are stronger); 2) the photons do leave the colder body in the direction warmer body, but can not get into the warmer body, because they can not overpower the photons streaming out of the warmer body; and 3) the photons from the colder body do successfully enter the warmer body, but this produces zero effect on the warmer body’s temperature.

    So, as you can see, although theoretically that “net” thing is possible, there are other possibilities as well. How can we know then, what is right here? I would say: only by conducting experiments. This is how physics works. The Laws of Thermodynamics, by the way, were not given us by Got like 10 commandments, they were formulated by scientists on the basis of experiments.

    I personally asked warmists many times on various blogs to present a real scientific experiment proving that “net” thing. They presented ZERO. The told me a lot of things, like about “thought experiments”, blankets etc, but not a single experiment proving that IR from a colder body can slow down cooling of a warmer body was presented. So, at best, this “net” thing is a science fiction with no evidence, but no way it is a scientific fact.

    00

    • #
      Jose_X

      >> the 2nd Law does not say “NET”

      “Net” is implicit in the definition of heat.

      In thermo, for a given body, heat is defined as a change of its internal energy after accounting for the work done on it (or by it). This definition applies to an arbitrarily sized body with arbitrary internal components. If we consider a body composed of two (smaller) component bodies and assume that these two component bodies only exchange energy with themselves and have no work done on them, then the definition of heat implies that the “aggregate body” has no heat flow even while each of the internal components does gain (or lose) heat. So heat gain (loss) for the “aggregate body” would be the sum of the heat gain (loss) of each of the two subbodies, that is, heat for the aggregate body is equal to the “net” heat flow of its internal components.

      Note that this definition doesn’t hypothesize that any subcomponent of a body can be giving off heat while another subcomponent losses, each only to the other, but the point is that if we do make that assumption then “heat” would be the same as “net energy flow”.

      Now, it turns out other theories beyond the macro thermo theory (eg, Quantum Electrodynamics) do hypothesize the mentioned assumption, that within any given delta instant of time, two (sub)bodies have a probability of exchanging energy solely with themselves.

      We can even appeal to Wikipedia (its “Heat” entry) for a use of “net flow of energy” as a surrogate to “heat”:
      > Heat in physics is defined as energy transferred by thermal interactions. Heat flows spontaneously from systems of higher temperature to systems of lower temperature. When two systems come into thermal contact, they exchange energy through the microscopic interactions of their particles. When the systems are at different temperatures, the result is a spontaneous net flow of energy from higher to lower temperature, so that the higher temperature decreases and the lower increases. This continues until the temperatures are equal. At that point the net flow of energy is zero, and the systems are said to be in thermal equilibrium.

      00

      • #
        Jose_X

        I have to clarify that the sub bodies don’t have to give energy exclusively to each other. Heat for the aggregate body would still be the “net” flows across all subcomponents.

        This idea embeds the principle of superposition (and note that the gain/loss of energy is a linear relationship).

        Also, remember that while we can come up with different interpretations of physics (superposition, etc), the simplest explanation that meets experimental evidence is the one that will be taught in textbooks and used by those practicing in order to arrive at results.

        In short, we can’t prove any theory to be correct without doubt. We can only (a) show that a given set of experiments is consistent with some theory or (b) disprove a theory by relying on a given set of experiments.

        I’ve argued that the idea of “net” is consistent with how we treat energy of a body as the sum of the energy of the subbodies. Now, my challenge to you is to find a contradiction or else to give an alternative explanation that is essentially as simple as that one.

        In any case, your [Greg House’s] beef is with physics that has been established and followed independent of the field of climate science.

        00

        • #
          Greg House

          Jose_X says: “…we can’t prove any theory to be correct without doubt. We can only (a) show that a given set of experiments is consistent with some theory or (b) disprove a theory by relying on a given set of experiments.

          I’ve argued that the idea of “net” is consistent with how we treat energy of a body as the sum of the energy of the subbodies. Now, my challenge to you is to find a contradiction or else to give an alternative explanation…”
          ===========================================

          I see, nothing can be proved (ROFL). And until I disprove your point, it is correct, right? Let us say, you are accused of committing a crime and therefore considered guilty until you find a “contradiction”.

          So, you are talking about “set of experiments”, then just present valid links to scientific experiments proving/confirming this notion of “net”. From my own experience with warmists I can temporarily conclude that there is none.

          00

          • #
            Jose_X

            Well, if your theory is that I am guilty just because I exist, then the implication might be that everyone who exists is also guilty. If you give a reason, then we can look at other cases of other characters and see if other people, by that definition, might not be guilty.

            Let me ask you, was Newtonian mechanics proven correct? But it isn’t correct as evidenced by quantum mechanics and relativity. Are the latter 2 correct? It might only be a matter of time until our best theory is found to fail in some way or where we have to add qualifications/assumptions that were not considered before.

            Some differences between the standard of proof in science vs in a court of law: The standard for many sciences is higher than for criminal cases, and, in science, the case is never closed. Science represents the best understanding we have of some facet of nature. Court cases require finality (a definite and usually relatively short time frame). Many sciences question many things that people take for granted or don’t ever concern themselves with in any court case.

            00

      • #
        Greg House

        Jose_X says: ““Net” is implicit in the definition of heat. […] We can even appeal to Wikipedia (its “Heat” entry) for a use of “net flow of energy”…
        =================================================

        This is misleading and simply not true. We can easily find definitions of “heat” on the internet, where this “net” thing is not even mentioned:

        1.http://www.chemicool.com/definition/heat.html

        Definition of Heat
        A form of energy that flows between two samples of matter because of their differences in temperature.

        2.http://www.chegg.com/homework-help/definitions/heat-5

        In thermodynamics, heat is the same as heat transfer, which is the movement of thermal energy from one thing to another thing of lower temperature. A difference in temperature is required for heat to exist.

        3.http://science.yourdictionary.com/heat

        heat science definition

        Internal energy that is transferred to a physical system from outside the system because of a difference in temperature and does not result in work done by the system on its surroundings. Absorption of energy by a system as heat takes the form of increased kinetic energy of its molecules, thus resulting in an increase in temperature of the system. Heat is transferred from one system to another in the direction of higher to lower temperature.

        4.http://chemistry.about.com/od/chemistryglossary/a/heatdef.htm

        By Anne Marie Helmenstine, Ph.D., About.com Guide

        Heat Definition: Heat is the form of energy that flows between two samples of matter due to their difference in temperature.

        5.http://www.physicsphenomena.com/Physicsdictionary.htm#letH

        Heat: quantity of energy transferred from one object to another because of a difference in temperature.

        So, Jose, you need yet to prove your “net” scientifically. Not just “explain”, how it works according to your imagination.

        00

        • #
          Jose_X

          You might be confusing heat with energy.

          Heat is a net transfer of energy; however, energy can go back and forth among bodies without there being heat (or heat transfer) if these migrations of energy among the subcomponents balance out.

          A cold body can impart energy to a warmer body and the warmer body impart a little more back to the cold body. The *net* change in energy would be from the warm to the cold and that is the direction of heat flow because the net change in energy defines which way heat flowed, by definition.

          Heat flow equals net energy flow by definition. [Where an energy flow can be measured as work or as an internal change in some form of energy.]

          00

          • #
            Greg House

            Jose_X says: “You might be confusing heat with energy. Heat is …”
            ==========================================

            What you are doing now is not far away from lying.

            You specifically referred to “definition of heat” from Wikipedia containing that “net” word. Then I gave you 5 other definitions of heat from various sources on the internet that do not contain the “net” word, thus debunking your assertion about “”net” being implicit in the definition of heat”.

            Now you are telling me that I confuse heat with something else. This is not nice, what you are doing. Actually, this looks very much dishonest to me.

            00

          • #
            Jose_X

            What is the definition of a hamster? A hamster is a creature with a very low IQ by human standards. This fact is implicit in the meaning of hamster (you won’t find an exception), yet you likely will not come across that phrase (IQ) in any short definition you are likely to find online of hamster.

            You can read a whole book to try and understand “heat” and its consequences. Finding a short sentence definition will not explain all the things it really means to be heat. If it did, people would not need to take whole semesters of courses to solve heat problems (like the greenhouse effect).

            Also, implicit means you won’t necessarily see it mentioned explicitly, yet that wouldn’t make it less true.

            I said you may have confused heat with energy because you seem to be thinking that just because heat must move from warm to cold (ie, if “work” is out of the question) then that would also mean photons can’t move from cold to warm.

            Each photon carries energy, and a photon released from a cold body towards a warm body could transfer that energy quantum. Regardless, heat would only exist as seen by the larger body under study (ie, heat transfer on the larger body would only exist) if there was a *net* gain or loss of energy among the parts releasing photons. Thus similar photons moving in opposite directions from the opposite bodies to the other would not mean there was a heat transfer, but we would know there was an energy transfer at the microscopic level.

            00

  • #
    Greg House

    “From Jo
    Here’s why talk about whether its convection or conduction vs radiative cooling is irrelevant
    The blanket analogy is perfect because we are discussing whether it’s possible for a cooler item to induce (somehow) an increase in temperature of a warmer item. NOTE: The cooler item has no internal heat source, but the warmer item (Earth or body) does have energy added in. The method of heat transfer is irrelevant. (Talk of two ice cubes misses the point unless one cube is heated by the sun, or burns fat. )

    The point is that YES, obviously in the real world, blankets keep us warm. Pink batts “lift the temperature of your home in cold weather”. They don’t do it by supplying energy, they do it by blocking energy loss. The cooler item is not supplying a single new joule of energy, but there another mechanism of increasing an objects temperature. It’s called insulation.”
    ==================================================

    Sorry, but the blanket analogy is not perfect, it is false. A blanket keeps us warmer (or colder, depending on the air temperature!) by preventing convection. The so called “greenhouse gases” are not said to prevent convection. And blankets are not generally said to warm by back radiation (although one warmist on WUWT did actually say that).

    These 2 things, blanket and “greenhouse gas”, are simply unrelated, they have nothing to do with each other.

    I could equally make an illogical statement like that: “Blanket warms by preventing convection, “greenhouse gas” does not prevent convection, hence “greenhouse gas” does not warm”.

    So, if clever people tell you about “greenhouse gas” warming by back radiation, do not buy their “analogies”, just ask them to prove it experimentally.

    01

    • #
      Jose_X

      >> just ask them to prove it experimentally.

      Remember, you can’t prove a theory. Instead you can find a theory that offers an explanation for an experiment (of possibly infinite many explanations) while at the same time also failing to find an example of an experiment that appears to disprove that theory.

      Or, if you want to try to prove a theory, prove that we need oxygen to live.

      >> The so called “greenhouse gases” are not said to prevent convection.

      Convection as a way to remove heat away from the planet is impossible (to first approximation) because there is no air in outer space, so, trivially, the atmosphere prevents convection from cooling the planet.

      >> And blankets are not generally said to warm by back radiation

      Radiation is given off by the blanket. That is accepted physics. A warm blanket removed from a space shuttle into cold space (and within the shadow of the sun’s rays) will continue to release heat for a little while until it has fully cooled. I have not carried that out, but NASA likely has (or easily could), and they are a supporter of climate science, which includes a belief in that standard physical theory.

      As another example, it has long been accepted that objects on earth emit infrared radiation. It’s this emission that allows infrared cameras to work or else we would just see blurriness as created by convection.

      It is true that convection dominates, but an accurate representation of the temperature of the blanket would include a component for convection as well as one for radiation and likely one for conduction.

      The “back” is just an adjective, used at ground level, to suggest that the radiation can go in any direction, including back towards the planet (or the measuring instrument). Note that near the ground, about half the time such radiation will be in the “downward” direction. [High into the atmosphere, then the planet’s direction will represent less than half of the total solid angle (of the total surface area around a virtual unit sphere centered at the molecule in question).]

      00

      • #
        Greg House

        Jose_X says: “…so, trivially, the atmosphere prevents convection from cooling the planet.”
        ==============================================

        Sorry, but I have rarely read such a senseless statement. Please, do me a favour and look up the definition of convection somewhere.

        01

        • #
          Jose_X

          You said: “The so called “greenhouse gases” are not said to prevent convection. And blankets are not generally said to warm by back radiation”

          So the analogy is between the atmosphere and a blanket. The analogy as applied to convection would be that a blanket (the atmosphere) prevents the outside air (space) from cooling by convection the person’s body (the earth).

          In other words, if we compare the atmosphere to a blanket, then the atmosphere does in fact (just as I stated) trivially keep outer space from cooling the planet by convection, and this parallels how a blanket keeps the outside air from cooling the person by convection. The difference between the atmosphere and a blanket (wrt convection) is that if we remove the blanket, then convection takes over, but if we remove the atmosphere convection still does not happen as we allow outer space to “circulate” around the solid planet.

          00

          • #
            Greg House

            Jose_X says: “then the atmosphere does in fact (just as I stated) trivially keep outer space from cooling the planet by convection,”
            ========================================
            Jose, your statement does not make any sense at all.

            Again, please, do me a favour and look up the definition of convection somewhere.

            00

          • #
            Jose_X

            To have convection you must have particles outside the body.

            If the body is the planet without an atmosphere, then by definition there are no particles to be able to convect heat away from the planet, assuming to first order that outer space doesn’t have particles as would be required to meet the definition.

            Thus, outer space cannot cool the solid “naked” planet by convection. [Naked, meaning without an atmosphere or insulation, aka without a “blanket”.]

            This is the parallel to the blanket analogy.

            1: The *blanket* prevents convection by the *outside* acting on the *naked body* inside.

            Or, using the planet example,

            2: The *atmosphere blanket* prevents convection by the *outside outer space* acting on the *naked planet body* inside.

            00

  • #

    It is very interesting to read the many comments on this thread which seeks to analyse the behaviour of a green house gas.

    As explained by Jo and many here, for any “body” (solid, liquid or gas) the basic “thermodynamics” of absorption and radiation of heat energy is exactly the same. The molecules, made up of electrical charges, absorb energy from the electric field of the incoming radiation. This energy is converted “electromechanically” into vibrational kinetic) energy of the individual molecules, which is what is correctly defined as “heat” – (there is sometimes discussion about this term as to what form the energy takes at the point where it is defined as heat, rather than radiation, kinetic energy etc., but this is of no relevance here.)

    When the body becomes “hot” which is really saying it is above its surrounding temperature, since all bodies at a temperature above absolute 0 K (-273 C) can be regarded as hot# relative to their cooler surrounds, the energetic, moving (vibrating) molecules, collide with their partners and the energy may be again transformed into the “internal” form of energy, the same as it was just after absorption from the radiation. The molecule will now again lose that energy by way of one of two things – it can again convert that energy back to kinetic energy by collision with its partners, or it can radiate it in any direction, including out from the body – remember this body may be solid, liquid or gas.

    The average rate at which this body loses energy by its own radiation depends on its temperature sand is totally independent of any bodies near it, hot or cold. The thermodynamic laws apply to NET transfer, not absolute transfer.
    However, if the heated body, e.g. a mass of oxygen, is not sufficiently hot to convert the energy available from a significant number of collisions into an allowed (quantum) level of internal energy, because of the particular electronic structure of the molecules in the body, then the heated body will NOT radiate. This is the case for Oxygen, Nitrogen and Argon in our atmosphere. The only gases which can absorb significant amounts of radiation from the earth and also can re-radiate that heat both out to space and back to earth, are the green house gases.

    The Green House Gases (GHGs) thus can help to retain the heat which is radiated from the ‘solar heated’ surface of the earth. With regard to the blanket analogy, this is actually quite good, because the heat from our body warms the blanket by both contact, radiation from our body (or our pyjamas!) and convection of the air between non-contact parts of our body and the blanket. Thus the blanket warms. Some of this heat is conducted through the blanket, which is why in the winter time, we add blankets to our bed and wear warmer jarmies. Some of the energy is radiated back to us from the inside surface of the blanket. (Think of going to bed late at night in winter when there has been no heating in the house – the bed is “cold” until we warm it up with our bodies.)

    Thus the green house effect (GHE) is very similar to the effect of a “blanket”. The argument of convection is not relevant here, as we are talking about a “whole of atmosphere” effect and convection is just an ‘add on’ within that whole of atmosphere.

    However, there is still one difference between the GHGs and the blanket. That is the selective “spectra” of the GHGs. Radiation from our body in bed (or elsewhere!) and from the surface of the earth, covers a wide range of wavelengths (from ~0 m to 10^-5 m – or 0.0 to 0.00001 m) . Our blanket in general absorbs all of those wavelengths. The GHGs only absorb a relatively narrow range and for CO2 the main part of its absorption is from 13.5 x 10^-6 to 15.5 x 10^-6 m. However, this absorption is sufficient to absorb very roughly 1/5th of the energy radiated by the earth. (Mind you, effects which appear to be ignored by those supporting the view of the IPCC, include the heat transferred to the atmosphere by the evaporation of water over the sea and lakes, which accounts for an estimated 60% of all of the energy from the sun, heat transferred to the atmosphere by wind over the surface (convective cooling) estimated at 20% and a further 20% by carbon dioxide.

    (Consider what might happen if there were no green house gases, no CO2, no water vapour. The air would still heat at the surface probably absorbing about 50% of the heat from the sun, the rest being radiated to space. But there are now no Green House Gases to radiate the energy from the warmed atmosphere to space! Does the earth worm or cool?)

    Thus CO2 contributes something to the warming of the earth and there is no doubt whatsoever about this fact. However, it is not this initial warming which concerns us. Warming, not that different from the present, would occur at a concentration of atmospheric CO2 only about 1/10th of the carbon dioxide in the atmosphere that there is today.

    The debate which is seldom clearly defined, is NOT about the Green House Effect (GHE), which involves the main part of the CO2 absorption/radiation spectrum, but about the Enhanced Green House Effect (EGHE) which involves the much weaker parts of the CO2 spectrum and which will increase very slightly the energy absorbed into the atmosphere if the concentration of carbon dioxide in the atmosphere is increased. It will increase the energy absorbed and change the relative temperature of different layers of air in the troposphere, but may not significantly raise the temperature of the earth.

    Initially, this is a not matter for climatology, but for spectroscopy. In 1985, Jack Barrett, Professor of Chemistry at Imperial College London, presented a paper to an environmental conference in Europe showing that the spectra of CO2 did NOT lend itself to causing the increase in atmospheric warming proposed by these environmentalists/climatologists. There was an uproar and delegations to Imperial College, to its eternal shame, caused restrictions on research funding and freedom to publish under the banner of Imperial to be placed on Jack and he was effectively drummed out of his job! So much for academic freedom! There are other parallel cases which have happened in Australia and no doubt elsewhere. Few “skeptics” in academia today will put their head above the parapet while developers of the AGW site “Skeptical Science” are lauded (sadly, by my own Alma Mater – with “A place of Light, Liberty and Learning” carved upon its main and oldest building!) and given positions which appear to have been created just for them!

    A fundamental and easily understood part of the spectroscopy which appears to be totally overlooked by any one discussing the effect of the increase in CO2, and in particular the well known James Hansen whose diagram of what and where radiation goes, from and to the earth, is that as carbon dioxide increases, more radiation is absorbed at a lower level, but the increased CO2 also inhibits the return of radiation to the earth, so the EGHE is all but nullified. For all of these reasons, it is not TOTALLY clear whether at some point, increased CO2 might not cool the earth!!! I believe that is extremely unlikely and it certainly would not be very significant, but it should not be totally ruled out. The spectroscopy plus some good sound atmospheric physics will one day give us the answer.

    At present I believe that increased carbon dioxide will cause very small increases in global temperature. But I also believe that the application of Le Chetalier’s Principle will apply and feed backs must be negative, (otherwise the earth would already be burnt to a cinder) and changes because of atmospheric composition must be much less than the effects of extraterrestrial forces from the other planets and the sun. If the drivers of the Milankovitch cycles can induce an Ice Age and return us to a Holocene, then minor changes in these mechanical perturbations wqhich are ever present with us, must surely be expected to drive the climate into these much smaller fluctuations than can be caused by CO2.

    #In outer space where T ~ 0 K, a body at 10 K (-263 C) is “hot”!

    01

    • #
      Jose_X

      [Sorry, if the info in this comment is old news to you. While I really liked the description you gave in the first half of your comment, I can’t tell from the bottom half what background you have.]

      >> It will increase the energy absorbed and change the relative temperature of different layers of air in the troposphere, but may not significantly raise the temperature of the earth.

      I agree. It may or may not. We would have to show the theory and math and let others critique away.

      >> The GHGs only absorb a relatively narrow range and for CO2 the main part of its absorption is from 13.5 x 10^-6 to 15.5 x 10^-6 m. However, this absorption is sufficient to absorb very roughly 1/5th of the energy radiated by the earth.

      That 1/5 value is only one of several variables that contribute to the temperature just like we could not tell how warm it would be simply by knowing we had a solid blanket on but would need to know its thickness (ie, 1 blanket on vs 10 blankets on).

      For example, let’s say that we have a blanket of a certain thickness that absorbs 90% of incident radiation. Now, let’s consider 10 such blankets stacked together. They will also absorb in total over 90% of the incident radiation (say, 1-(.1**10)), but they possibly might lead to a much higher temperature or to only slightly higher. To know the temp, we would have to have a good physics model with accurate details and crank out the math. The answer certainly may not be obvious to mortals.

      To get an idea of the radiative effect, let’s assume there is no convection. A photon can leave the human body and be absorbed by the first blanket. This gets repeated continually as the body releases more photons (ie, as the sun keeps radiating on the planet). In equilibrium, however, we will have photons released by the first blanket towards the body but also towards the outside and 99% of the time be absorbed by the second blanket. This repeats for all blankets. At any point in time, the photons hitting the skin will come from the first blanket and also, every now and then, from the second blanket directly, and, very rarely, from each of the other blankets in turn (with each further blanket being a rarer phenomenon). A simple radiative shell model might show that adding a second blanket has a significant effect on temp near the body while adding the 101st blanket almost surely hardly affects equilibrium temp (with 100 blankets already there).

      Then, to *really* solve the problem, we have to look at the convective contribution in addition to this radiative contribution. The heat flow equations involved are partial differential equations. This is why computers are required to get “accurate” results. [Not to mention the many assumptions that must be made because particles of all types, including clouds, play a role, and the future evolution of these things are not easy to predict too many years out.] The source code to the main computer models are open source and many papers study and contribute to their incremental improvement. A breakthrough paper was produced in 1978 by Ramanathan and Coakley. Additionally, lots of global data is used to properly set up the boundary conditions on the segmented earth computer subproblems. Remember, we not only want models that work, but we need to predict many decades into the future despite the many unknowns, and these guessed variables have complex interrelationships over time. [Computer model runs relying on different random number generation lead to a distribution of values.]

      Now, to offer a competing thery, one would have to at least match a great bunch of study that has been done on these physical and computer models or otherwise explain why the model of the climate can be approximated much more simply and then offer those simpler calculations yourself.

      Note from Wikipedia that Ramanathan is not a climate “cool-aid” researcher as some people believe many climate scientists are, where one allegedly only listens to what s/he likes or what pushes forward the “AGW” theory.

      > His focus then shifted to the radiative effects of clouds on the climate. This was done using the Earth Radiation Budget Experiment (ERBE), which showed that clouds have a large cooling effect on the planet.[7][8] ERBE was also able to measure the greenhouse effect without the use of climate models.

      >> In 1985, Jack Barrett, Professor of Chemistry at Imperial College London, presented a paper to an environmental conference in Europe showing that the spectra of CO2 did NOT lend itself to causing the increase in atmospheric warming proposed by these environmentalists/climatologists. There was an uproar and delegations to Imperial College, to its eternal shame, caused restrictions on research funding and freedom to publish under the banner of Imperial to be placed on Jack and he was effectively drummed out of his job!

      I looked for that paper online once and did not find it available for free. We can speculate about how bad the institute might have been by assuming the paper was a work of great science or we can analyze the paper and perhaps instead conclude that it represented a potential money sink hole for the school.

      >> as carbon dioxide increases, more radiation is absorbed at a lower level, but the increased CO2 also inhibits the return of radiation to the earth, so the EGHE is all but nullified.

      You would gain new fame if you could show it truly was a wash, but I really don’t think that falls out of the equations (ie, to 0). What is true is that the incremental gains are diminishing, but the counter effect never undoes completely the extra bits added any more than a little CO2 in relation to 0 GHG would lead to no warming at all.

      The diminishing returns effect is already articulated rather explicitly in the long-established logarithmic relationship believed to exist between the quantity of GHG and the temperature near the surface of the planet.

      [If you can, please give an example of tracking photons (in a hypothetical arm waving fashion would be good enough to start the conversation) that would show that the number of photons of some energy (or the total energy) arriving at the earth would not go up if there was more absorptive capabilities in the atmosphere. Remember, even if you absorb more to block downward photons, you don’t absorb 100%, AND even what you absorb extra (even if 100%) offers another chance to emit but perhaps this time in the downward direction. Bottom line, more absorption (aka, longer path length * pressure) means more energy stuck in the atmosphere bouncing around and this very likely means higher temp in the atmosphere.]

      >> The spectroscopy plus some good sound atmospheric physics will one day give us the answer.

      I think that is what the community studying climate science at the forefront generally believed many decades back and by now have concluded something along the lines of what the IPCC has been capturing periodically in its publications.

      >> and feed backs must be negative, (otherwise the earth would already be burnt to a cinder)

      This confused me as well some time back. The definition of negative feedback does not refer to the same quantity one normally expects of a system labelled as such in feedback theory. Instead it refers to the net feedback of a certain inner loop and excludes the obvious negative feedback effect expressed by blackbody radiation of the planet. Also, it’s possible that the reference point might sometimes be taken relative to what we would expect from a simple model that only includes GHG warming effects (eg, ignoring effect of aerosols, clouds, volcanoes, etc). I agree that this use of “negative feedback” is confusing to outsiders, but likely it is convenient to those studying almost uniquely this particular system.

      >> If the drivers of the Milankovitch cycles can induce an Ice Age and return us to a Holocene, then minor changes in these mechanical perturbations wqhich are ever present with us, must surely be expected to drive the climate into these much smaller fluctuations than can be caused by CO2.

      I think this is called a gut feeling devoid of mathematical support.

      For example, the Milankovitch cycles might affect the temperature in my oven when I turn it on full blast with the oven door closed, but I doubt it would ever match the effect from having two glass panes on the door vs say only one. We can surmise that CO2 has a negligible effect relative to the Milankovitch cycles, but we would have to show some math that the CO2 effect isn’t in fact instead something as powerful as adding another pane of glass on the oven door.

      00

    • #
      Greg House

      John Nicol says: “The thermodynamic laws apply to NET transfer, not absolute transfer.”
      ==============================================
      Who exactly, how and when proved it, John?

      I will take “don’t now” for answer, too.

      00

    • #
      Greg House

      John Nicol says: “The only gases which can absorb significant amounts of radiation from the earth and also can re-radiate that heat both out to space and back to earth, are the green house gases.
      The Green House Gases (GHGs) thus can help to retain the heat which is radiated from the ‘solar heated’ surface of the earth.”

      =================================================

      This “help to retain the heat” is exactly the point, warmists apparently can not prove by a real scientific experiment. More exactly, they can not prove that this back radiation warms or slows down the cooling of the surface. I asked them repeatedly to present such experiments and the result was ZERO. It is just a narrative.

      00

      • #
        Jose_X

        >> This “help to retain the heat” is exactly the point, warmists apparently can not prove by a real scientific experiment.

        Before anyone had been to the moon, we had many theories on how matter behaved, and these theories (based on experiences with simple cases) allowed us to achieve the feat and with relatively few mistakes at that. Generally, scientists make many accurate predictions that only much later are verified (eg, such as when the moon is reached). We assume matter works the same under many different but hypothesized to be related contexts and only carry out experiments for a very small subset of the total number of contexts and possible scenarios.

        We cannot run an earth experiment for a hundred years to verify if the theories were accurate. You are not going to be given that type of proof by anyone on any forum anywhere. On the other hand, many parts of the theories were borrowed from other fields in physics, and those theories have various degrees of simpler experiments that have been conducted in support.

        For example, the absorption/emission of radiation of numerous GHG has been measured very accurately. These spectra and the related physics of radiation are assumed to hold in the atmosphere. There are instruments used to measure “downward longwave radiation” that have their own physics justifying them. The totality of many experiences are what give the knowledgeable scientists confidence in a particular theory that cannot currently be measured directly. Few people understand all the components and could describe the totality of experiments that might convince say someone like you.

        My guess is that if you read up more on the matter or ask simpler questions and keep an open mind, you will gain insight. In the end, you have to construct your own particular experiment to convince yourself. Asking even a knowledgeable person may prove unsatisfactory (assuming they take the time) because there might be a significant gap between their understanding of the material and what constitutes key evidence to them and your knowlege and requirements (which likely requires more basic experiments to gain confidence with components that they have already accepted and perhaps once convinced themselves of by hand).

        >> More exactly, they can not prove that this back radiation warms or slows down the cooling of the surface.

        Many who understand the physics and math would simply write a word problem and solve it. But to prove each of the equations they used itself would require many more experiments and explanation. If you first take classes in this material (do the leg work) and then still have questions, you will be much better positioned to get a response you’ll value and be able to believe or convincingly critique in front of other knowledgeable peers.

        Let me ask this, would you be willing to read and understand this paper: http://www.google.com/url?sa=t&rct=j&q=&esrc=s&source=web&cd=1&ved=0CDAQFjAA&url=http%3A%2F%2Fclimateknowledge.org%2Ffigures%2FRood_Climate_Change_AOSS480_Documents%2FRamanathan_Coakley_Radiative_Convection_RevGeophys_%25201978.pdf&ei=fFa9UKDeOImC9QS-v4Bw&usg=AFQjCNHiGBDcpf984Tq7utHa_-SSmjgAFg&cad=rja ? It offers one candidate solution to the general question on atmosphere greenhouse effect. They use accepted physics for the most part, but you can obviously question their interpretation and application.

        As a simple reply, radiation works independently of the temperature of the destination. Energy is conserved. It is perfectly acceptable that a photon will radiate from a colder object to a warmer object and thus impart energy to the warmer object. A proof? Well, that theory helps explain things like what we measure when we aim a pyrgeometer upwards. We get infrared radiation at levels that are explained by greenhouse gas theories. Feel free to carry out that experiment and explain the results your way. [Of course, you might start by figuring out what is a pyrgeometer and how does it work.] There are experiments with lasers, with radiation in chambers holding a particular GHG, etc, that all work together to help explain the greenhouse effect. It’s not a short answer and the path taken in order to be convincing largely depends on your own background in the subject.

        Like I said, we can’t run an experiment on the single planet we have in order to first see if we are right. We use theories and try to quantify our chances of being right or wrong.

        00

        • #
          Greg House

          Jose_X says: “For example, the absorption/emission of radiation of numerous GHG has been measured very accurately.”
          ==============================================

          Yeah, but this is not what has been questioned. What has been questioned is the narrative that this radiation affects temperature of a warmer object. Get it now?

          Again, there is apparently not a single experiment confirming that.

          The problem of our time is that warmists sell this unproven fiction to the public as a scientific fact. Very bad, Jose, very bad.

          10

          • #
            Jose_X

            You are questioning mainstream physics rather than climate science. Photon/energy transferred from cold to warm is mainstream physics. This effect is consistent with thermodynamics. Thermodynamics doesn’t say a photon can’t do that. It says that heat (net change in energy) will be from a warm to a cold body.

            00

        • #

          Well, that theory helps explain things like what we measure when we aim a pyrgeometer upwards. We get infrared radiation at levels that are explained by greenhouse gas theories.

          The above statement may be true, but we should also add that pyrgeometres are notoriously inaccurate in measuring long wave radiation (for the purposes of measuring LWR due to the enhanced greenhouse effect}

          e.g. Stoffel et al 2006 from the abstract..

          The calibration of ARM pyrgeometers continues to be a topic of intense research to achieve the goal of accurate field measurements that are traceable to a recognized reference standard……..Recent results of data analyses by the Broadband Heating Rate Profile, including the Longwave Quality Measurement Experiment comparisons involving the Atmospheric Emitted Radiation Interferometer (AERI), indicated a significant and consistent pyrgeometer measurement bias of about -12 Wm-2 ± 5 Wm-2 under clear-sky conditions.

          EKO Instruments manufacture pyrgeometres. They state that…

          The measurement of long wave radiation is more difficult technically than the measurement of solar radiation. For this reason, International Standards for the measurement of long wave radiation have not been developed.

          Considering a doubling of CO2 from pre-industrial era is supposed to increase LWDR by something less than 4Wm2, I can’t see how one would develop an experiment accurate enough to be meaningful.

          p.s. At 152.1 you were looking for a Barrett paper. I provided it for you at 152.1.1

          00

          • #
            Jose_X

            Thanks for the Barrett link. I downloaded it but haven’t found time to read it, although a glance gave me the impression it covered information already discussed here a bit. I haven’t forgotten about it.

            Having an instrument be difficult to calibrate and use is nothing new for CERN and many groups that deal with this all the time even if Joe Schmoe won’t be able to get a good reading. In any case, these aren’t the primary measurements to test warming (temperature is). Of course, in the context of our discussions here, the point is to find an effect even if you can’t measurement as precisely as is ideal. [eg, an galvanometer is not intended to tell you the value of a current but rather if a current exists] Charge coupled devices (CCDs and numerous semiconductor configurations) respond to visible (or other) radiation sources coming from a particular direction and within some range. Simply, there are many ways we have devised to detect and roughly measure what we call E&M radiation through a wide range of frequencies. Our theories of photons allow us to build sophisticated devices used in medicine, science, and consumer and industrial settings to great success. So one of the main points (to Greg House) is that we have theories that imply that cold objects always radiate (losing energy) regardless of whether a warm or cold object is nearby and absorbs, reflects, or passes through that radiation (there is symmetry in absorption and emission). We can account for the energies, and, despite cold object emissions absorbed by warmer objects, cold objects will gain heat (net energy) from warmer objects at least in the cases where work is not done. If someone has an example that they think cannot reconcile these theories of radiation with the laws of thermodynamics, let them give the example.

            00

        • #

          Jose,

          Do you understand the difference between a continuous exchange and a net exchange between EM and non EM forms of energy?

          Would you agree that most of the non EM flux entering and leaving the atmosphere is carried by water?

          Would you agree that the average temperature of the surface is dominated by the temperature of the oceans?

          Do you agree that evaporation is equal to precipitation?

          Do you agree that the water in the atmosphere is thermally connected to the water in the oceans via evaporation and precipitation?

          Would you agree that most of the energy that drives the weather heat engine comes from Sun via the latent heat of evaporated water?

          If there is a net exchange of radiant energy with water, i.e. a net exchange between the total photon energy and total matter energy, the temperature of that water will increase or decrease, depending on the direction of the imbalance, until the planet is again in radiant balance.

          Therefore, the net non radiant balance must be zero. The balance can certainly be pushed in one direction or another, but the harder its pushed, the harder the system pushes back. I should point out that diurnally, seasonally and over other periods, there is an oscillation of accumulated energy transferred between the 2 forms, but the long term average is a net zero sum. In other words, for all photons converted into energy transported by matter, an equal and opposite amount of energy transported by matter must be converted into photons.

          You also do not seem to grasp that the vibrational energy has a high probability of remaining with the CO2 molecule upon collision with an N2 or O2 molecule and has a somewhat larger probability of exchanging energy when colliding with another energized CO2 molecule and of course, its translational energy is equalized upon collision as expected. The energy never disappears as you suggest. While vibrational energy can manifest temperature in a solid, the GHGs in the atmosphere are not solid. In a gas, its translational kinetic energy that manifests temperature.

          The div2 test was just the first test and I’ve already discounted your specific objections. The second test is the emitted spectrum. Have you examined the power spectrum measured from space and the one predicted by this hypothesis (links in another post)? Your vague explanation has a significant problem, which is that there isn’t enough matter in the upper atmosphere to emit that much power in those bands. Remember, the intrinsic emissivity of low pressure O2/N2 gas is quite low.

          George

          00

          • #
            Jose_X

            My comment from a few hours ago (the last one) forms a part of my reply here (ie, is included by reference).

            >> I should point out that diurnally, seasonally and over other periods, there is an oscillation of accumulated energy transferred between the 2 forms, but the long term average is a net zero sum.

            And just the same (as stated earlier), the (space) local values need not be in balance even if globally they basically are.

            >> The energy never disappears as you suggest.

            I don’t think I have suggested that, but maybe I wasn’t clear (can you quote?).

            >> In a gas, its translational kinetic energy that manifests temperature.

            Equipartition of energy means that in the aggregate the average translational kinetic energy is balanced by vibrational and other energy means in equal portions.

            >> the vibrational energy has a high probability of remaining with the CO2 molecule upon collision with an N2 or O2 molecule and has a somewhat larger probability of exchanging energy when colliding with another energized CO2 molecule

            I don’t know these probabilities. Can you quote a source or give more details?

            What is your opinion when we consider the probabilities in the context of the large number of interactions that happen prior to any likely emission?

            What about the probability of CO2 translational energy changing as it interacts with almost any N2, O2, etc, and then that change simultaneously or soon thereafter affecting the vibrational energy?

            >> I’ve already discounted your specific objections.

            I see an unproven hypothesis that to work must ignore experimental evidence of downward longwave radiation and ignore energy conservation at ground level while invoking a form of conservation that you recognize does not apply locally.

            >> Have you examined the power spectrum measured from space and the one predicted by this hypothesis (links in another post)?

            Could you re-express that argument again?

            >> Your vague explanation has a significant problem, which is that there isn’t enough matter in the upper atmosphere to emit that much power in those bands.

            Can you be more specific? [I want to make sure I am following the right argument).

            00

          • #
    • #

      Hi John;
      I noted this post and your comment only yesterday, and would like to join the discussion to offer an alternative interpretation as follows:

      1) The second law of thermodynamics is not a law about thermal convection, conduction or radiation, it is about the second kind perpetual motion machines. We shall not confuse the second law of thermodynamics with the Newton’s cooling law. An object absorbs radiation waves according to its own property (i.e. absorptivity), regardless whether the radiation waves are originated from a cooler or warmer body. The second law of thermodynamics is simply irrelevant, there is no need to add “NET”. A warm body can of course absorb radiation waves emitted from a cooler body.

      2) The word and its associated physical concept “re-radiate” in the statement that green house gases can absorb significant amounts of radiation from the earth and also can re-radiate that heat both out to space and back to earth is very questionable, as it implies the GHGs emit radiation waves only after they have absorbed radiation, and/or emit only the portion of energy they have absorbed. According to the Stefan-Boltzmann’s law (or the Planck’s law), all black (and grey) bodies emit radiation 24/7 under any circumstances as long as their temperatures are higher than 0 K. A good question for brain is what temperature will be if the GHGs do not have a radiation source to absorb?

      3) The GHGs emit radiation constantly as long as their temperatures are higher than 0 K. The downwelling portion of the radiation energy will be absorbed by the earth ground surface, which then raises its temperature to emit more radiation. The total outgoing radiation to the space is the same – no heat is retained in the earth-atmosphere system. In other words, the heat energy blocked in the GHG absorption bands go to the space via other wavelength bands. The GHGs function as a half-mirror hanging on the sky to the earth ground surface.

      There arguments may appear foreign to many. Welcome comments and criticisms. Regards

      Jinan Cao

      00

      • #
        Greg House

        Jinan Cao says: “An object absorbs radiation waves according to its own property (i.e. absorptivity), regardless whether the radiation waves are originated from a cooler or warmer body. …A warm body can of course absorb radiation waves emitted from a cooler body.”
        ==============================================

        The bone of contention is not absorption as such, it is if there is an effect on temperature or not.

        Do you have anything real experimental on “IR from colder bodies slows down cooling of warmer bodies”?

        00

        • #

          Greg;

          Radiation energy is measured in W/m2 (or J/s m2). Assuming an object with surface area A, volume, V, density d, specific heat capacity cp, one then calculates the temperature increase of the object, DeltaT from following equation:

          Radiation flux x A = V d cp DeltaT

          How can radiation energy is absorbed without an effect on temperature ?
          Regards

          00

          • #
            Greg House

            Jinan Cao says: “Radiation energy is measured in W/m2… How can radiation energy is absorbed without an effect on temperature?”
            =========================================

            I see. OK, let me reformulate my question then.

            Do you have anything real experimental on “IR from colder bodies that is measured in W/m2 slows down cooling of warmer bodies”?

            Such a simple question. And it is very relevant, because if that “warming back radiation” notion can not be confirmed experimentally, then it is at best an unproven fiction and not a scientific fact. I think the mankind would be happy to learn that.

            00

          • #
            Mattb

            You would measure the rate of cooling of a warm object, and then the rate of cooling of the warm object when a second object, not as warm as the 1st brick, is placed nearby.

            Look all objects emit IR. an object that is in the way of the IR cannot choose to just pretend it is not there depending upon whether the other object is warmer or cooler.

            00

          • #

            Greg;
            It is expensive to carry out an experiment. For most scientific and technical inquiries, we can find answers from existing knowledge base and scientific methods including induction, deduction, formal logic etc. If they do not provide an answer, experiments are designed and carried out to develop new pieces of knowledge.

            Now for the argument that “colder warmer objects do not absorb radiation emitted from warmer colder objects because it violates the second law of thermodynamics,” we know:
            1) It does not violate the second law of thermodynamics that is irrelevant;
            2) The argument itself violates the absorption law. Radiation waves have only two parameters: wavelength (frequency) and intensity. Objects have no way to tell whether a beam of radiation waves was originated from a cooler or warmer body.

            If you are not convinced and you prefer to see experimental results, why not? You can easily design an experiment to put your inquiry to the nature. Regards.

            [I made an assumption and adjusted your post. mod oggi]

            00

          • #
            Jose_X

            Why don’t you (Greg) come up with a theory that works through “action at a distance”. Maybe you can find a consistent re-interpretation of all of our current physical theories. If you do, then you can argue that cold objects can never give energy to a warmer object. Until then, the idea of each object radiating is accepted theory and describes a lot of experiments. Plus, “action at a distance” might be a plausible alternative if thought out carefully, and, even if bulky or confusing, you could end up re-deriving climate science with a different explanation and maybe new equations.. but perhaps result in the same conclusion as what we have now, that the temp is biasing upwards because of fossil fuel burning. Or maybe not.

            BTW, the theories of “virtual photons” I think include these ideas of potentially “action at a distance” and of travel faster than the speed of light. These theories describe effects as a result of particle interactions we don’t/can’t see. So in the end, the theory can be crazy, but that is OK if it can explain something consistently and have it be useful to us.

            If you can come up with such a set of theories and show they they explain something better than current theories, you can become very famous.

            BTW, a thermocouple (and thermopile) are tools that allow us to measure radiation (or what we call “radiation”). We also have our eyes and many other sensors for radiation. Sure, we can find an alternative description to photon behavior and to matter and energy if we try enough, but the current theories work. You have the burden to find something better; you have the burden to show how your re-interpretation works and explains everything else we currently can describe.

            [If I’m not being clear, what I am saying is that I can pick instruments and theories, but you can always say “well, what if that isn’t a photon or an atom or x or y and instead it is something else as such …. However, you then have the burden to carefully explain all the “as such” rules and how they help us solve the same (or a new) set of problems we already know how to solve by hypothesizing that photons exist and are emitted by cold and warm objects regardless, etc, etc.]

            00

      • #

        Jinan,

        On 1), you are correct that warm bodies can absorb photons emitted by cold bodies, however; on 2) and 3) you are missing information.

        The GHGs in the atmosphere are rarefied and as a result, most are already energized. The probability that a photon will be emitted upon absorption is very high for energized GHG molecules, approaching 100% after the absorption of a very small number of photons. The claim that the probability of re-emission is low applies only to ground state GHG molecules, which of course, most are not.

        The ‘path’ through the atmosphere of a photon in a CO2 absorption band is a random walk bouncing between CO2 molecules until it eventually exits to space or is returned to the surface. At each CO2 molecule, a different photon leaves then arrives and it may even shift frequency, but will always be a frequency in a CO2 absorption band and will be emitted a short time after the last absorption event. Given that re-emission is also in a random direction, there is a net 50/50 distribution up and down and which is independent of lapse rates.

        Given a flux of photons in CO2 absorption bands entering the atmosphere, a corresponding flux must be leaving. However, it leaves over twice the area (above and below) then it arrives (below) which is another indication of the 50/50 split. The evidence of this is in the emitted spectrum of Earth and the other planets which show a consistent 3 db drop in power around narrow lines where the probability of a surface photon at that frequency being absorbed by a GHG molecule is otherwise 100%. If re-emission was at other frequencies, we would see an infinite power drop.

        This is consistent with a hypothesis of mine which states, “The atmosphere has an EM energy balance transported by photons and a non EM energy balance transported by matter, whose net fluxes must be balanced independently”. The matter is air and water, includes latent heat, convection, etc. and the hypothesis is tested here . The test is that if the long term average split is not 50/50 as dictated by BB physics, the hypothesis is falsified..

        George

        10

        • #

          George;

          Thanks for your comment. On 2) and 3), my interpretation differs from yours.

          There are two simple governing physical laws: one is the absorption law describing how much radiation energy, I, an object absorbs; the other is the Stefan-Boltzmann law (or the Planck’s law) expressing how much radiation energy, J, the same object with surface temperature, T, emits per unit area and unit time. (note that I and J are in unit W/m2 or J/s m2)

          (1) I = a I0
          (2) J = ε σT4

          An important message from Eqs (1) and (2) is that absorption of heat energy relies on and only on EXTERNAL factor – radiation source, I0, whilst emission is determined by and only by INTERNAL factor – surface temperature, T, of the object. In other words, with or without absorption, a radiative species keeps emitting radiation as long as its temperature > 0 K.

          Your explanation sounds that CO2 does not emit photons before absorption and CO2 emits only photos that have been absorbed. This does not appear consistent with Eq. (2). I reckon the question “what temperature will be if the GHGs do not have a radiation source to absorb?” is worthy of consideration. Regards
          Jinan

          10

          • #

            Jinan,

            Yes, I know how black bodies work, but the narrow band absorption and emission of GHG’s is not governed by the same physics as the absorption and emission of broad band Planck radiation from matter in equilibrium with its energy source. A similar difference in physics is behind the misinterpretation of the Second Law. If the energy flux is carried by matter, then an interpretation of the Second Law would suggest that energy does not flow from colder bodies to warmer ones, If the energy flux is carried by photons, the Second Law can not be interpreted this way. This is more corroborating evidence for my hypothesis that the energy carried by photons and the energy carried by matter must be conserved independently.

            It’s also not particularly meaningful to talk about the temperature of a specific trace GHG species in the context of the entire atmosphere. After absorbing just 3 photons, a CO2 molecule would have an equivalent temperature many 100’s of degrees C warmer than the O2 and N2. Although, even after absorbing just one photon, the probability of re-emission is already close to 100%. GHG absorption is not warming the N2 and O2 in the atmosphere which then radiates the absorbed energy back to the surface, but is acting like an optical splitter redirecting photons emitted by the surface up and down in equal amounts. The up/down fraction is modulated by clouds to affect the photon energy balance and drive it towards equilibrium. BTW, any measurement of the temperature of atmospheric O2/N2 will be dominated by measuring surface radiation and GHG re-emissions and emissions by clouds.

            The photons re-emitted by CO2 molecules are not the same photons that arrived, nor are they even necessarily the same joules as received from a prior incident photon. The photons energy is absorbed by the electron shells becoming part of the electron and emitted photons are just the reverse of the absorption process.

            George

            00

          • #
            Jose_X

            1:

            I disagree with the idea that energy in a certain form cannot change into energy in another form (ie, what you are saying about ignoring everything but radiation). I don’t think you could find a mainstream university course or text to support that view.

            Let’s ask some questions maybe you can answer. Using robotic arms with low conductivity mittens, we place a metal cube on top of a hot plate, within a vaccum chamber, for 10 minutes so the cube gets very hot. We quickly move the cube to a ceramic material that conducts very little. We measure and observe the temperature of the cube drop over time.

            Questions:
            – Do you believe (a) the cube first got hot and then cooled or (b) do you think that is wrong and, eg, remained at about the same temperature the whole time?
            – If (a), do you think the cube acquired most of its extra energy (a) via conduction from the hot plate or (b) through some other means? [I’ll assume you believe in conservation of energy, btw.]
            – If (a), do you think the energy was lost as it cooled primarily via (a) conduction with the ceramic surface or (b) radiation, eg, as per the Stefan Boltzmann law, or something else?
            – If (b), then how do you explain how energy was added through conduction primarily but lost primarily through radiation? Doesn’t this violate your hypothesis?

            2:

            At normal pressures, CO2 molecules acquire photons and then are much more likely to hit other molecules and change their energy that way than to emit first.

            Let me quote from Raymond Pierrehumbert explaining (his understanding, which agrees with what I have said) of what happens amongst the ghg and on-ghg [http://climateclash.com/g6-infrared-radiation-and-planetary-temperature/ ]:
            > …An IR photon absorbed by a molecule knocks the molecule into a higher-energy quantum state. Those states have very long lifetimes, characterized by the spectroscopically measurable Einstein A coefficient. For example, for the CO2 transitions that are most significant in the thermal IR, the lifetimes tend to range from a few milliseconds to a few tenths of a second. In contrast, the typical time between collisions for, say, a nitrogen-dominated atmosphere at a pressure of 104 Pa and temperature of 250 K is well under 10−7 s. Therefore, the energy of the photon will almost always be assimilated by collisions into the general energy pool of the matter and establish a new Maxwell–Boltzmann distribution at a slightly higher temperature. That is how radiation heats matter in the LTE limit.

            Conclusion from 1 and 2 is that I think you have to account for convection, evaporation, etc, and all forms of energy. This would really throw off your numbers.

            3:
            [This point is a side track to discuss a bit multiple shells and make a few other observations.]

            You got an under-determined system because you needed some way to come up with T. Your 3rd equations just a restatement of what you used implicitly to come up with eqns 1 and 2.

            So I agree it is acceptable to derive T from some data set (just as you did). I have not looked at your source, so I don’t know if 24.1% makes sense or not.

            Now for some words on multiple shells.

            For 1 shell, F<.5 generally, but F=.5 (approx) if the shell location is very close to the earth's surface so that the area of that shell is about the same as the surface area of the earth. But we can't just invent this. To figure that out we need more variables and physics and to consider more details of the atmosphere.

            There are many approximations all over the place, obviously, and we can try to keep simplifying by assuming F=.5, if we assume the atmosphere is consistent with this (so it may or may not be true on earth). This keeps things simple (but we may not be solving an earth problem). The question now is what about 2 shell approx?

            First point is that we would need more Ts, and we have to look at 2 indices (from and to). Here, the old T would be T(from ground to space) and 1-T would be T(from ground to atmosphere). Anyway, now we would need T(from ground to layer 1), T(from ground to layer 2), T(from layer 1 to layer 2), etc.

            Anyway, if we assume the shells are thick wrt infrared, then Txy=0 or 1 depending on whether the layers are far away or next to each other. This would be a nice simplification we might want to use.

            Anyway, to skip to the end, having more shells boils down to approximating a continuum with higher and higher precision. This would allow a coarse profile of the atmosphere to be determined; however, if we want the approximation of optically thick, then this presents a limit because, eg, 100 meters of low earth atmosphere is probably not approximately opaque (optically thick) to infrared. We probably need a longer path length. Also, the higher we go, the more atmosphere we need in order to meet that criteria. Thus our profile would be at least this coarse.

            OK, I'm almost done with this sidetrack.

            The point is that when I spoke of multiple shells, I was assuming something like this scenario I'm describing. In this case, the more layers there are, then the layer nearest to the earth will be closer and closer in temperature to the earth. That is analogous to modeling a bar of steel as cross-sectional slices next to each other, where the bar is held at each end at a different temp. If we use more slices, then the nearest slice to an end will come up having a temp closer to that end. And a slice temp really is like an average of the steel chunk temperature all throughout that slice. [Note that each slice has to meet constraints (eg, minimal thickness) if we want to use a simple slice model .. so for the atmosphere case, we wanted to use a model of optically thick, which implied constraints among the slice distances from each other. For steel, we might have a similar requirement if say the steel was not homogeneous.]

            OK, sorry for the diversion. The important points are 1 and 2 above.

            00

          • #

            George;

            I used the Stefan-Boltzmann equation to highlight the physical principal of emission of radiation by matters. This physical principle is the same for the Planck’s equation apart from the fact that we need to replace the emissivity by the spectral emissivity etc. I do not think your hypothesis that energy carried by photons and energy carried by matter must be conserved independently is physical. In addition, you have over interpreted the 2nd law of thermodynamics. In my opinion, you’d better to dispose the hypothesis to re-formulate a new one to move forward.

            00

          • #

            Jinan,

            Perhaps you are unaware of how the scientific method works. You come up with a hypothesis and you test that hypothesis. If the tests do not agree with the hypothesis then you adjust the hypothesis. Its foolish to change the hypothesis if the tests continue to confirm it. BTW, the div2 test not only confirms my hypothesis, it falsifies Trenberth’s hypothesis that doubling CO2 has a 3C +/ 1.5C effect on the average temperature.

            Stefan-Boltzmann applies to matter emitting BB radiation, for example, the water in the oceans and clouds, but only applies to bodies radiating Planck distributions of photons. The GHG gases in the atmosphere do not radiate Planck spectrums and are more like lasers emitting narrow band energy constrained to the absorption bands. However, things like steradian effects and COE certainly still apply and relative to BB radiation are equivalent, moreover; while its possible to arrive at an equivalent BB representation of GHG absorption/emission, the assumption that the O2 and N2 are the radiating body and the temperature metric itself are where the equivalent model doesn’t match the data.

            Look at the detailed physics to understand the connection between BB effects and the Planck spectrum. Look at the emission spectrum of the Earth and other planets. How do you explain that the net attenuation of emissions in the absorption bands is limited to about 3db (50%)? The tenuous layer of atmosphere above where GHG absorption is only 1/2 of average doesn’t have any where near enough mass or temperature to produce this much energy.

            George

            00

          • #

            George;
            Your hypothesis that energy carried by photons and energy carried by matter are independently conserved is a stranger to most people who know some physics. You will suffer more difficulties than “successes” in explaining various physical phenomena. The flaws in current climate science can all be explained by existing physical laws we have already known; there is no need to rely on any “new” physical laws that are not yet to be discovered. For me, it is understandable why and how one loves his/her own hypothesis/theory so much as a baby.

            I showed you following equations on December 6 in hope that the key message can get crossed.
            (1) I = a I0
            (2) J = ε σT4
            An important message from Eqs (1) and (2) is that absorption of heat energy relies on and only on EXTERNAL factor – radiation source, I0, whilst emission is determined by and only by INTERNAL factor – surface temperature, T, of the object. In other words, with or without absorption, a radiative species keeps emitting radiation as long as its temperature > 0 K. When we apply the Planck equation, this principle is not changed. I know that nothing is a blackbody including water in oceans and clouds.

            00

          • #

            Jinam,

            I’m not disputing your points that all of the energy that heats the surface originates from the Sun, or that the Stafan-Boltzmann equation quantifies how much radiation is emitted by a body at a specific temperature. My point is that the molecules of GHG gases do not behave as black bodies, although they do have an indistinguishable macroscopic behavior relative to the flow of energy. Absorption and re-emission by a GHG molecule is governed by Quantum Mechanics, not the continuum physics of translational motion and elastic collisions. The difference is not in the magnitude of the power flux, but in its spectral characteristics, thus leading to my next test.

            Here is a plot of what my hypothesis predicts the average radiated spectrum of the planet should look like at nominal GHG concentrations.

            Find figure 7-8 in this paper which shows a measured spectrum.

            The major features of the two plots are nearly identical in position and relative magnitude. Even minor features like the small peaks at 800 and 900 as well as the relatively sharp drop at 1200 are present in both. The only significant difference is the magnitude, which as expected is higher at noon in Africa, than the average emissions of the planet. Unfortunately, the paper only plots from wave number 400 and the sharp drop between 300 and 400 is missing from the measured data. As a final cross check, the calculated equivalent temperature of the integrated spectrum is about 255K.

            Using the SB model for emissions resulting from GHG absorption would result in far more power in the transparent regions of the spectrum and almost no power in the absorption bands. This plot shows the probability that a photon at a specific frequency will be absorbed by a GHG molecule in the atmosphere. The inverse of this would be close to the expected power output spectrum if GHG absorption is translated into BB radiation of heated atmospheric gases.

            George

            00

          • #
            Jose_X

            >> Its foolish to change the hypothesis if the tests continue to confirm it. BTW, the div2 test not only confirms my hypothesis, it falsifies Trenberth’s hypothesis that doubling CO2 has a 3C +/ 1.5C effect on the average temperature.

            If the “confirmation” of your hypothesis is what I read online, then I pointed out problems with it. I don’t see how you can say you got confirmation given what I pointed out.

            The main problem I have is with this thing called matter flux and how it doesn’t turn into this other thing called photon flux and vice-versa.

            If you can show that you are correct (that these independent quantities exist and work according to x or y equations), then some of the rest of what you claim might follow (I haven’t considered it that carefully and would need more details), but you do have to show how you are correct and how physicists far and wide for many years have not been.

            Now, let me point out the position you are up against.

            Perhaps at its simplest, I think you are claiming at least that molecular collisions are all elastic. An inelastic collision of two objects going at each other very fast might leave the two mostly stationary. Where does the energy go? There was lots of kinetic energy before (mechanical) that can’t just disappear unless you don’t believe in conservation of energy.

            It goes to affecting the internal energy (eg, maybe chemical bonds are broken or changed or electrons excited or higher vibrational modes entered)) or maybe photons are released.

            I don’t know exactly what you mean by “matter flux” and “photon flux”, but please answer how an inelastic collision, eg, of the extreme case I just mentioned, can exist in your world.

            From Wikipedia Inelastic_collision
            > An inelastic collision, in contrast to an elastic collision, is a collision in which kinetic energy is not conserved.

            > In collisions of macroscopic bodies, some kinetic energy is turned into vibrational energy of the atoms, causing a heating effect, and the bodies are deformed.

            > The molecules of a gas or liquid rarely experience perfectly elastic collisions because kinetic energy is exchanged between the molecules’ translational motion and their internal degrees of freedom with each collision.

            >> How do you explain that the net attenuation of emissions in the absorption bands is limited to about 3db (50%)? The tenuous layer of atmosphere above where GHG absorption is only 1/2 of average doesn’t have any where near enough mass or temperature to produce this much energy.

            I don’t understand what you are saying. Can you elaborate?

            00

          • #
            Jose_X

            >> There was lots of kinetic energy before (mechanical) that can’t just disappear unless you don’t believe in conservation of energy.

            Replace “kinetic” with “” [nothing] and replace “mechanical” with “kinetic energy”.

            00

          • #
            Jose_X

            If you want to pursue the “matter flux” vs “photon flux”, consider first reading:

            http://people.virginia.edu/~rej/papers-scan/Johnson-AtomMol87.pdf You can glance at this or study aggressively. It covers math related to collisions. Energy forms are exchanged. {I haven’t read it]

            Wikipedia, Thermal_energy

            > Microscopically, the thermal energy is the kinetic energy of a system’s constituent particles, which may be atoms, molecules, electrons, or particles in plasmas. It originates from the individually random, or disordered, motion of particles in a large ensemble. In ideal monatomic gases, thermal energy is entirely kinetic energy. In other substances in cases where some of thermal energy is stored in atomic vibration, this vibrational part of the thermal energy is stored equally partitioned between potential energy of atomic vibration, and kinetic energy of atomic vibration. Thermal energy is thus equally partitioned between all available quadratic degrees of freedom of the particles. As noted, these degrees of freedom may include pure translational motion in gases, in rotational states, and as potential and kinetic energy in normal modes of vibrations in intermolecular or crystal lattice vibrations.

            00

          • #

            George;
            By reading the absorption and earth to space spectra you show me, I’ve realized that you may have a problem that have lead you to your hypothesis in understanding what is surface for the earth-atmosphere system.

            The earth-atmosphere system consists of the ground surface, non-radiative gases as well as GHGs. There is a complication: over the absorption bands of GHGs (e.g. the absorption band 667 cm-1 for CO2), the surface is a layer of atmosphere starting from the TOA with thickness equal to its absorption depth (aka optical length); and the surface temperature is the mean temperature of CO2 in this air layer, Tco2(h). One can similarly find out the surface and surface temperature for any other absorbing bands of the GHGs. For the rest of wavelength bands, the surface and surface temperature are the ground surface and its mean temperature, TGSurf. Therefore we understand that the flux over the CO2 absorption band (~667 cm-1) of the spectrum shown in Figure 7-8 in your link is actually the emission flux of the CO2 molecules in the air layer of the first absorption depth from TOA.

            c.f.
            There is a blackbody large sheet at temperature T1, and a detector picking up the emission spectrum.

            Now, we insert another bb patch at temperature T2 in between the large sheet and the detector, what the detector would tell us: over the patch, it is T2, over the rest areas, it is T1.

            If we calculate the absorptivity of the patch, it is of course 1.0. Then you are saying the detector should detect nothing over the patch, i.e. 0 K.

            Another comment for your 3rd paragraph: it is better to say that the integrated flux over the whole spectrum is 240 W/m2 instead of 255K. A black body at temperature 255K emits 240W/m2; for a grey body, it must be at a temperature higher than 255 K, depending on the emissivity, to emit 240 W/m2.

            Regards. Jinan

            00

        • #
          Jose_X

          co2isnotevil, I haven’t read the whole page you linked to, but I’d like to point out some problems I found with the approach.

          I think you derived using a 1 shell (pure radiative) model of the atmosphere. I believe this was among the first models done by people first studying the climate seriously in the 1950s or thereabout. That model includes interesting elements and gives first guesstimate but is definitely not more accurate than what we have had for many decades now (eg, glance over the Ramanathan and Coakley 78 paper I mentioned earlier as a more accurate starting point, which includes the important details of convection).

          If you were to extend the atmosphere to a 2 shell model, you would get an extra set of equations and you would get a bit different value (remember, I haven’t gone over the math, so I won’t comment on that accuracy of other details).

          A minor flaw is that you assumed the radiation leaving was equal to that entering (post albedo). The Trenberth et al paper I think says that based on measurements from satellites (and I’d guess maybe derivations, since simply interpreting satellite data implies derivations and extra analysis) that the planet is gaining about 0.5 W/m^2 currently. So there isn’t a balance. This is a relatively minor point but worth mentioning.

          A more serious problem is that you can’t assume radiative fluxes balance (ie, energy per unit time and unit area balance). Energy balances (ie, conservation of energy), but convection (and the next point) really spoils the situation for that simple assumption of radiative balancing.

          [This next point also addresses other comments I read.] As Jinan Cao stated, you don’t have emission exactly after and because of absorption. From what I read, at our ground pressure, much more frequently than when a GHG molecule absorbs a photon and then emits one (which would be at a different energy level, btw, as dictated by quantum probability wave), is when it absorbs and then continues colliding with other molecules (eg, with non-GHG) almost surely resulting in a different energy distribution than before (sorry, don’t have time constant calculations/measurements handy). This condition is known as local thermodynamic equilibrium (LTE) and results in preserving a Maxwell Boltzmann energy distribution (which is just what you’d get as the limiting value at a very large number of particles, with a few simple rules, and perfect chance). I haven’t done the math (or studied this very carefully), but later I will point you to a Physics Today (online) article that at least covers this ground but in more detail.

          Also addressing Greg House, a major problem with this simple model is that we can in fact measure back radiation (downward longwave radiation) near the ground (or anywhere else.. as I think also can be pulled from HITRAN) and the values agree (more or less) with the Trenberth et al diagram. In fact, I think Trenberth et al overviewed those sort of results and commented on how close they were to the graph and explained some of the differences.

          Since I know you covered other points in the analysis, I will try to read through it all a bit later when I find time.

          00

          • #
            KinkyKeith

            Jose

            Your comment:

            “The Trenberth et al paper I think says that based on measurements from satellites (and I’d guess maybe derivations, since simply interpreting satellite data implies derivations and extra analysis) that the planet is gaining about 0.5 W/m^2 currently. So there isn’t a balance.”

            You seem to have a lot of faith in this guy Trenberth et al.

            He must be an amazing scientist to be able to calculate that there is a residual “accumulation” of heat of that magnitude.

            A suggestion.

            Instead of getting mesmerized by other peoples propaganda why don’t you go to University and invest some time and effort over a few years to educate yourself.

            Then you can make your own Scientifically Correct statements.

            00

          • #
            Jose_X

            A few more points:

            a:
            Note from Fig 1 (page 4) of the Trenberth paper (linked from the analysis) that:

            161+333 [494] = 17+80+396 [493]

            give or take (since those values are rounded off).

            This is an example of how the “back radiation” number [333] is balanced out in the energy equations and also likely has a significant effect in warming. This back radiation value does not change too much during the night time (from its value during the day.. assuming same degree of cloud cover) and is distinguished by its spectra (baked into the measurement instrument) from the energy coming directly from the sun. [I may have linked, within a comment on this article many months back, to some sample charts of these results from a location in California taken at various times of the year I think around 2005.]

            b:
            I don’t want to get much more detailed with how your analysis varies in assumptions from the Trenberth results because it’s obvious they do a fair amount so we should expect differences. For example, the 239 you say hits the ground (from the “measured” R of .3) is shown on the Trenberth diagram to be 161 hitting the actual earth and 78 getting absorbed into the atmosphere, which would be straight into your “A” but you would not be accounting for that in the “A=” equation.

            c:
            Actually, if you were doing what I am calling a one-shell radiative model as I had earlier thought, than you would probably assume .5 up and .5 down (isotropic) instead of calculating it. I suppose it might be luck that your numbers come close to .5 (and are impacted by the non-radiative energies you decided purposely by assumption to ignore and issues like the “A” component mentioned above). Well.. looking further into your math, it seems it’s not luck but a mistake.

            d:
            The system of equation you came up with is in fact solvable (if we ignore the constraints on the variables). You can get a quadratic in T out of that, but in neither case do the values meet your constraints. For example, I got the following 2 approximate sets of values which you can substitute back into your 3 equations to verify for yourself:

            Solving,

            239 = T*385 + (1-F)*A
            385 = 239 + F*A
            A = 385*(1-T)

            can lead to the following quadratic,
            385*385*T*T – 2*385*385*T + 385*385-1 = 0

            and these 2 candidate solutions (approx):

            T = 0.9974
            A = 1.001
            F = 145.9

            T = 1.0026
            A = -1.001
            F = -145.9

            Neither of these potential solutions meets the constraints. F is supposed to be between 0 and 1 (inclusive). Further, A is supposed to be positive while T must also be between 0 and 1 (inclusive).

            e:
            Anyway, I’m trying to give my input into why I think there are differences between your results and the Trenberth paper. Trenberth relies on a number of earlier research results that I won’t bother to examine, but if you want to discredit Trenberth you probably should look at those at some point in time.

            00

          • #
            Jose_X

            KinkyKeith,

            I don’t know the basis for them concluding the 0.5 value (the 2009 paper has 0.9 so I may have been thinking of the first paper). I think different groups have come to different mean values and error ranges (with some leaving a little wiggle room that there might be a net decrease in flux). I also said it was a minor point and actually wasn’t relevant to the website analysis.

            Do note that the Trenberth et al paper is widely referenced by professionals and has been peer-reviewed, so that means the burden is on you (or me or anyone wanting to claim otherwise) to show that anything in that paper is incorrect. I also do think “Trenberth et al” have spent a fair amount of time at universities learning, researching, and possibly even teaching and so would probably know how to make “scientifically correct statements”. Of course, if I were to go (back) to a university in order to engage like they have or like you suggest, I might end up just producing the same propaganda.

            00

          • #
            Jose_X

            Point d above is trash. I made a math mistake and did not get the correct observation co2isnotevil had stated in that paper that the system is undersolved. This also means the last sentence of point c is also wrong.

            00

          • #
            Jose_X

            f:

            Despite not using some terms (evapotranspiration) and shifting the 78 to the earth rather than keeping it in the atmosphere (which would make a huge difference), I found it interesting that you ended up with .500 for F (ie, .4996…).

            But, you computed Stefan Boltzmann on the average of the different temperatures around the planet during a year, and this is not the same as taking the average of the SB values of the different temperatures around the planet during a year (ie, the f(x_ave) != AVE(f(x_i)) for f=SB). This was explained in the Trenberth paper (pg.5, “Spatial and Temporal Sampling”) and is why the value derived is 396 and not 385. This 396, with T=.241, would lead to F=.522, which is close but not nearly so exact (which makes it not nearly as interesting from my perspective).

            So, as interesting as it is to get .500 exactly, the real value is likely nothing like that in the worst case or in the best case perhaps a few percentage off (eg, maybe about 5%).

            g:

            Also, out of curiosity, your 38%, 83%, and 66% ISCCP data covered what time period? I am not sure about the Trenberth et al 2009 coverage time, but that might be another source you should carefully consider (it may work in your favor or not) if you are interested in seeing what F value exactly comes out.

            h:

            And another small detail is that, using your assumptions, the derived (but with the changes specified here) F>.5 may not be correct since it is likely that F should be less than .5 because, a few kilometers above the earth’s surface (if we assume this is where the shell approximation would lie), the earth would receive a little less than a hemisphere’s worth of radiation. Of course, if we wanted to be so precise, we would probably consider using error bars all throughout the analysis.

            00

          • #
            Jose_X

            Scratch point c altogether since it turns out that you did two separate analyses, and in one you did assume the .5 (while in the other you tried to derive it).

            10

          • #
            Jose_X

            Raymond Pierrehumbert http://judithcurry.com/2011/01/19/pierrehumbert-on-infrared-radiation-and-planetary-temperatures/ wrote the Physics Today article I mentioned earlier.

            00

          • #
            KinkyKeith

            Jose

            Thank you.

            As to the 0.5 why CALCULATE it theoretically.

            Why not just MEASURE it.

            Or is the system too variable to allow that?

            I just don’t believe that Harry Trenberth knows what he is doing.

            KK

            00

          • #
            Jose_X

            And what if there isn’t a single instrument we can create that can measure that value directly? Will global warming (if we consider that it might be real) just pause until someone smart enough comes along and can build that device?

            We (as a society) do try to use our equations and theories that have served us well in the past in order to analyze and measure what and where we can and then draw conclusions about what is of interest but what we can’t directly measure.

            Hope for my great grandchildren’s sake people don’t pick the wrong horse and find out too late.

            00

          • #
            Bryan

            Jose_X

            …..”what if there isn’t a single instrument”

            A very good instrument is the Earth near surface temperature record.

            Manns hockey stick predicted an accelerating rise in temperature with increasing CO2.
            The last 16 years of flat temperatures shows no correlation between temperature and increased CO2.
            This is also confirmed by the typical historic CO2 lag of 800years BEHIND temperature rise.

            “Hope for my great grandchildren’s sake people don’t pick the wrong horse and find out too late.”

            Some consider the Earth being hit by an asteroid to be an urgent problem.

            After a series of very low temperatures in the UK others think we are entering a new ice age.
            These include Joe Bastardi and Piers Corbyn both have a good record of past predictions.

            Which horse to back?

            All we can do is to be sceptical of any alarmist scare.
            Look for evidence to back up any assertion.
            This used to be known as the scientific method

            00

          • #
            Jose_X

            >> The last 16 years of flat temperatures shows no correlation between temperature and increased CO2.

            You may not realize that scientists understand that numerous factors affect the climate that cannot be boiled down to the “maximum temp within a 16 year period”.

            For example, while CO2 adds warmth bias (a rising contribution to temperature), there are other factors, like the sun, volcanoes, aerosols, and “natural variability” that contribute on top of the CO2 effects and on some years contribute more than on other years and by a wide margin perhaps. For example, the oceans absorb much more heat than the atmosphere and modify what we feel in the atmosphere as the oceans turn over warming surface water with cold deep waters.

            That said, you are cherry-picking. For example, look at the last 17 years and you get a very different picture (every year since then has been warmer). Look at the last decade and you will see the current decade has been warmer (the average).

            You should consider taking a course in statistics if you haven’t so that you understand the value of means (averages) and that stray points happen all the time.

            If you took climate science courses, you would understand that a number of factors that affect warming were lined up in 1998 so it may take a while to get that high again.

            But if you looked at the most updated data, you would see that 1998 was not the warmest year. From NOAA’s website:

            > For 2010, the combined global land and ocean surface temperature tied with 2005 as the warmest such period on record, at 0.62°C (1.12°F) above the 20th century average of 13.9°C (57.0°F). 1998 is the third warmest year-to-date on record, at 0.60°C (1.08°F) above the 20th century average.

            > The global land surface temperature for 2010 tied with 2005 as the second warmest on record, at 0.96°C (1.73°F) above the 20th century average. The warmest such period on record occurred in 2007, at 0.99°C (1.78°F) above the 20th century average.

            >> This is also confirmed by the typical historic CO2 lag of 800years BEHIND temperature rise.

            Show me a paper or write-up that “confirms” what you just said.

            There is no set of decades we know of around 800 years back where we can say that the temp at that time saw a significant rise to match the significant rise we have seen with CO2 over the past few decades.

            The Medieval Warming was on which year? That’s right, it wasn’t on any given year. Supporters of MWP cherry-pick years around the world and within a list of centuries and try to put them all together as if they all happened at the same time. That’s a no-no and is why serious climate experts generally don’t put much into the MWP theories.

            Also, CO2 has lagged temp, but humans have only been burning loads of dinosaur fossil fuels for a short time, so the historical paleoclimatic record does not include an applicable scenario.

            >> After a series of very low temperatures in the UK others think we are entering a new ice age. ..These include Joe Bastardi and Piers Corbyn both have a good record of past predictions.

            Going back how many years now? How many times has either of them been wrong vs right? Maybe we should track their predictions, do they have a site where he posts his official predictions? At any given point in time a fraction of the guesses out there will be right on the mark.

            Have 1 million people guess a sequence of 20 coin flips, and we can expect that 1 of them will nail it exactly — all 20 in a row. But that person will still have a 50% chance of being wrong on the next coin flip.

            From Wikipedia, Bastardi doesn’t seem to understand the value of probabilities. That doesn’t inspire much confidence in me. Also, he probably keeps his private sector research mostly secret. If not, I’d like to see it. If so, that doesn’t inspire much faith.

            Weather is very hard to pin down. Probabilities arise as we try to nail down a range of very likely scenarios but are not 100% sure of one, so we take averages.

            His “prediction” of a return to 70s temp is for 20-30 years down the line, and is surely possible even with the climate scientists being correct. For one, there are variables we have to guess out into the future, like the sun’s radiation, volcanoes, and levels of CO2 release by humans. For another, even if very unlikely, those predictions might be within the 95% error range of what the theorists predict (I’m not sure).

            The US has experienced in the last few years significant drought, wetness, and warmth that stand out statistically speaking. Your cold is our warmth, in part I think because of changes in the ocean currents due in part to extensive Summer melting of Arctic ice.

            >> All we can do is to be sceptical of any alarmist scare.

            Yeah, like ignoring that the earth might end this year without providing any scientific evidence,

            .. but if scientific evidence suggests we will get cancer and die many years earlier than average if we smoke 2 packs of cigarettes a day from our teenage years onward or die sooner if we shoot heroine or any of zillions of other predictions that are born out by the stats and science, then I will pay attention to that alarmism.

            >> Look for evidence to back up any assertion…This used to be known as the scientific method

            Exactly. The climate scientists have reams of evidence to support their points. The lucky weather analysts you mentioned have almost nothing except arguably the temporary luck found within a large numbers of guessers.

            Anyway, so do you have a list of papers that support Bastardi and other guessers?

            00

          • #

            Jose,

            Extending to N atmospheric layers makes no difference. If you get a different answer when you go from 1 to N layers, then you have done something wrong in your analysis. What I have is an equivalent model whose behavior at 1 layer is identical to the behavior at N layers. I’ve seen the analysis you speak of and as you extend to N layers, the difference from the 1 layer case approaches zero. In other words, the 2 layer model is wrong, not the 1 layer model.

            Keep in mind that my model exhibits the measured behavior at the top boundary with space and at the bottom boundary with the surface. As long as the boundary conditions are matched as the system changes, the number of layers you use in the model is irrelevant.

            The Trenberth paper is wrong. He improperly lumps together the energy carried by matter (latent heat, thermals etc.) and that carried by photons, allowing an arbitrary interchange between the 2 forms. He also improperly conflates cloudy skies and clear skies, which basically short circuits the climate systems use of clouds to modulate the energy balance and seems to have no clear purpose other than to inflate inferred GHG effects with cloud effects.

            The frequency of collision is on the order of the rate that photons are received, but that doesn’t matter. When energy is transferred between a hot gas and a colder one, translational energy is exchanged by elastic collisions and velocities equalized. The energy stored by GHG absorption is in the form of vibrational energy which must be exchanged in quantized units, just like emitted photons. Just because a GHG molecule collides with an O2 or N2 molecule doesn’t mean that the N2 or O2 ends up with any of the vibrational energy as absorption has no influence on velocity.

            The change in energy states you speak of occurs when an energized CO2 molecules collides with another energized CO2 molecule. However, there’s only a 1 in 2000 chance that any single collision would do this and the result just equalizes the vibrational energy among GHG molecules which eventually plateaus at a level where the input photon flux in the absorption bands is equal to the output photon flux in the absorption bands.

            George

            00

          • #
            Jose_X

            My prior comment addressed the n layers a bit, but that is a more complex model. I’m not going to think about it much more, and it’s tangent to the main questions.

            I agree with Trenberth and others. You cannot assume energy stays in one form. Our modeling of matter almost never assumes that. I gave an example in the prior comment (hot plate and cube). I think that example shows how little sense it makes to assume radiation doesn’t mix with conduction (and this would apply to convection and evaporation as well).

            There is no conservation of “radiant energy” or of “conductive energy”, etc, but simply of energy. In physics problems I’ve seen, energy is expected to change forms. As indicated by Raymond’s quote in my prior comment, vibrational energy dominates among the ghg close to the ground, with emissions and absorptions being spaced out far in time. The values for these expected times can be measured ,and I think also derived (eg, Einstein coefficients may be derivable using other basic numbers via QED).

            Bunching up all atmosphere types as vanilla atmosphere (ie, ignoring clouds) is fine. It just means hat he didn’t try to apportion attribution.. because it is a tough problem. It’s like saying, “the commenters on this greenhouse effect article have been writing for a while”, without specifically trying to attribute time to each specific writer. The assumption is that cloud and no cloud cases all can be fit fairly neatly within the scope of “atmosphere”, eg, without overlapping the surface effects.

            >> Just because a GHG molecule collides with an O2 or N2 molecule doesn’t mean that the N2 or O2 ends up with any of the vibrational energy as absorption has no influence on velocity.

            Since you want to make that claim, can you provide backup support (eg, mainstream text, university course, experimental evidence, or personal detailed theoretical derivation)?

            00

          • #

            Jose,

            You’re not getting it. The point is that while instantaneously, energy is continually transforming, it’s the NET flow dependent on a specific transport that must be conserved. Furthermore, this is just my hypothesis. You can object all you want, but the data is consistent with the hypothesis and that trumps your objection.

            Consider 2 insoluble fluids at the same temperature, entering opposite sides of a mixing bowl with a drain in the center allowing the mixture to leave at the same rate the inputs are arriving. The more output is collected, settled and separated, the closer the amount of each fluid will be to each other. The point being that there is no net exchange of form or energy from one fluid to the other. Otherwise, one fluid would heat without bound as the other one cools. If the 2 fluids were different temperatures, energy would be transferred, but in the case of the Earth, both fluids (the photon flux and the matter flux) originate from the surface whose temperature is invariant between them.

            Look here, figure 7-8. How can you explain the very high measured levels of radiation leaving the planet in the strongest of the CO2 (15u) and O3 (10u) lines, which otherwise absorb 100% of the energy emitted by the surface in those bands? It’s a linear scale and the attenuation averages only about 50%, again confirming my hypothesis.

            I also suggest you read this.

            I also must also insist that conflating cloudy and clear sky conditions is invalid since the fraction of the planet covered by clouds sets both the amount of energy entering the system and the amount leaving by modulating the relative fraction of warn surface and cold cloud tops presented to space whose average modulation drives the energy balance. The ISCCP data shows this conclusively. All of my plots, some of which are here are presentations of ISCCP data aggregated by GISS. http://isccp.giss.nasa.gov

            George

            00

          • #
            Jose_X

            I have not seen a good description for “matter flux” or for “photon flux” from your hypothesis. I can’t argue about something whose meaning I don’t know. For example, how do I measure or derive “matter flux”?

            >> How can you explain the very high measured levels of radiation leaving the planet in the strongest of the CO2 (15u) and O3 (10u) lines, which otherwise absorb 100% of the energy emitted by the surface in those bands?

            I haven’t seen the answer to this, but I can think of a major reason for this which (not having done calculations) appears to nicely support the perspective I’m sharing.

            CO2 is the dominant GHG in the higher atmosphere, I believe. For absorption that happens from H2O (nearer the ground) and also from other GHG anywhere, that energy is likely to be divided among the other gas molecules after contact with them (I think something like 99% of the time this happens instead of emission near ground level (?)). So while H2O, dust, whatever, absorbs a lot, the energy gets dispersed among all molecules. CO2 dominates as GHG in the higher atmosphere. So emission at low levels is dominated by H2O but a lot of the energy makes it way upwards through contact and the majority of that is emitted by CO2. I haven’t written a computer program to try and calculate these things nor even done a back of envelope guesstimate, but the measurement you mention appears to make sense if a fair amount of the energy absorbed near the ground is transferred upward through means other than radiation. In addition to this, H2O and CO2 overlap in many places, so some of the emissions upward from H2O can be absorbed directly by much higher up CO2.

            If all absorbed by a molecule was emitted by the same molecule, then the CO2 fraction of what leaves to space would appear to be closer to 20% or no?

            As for O3, that absorbs directly from the sun (ozone protective layer) and high up in the atmosphere where there is less contact with other molecules so there is a much better chance of emission from the absorbing O3, and, when this happens, there is >50% that the
            direction will be away from the planet, towards space.

            >> I also must also insist that conflating cloudy and clear sky conditions is invalid

            How can putting the contributions from A and from B together into one bag and labeling the bag “contributions from both A and B” be invalid?

            00

          • #

            Jose,

            The photon flux is comprised of the photons entering and leaving the surface, atmosphere and space. There is no discernible matter flux between the atmosphere and space (meteorites and space vehicles not withstanding) and it exists only between the surface and the atmosphere, It enters the atmosphere as latent heat and leaves as wind, rain, lightning, erosion and other weather related effects. One form is electromagnetic and the other is not and this is the distinguishing characteristic that separates the 2 energy fluxes. I call the non EM a matter flux because energy is carried by matter including liquid and gaseous water and the rest of the gases in the atmosphere, but mostly water.

            Both O3 and CO2 have a stronger influence in the upper atmosphere, but there is a signficant H2O influence since H2O is so powerful and has a significant effect even at very low concentrations. Ozone has a strong absorption band at 10u which is at the peak energy density of the power leaving the surface. But enough ozone is present in the lower atmosphere to result in significant absorption. Of course it doesn’t matter at which vertical layer of the atmosphere absorption occurs, the 50/50 split up and down of the corresponding emitted radiation still applies.

            Where do you get 20%? It’s 50% up and 50% down since the emitted photons have a 50/50 chance of being emitted up or down.

            You still don’t seem to see the difference between a steady state with an equal and opposite exchange between the 2 forms and a steady state with a net transfer of energy from one form to the other. The fact is the later is not a steady state condition, for if it was, the planets water would be either heating or cooling without bound as water is the primary transport of non electromagnetic energy.

            Consider all of the planets water as a single entity. If there was a net transfer of EM energy in to or out of this water, the water would heat and/or cool until it was again in radiative balance. You are probably also missing the fact that the oceans and clouds are thermally connected via evaporation and precipitation driving the heat engine we call weather.

            George

            00

          • #
            Jose_X

            >> The photon flux is comprised of ….

            I’m trying to understand what you mean by matter vs photon fluxes. If I don’t use the term correctly at some point, let me know. I don’t think the explanation you give is comprehensive enough to clarify some questions I might have, but I have enough to go on to try and address some of your points perhaps a little better.

            >> Where do you get 20%? It’s 50% up and 50% down since the emitted photons have a 50/50 chance of being emitted up or down.

            I said: “If all absorbed by a molecule was emitted by the same molecule, then the CO2 fraction of what leaves to space would appear to be closer to 20% or no?”

            I was addressing a different issue: I believe a large fraction of the radiation seen by satellites as coming from below is in the CO2 band. I thought one of your references to 50% was referring to that (to satellites picking up about 50% in the CO2 band).

            If there wasn’t sharing of energy via contact with other molecules at LTE after CO2/H2O/etc absorbed a photon but before emission, then satellites should see the same spectra as the earth emits, I think. The fact it sees a higher concentration around CO2 band is consistent with the energy sharing, since that means that a bunch of energy picked up by (eg) H20 would not necessarily be released by H2O but would help ultimately “heat up” most other nearby molecules, many of which would move their way up above the H2O vapor altitude boundary and then mostly have their energy released by CO2, which is the ghg that dominates in that region high up (even though the pressure is much lower). So we would see a shift in measured emission % towards CO2 even if H2O did most of the absorption initially. [I think CO2 bands cover about 20% of spectral energy emitted by earth but satellites see about 50% or something like that.]

            >> 50% up and 50% down

            But note (as a point of interest) that near the ground, after an H20 gas absorption of a photon from the ground, almost 100% of these H2O emitted photons are going back into the planet, because whether up or down, the emitted photon has a great chance of getting absorbed by a part of the planet (eg, H20 higher up or the ground itself), repeatly, for these low lying H20.

            In some sense, the sun is cooking, not just the shell of the planet as happens with the atmosphere-less moon that is super hot during the day and freezing cold at night because the acquired day sun energy was only on the outer surface right next to freezing space, but is cooking the earth a little bit from the “inside” out, so that there is always a region that remains hot even when you remove the cooking during the night. The section in between the ground and the top of the atmosphere shows a gradiant of temperatures anchored by near 1000 W/m^2 very hot surface and near 0 K space.. modified.. of course.. by convection currents that keep an average balance during the 24 hours. The sun power cooks straight past the atmosphere. Then the atmosphere then just has to slow down the loss of that “deeply” embedded heat-cooking long enough for the next day to arrive and add the power cook again. This is possible because of the properties of ghg, to allow pass through during the day “deep” into the planet and then to stop the outgoing warmth from escaping that “deep” point nearly instantaneously (while also using convection to keep everything moderate). If the power cook was only at the surface, that energy would leave fast. Not only could we not live on that same surface next to cold space, but warming deep down via conduction (from the outside of the rock towards the inside) is much less efficient than warming down deeper via radiation that passes straight through without getting intercepted by the shell and radiated right away into space. [It’s “deep” in relative terms, enough to create a warm region for us humans] This injecting of heat so that radiation “upwards” happens “into” the planet rather than directly into space, is a key aspect of the atmosphere greenhouse effect and also why cars with transparent windows get much hotter faster than those with covered windows (where cooking happens at the outside layer first).

            00

          • #
            Jose_X

            >> You still don’t seem to see the difference between a steady state with an equal and opposite exchange between the 2 forms and a steady state with a net transfer of energy from one form to the other. The fact is the later is not a steady state condition, for if it was, the planets water would be either heating or cooling without bound as water is the primary transport of non electromagnetic energy.

            You are missing something, but let me first add that your hypothesis fails if it violates conservation of energy (and I don’t think you want to attack conservation of energy). The latent heat is energy that must be accounted for.

            I think you might not be treating the diagram values at the bottom as representative of ground level (or very very low atmosphere). At that level energy must be balanced. If energy leaves the ground through evaporation at some rate, that is less energy available to leave via some other mechanism.

            Also, the balance you speak of is global, not local. The figure values shown are local, so they need not balance. They are at ground level and have a counterpart throughout the lower atmosphere where there is extra downward radiation (than otherwise) in areas where there would be a greater amount of condensation than evaporation, but this region is not specifically described in that diagram. We simply have a snapshot of the lowest part of the earth+atmosphere, where there would be a clear net amount of evaporation over condensation.

            00

          • #

            Jose,

            There is no COE violation. The EM energy is conserved and the non EM energy is conserved and of course, the sum of the 2 is consered.

            Latent heat is the predominate source of non EM energy entering the atmosphere from the surface. This is returned to the surface in a variety of ways involving mechanical work, rather than radiation. These include water lifted against gravity, temperature and pressure differentials, currents, wind, weather, enthalpy of fusion and even the temperature of precipitation.

            To the point of distinguishing between energy transported by photons or by matter is that the former is a propagating electromagnetic wave and the later is not.

            You are correct that a significant amount of the energy leaving the planet is in the absorption bands of atmosphere GHGs and the 50/50 split quantifies this. The GHG’s in the atmosphere are acting as an energized conduit for photons emitted by the surface to either leave the planet, or return to the surface, after some delay. If we were seeing significant sharing between captured GHG energy and the rest of the atmosphere, we would see less in the GHG absorption bands and more in the transparent bands of the spectrum.

            At any layer in the atmosphere a photon emitted by an energized GHG molecule has an equal probability of being emitted in any direction yielding a 50/50 chance of returning to the surface or eventually leaving the planet, independent of density variability or lapse rate concerns. Such issues only affect the probability that any single photon will be absorbed and how long it takes for the energy to eventually escape.

            You can think of photons in GHG absorption bands as following a much shorter version of the paths photons from a stars core takes in order to leave the star.

            George

            00

          • #
            Jose_X

            I said > I think. The fact it sees a higher concentration around CO2 band is consistent with the energy sharing, since that means that a bunch of energy picked up by (eg) H20 would not necessarily be released by H2O but would help ultimately “heat up” most other nearby molecules, many of which would move their way up above the H2O vapor altitude boundary and then mostly have their energy released by CO2, which is the ghg that dominates in that region high up (even though the pressure is much lower).

            Seems, whether consistent with this analysis or not, that I got the evidence backwards, so either other dominating factors are at play or instead perhaps the analysis I gave here is wrong (or both.. or something else).

            Sorry about that, and do you have any comments? Supposedly, seeing less CO2 band longwave from the satellites means stronger greenhouse effect by CO2.

            The countering effect I used to think was the case (and maybe is) is that every time CO2 absorbs, it lowers the contribution from CO2 band. Every time CO2 emits back to the ground, gives more opportunities next time around for the ground emission to be out of CO2 band.

            There are some other points here. I’ll think about it later.

            00

          • #
            Jose_X

            >> There is no COE violation.

            Thought experiment assuming we could work at the 1 photon, 1 particle, and approx 0K level without measurement or control issues (ie, this represents a reduction in scale to get a point across about energy conservation at a larger scale): We fire 1 photon at material at absolute 0 and observe no photons but instead a single molecule leave the surface. That molecule needed just that photon energy to eject. Energy is conserved if energy in = energy out but not otherwise. Here we have 1 photon (say 100eV) coming in and the equiv in latent energy coming out and observe no other energies.

            In other words, if we see so much latent energy coming out from the surface of the planet (let’s assume for a moment that that figure is correct) and know the energy coming in, we expect the remaining significant energy coming out (radiation) to be the difference of all of the other energies in that simple relation (in the implied way), more or less.

            But that is not what you have done. We can’t assume radiation in will be balanced solely by radiation out. Radiation is not generally a conserved quantity locally (and only approx in the global case as you pretty much have been explaining).

            Are you looking at that diagram’s near-ground numbers as representative of very near the ground value and nothing else?

            There may be valid criticisms of that diagram, and since I haven’t read the paper and don’t feel I could judge it for correctness — I really don’t know; however, I think it’s a COE analytical flaw to ignore evaporation and other “matter flux” within the system of equations I saw in div2.

            >> This is returned to the surface in a variety of ways involving mechanical work, rather than radiation.

            Based on the diagram, this work is returned above ground level and so do not form part of the COE at surface level. Presumably the 80 and 17 values represent net such energy differences at surface level (and would exclude exchanges that happen in clouds and already account for wind effects near the ground, for example).

            You can argue those values are wrong in that diagram if you want to undertake that challenge, but you can’t say they should be excluded from the system of equations you wrote there. Locally, radiation is not generally conserved. If it is treated as conserved, in numerous cases that would violate COE (see experiment above).

            [When I started to look at shell models and wanted to extend the argument, I eventually realized that same problem that I couldn’t just look at radiation.]

            If you think I am wrong, tell me how. Otherwise, the system of equations seems to require changes.

            >> The GHG’s in the atmosphere are acting as an energized conduit for photons emitted by the surface to either leave the planet, or return to the surface, after some delay.

            Like all or most of the convection insulation examples I have given before (and augmented by some tub examples contrasting rates (in/out) vs storage (water in the tub), the insulation does provide the conduit, but its presence enables the average temp within the system to remain elevated (vs the no insulation case) after equilibrium is established.

            >> If we were seeing significant sharing between captured GHG energy and the rest of the atmosphere, we would see less in the GHG absorption bands and more in the transparent bands of the spectrum.

            If you want me to analyze this issue, which is separate from div2, I would want the basic data (I did find a page via google listing papers on this topic) and then a clear hypothesis suggesting some non greenhouse effect version of life. I already messed this up a little and don’t want to get distracted by it unless I can focus on it with some care (as I find time).

            >> yielding a 50/50 chance of returning to the surface or eventually leaving the planet

            That represented energy bundle can go up or down, but, in either case, it has a high probability of being reabsorbed by the planet if emitted near the ground. Eventually, that energy bundle will leave the planet regardless of how much zig-zagging that hypothetically taggable photon does. The atmosphere is optically thick, at least in CO2, probably several times over. The 50-you-say-50-you-leave only applies near “TOA” (I did at some point get the impression climate scientists define TOA like that, based effectively on optical thickness of ghg or perhaps even of CO2 in particular).

            00

          • #
            Jose_X

            >> but, in either case, it has a high probability of being reabsorbed by the planet

            planet = earth, water, air (as used here)

            >> The 50-you-say-50-you-leave

            50-you-stay-…

            >> only applies near “TOA”

            In other words, as long as we are within the thicker part of the atmosphere, photons emitted by ghg have a high chance of being absorbed again (by the earth-water-air system) no matter the direction.

            Eventually, as the energy makes its way to TOA, then it has a significant chance of leaving into space.

            The more ghg, the higher is TOA.

            I am not sure about the physics (or math), but I think (in contrast to the potential temperature) that the real temperature going up the atmosphere is linear with a slope based on planetary parameters that are independent of the total quantity of gases or type of gases. What having more ghg in the atmosphere does, I think, is to raise the “TOA”, the level where the photons released upwards do leave the planet once and for all. At this TOA, we have to balance the radiation coming in from the sun, and that level is associated with a certain temperature. A higher TOA then means that the temp on the ground would be higher as we follow the linear effect downward from that higher TOA and the certain temperature at it.

            00

          • #

            Jose,

            Less emissions in absorption bands is an indication of more atmospheric absorption, however; while 0% absorption corresponds to 100% net transmission, 100% absorption corresponds to 50% net transmission, not the 0% often implied and that leads to the need for a ‘run away’ effect to explain Venus. The physical reality is that the net transmission between an energy storing mass (the ocean) and the environment (space) through a semi transparent medium (the atmosphere) must be at least 50% and is currently 62%.

            The fallacy of a run away greenhouse effect applied to Venus has likely distracted you from understanding the 50% limit. The temperature of the surface of Venus is as independent of GHG effects as the 70% of Earth’s solid surface under the oceans. Venus has an Earth equivalent surface, high up in the atmosphere, enclosing most of the mass storing solar energy whose equilibrium with the Sun is readily explained without run away GHG effects. On Earth, the thermal store is the oceans and on Venus, its a dense ocean of super critical CO2, whose bottom temperature is dictated by the PV profile of the CO2 ocean and the super critical temperature. The Earth equivalent surface will experience day, night and seasons, while the Venusian solid surface and the Earths solid surface under the ocean do not. The last point is that the emission spectrum of both Mars and Venus show the same 3db attenuation in the absorption bands.

            Latent heat is about 77 W/m^2. About 5% of this is converted into the potential energy of water lifted against gravity. More than 10% overcomes the enthalpy of fusion required to melt the ice which would otherwise form upon condensation and more heats the condensed water. Even small storms send 100’s of W/m^2 to the surface while hurricanes can send 10’s of Kw/m^2 to the surface. Maintaining pressure and temperature gradients requires work and the energy driving that work comes from latent heat. The climate system is a giant heat engine which requires input energy that arrives as latent heat. You are probably confused because Trenberth includes kinetic return components in his hypothetical ‘back radiation’, which is a theoretical quantity and nearly impossible to measure for confirmation or falsification.

            The main point I’ve made is that the radiant components of the system are already in balance without accounting for latent heat, so to the the extent latent heat puts energy into the atmosphere, all of this energy must be returned to the surface, as none needs to leave in order for the planet to be in a steady state radiant balance. This is not an assumption, but an emergent property of the data. There can’t be net radiation of latent heat energy because that must be radiated isotropically and no additional power is required in the up direction for the planet to be in radiant balance.

            One more point is that the thermal conduction of an insulator is not how GHG’s pass energy. The column of energized GHG molecules is more like a very low resistance conductor of photons.

            George

            00

          • #
            Jose_X

            >> must be at least 50% and is currently 62%.

            Is this another hypothesis? Can you put a little more math and argumentation behind those numbers? I read div2, and I think it is wrong because I was able to look at the math and logic. Or reference something so that I can follow.

            It’s easy to show otherwise, in fact. Take Venus.

            >> The temperature of the surface of Venus is as independent of GHG effects as the 70% of Earth’s solid surface under the oceans.

            Venus’ atmosphere (object if you think I am wrong) allows a significant amount of shortwave through it to the ground second after second, while its atmosphere reflects or achieves a greenhouse effect.

            How much shortwave or longwave from the sun is reaching the ocean floor directly? 0.00. The ocean floor does not heat up from the bottom up as does the atmosphere on Venus to a large degree because of the heating of Venusian ground by direct sunlight. There is no greenhouse effect for the Earth’s ocean floor. [Further, most IR travels no further than millimeters or at most a few meters in the ocean.]

            So, no, that is not true. And if we divide the radiation on the surface of Venus by the radiation leaving into space, we get a number clearly smaller than 50% (around 2% I think).

            >> Venus has an Earth equivalent surface, high up in the atmosphere, enclosing most of the mass storing solar energy whose equilibrium with the Sun is readily explained without run away GHG effects.

            Yes, I know, this “Earth equivalent surface” up in the clouds of Venus can be explained with fairy dust evolution and dragon magic. If you know of a second alternative to the greenhouse effect, I am in the mood for a good story.

            >> The Earth equivalent surface will experience day, night and seasons, while the Venusian solid surface and the Earths solid surface under the ocean do not.

            Right, a castle on a cloud on Earth that I once saw experiences day, night, and seasons up there as well; hence, that is just like the Earth’s surface.

            No.

            And the Venus ground does experience day, night, and seasons (if the seasons are “boring” and day and night is in a shifted frequency range at still very significant intensity contrast). Eg, the Earth sees IR day and night but sees significant shortwave almost exclusively during the day. Same is true for Venus but false for the Earth ocean floor.

            >> You are probably confused because Trenberth includes kinetic return components in his hypothetical ‘back radiation’,

            Is this another hypothesis of yours that Trenberth back radiation includes significant quantities (70-100 range) of anything but photon radiation? Do you have data and math so that I can judge your hypothesis for myself?

            So I suppose my overall judgement on the div2 hypothesis may be improved with regards to the legitimacy of ignoring the “matter flux” if you prove this other hypothesis.

            >> The main point I’ve made

            You’ve stated your belief. I have beliefs as well and can tell you about how powerful pixie dust is. I hope you can back up your beliefs with links that include math and physics, just as physicists have been doing in support of the greenhouse effect now for many years.

            >> so to the the extent latent heat puts energy into the atmosphere, all of this energy must be returned to the surface

            .. as, eg, back radiation.

            I recently told you. When, eg, water vapor releases photons downward to reach the ground as it converts to liquid water up in the atmosphere, the water has lost energy above ground (not at ground) and that energy is returned to ground as 100% bona fide (photon) back radiation, as has been measured by many groups many times.

            >> ..as none needs to leave in order for the planet to be in a steady state radiant balance This is not an assumption, but an emergent property of the data.

            Who is talking about leaving? “Back radiation” is radiation coming back to the ground.

            And your radiation equations can be assumed to be in violating COE, for you have not proven that the ground’s non-radiant net energy is 0. The Trenberth diagram for 2009 update of the earlier paper shows 80 and 17 as other energy figures. Talk of condensation and water being an engine is consistent with Trenberth, but using that as an excuse to ignore the nonradiative ground energy fluxes is what you are doing and is not consistent with reality unless you can show that the net non-radiative energies at ground level are 0 (a tough task considering the well known behavior of H20 to condense up in the atmosphere while releasing energy).

            >> There can’t be net radiation of latent heat energy because that must be radiated isotropically and no additional power is required in the up direction for the planet to be in radiant balance.

            The radiation from, eg, H2O condensation happens below TOA, but even if it happened at TOA, you haven’t shown that this isn’t included in the 239.

            >> One more point is that the thermal conduction of an insulator is not how GHG’s pass energy.

            Thankfully, I did not claim or depend on that point in anything I have said.

            However, again, like an insulator, the ghg effect in our atmosphere allows the ground to stabilize at a higher temperature than would be the case without ghg (or insulation).

            00

          • #
            Jose_X

            There was an analogy I thought I had posted a few days back, but it was for a different article. Basically we use a heater that is on for only 3 hrs each day to keep a super well insulated house at a very warm temperature despite freezing temp outside. If the insulation were weaker, that same intermittent heater would instead establish a colder (but still nonfreezing) equilibrium temperature. The heater only needs to add a little energy directly inside the house to help maintain a temp well above freezing. We can’t calculate the final temp merely from knowing the heat input boundary conditions. We need to know the insulation material as well. … We need to know the ghg effect in the atmosphere and not just the sun’s energy at the boundary in order to get our answer. [so if we had a planet with no convection or evaporation and used a shell radiative model to find the surface temp, then we could account for the ghg effect by adding extra shells to match the optical depth of the ghg]

            Let me add another analogy.

            If a sun injects its energy deep inside a planet on an ongoing basis, then that planet will reach a warmer temperature throughout than if the sun’s energy is simply cast at the surface of the planet right next to cold space. Inside the planet the heat cannot dissipate that quickly. The temp on the outer neighboring side of the injection point rises as more heat is added at the injection point. This is true until the internal temp is so hot that the migration of heat outward is in sufficient quantity so that the heat leaving at the surface will be able to match the energy coming into the center. Once that equilibrium is established, the internal really high temp will be maintained so that the release rate into space will stay steady and equal to the injection rate. At equilibrium we have energy in = energy out, it’s just that a high temp is needed to make sure energy out is large enough to match energy in. In contrast, if the injection was at the surface, radiation into empty space would carry away the heat much faster than conduction would internally. This is what we see on the moon. Very hot during the day and very cold later at night.

            Now the Earth is like that internal injection, except that instead of the outer layers being also solid earth, they are gas (which works differently than solid). The ghg slow down the escape of energy away from the ground so that there is accumulation of energy near the ground meanwhile the sun keeps pounding its energy injections. Eventually, a higher temp is reached where the amount of slowed energy released at the space side of the atmosphere matches the injection boosts we get from the sun. Only if the surface temp is high enough will the consequent lower temp at TOA also be high enough to compensate for the sun’s injection rate.

            It’s a simple concept. What is confusing in part, I am understanding, is that the solid ground and liquid water release some of that energy via evaporation into the atmosphere gas and don’t just release all the energy by radiation. There are two forms of energy we must look at. If we just look at the radiative component, it seems like the atmosphere is coming up with extra radiation to warm the planet back.

            Anyway, the radiative component of released energy on the surface of our Earth is lower than would otherwise be the case thanks to the water evaporation cooling effect.

            00

          • #
            Bryan

            Jose_X says

            “Basically we use a heater that is on for only 3 hrs each day to keep a super well insulated house at a very warm temperature despite freezing temp outside. If the insulation were weaker, that same intermittent heater would instead establish a colder (but still nonfreezing) equilibrium temperature. The heater only needs to add a little energy directly inside the house to help maintain a temp well above freezing. We can’t calculate the final temp merely from knowing the heat input boundary conditions.”

            We need to know the insulation material….very important!

            There’s nothing wrong with your analogy…here’s another.

            The Earth represented by a resistor
            Solar Energy input represented by a current
            Radiative output losses to space at -273K

            The Earth system (Earth + Atmosphere) has several insulating properties.

            If input > output losses => the Earth surface temperature will rise.

            There is no arbitrary limit at 255K for the surface temperature it all depends on the insulating properties.
            Indeed some speculate that more CO2 will increase the insulating properties leading to higher surface temperature.
            The so called GHE.

            But there are several other heat retention methods.

            To name one, evaporation over the 70% of planet Oceans absorbs masses of thermal energy.
            This energy does not disappear but largely returns to the Earth surface in the form of precipitation.
            Indeed as George pointed out hydroelectric power stations intercept a fraction of the rain to provide us with a useful source of power.
            The clouds formed by precipitation interfere with the radiative processes in an unpredictable way.

            The ground heat flux is another source of heat storage.

            The N2 and O2 atmosphere heated by conduction with Earth surface cannot lose their energy unless by conduction back to a colder Earth surface or collision then radiation from CO2 and H2O.

            Short wave (or light) radiation on entering the Oceans cannot be absorbed by pure water.
            Any absorption must be by the mainly organic particulates in the water.
            This energy is stored by method of photosynthesis and photochemistry.
            ……and so on.
            I have hardly scratched the surface of the different types of heat storage in the Earth system.

            The IPCC with its overemphasis on radiation results in ignoring all other methods of heat retention.

            This creates a false theory of climate.

            The computer models that use IPCC science fail all the time.
            The last 16 years temperature record shows no correlation between increasing CO2 and temperature.

            11

          • #

            Jose,

            The 62% is not a hypothesis, its a measurement. The 50% limit is not a hypothesis, but a fundamental property of a system whose topology is an energy storage mass below (Earth ocean, Venus supercritical CO2 ocean) an energy source and sink above (Sun/space) and a semi transparent medium between the 2 (Earth atmosphere or Venus upper atmosphere) that passes energy in and out of the storage mass while storing a small amount itself. For Earth, the temperature at the top of the storage mass is the relevant surface in direct equilibrium with the energy source and is the surface whose temperature we want to characterize. The specific topology must match exactly before any kind of equivalences can be inferred between systems. Invoking highly speculative run away feedback effects to overcome a difference in topology is not justifiable.

            The specific IR transparency of the Venusian atmosphere is largely irrelevant to its solid surface temperature. The surface of Venus is at about 93 bar. Anything above 74 bar is a supercritical fluid making more than 1/4 of the Venusian atmosphere a super critical fluid ocean. From PV = nRT, when you get closer to the surface, the pressure increases due to gravity and the temperature must increase in response. At a pressure of about 35 bar, about 3/4 of the mass of the atmosphere is towards the surface and this is also about where the saturated vapor density of water becomes greater than the CO2 and clouds can start to form a discernable surface. If the temperature of this cloud surface is about 277K, then the solid surface would have to be about 735K owing to the ideal gas law and nothing else. How it gets that hot is irrelevant, the point is that it must be that hot and this would be true even if the Venusian CO2 was replaced with N2.

            Your insulation analogy is based on false logic that arises from mis-characterizing the effect of water vapor and CO2 as a greenhouse effect. The roof panels in a greenhouse are like diodes that allow visible energy in but are broad band reflectors of IR energy keeping that from leaving. A better analogy would be that GHGs act like a greenhouse with half the rook panels removed. GHG’s only operate at the frequencies in their absorption bands. Photons with frequencies outside of the absorption bands pass right through the atmosphere with no interaction at all. GHG captured energy returned to the surface is transformed into new broad band Planck emissions, about half of which are outside the CO2 absorption bands and pass through the atmosphere untouched. It doesn’t take long for all to escape which means that GHG’s act on very short time frames. Radiation cooling on a clear winter night is another example of short time constants relative to GHG effects.

            The only insulation effect on the Earth is that oceans are so deep that they insulate the deep ocean cold from the warm surface waters. The oceans temperature profile is evidence of this which looks identical to an insulating wall whose thickness is that of the thermocline. Consider that the temperature at the bottom of the ocean at the equator is 0C, just like that below the N pole. Are you trying to say that this is dependent on GHG effects? How would this temperature change if CO2 quadrupled?

            Finally, CO2 represents about 1/3 of all GHG effects and the influence of all GHG’s combined is still small relative to the heat trapping contribution of the clouds covering 2/3 of the planet. This makes the GHG’s over 2/3 of the planet far less relevant to the surface temperature than the anti-oil, fear driven climate science favored by the consensus would suggest.

            George

            00

          • #
            Jose_X

            >> The 62% is not a hypothesis

            I agree. It is a measurement you decided to bring up, perhaps because the div2 hypothesis needs it to be true. I don’t know.

            >> The 50% limit is not a hypothesis

            On Venus, despite the distraction into “supercritical fluids” (for which I believe, eg, pv still equals nrt like it does on earth) and the distraction into the Venusian greenhouse effect and Venusian runaway greenhouse theory (two issues which are likely irrelevant to that measurement) and unless you don’t believe in Stefan B applying over there, the ground radiation is probably very high and could be near 17000 W/m^2. That specific value or any within the ballpark would put the percent value at the space boundary at a few percent and certainly much under 50%. This is called experimental data creating obstacles to a hypothesis.

            >> The specific topology must match exactly before any kind of equivalences can be inferred between systems.

            I don’t know what you are talking about. If you are talking about analogies I have given, then it comes as news to me that analogies require “same topology”. Certainly, an analogy is not an analytical solution but is instead an explanatory device sustained by formal analysis elsewhere.

            >> Invoking highly speculative run away feedback effects to overcome a difference in topology is not justifiable.

            And I still don’t really know why you are bringing irrelevant issues into play as concerns SB/radiation on Venus.

            And I have no idea where you think I have appealed to the runaway effect in our conversation.

            And I still don’t see in what way the runaway theory would be important to finding that ratio (which certainly can be below 50% if I understand correctly).

            Maybe you can define how you calculated that 62% number because it feels like we are talking past each other.

            >> From PV = nRT, when you get closer to the surface, the pressure increases due to gravity and the temperature must increase in response.

            Because you haven’t done much physics or math here, I can’t be more precise (also because I have not studied this problem well).

            However, “we” can certainly point out some problems with what you did offer. First, n is larger near ground for a given volume. So if I just look at this equation and consider a particular constant volume at two different heights, I see nothing that suggests T should change in any specific way. Below, P is higher but so is n. Meanwhile V and R are the same. So T is not forced into anything, except that it definitely cannot go up by as much as P goes up if n is going up as well.

            If we instead consider a volume above and bring it down, then P goes up, V goes down, n and R are constant and that still doesn’t say anything about T.

            So, no, I don’t see what point you are trying to make, nor have you proved that T goes up or to what degree.. You need more math/physics, and you surely haven’t shown that T should be 750 K or 10000K or 400 K or the actual measured value.

            In other words, you haven’t made a useful statement for or against the greenhouse effect nor for or against the hypothesized 50% limit.

            Until you address these or find something else that helps your case, the div2 analysis and various other hypothesis don’t follow and appear to be unsupported, not to mention inconsistent with widely accepted physical theories.

            >> The specific IR transparency of the Venusian atmosphere is largely irrelevant

            You aren’t serious are you?

            So the significant amount of solar radiation reaching the ground doesn’t contribute to heating the ground, in your opinion?

            Remember, you haven’t shown T should be x or y, and now you want to ignore physics that does actually does help define ground T?

            >> If the temperature of this cloud surface is about 277K, then the solid surface would have to be about 735K owing to the ideal gas law and nothing else.

            Unless you can rebuttal the above, you haven’t shown that pv=nrt is sufficient to explain 735K.

            >> How it gets that hot is irrelevant, the point is that it must be that hot and this would be true even if the Venusian CO2 was replaced with N2.

            And if you can’t explain the 735K, then you can’t use that uncertainty to point blank reject the possible contributory effects from CO2 vs from N2. You need further physics/math, and I look forward to seeing that physics (and it might be hard to do, so feel free to point to a paper done already by others and which I can read online for free).

            >> a fundamental property of a system whose topology is an energy storage mass below (Earth ocean, Venus supercritical CO2 ocean) an energy source and sink above (Sun/space) and a semi transparent medium between the 2 (Earth atmosphere or Venus upper atmosphere) that passes energy in and out of the storage mass while storing a small amount itself.

            In this short discussion, I see no evidence to support the 50% alleged limit nor why SB should be ignored at ground.

            >> Your insulation analogy is based on false logic

            I saw nothing in that paragraph (or in anything earlier) that shows why the ghg atmosphere doesn’t achieve an insulation effect that allows the “internal” temperature to rise.

            >> A better analogy would be that GHGs act like a greenhouse with half the rook panels removed.

            I disagree, especially since removing half the panels might destroy near 100% of the greenhouse effect. While that result might concur with your belief about the atmospheric ge, I have seen no analysis that would rebuttal the existing scientific support for the greenhouse effect as I understand it.

            Remember, since the atmosphere ge and the ordinary ge work differently, we don’t have apples to apples, so can’t just infer a simple same % reduction in components.

            >> that GHG’s act on very short time frames

            You have not quantified this in any way. Regardless, I don’t see how that would contribute to the 50% question or to the position that the ge is a myth or irrelevant (on earth or in venus).

            >> The only insulation effect on the Earth is that oceans are so deep that they insulate the deep ocean cold from the warm surface waters.

            Funny you say this, yet essentially claim the following statement is false: “the [atmospheres] are so deep that they insulate the deep [atmosphere] cold from the warm surface [air].

            >> Finally, CO2 represents about 1/3 of all GHG effects and the influence of all GHG’s combined is still small relative to the heat trapping contribution of the clouds covering 2/3 of the planet.

            I see no evidence to support the insinuation that the clouds don’t contribute to the ghg effect.

            And let me add:

            Botulinum toxin representing less than 1/3 of the 1 microgram of toxins in a cup of water is still small relative to the poisoning contribution of something greater, yet it is sufficient to kill a man… for whatever that’s worth.

            00

          • #

            Jose,

            OK. At least now you seem to get that the while the 50/50 split is a constraint of radiative thermodynamics, there is also enough data to confirm that this constraint is all that needs to act on the pure radiative balance in order to match the measured data. Its only a small leap from here to my hypothesis.

            Also, clouds to not contribute to the GHG effect, they contribute to trapping heat at the surface. The GHG effect is limited to the absorption and emission of photons that happen to resonate with molecules of GHG gases. The water in clouds absorbs and emits radiation as black bodies with an average emissivity of about 0.8 as do the oceans with an emissivity closer to 1. The relevant difference is spectral. BB is broad band Planck radiation and broad band absorption, while GHG effects operate only on photons in very specific bands of energy.

            Look at the thermal properties of liquid water and air to see at what depths they can become a thermal insulator.

            I also suggest that you explore system modeling and system analysis and try to understand what an LTI is and why this describes the thermodynamics of the planet so well.

            Happy Holidays,

            George

            00

          • #
            Jose_X

            >> The GHG effect is limited to the absorption and emission of photons that happen to resonate with molecules of GHG gases.

            This is a silly sidetrack point, but I’ll clarify that (a) you are generally right in your rebuttal, but (b) I was considering “clouds” to include the water vapor in the vicinity of clouds (including “within”) and minute components that might not absorb strongly shortwave but would longwave. I was stretching but did so because we were talking about “contributing”, so any contribution would qualify.

            >> I also suggest that you explore system modeling and system analysis and try to understand what an LTI is and why this describes the thermodynamics of the planet so well.

            Another “I said so” statement. I do hope you read the 6 trillion degree problem from first order diff eqn solutions.

            Let me ask, what would you consider to be the impulse and impulse response to your RC climate model and their actual manifestations in the climate system? How would you initiate and measure these in the actual climate? How do you plan to infer them, otherwise?

            And do you have any data that you believe shows an RC response over time, say over the last 30 years? Provide the input and output sequences (for a discrete approach) or a sufficiently decent analytical representation of that data. And please don’t replicate Spencer’s paper with the “6 trillion degree” issues.

            Or demonstrate some physical and mathematical analysis that argues that the thermodynamics of the planet and the forecasting of future surface temperatures is described “well” by an LTI system (in particular, by a first order “RC” solution). Feel free to reference a text book or other resource that directly addresses this question with clear physical/mathematical analysis.

            See, if an LTI model was going to be “sufficient”, a first order model would not cover the complexity in our climate. But if you disagree, feel free to show the input, output, and system functions or series that you think argue that point for you.

            00

          • #
            Jose_X

            (co2isnoteveil) BTW, I don’t think I am asking anything of you, in terms of providing supportive evidence, that the physics/climate scientific/educator community as a whole — your competition — hasn’t already provided. Obviously, I am asking for a lot, but you are the one that has written off all the analysis and research on greenhouse effect and what not, so you have effectively volunteered to rise to that challenge.

            PS: This is my 3rd exception.

            00

        • #

          Jose,

          The div2 model doesn’t require 62% (385 @ 287K / 239 @ 255K), it is a consequence of it. From first principles, the steady state planet emissivity can be derived as,

          e = T + F*(1 - T)

          where e is the net emissivity of the planet, F is the fraction of absorbed power emitted to space and T is the transparency of the atmosphere given by,

          T = t*(1 - c*(1 - p))

          where t is the size of the transparent window of the clear sky atmosphere, c is the fraction of the planet covered by clouds and p is the fraction of surface power leaking through the clouds. Plugging in measured values of t, c and p and e, F must be 1/2 for e to be 62%.

          The topology I’m referring to is that on Earth, the surface whose temperature is affected by GHGs is coincident with the boundary of the planets energy store across which power from the Sun enters, stored power leaves per SB and which is in direct equilibrium with the incident solar power. On Venus, the surface you claim is dependent on GHG effects is the solid surface at the bottom of a dense CO2 atmosphere whose mass is on the order of the mass of the Earths oceans. The heat capacity of the surface dwarfs that of the CO2 above and the surface contribution to emissions by the combination of the surface and the atmosphere is insignificant. How important to the radiation emitted by the planet is the radiation emitted by the Earths solid surface below the deep ocean?

          The n is relatively constant at the bottom of the Venusian atmosphere. As you get closer to the surface, the pressure increases, but so does the temperature and the density decreases with increasing temperature. The assumption that the density increases, i.e. n increases, assumes both T and V to be constant. In fact, the surface T seems close to whats required to keep n constant between 74 ATM where it becomes supercritical and the 93 ATM at the surface, although I don’t have a precise enough temperature/density plot to verify this. This is also a contributing factor for why the Earths atmosphere gets colder as you get higher up. The difference is that on Earth, the atmosphere is mostly heated by the surface and surface is mostly heated by the Sun, while on Venus, the atmosphere is mostly heated by the Sun and the surface is mostly heated by the atmosphere.

          Very little solar energy reaches the Venusian surface. The cloud layers are very thick and little solar energy passes through. moreover; the clouds are almost completely opaque to the LW IR radiation emitted by the surface, so little, if any, power emitted by the surface passes out in to space. To top it off, most of the matter storing heat between the cloud layer and the surface is not the surface, but the ocean of CO2 in between.

          Clouds do not contribute to the GHG effect, but displace and in effect cancel GHG effects acting on the surface.

          Your objection to the ‘half of the roof panels removed’ analogy is because you think a greenhouse and the GHG effect are based on the same principles, when in fact, they couldn’t be any more different. The fact that this is even called the greenhouse effect is part of the problem.

          George

          00

          • #
            Jose_X

            co2isnotevil, I am going to try and eject myself from this conversation. It is taking too much time. As the greenhouse effect is accepted by mainstream, I suggest, if you want to convince scientists otherwise, that you take one of the really advanced texts that covers the detailed math and physics of atmosphere and write a paper that explains why x or y is incorrect or inferior to something else. Simply coming up with alternative hypotheses without carefully detailing how the existing is problematic is something anyone can do by inventing stuff, so you are not going to convince many people to take time off what they are doing to address every part of your ending stream of hypotheses. The ideal course is to find what part of the existing published (respected) research is wrong and address it precisely in a formal paper and perhaps in bite sizes where you tap into a common pool of physics that is widely accepted; however, just carefully critiquing something like an advanced bona fide text in the subject and doing the writeup online may turn heads if you actually do an accurate job.. or write a book that covers all the hypotheses in an organized fashion.

            I have not studied atmospheric physics and dropped out many years ago of what could have been a physics major, so I am not the ideal person to debate this topic (specially since I just am not very interested in that right now). The more side topics that get drawn into the div2 or other basic discussion, the worse it gets for me, and I see only slow progress. I am sorry you cannot accept accepted science and want to claim your hypotheses are the truth and mainstream science is wrong without taking the time to do a formal writeup of what is wrong. It’s a shame that attitude is trendy right now. I wish you luck.

            ****
            Very quick reply:

            >> The div2 model doesn’t require 62% (385 @ 287K / 239 @ 255K), it is a consequence of it.

            Yes, I am not sure now why I worded the other statement the way I did. I may have been thinking about how I don’t see this particular calculation as important to the main issue I see or maybe I was thinking about the 50% alleged boundary.

            As for Venus, I am not going to keep arguing there. I disagree with a few key claims there. For example, (a) a gas does not behave like a liquid, and (b) venus surface gets a lot more direct sun irradiance than does the bottom of the earth ocean, yet you keep comparing to the bottom of the ocean.

            As for the pv=nrt, that equation does not imply a rise in T as you suggested it did and hence that such would be sufficient to discard the greenhouse effect. We would need to look at more physics to see just how much T rises and why. Since that equation by itself is not sufficient, we’d have to look at more analysis, and I said I am not interested. I suggest you get a good book on the subject and write a paper explaining meticulously why you think that book is wrong. If you don’t or can’t do that, you will not turn many heads of people doing serious scientific work (serious at least in the sense that they don’t pretend they can write up their own physics and claim that is the truth without first, before their peers, formally take apart the necessary existing theories or applications of them). I had 2 cents to throw in on div2 and on some other minor issues, so I did.

            Clouds (and their water) absorb and emit and participate in thermodynamic exchanges, so they *contribute* to the ghg effect. That is what I meant with that.

            The atmospheric ge and the ordinary ge have common elements. Again, if you want to fight what is widely accepted, there is a process for that. My point is that since the physics is different enough, you can’t just draw the parallel you were drawing. You are the one who appears to have been drawing the tight parallel by wanting to remove 50% of panels to match the CO2 spectrum holes and then use that perhaps to dispel the greenhouse effect ..or whatever. I don’t really care. It was a little side comment and too far removed from the div2.

            I like engaging you, but this is not the time for me to run all over the place on this issue when it starts appearing that the conversation might just keep getting longer and longer and when you seem convinced your many hypotheses are correct and mainstream is wrong.

            Bryan, sorry I didn’t have much to say. You didn’t prod me that aggressively this last time out, and I was distracted and not wanting to get further engaged without good reason.

            There is a paper I didn’t get to that came up this last week. I’ll go back and see if I look at that and maybe comment later on.

            00

          • #
            Truthseeker

            Jose, if you believe that the greenhouse gas effect is “mainstream”, you may want to look at this. Of course, you can try and cherry pick which mainstream you want to follow …

            00

          • #
            Jose_X

            So over 30 years ago, before volumes of research, the academy was not sure about “greenhouse effect”. And from this, we are to conclude that the brains knew as far back as 30 years ago that ge is not real because these people back then said they weren’t sure? That doesn’t compute.

            But wait. Did you read the report?

            ***
            Carbon Dioxide and Climate: A Scientific Assessment

            Report of an Ad Hoc Study Group on Carbon Dioxide and Climate

            Woods Hole, Massachusetts

            July 23–27, 1979

            to the

            Climate Research Board

            Assembly of Mathematical and Physical Sciences

            National Research Council

            NATIONAL ACADEMY OF SCIENCES

            Washington, D.C.

            1979
            ***

            Let’s read the report, but let me provide a spoiler clue: anyone who says that the nas did not believe in the ge has a hill to climb. These people you linked to are doing nothing for their credibility.

            > When it is assumed that the CO2 content of the atmosphere is doubled and statistical thermal equilibrium is achieved, the more realistic of the modeling efforts predict a global surface warming of between 2°C and 3.5°C, with greater increases at high latitudes. This range reflects both uncertainties in physical understanding and inaccuracies arising from the need to reduce the mathematical problem to one that can be handled by even the fastest available electronic computers. It is significant, however, that none of the model calculations predicts negligible warming.

            > The primary effect of an increase of CO2 is to cause more absorption of thermal radiation from the earth’s surface and thus to increase the air temperature in the troposphere. ..We have examined with care all known negative feedback mechanisms, such as increase in low or middle cloud amount, and have concluded that the oversimplifications and inaccuracies in the models are not likely to have vitiated the principal conclusion that there will be appreciable warming.

            > To summarize, we have tried but have been unable to find any overlooked or underestimated physical effects that could reduce the currently estimated global warmings due to a doubling of atmospheric CO2 to negligible proportions or reverse them altogether. .. It appears that the warming will eventually occur….

            From Ch 3:

            > An increase of the CO2 concentration in the atmosphere increases its absorption and emission of infrared radiation and also increases slightly its absorption of solar radiation. For a doubling of atmospheric CO2, the resulting change in net heating of the troposphere, oceans, and land (which is equivalent to a change in the net radiative flux at the tropopause) would amount to a global average of about ΔQ=4 W m−2 if all other properties of the atmosphere remained unchanged. This quantity, ΔQ, has been obtained by several investigators, for example, by Ramanathan et al. (1979), who also compute its value as a function of latitude and season and give references to other CO2/climate calculations. The value 4W m−2 is obtained by several methods of calculating infrared radiative transfer. These methods have been directly tested against laboratory measurements and, indirectly, are found to be in agreement with observation when applied to the deduction of atmospheric temperature profiles from satellite infrared measurements. There is thus relatively high confidence that the direct net heating value ΔQ has been estimated correctly to within ±25 percent.

            Well, at least we now know from that this 1979 report can be trusted because it was before the greenhouse theory became political, as the linked to group appears to believe.

            00

          • #
            Jose_X

            >> Readers can browse for themselves online the 13,000-word 33-year-old U.S. government report that details the role of carbon dioxide and how it might impact climate. You will see that while CO2 is mentioned no less than 112 times, as you’d expect, nowhere in those 13,000 words will you find ANY mention of the greenhouse gas effect/theory. Scientists at PSI who have carefully studied the document assert this to be the most compelling physical evidence ever found proving the GHE as nothing more than a modern (post-1979) political construct – a veritable sky dragon now well and truly slain. After studying the report PSI expert Hans Schreuder adroitly characterizes the tone of it’s authors: “the main theme that jumps out at me is “we don’t know enough.””

            Oh, those studious resident scientists at principia-scientific!!!

            What climate skeptics have been unable to do for years and “mainstream” scientists have been hiding for decades, they resolved for us and for the scientific establishment — nay, for all of humanity!!! — right here on this 20th day of December 2012, just hours before the warmist sinners’ last chance to keep their souls from a veritable fiery CO2-fortified furnace.

            00

          • #
            Bryan

            Jose_X

            I have to agree with you about the 30 year old report.
            John O’Sullivan seems to have some difficulty in reading.
            Any reader would come to the conclusion that PSI shot themselves in the foot.
            PSI are a mixed group, some like Joseph Postma are worth reading others like John are not.

            01

        • #

          Jose,

          The impulse response characterized is that of a system absorbing and releasing energy in response to changing input power where the total energy stored is E as described by the equation,
          Pi = E/tau + dE/dt
          whose impulse response is given by
          E = tau*Pi*(1 - e^(-t/tau))
          Since dT/dt is proportional to dE/dt we can infer how T will respond to an impulse as well. Keep in mind that the simplest form is characterizing the combination of oceans and land as a single pool of energy with an average response. Also, Po (the E/tau term) includes reflection and Pi has a bias. But relative to the response none of this matters much, just as the T^4 dependency is independent of emissivity.

          The impulse response is not very useful since an impulse is an instantaneous change, which at the scale of climate variability only applies to volcanoes and impact events. While this behavior can be seen between day and night, a more useful response to observe is that to a sinusoid. http://ptolemy.eecs.berkeley.edu/eecs20/week9/lti.html.

          This can be measured by considering local seasonal change a sinusoidal Pi as shown here. Note that the variable called albedo in the flux plots is actually the average global reflectivity, while the albedo plots show the albedo which is reflection weighted by relative input power.

          Regarding the 6 trillion degree stuff, the PDO is irrelevant to this analysis. This DE describes the average behavior of a thermodynamic system which is the average of the noise. The PDO is an oscillation in the chaotic transitions between states. This DE only describes the end state.

          I don’t expect to be able to respond again for about a week and a half.

          George

          00

          • #
            Jose_X

            Like I said, I don’t want to spend much time on this at all unless I see something significantly new or if you write something up that is formal. Nevertheless, this reply isn’t exactly short…

            I asked for impulse and impulse response to get an idea of how you would derive the coefficients of your system function. We can do this for LTI.

            You need to pin down the constants in order to compete with the computer modelling. You need to define your system parameters.

            For example, in defining the system, an RC system with a sinusoid disturbance at one end creates a simple lag at the other end, but you have to explain these details. What is this “sinusoid” in nature and how do you measure it or derive it? What is the capacitance value? What is the resistance value? What is the temperature formula that comes from this model?

            Remember, the goal is to come up with a “formula” (eg, as implied by a computer program) that gives values of temp into the future based on information we have now or that are reasonable assumptions.

            The 6 trillion issue arises when you define your T(t) as a simple exponential (as Spencer did in one paper, and as one might from an RC model) and you work your way to historical time periods to see how well your function describes the temperature at those times in the past. An exponential will fail miserably either in the past or in the future, so you may want to avoid making that particular choice of function.

            Write something up formally, so we can read it and judge/critique it more usefully.

            Also remember that even something like a fairly simple but useful digital filter used in sound applications still requires a model that is of much higher complexity than a simple RC circuit. This is one reason why I am surprised you want to model the Earth’s temperature via RC complexity. If you don’t mind being off by say 10% of the final Kelvin temperature in 100 years, then you can come up with a simple model based on ridiculously simplified physics. But such inaccuracy is useless. On the other hand, being even 50% off the delta temp change at least gets you a dog in the race.. if you can argue convincingly that you likely are within 50% and in the right direction.

            If you simply don’t think natural variability can be tamed (understood sufficiently to make useful predictions), then you don’t really have a model and aren’t competing with the computer models. That criticism would be welcomed if based on solid physical theories.

            Also, if you say the ocean and air are simple storage like a capacitor, you should probably provide data to back this and derive the implied capacitance (and/or resistance). Make sure to be precise enough to deal with error bars so that we have an idea if the 100 year projection is useless or if it might be useful.

            00

      • #

        Jinan,

        Therefore we understand that the flux over the CO2 absorption band (~667 cm-1) of the spectrum shown in Figure 7-8 in your link is actually the emission flux of the CO2 molecules in the air layer of the first absorption depth from TOA.

        This is correct, moreover; the emitted flux is half of what was absorbed from the surface where the remainder is returned to the surface. My point is that the origin of this energy is not BB radiation from the N2 and O2 in the atmosphere which was heated by GHG absorption, but re-emission of an energized column of GHG’s acting as a conduit between surface emissions and space (and back to the surface). Consider this in the context of electrons flowing through a conductor. The electrons entering one end of the wire are not necessarily the same ones coming out the other side, but they are coming out the other side at the same rate that they are going in. The energy of photons entering and leaving the atmosphere exhibits the same property which leads to my hypothesis.

        There is a blackbody large sheet at temperature T1, and a detector picking up the emission spectrum.

        Now, we insert another bb patch at temperature T2 in between the large sheet and the detector, what the detector would tell us: over the patch, it is T2, over the rest areas, it is T1.

        If we calculate the absorptivity of the patch, it is of course 1.0. Then you are saying the detector should detect nothing over the patch, i.e. 0 K.

        It T2 was a dense cloud with near unit emissivity and T1 the surface, then T2 would be detected. If T2 is the O2 and N2 in the atmosphere then T1 would be detected since the emissivity of the O2 and N2 in the atmosphere is close to zero. Relative to clouds, T2 is a linear function of T1, thus T2 effectively reduces the emissivity of T1, which can never be reduced to less than 1/2. The current net emissivity is 240/385=0.62 and no amount of incremental GHG’s or clouds can reduce this below 1/2.

        George

        00

        • #

          George:
          Have you ever thought about following scenario: If we send two balls with initial temperature both 15°C, one literally white and the other literally black, into space, what temperature will be for the two balls after a certain period of time.

          Of course the answer is: white ball remains 15°C, while the black one -273.15°C (0 K).

          Now thinking about what temperature CO2 molecules should be if they do not receive radiation from the earth ground surface and if they do not gain heat from neighboring molecules. The answer is -273.15°C (0 K).

          Go further to calculate how much radiation from the earth ground surface can warm up CO2 molecules, if they do not gain heat from molecular collisions with neighboring molecules. You will find the truth.

          ***
          Your calculation of the earth-atmosphere emissivity 0.62 = 240 /385 is not correct. 385 = sigma * (273.15 + 14)^4. However, this 14°C (or 15°C) is largely the temperature of N2 and O2 in the near ground surface whose emissivity is literally 0. So it does not have any relevance with the SB equation. The meaningful surface temperature in this case is the mean temperature of the earth ground temperature and the GHGs in the top air layer (optical thickness from TOA) averaged in terms of radiation. This will lead to a value close to 0.7. Jinan

          00

  • #

    Thanks to Jose X for a very comprehensive reply to my starements last night (3/12/2012).

    I will prepare a fuller response to each of the matters you have raised so clearly. I do enjoy a discussion with someone who is able to make points in the ay you have.

    I agree that much of what I said is speculative and requires mathematical justification. I have worked as a physicist for over thirty years, mainly in electromagnetic theory and spectroscopy, but have taught most university physics courses from relativity to accoustics at al levels. I disd some work also in ionospheric physics earlier on, which of course involved some atmospheric physics and did some things associated with upper atmsopheric air glow – not very relevant to the tropospohere which is the most important part for the green house effect, although radiation from the stratosphere of course is also significant.

    More recently, I have done computer modelling of the absorption of IR in the atmosphere and am still working on the full radiation transfer, including radiation back to earth, involved with the varying density of CO2 with heighrt etc. and studying the effects both in the centre of the spectrum and in the wings of the broadened lines which make up the absorption/radiation bands. The variation of the spectra with height because of changes in pressure broadening is quite significant but is hopefully taken into sccount in some way in the IPCC models.

    I spent quite some time some years ago (like about 8 years) communicating with the various climate groups at ANU, Melbourne, UNSW and CSIRO trying to obtain information on the basic physics they used in making the estimates of CO2 effects for their models. None was able, apparently, to refer me to any significant papers beyond the original speculative hypothesis by Arhenius and a few similar though more modern papers by Hansen, Callendar and others. In the end, Dr Penny Whetton, Head of CSIRO Climate Group in Melbourne which models for the IPCC said only that: “We believe that most of the increase in global temperatures during the second half of the twentieth century, was very likely due to increases in the concentration of atmospheric carbon dioxide”. This statement, from the head of our most prestigious climate group, without any other explanation, left me somewhat perplexed and was the main reason that I decided to do some work of my own, which may of course be of no consequence, and turned me into a fairly committed skeptic.

    I’ll talk to you again soon. John Nicol jonicol18ATbigpondDOTcom

    [Edited your email address to avoid inevitable spam. Mod oggi]

    10

    • #
      Jose_X

      Months ago I downloaded the source code to one of the more referenced general circulation models[*] (I’ll look for the link later). It was a very large amount of computer source code from which I would probably have to infer the physics (which makes it harder yet) if I had no help. If you are really interested in getting to the bottom of this, you may gain a lot by seeking out some of the more specialized researchers in this aspect of the field.

      I would definitely look at the Ramanathan and Coakley (78) paper, which can be found online and downloaded for free. I would also look for books on the topic (eg, radiative transfer, and anything that covers some amount of QED or the like in order to have a stronger appreciation for the details).

      It’s very possible many climate scientists might not be particularly strong in these areas. Also, skepticalscience folks talk periodically of how a lot of progress in the field has been made in the past decade, meaning that more of the folks who know this stuff well might be among the younger researchers and they might not have top positions as some of these institutions.

      My journey has slowed down significantly because I gained enough awareness to allow me mostly to trust the science. It is a lot of work to “prove” this stuff to yourself beyond all reasonable doubt.

      [*] From wikipedia: “A general circulation model (GCM) is a mathematical model of the general circulation of a planetary atmosphere or ocean and based on the Navier–Stokes equations on a rotating sphere with thermodynamic terms for various energy sources (radiation, latent heat). These equations are the basis for complex computer programs commonly used for simulating the atmosphere or ocean of the Earth. Atmospheric and oceanic GCMs (AGCM and OGCM) are key components of global climate models along with sea ice and land-surface components. GCMs and global climate models are widely applied for weather forecasting, understanding the climate, and projecting climate change. Versions designed for decade to century time scale climate applications were originally created by Syukuro Manabe and Kirk Bryan at the Geophysical Fluid Dynamics Laboratory in Princeton, New Jersey.[1] These computationally intensive numerical models are based on the integration of a variety of fluid dynamical, chemical, and sometimes biological equations.”

      00

  • #

    Thanks for your comments Jinan. One suggestion I would make though is that the downwards radiation from the GHGs does not all reach the earth because it is reabsorbed in the lower regions where CO2 density is higher. The result of an “analytic’ calculation which uses a severe approximation that the density of the atmosphere is constant with height, shows that 1/3 rd of the energy radiated by the earth and absorbed by a Green House Gas returns to the earth after much re-absorption and re-radiation. This does not change significantly when the concentration of GHGs changes, which is one of the big fallacies in the global warming hypothesis. Since the atmosphere in fact does vary in density with height, or more importantly the CO2 in the atmosphere varies with height, being denser and hence more absorbing at lower heights, the radiation returned is less than that calculated for a constant density.

    10

    • #

      John;
      Thank you for your reply. To be precise, one needs to calculate the optical length (aka absorption depth) of CO2 for the lowest near surface atmosphere to obtain downwelling radiation. It can also be measured using a photomultiplier. Regards Jinan

      00

  • #
    Greg House

    Jinan Cao says: “Greg; It is expensive to carry out an experiment. […]
    Now for the argument that “colder warmer objects do not absorb radiation emitted from warmer colder objects because it violates the second law of thermodynamics,” we know:
    1) It does not violate the second law of thermodynamics that is irrelevant;
    2) The argument itself violates the absorption law. Radiation waves have only two parameters: wavelength (frequency) and intensity. Objects have no way to tell whether a beam of radiation waves was originated from a cooler or warmer body.

    If you are not convinced and you prefer to see experimental results, why not? You can easily design an experiment to put your inquiry to the nature. Regards.”
    ===============================================

    Jinan Cao, in real science it is not so that a fiction is considered a scientific fact until proved otherwise.

    Second, I understand your ambiguous “it is expensive to carry out an experiment” as “there is no experiment confirming colder bodies warm warmer bodies”. Given enormous amount of money warmists have at their disposal, this “expensive” sounds simply ridiculous. Probably, this assertion is false and therefore impossible to prove right, this must be the real reason, and therefore neither you nor warmists I talked to on various blogs can present anything real.

    Third, the talk about a “beam of radiation waves” and “objects telling the origin” is just a product of imagination. The cardinal point is, what effect IR radiation really has depending on certain conditions, not how you imagine it works. I hope you can understand the difference. So, if you can not prove what you imagine to be right, then you have only produced a fiction, a science fiction, if you like, but not a scientific fact.

    00

    • #

      Greg;
      Your theme that warmer objects do not absorb radiation emitted from colder objects because it violates the second law of thermodynamics,” is an interpretation of yours. I offered a different interpretation saying it does not violate the second law of thermodynamics for warmer objects to absorb radiation emitted from colder objects, because the second law of thermodynamics is an irrelevant physics law in this case – you have misinterpreted the second law of thermodynamics.

      Physics laws are all a result of numerous experimental and theoretical studies. With regard to the argument about experiment being expensive, I meant any specific experiments in your mind that you would like to see. When one can work out the answer to a specific inquiry, there usually is no need for experiments. If you will not satisfy without seeing the result of any specific experiment in your mind, nothing prevents you from doing so.

      In your third paragraph, you state “The cardinal point is, what effect IR radiation really has depending on certain conditions, not how you imagine it works.” This is not clear enough in term of physics. Would you please spell it out what those certain conditions are? Regards

      00

      • #
        Greg House

        Jinan Cao says:”Greg;
        Your theme that warmer objects do not absorb radiation emitted from colder objects because it violates the second law of thermodynamics,…”

        ===============================================
        No, this is not my theme at all, and I never said that. What I said was that the warmists’ interpretation (the “net” thing) was unproven.

        Second, if a statement does not violate the 2nd Law it does not prove that this statement is correct. Example: the statement “there is life on Mars” does not violate the 2nd Law, but is it therefore a scientific fact? It is a simple logical issue, Jinan Cao.

        If you claim that IR from colder bodies slows down cooling of warmer bodies, you have to prove it before selling it as a scientific fact. Maybe there is some negligible effect, or a strong effect, or no effect at all. If you have nothing experimental to present, just admit it, it would be nice. Or present an experimental proof that it REALLY works.

        00

        • #
          Jose_X

          >> What I said was that the warmists’ interpretation (the “net” thing) was unproven.

          Be more specific, with math preferably, and say what you believe is incorrect. Show the experiments that you think contradicts.

          I’ll be waiting.. otherwise, I would hope you send me some money if you want me to keep repeating the points I have been making or else bring up a new point.

          Oh, and prove you are alive while you are at it if you think it’s so easy to prove things beyond all doubt because I am starting to wonder if you aren’t a very stubborn computer program.

          >> If you claim that IR from colder bodies slows down cooling of warmer bodies, you have to prove it before selling it as a scientific fact.

          It’s not just us, many scientists make those claims.

          Prove the second law of thermodynamics. I’ll be waiting. And until you prove it, you should consider no longer invoking it. In fact, you should perhaps consider applying that standard to yourself and first proving everything you want to claim or else don’t claim it.

          Anyway, so if you don’t prove the second law of thermodynamics, it will seem to me you are invoking a lie or else are just going by faith and what you want to believe. That is not how the scientific method works.

          BTW, as for colder slowing down rate of warmth of warmer, consider this thought experiment (a thought experiment I would love to carry out for you in real life if you give me a few billion dollars so I can bribe NASA or any of those going into space.. yes, it can be expensive to carry out some experiments.. let me know if you know how to get a good discount on that one):

          A body of mass 100 kg and temp 100 C and the shape of a prism 100 cm x 100cm x .01 cm is in space isolated from other sources of significant radiant energy except for another equally shaped body at 100 kg and at temp a little higher, 100.01 C.

          If only the second body was there, it would radiate heat so that after 10 seconds it might be at temp T1 (T1 < 100.01 C).

          Now, with this first body near it, we can now assume they just about touch. In this case, we can expect that the second body won't cool down that fast — just as we are claiming and you believe can't happen. Why would it cool down slower when the cooler first body is near it?

          .. because we effectively have a mass of 200 kg (both together) at 100 C (100.005 C). The surface area of radiation in the first case was 100*100 * 2 sides + .01*100 * 4 sides = 20004 cm^2 and in the second case would be 20008 cm^2 or virtually the same.

          So we have (a) an object of mass 100, temp 100.01 and surface area 20004 vs (b) object of mass 200, temp 100.005 (average) and surface area 20008.

          If the first goes to T1 in 10 seconds, the second will have dropped not as far in temp because it has about twice as much energy it would need to radiate to cool to T1 but can only radiate through an "opening" that is about the same size as before.

          I skipped the math (calculus) of the last steps, but let me know if you really don't believe my conclusion.

          Remember, total energy radiated in a split second for a body as specified by Stefan Boltzmann depends on temperature, surface area, and material (emissivity) only, so we can expect that in a very small instant of time these two cases (a and b) radiated the same amount of energy just about (we can reduce the 20004 20008 discrepancy further if you want). Meanwhile, b will nevertheless require twice the energy loss in order to go down the same temperature in that instant of time. This implies b will cool down slower in that first instant and ever thereafter.

          I hesitate to give more details at the moment since I don't know how comfortable you are with limits of series or with calculus.

          So, please let me know if you are convinced and now agree with the "warmists" or if you still are not convinced and then please specify why not.

          00

          • #
            Jose_X

            In case the experiment above is a little unclear, let me summarize the point:

            If a body in space losses heat at a given rate, then if we double the mass, it will take longer to radiate its heat away. I simulated the doubling of mass by considering the object to be something thin (think of a sheet of paper) and then putting two pieces next to each other to resemble the first. In the new case, I hardly changed the surface area and hence hardly changed the rate of heat loss, but I doubled the mass so it should take measurably longer to drop in temperature.

            If you believe the above (if not, please specify why not.. but you can probably imagine that a thin cold steel spherical shell would heat up, in a warm room or if we touch a hot plate, faster than would a cold solid steel sphere).. if you believe the above, then we see that the second case can have the sheets close enough not to touch and conduct or convect in space (so only radiation is at play). Also, one of the sheets is a dash cooler than the other.. yet that won’t change the fact that the hotter sheet will radiate slower (aka, cool slower).

            In isolated space, a hot sheet and a cold sheet really close to each other drop in temp slower than if only one sheet is by itself. The nearby cold sheet slows down the temperature drop of the second warmer sheet.

            00

        • #

          Since the 2nd law of thermodynamics is irrelevant with absorption of radiation, the argument that the 2nd law of thermodynamics states “NET” heat flow is always from the warmer to the colder” is equally incorrect. It is an over-interpretation of the 2nd law of thermodynamics.

          For your second paragraph: I know that, Greg. Disproof of the statement that the colder/warmer violates the 2nd law of thermodynamics does not constitute of a proof of the GHG warming theory. They are two separate issues. However, we cannot use false arguments to disprove anything including the GHG warming theory.

          3rd: no need to run a circle.

          00

          • #
            Jose_X

            With respect to the second part, the fact remains that no one here is proving very much. Generally, scientists gather evidence and mathematical/logical reasoning that appears to be consistent with evidence and with itself. That is the best science can do.

            I will support the theory that appears complete enough and superior. From what I have seen (limited no doubt), AGW appears to fall into this camp. In contrast, I see no coherent theory that challenges that so as to replace it. At most, and as is good, the skeptical observer contributes to improve something, but it is rare to just overturn something or make a monumental change to it. Regardless, until that day comes with a competing theory that is substantial, I’ll stick with what is already acceptable in quality… even if it can’t be “proven beyond a shadow of a doubt”.

            00

          • #
            Greg House

            This is better, Jinan Cao, thank you. To your “we cannot use false arguments to disprove anything including the GHG warming theory”, well, this is self evident. And yes, the argument “GHG warming contradicts the 2nd law (or Clausius statement)” does not seem convincing to me, but for a different reason. The laws of thermodynamics were formulated on the basis of experiments and I do not know, whether they experimented with IR radiation or not. Just for that reason. But similarly, we can not use illogical and therefore false arguments to “prove” the alleged GHG warming, like the one about “not contradicting the 2nd Law”.

            Anyway, the main point is whether back radiation can warm (slow down cooling, whatever) or not, in other words, whether the effect is negligible or strong or zero. “Warming via back radiation” is the political relevant version of the so called “greenhouse effect” as presented by the IPCC, I guess you know that. And there is apparently no single experiment proving the warmists’ assertion.

            I look forward to the “expensive”-counterargument (lol).

            00

          • #
            Jose_X

            >> And there is apparently “no single experiment proving the warmists’ assertion”.

            But the evidence and theoretical analysis supports that idea more than most any other that has been put forth.

            00

          • #
            Jose_X

            Climate scientists don’t claim something bad will necessarily happen. They have produced a probability distribution, with a significant part of that representing a likely danger zone to many forms of life and human subpopulations. True, along the way, humans are doing plenty to hurt ourselves (overpopulation, fresh water exhaustion, environmental and body poisoning, ..).

            00

          • #

            Greg;

            Now you are convinced that it does not violate the 2nd law of thermodynamics for a warmer body to absorb radiation emitted from a cooler body; no need to see any experimental results any more. It is a progress. The conversation I made with you was all about this specifically defined theme. Discussions of different issues can take place in other occasions.

            00

    • #
      Jose_X

      Greg House:
      In real science… you respect the peer reviewed and widely taught literature; you understand that those ideas do have laboratory and theoretical backing; you make a diligent effort to figure out what that backing is (you read books and ask in a school not in random Internet forums where the right people are not hanging out and the people who are only have so much time to help you for free); and you understand that the burden of proof lies with you who is claiming differently than those widely accepted theories.

      >> Given enormous amount of money warmists have at their disposal,

      The fossil fuel industry makes in profits hundreds of billions yearly, and they spend (and write off) at least that much, some of which presumably is to fund skeptics to theories that threaten their very lucrative business.

      Government money set aside for climate science is much less than that.. not to mention it is also to provide services that even skeptics use (eg, the data from satellites and what not).

      >> this “expensive” sounds simply ridiculous.

      Do you know how much money it would cost to replicate the earth’s atmosphere and biosphere in a laboratory? Hint, it’s more than 1,000,000 times a billion.

      >> and therefore neither you nor warmists I talked to on various blogs can present anything real.

      I think you can gain by re-evaluating your understanding of science and the money that flows to it. Then your gain in that area might be converted into greater understanding of what “warmist” are saying.

      >> The cardinal point is, what effect IR radiation really has depending on certain conditions

      You seem not to know. Nor are you offering a theory with experiment to support it.

      Meanwhile, “warmist” are generally just telling you that great theories exist and have backing and if you would bother to read and study and recreate those experiments (and it would be costly and time consuming), then you would have a better idea about what works and has been used to drive our society into the current information age and advancement.

      Pay me, and I will take the time to walk you through the experiments in some of my past lab courses and walk you through the mathematics in some of my theory courses.

      Meanwhile, I wait for you to present your alternative theory and the results from your alternative experiments. [I do have other things to do besides post comments here, so I’ll try to resist the temptation to keep repeating myself, at least not until you send me over some money.]

      01

      • #
        Greg House

        Thank you, Jose_X, I do not even need to touch this posting of yours, it is so wonderfully revealing.

        11

        • #
          Jose_X

          That’s your choice, but this and my other recent comment (with some basic math) is in my opinion a legitimate rebuttal to your claims that “warmist” are not putting up and so should be ignored or whatever it is you are suggesting.

          I asked of you what you have been asking of “warmists”. Until you come through, I don’t think your words mean an iota more than the “warmists” you criticize.

          If you decide to get serious and raise the argumentation to something workable, then I’ll be willing to try some more (for free). But if you require “proofs” for everything basic that is already accepted by science, make sure you come prepared to prove everything you say or you will not have credibility.

          00

  • #
    Greg House

    Jose_X says: “Climate scientists don’t claim something bad will necessarily happen. … True, along the way, humans are doing plenty to hurt ourselves (overpopulation, fresh water exhaustion, environmental and body poisoning, ..).”
    ====================================================

    And that should justify lying about “greenhouse effect”?

    I guess I know how green radicals are going to help us. This is what Jacques-Yves Cousteau said in an interview to the Unesco Courier (The Unesco Courier, November 1991, p.13 http://unesdoc.unesco.org/images/0009/000902/090256eo.pdf):

    “World population must be stabilized and to do that we must eliminate 350,000 people per day.”

    00

    • #
      Jose_X

      >> And that should justify lying about “greenhouse effect”?

      Again, find proof that anyone is lying. If you haven’t studied and looked at the evidence and theory details, you should not accuse others of lying.

      00

  • #
    Greg House

    Jinan Cao says: “Greg;
    Now you are convinced that it does not violate the 2nd law of thermodynamics for a warmer body to absorb radiation emitted from a cooler body;”

    ==============================================

    This is absolutely not true.

    I did not say that and it can not be derived from what I said. Is it possible, Jinan Cao, that you have some serious logical issues? Sounds unusual for a scientist.

    I said that from what I know the alleged violation of the 2nd Law by warmists did not seem proven to me. At the same time their statement that their “greenhouse gas warming” does not violate the 2nd Law seems unproven to me as well.

    OK, let me give you a simple example to illustrate that. I have no evidence that you drink coffee, but it does not mean that I am convinced that you do not drink coffee. This is simple logic, Jinan Cao.

    00

    • #

      Greg;

      We now understand your argument better:

      The argument that it violates the second law of thermodynamics for a warmer object to absorb radiation emitted from a cooler object is not proven to you on one hand;

      Yet you believe the argument is valid, and has good chances to be proven later on the other.

      Why do not you carry out a theoretical and/or experimental study to either prove or disprove the argument?

      00

      • #
        Greg House

        Jinan Cao says: “Greg;
        We now understand your argument better:
        The argument that it violates the second law of thermodynamics for a warmer object to absorb radiation emitted from a cooler object is not proven to you on one hand;”

        =============================================

        I just hope that you keep distorting my argumentation unintentionally.

        I talked earlier on this thread in my reply to you specifically about the alleged effect on temperature and not about absorption: “The bone of contention is not absorption as such, it is if there is an effect on temperature or not. Do you have anything real experimental on “IR from colder bodies slows down cooling of warmer bodies”?”

        And let me assure you on this occasion: the issue about warmist being apparently unable to present a single real scientific experiment confirming their key assertion about warming via back radiation will not go away.

        00

        • #
          Jose_X

          Greg House,

          Explain the readings on pyrgeometers even within a 10% error marker. Why do those readings show when pointed up at the sky both during the day and night?

          It’s not clear from your comment if you want (a) direct evidence of back radiation, or (b) of greenhouse effect, or (c) that 2nd law in not violated. All of these things are different. Pick something and hopefully explain what experimental evidence would convince you based on your knowledge of modern physical theories used in climate science and in many other sciences.

          Assuming you want to attack the problem first of whether back radiation exists:

          Google “scienceofdoom pyrgeometer” and most things in the first page will be useful to the discussion.

          Eg, see the article titled:

          “Understanding Atmospheric Radiation and the “Greenhouse” Effect – Part One” Measurements supporting radiation formulas.

          “Heat Transfer Basics – Part Zero” if you are in the mood for some math and numerous experiments (on paper.. but at least some should extend to real life if desired). There is a bit of coverage of radiation.

          “Back Radiation” this provides 3 links to each of 3 parts of the series “The Amazing Case of ‘Back-Radiation’ “. You will see graphs of measurements done by professionals.

          I posted earlier on this article many months ago data from pyrgeometers (eg, value by California coast was similar to Trenberth numbers; day and night, suggesting radiation is coming from atmosphere) and from pyranometer (very high during peak sun and about 0 during the night) and asked some questions key to this discussion. I would have to search for those comments and links if you want.

          The evidence is strong and diverse for back radiation. For example, precision spectroscopy
          of gas emissions and absorption helped support and develop theoretical basis and provides data useful to computers to calculate things like the greenhouse effect much more accurately than can a human with calculator.

          I already mentioned the Ramanathan and Coakley 78 paper which you appear not to want to read, although this is more for looking at how to derive atmosphere temperatures that approx the real thing.

          00

          • #
            Greg House

            Jose_X says: “Greg House,
            Assuming you want to attack the problem first of whether back radiation exists: …”

            ============================================

            There is no need to assume that, because I never questioned existence of back radiation.

            it is funny to watch, how you and Jinan Cao keep distorting my argumentation.

            I just asked you in my previous comment (#157.2.1): “Please, do not waste your time and patience of the readers by talking of anything other than experimentally proven changes in temperature.”

            00

          • #
            Jose_X

            Greg House
            I stated that it would be prohibitively costly to simulate the earth environment at a realistic scale in a lab in order to control the knobs and actually measure the effects on temperature from 2xCO2, after the fake earth has fully adjusted, and come up with 3C or whatever. So when you repeat that you want an experiment, and I tell you it costs too much money for any nation to undertake it, you reply still asking for that evidence.. is this right?

            00

        • #

          Greg;
          Your point of view is getting harder to understand. Can I confirm that you mean:

          You agree that it does not violate the 2nd law of thermodynamics for a warmer object to absorb radiation emitted from a cooler object;

          But such absorption of radiation emitted from a cooler object does not have any temperature effect for the warmer object.

          If this is your argument, where does the absorbed radiation energy for the warmer object go in your opinion?

          00

    • #
      Jose_X

      Greg House,

      Let’s say you want to put effort behind a theory that the photons from the cooler object don’t jump to the warmer one but maybe just go up to it, sniff its temperature, and return if it’s too hot .. but there has to be an exception, right? For example does an infrared camera have to be colder in order to sense (see) objects? If I use a warm camera will it stop working? If it does work, then do you believe in conservation of energy or not? If so, then photons or some form of energy from the colder object have to be absorbed into the camera, right? But maybe that is OK if there is a circuit to draw in the photons; however, this would lead to more work being done by the colder camera object than is done to it, which is the opposite of the 2nd law.. unless there is an “ether” that senses all of this happening and then compensates by adding the right work or energy balance so that the second law is satisfied, but does this mean the ether is not subject to the 2nd law? Maybe a deiti causes this to happen behind the scenes. .. Of course, maybe you don’t believe in conservation of energy.

      Whatever theory you want to believe, unless you develop it precisely (eg, using math and how this math connects to the real world) and allow it to be tested in many conditions by others, and unless it passes, you will be one of the few people walking around believing it. Scientists won’t if they can’t use your theory to get correct answers about nature.

      So if you decide to believe a theory that cold cannot give photons to warm, then you will not be able to have a meaningful conversation with the majority of scientists who follow the other and well developed theory.. at least not until you first develop your theory, math and all, and it can be shown to pass experimental test after experimental test.

      For example, I spoke of the 2 sheets in space radiating. That is an experiment that could be tried if people doubted the physics (that has already been shown to “work” under different scenarios). Seeing that in fact the double sheet drops in temperature slower (after 10, 20, … seconds) would be consistent with the photon radiation ideas of modern physics and increase the confidence in that theory further.

      Let me ask, does that sheet experiment violate the 2nd law? Well, to be precise, we would have to be more detailed in describing how we got those sheets in place (since that requires work) and at what point in time we measured their temperature and how. We would use error bars (uncertainty interval) and carry out the math to end up with an answer that presumably would end up entirely within the range expected even when considering the uncertainty. Anyway, the theoretical prediction is that the warmer sheet gives the slightly cooler sheet more photons and so heat would be flowing from that warmer to the colder, just as required by the 2nd law.

      Good luck developing your theory and experiments so that one day maybe we can test it out and maybe even attack the greenhouse theories to see if with your theory we get the same conclusion or a different one. If we get the same, that will help validate both your theory and the modern one used by climate scientists. If you get a different conclusion from applying your equations and math, then we have a real challenge on our hands and someone will come up with an experiment to help pick the winner. Good luck!

      PS: Remember, if you don’t want to study, analyze, experiment, or instead simply accept the science used by climate scientists and if you won’t develop your competing theory so that climate scientists can study it and test it out, then you and climate scientists will not be able to communicate very well. This may sound cold, but it’s the truth.

      00

      • #
        Greg House

        Jose_X says:Greg House,
        Let’s say you want to put effort behind a theory that the photons from the cooler object don’t jump to the warmer one but maybe just go up to it, sniff its temperature, and return if it’s too hot .. but there has to be an exception, right? For example does an infrared camera have to be colder in order to sense (see) objects? If I use a warm camera will it stop working?

        ===============================================

        I know argumentation about IR cameras, IR remotes etc. IR is obviously detectable, but the point in question is about changes in temperature, not about absorption and detection.

        Second, as I said earlier on this thread to Jinan Cao: “If you claim that IR from colder bodies slows down cooling of warmer bodies, you have to prove it before selling it as a scientific fact. Maybe there is some negligible effect, or a strong effect, or no effect at all. If you have nothing experimental to present, just admit it, it would be nice. Or present an experimental proof that it REALLY works.”

        So, please, tell the audience how exactly colder objects slow down cooling of a warm IR camera or it’s sensor, go ahead, do nor forget, please, to present scientifically proven numbers.

        By the way, if you guys e.g. announced worldwide that your “greenhouse effect” warms by like, let us say, 0.0000000000000001C, I would be ready to kind of “settle” and not further torture you with my legitimate scientific questions.

        Please, do not waste your time and patience of the readers by talking of anything other than experimentally proven changes in temperature.

        00

        • #
          Jose_X

          So you are saying you believe back radiation of infrared comes down from the atmosphere, both day and night? Do you trust that this value (global average) is around what Trenberth et al show.. namely around 333 W/m^2?

          Are you also saying that, in believing that this really exists in the real world, that it does not violate the 2nd law of thermodynamics?

          And you believe that back radiation is photons emitted from GHG, right?

          I don’t know what experiment to think of that will satisfy you if I don’t know what physics you accept (eg, as asked just above). If you don’t want to come up with an experiment yourself, I will need help figuring out what your mind will understand.

          So to proceed further, given how you don’t seem to understand or want to read about the evidence in books about the greenhouse effect being taught all over the world (and it is difficult reading, eg, quantum mechanics), I will have to ask you questions like the above and then require you answer for me to know what experiment to describe to you that you will hopefully understand and believe (and I may very well have to read and learn as well.. so as I have time and interest, I will try to find for you this simple experiment).

          00

          • #
            Greg House

            Jose_X says (#157.2.1.1): “I will have to ask you questions like the above and then require you answer for me to know what experiment to describe to you […].. so as I have time and interest, I will try to find for you this simple experiment).”

            Jose_X says (#157.1.1.1.2): “Greg House
            …So when you repeat that you want an experiment, and I tell you it costs too much money for any nation to undertake it, you reply still asking for that evidence.. is this right?”

            ===========================================

            The time interval between those 2 comments of yours was only 7 minutes. And you so blatantly contradict yourself. Unbelievable.

            00

          • #
            Jose_X

            There is no contradiction.

            You are not clear about what you want. [Notice you are claiming the greenhouse effect from more CO2 does not produce significant temp changes on the surface of the planet even though all the major universities and scientific programs accept it. Take that as a clue that perhaps you don’t understand scientific experiments as well as you think you do.]

            And I notice you answered none of my questions, again.

            I have to take care of things tonight and get to bed to see if I can sleep a little.

            If you are serious, please pick some questions I have asked and answer them or it. I will try to read up tomorrow.

            Start with this one that I already asked:

            > I stated that it would be prohibitively costly to simulate the earth environment at a realistic scale in a lab in order to control the knobs and actually measure the effects on temperature from 2xCO2, after the fake earth has fully adjusted, and come up with 3C or whatever. So when you repeat that you want an experiment, and I tell you it costs too much money for any nation to undertake it, you reply still asking for that evidence.. is this right?

            00

          • #

            Jose,

            But as I said before, Trenberth’s conflation of the cloudy sky and the clear sky obscures the fact that most of the radiative component of what he euphemistically calls ‘back radiation’ is Planck radiation originating from the liquid water in clouds as they radiate a flux back to the surface equal to half of what they capture as strong broad band absorbers of LWIR energy emitted by the surface and as weak broad band absorbers or solar energy.

            The radiative component of ‘back radiation’ is not much more a than 385 – 239 = 146 W/m^2, less than half of this has a GHG origin and only a third of this is CO2 where the rest is from clouds. The remainder of his ‘back radiation’ is not returned to the surface as photons, but is transported by the matter flux as the temperature of rain returning to the surface, the potential energy of water lifted against the force of gravity and weather. A tiny portion of this return path has been tapped for wind and hydro power.

            George

            00

          • #
            Jose_X

            [I posted most of this comment in the wrong place (sorry), so I am re-posting here.]

            >> The radiative component of ‘back radiation’ is not much more a than 385 – 239 = 146 W/m^2

            This is your hypothesis in action — perhaps believed by just one person and perhaps without documentation to clearly explain to others that alternative science.

            Meanwhile, scientists who have built up collectively a huge amount of theory and experiment believe energy changes forms and that there is no unmixable matter flux vs photon flux. Using the tools of the widely accepted science (including measuring instruments), the back radiation levels are above 300, and this makes sense because going up at ground level is not just 396 (as per Trenberth diagram) but also 17 and 80, and these all mix.

            161+333 [494] ~= 17+80+396 [493]

            Where 161 is the part of 239 that reaches the ground and is not absorbed mainly by O3 I think.
            333 is the GHG backradiation. The two together represent the total radiation hitting the ground.

            Where 17 is the “net” from thermals (convection, etc), 80 is the “net” from evaporation (and plants I think), and 396 is ground/water radiation. Together, these 3 represent radiation leaving the ground.

            Note that 17 and 80 represent the net result since we do have convection currents bringing heat to the ground as well as removing it, depending on the location on the planet and time of day. And similarly there would be some heat from rain, etc. [Note how rain tends to be cool since I think some the energy has usually been radiated away higher up in its trip above ground.]

            co2isnotevil, I think if you are willing to take the time to try and develop an alternative theory, then we all benefit from that in the long run, whether you fail or succeed; however, until you succeed, it doesn’t make too much sense to expect people to believe your hypothesis instead of the science practiced and shared by many many scientists (not just climate scientists since many of these ideas are fundamental to physics).

            00

        • #
          Jose_X

          >> The radiative component of ‘back radiation’ is not much more a than 385 – 239 = 146 W/m^2

          This is your hypothesis in action — perhaps believed by just one person and perhaps without documentation to clearly explain to others that alternative science.

          Meanwhile, scientists who have built up collectively a huge amount of theory and experiment believe energy changes forms and that there is no unmixable matter flux vs photon flux. Using the tools of the widely accepted science (including measuring instruments), the back radiation levels are above 300, and this makes sense because going up at ground level is not just 396 (as per Trenberth diagram) but also 17 and 80, and these all mix.

          161+333 [494] ~= 17+80+396 [493]

          Where 161 is the part of 239 that reaches the ground and is not absorbed mainly by O3 I think.
          333 is the GHG backradiation. The two together represent the total radiation hitting the ground.

          Where 17 is the “net” from thermals (convection, etc), 80 is the “net” from evaporation (and plants I think), and 396 is ground/water radiation. Together, these 3 represent radiation leaving the ground.

          Note that 17 and 80 represent the net result since we do have convection currents bringing heat to the ground as well as removing it, depending on the location on the planet and time of day. And similarly there would be some heat from rain, etc. [Note how rain tends to be cool since I think some the energy has usually been radiated away higher up in its trip above ground.]

          00

  • #

    I note a lot of comment regarding the change of energy from one form to another.

    It is an axiom in physics that energy is only “observed” when it changes from one form to another. Think about it. Any detector of temperature, radiation, pressure increase etc, requires the expenditure of some (small) amount of energy from the source.

    So yes, energy does work only when it changes form, from kinetic to potential, potential to kinetic, acceleration of charges to radiation, radiation to internal energy of molecules and so on. There are no exceptions!

    00

  • #

    Second law does not say that the time-averaged heat flow will allow the cooler reservoir to become COOLER and another reservoir to become WARMER, not without the expenditure of work, anyway.

    Yeah, and there is work done to produce heat by a little thing called the Sun and to a lesser degree by decay of radiactive elements in the Earth’s core and elsewhere.

    I have no idea how anyone can claim that insulation (through absorption and re-emitting in the case of CO2) doesn’t cause a heated body (in both senses of the word) to be warmer than it otherwise would.

    Throw on a few jumpers (i.e., sweaters) and get back to me with the results of your science experiment.

    01

    • #
      Mark D.

      I have no idea how anyone can claim that insulation (through absorption and re-emitting in the case of CO2) doesn’t cause a heated body (in both senses of the word) to be warmer than it otherwise would.

      Here is one idea for you: water vapor, convection, condensation, repeat.

      Throw on a few jumpers (i.e., sweaters) and get back to me with the results of your science experiment.

      How exactly is a jumper like the atmosphere? How is it modeling water vapor, convection, condensation? Try a little harder because the atmosphere is just a bit more dynamic than your jumper analogy.

      00

      • #
        Jose_X

        >> How exactly is a jumper like the atmosphere? How is it modeling water vapor, convection, condensation?

        To analyze the physics, we don’t assume something is something else rather different; however, people have analyzed the physics and have concluded the atmosphere behaves a certain way. The result appears to show that the atmosphere behaves like an ordinary insulator in many ways.

        I used those analogies because of a number of claims made in comments here that thermo 2nd law was being violated (the title of this article), but when we look at the insulation cases, we see similar patterns and how thermo 2nd law is potentially no more violated in the one case as in the other.

        00

        • #
          Mark D.

          Jose-X

          Is it too hard for you to use the b-quote function?

          Someone that claims to know so much about the complex climate really ought to be able to click on one fricking button yes?

          Now you start to argue about “insulation” but fail completely to address my question.

          Two strikes for you.

          00

          • #
            Jose_X

            How is a jumper like the atmosphere? The atmosphere, via ghg effect keeps air surface temperatures inside it (averaged around the globe) warmer than they would otherwise if the surface were exposed directly to space. I thought you knew that, so I didn’t think it was necessary to state it again. A lot of research and math has been done on this problem. If you don’t see or believe this effect, you have an uphill battle in writing mathematically and physically sound papers that will pass peer muster. ..Oh, and these people have modeled water vapor and the rest, though some areas are more difficult to model. Indications are that clouds and what not are not the overriding influence or that the likelihood is towards them enhancing the effect rather than diminishing it.

            00

          • #

            Oh, and these people have modeled water vapor and the rest, though some areas are more difficult to model. Indications are that clouds and what not are not the overriding influence or that the likelihood is towards them enhancing the effect rather than diminishing it.

            There are differing opinions on the effects of clouds. My understanding that low-cloud cover, particular the vast areas of clouds you often see beneath you when you fly in an airplane, primarily add to cooling by increasing the albedo of the Earth, since they are white and reflect Sunlight back to space.

            Personally I think Henrik Svensmark’s model, which is based around clouds, describes the two drivers of at least two important climate-variation cycles. Of course, there are others.

            00

          • #
          • #

            That’s an interesting article, JoseX. Thanks for passing it along.

            One thing I’ll point out is that Henrik Svensmark’s model primarily focuses on variations of low-clouds (mostly over the oceans). These would be the ones which lend themselves to a cooling effect.

            Of course, the mere fact that’s where he focuses doesn’t mean that his model is right, but he is talking about variability in low-cloud cover specifically.

            00

          • #
            Mark D.

            Gee did you two forget the title of this thread? It is about how back radiation wouldn’t violate the second law. Now are you both saying jumpers back radiate like Co2? Is a gaseous mixture the same as fabric woven ut of solids? Of course there is really no comparison to be made between the Earths atmosphere and a jumper.

            And back to my point and it isn’t just about clouds. Heated air is free to move and does so rather impressively. Water present adds substantially to the heat carrying capacity of the air. At altitude radiant energy to space has much less “insulation” than it does at the surface. Clouds are yet another variable that work as insulators, heat storage “vessels”, and can be radiant absorbers, emitters as well as reflectors. Movement of energy is done through the phase change of water and the compression/expansion of air as it is forced up and down elevation. What you have is a heat engine of immense proportion. Anyone that claims all this has been adequately modeled much less measured is simply ignorant.

            What little bit part Co2 plays would be impossible to measure given what is described above

            Jose, the box experiment is useless in any attempt to simulate the atmosphere. Both of you, the jumper analogy is worse than wrong it is misleading. Sounds real good to a half wit, good enough that they go right on believing the crap obscured by it.

            01

          • #
            Jose_X

            >> Now are you both saying jumpers back radiate like Co2?

            >> Of course there is really no comparison to be made between the Earths atmosphere and a jumper.

            Riiight. We can’t compare cats and dogs because they are so obviously different. But wait, they are both pets, have 4 legs, etc.

            Where did I say that jumpers and CO2 work the same way? In calling them insulators, I am saying that they each achieve, within their respective domains, an effect that have numerous similar traits (insulation).

            >> Anyone that claims all this has been adequately modeled much less measured is simply ignorant.

            If you want to take it up with the climate science community that they can’t possibly know what they are doing or say anything meaningful about average temp rises from CO2 because the great water engine is so complex, then go for it. Unless you provide a better analysis than that paragraph you just wrote, you will be laughed or more likely ignored.

            >> What little bit part Co2 plays would be impossible to measure given what is described above

            Again, if your “proof” is that paragraph, prepare to be ignored by the science community and even most laypeople whom you might want to impress or convince.

            >> Jose, the box experiment is useless in any attempt to simulate the atmosphere.

            Well it’s nice you agree with me.

            >> Both of you, the jumper analogy is worse than wrong it is misleading.

            Your opinion backed by all of that one paragraph and no measurements, analysis, or math doesn’t worry me much, and I expect few climate scientists will worry either. Thanks.

            10

          • #

            While one can argue about the effects of this factor or that factor in a complex system, I believe we can all agree that the central point of this post, to wit that GHGs are not a violation of the second law of thermodynamics, is a true statement — that they can, in principle, exist.

            Now, whether a given gas is a GHG or not is a matter for experimentation.

            00

          • #
            Mark D.

            Jose:

            If you want to take it up with the climate science community that they can’t possibly know what they are doing or say anything meaningful about average temp rises from CO2 because the great water engine is so complex, then go for it. Unless you provide a better analysis than that paragraph you just wrote, you will be laughed or more likely ignored.

            Really all of the content in your reply (that this quote is taken from) is rather thinly veiled Argument from Authority.

            If you think I’m going to type a thesis here in a blog you’re mistaken’s not my job and my Big Oil check has been lost by the mail. That a warmist or climate scientist would laugh or ignore the elephant in the room is precisely THE point. The elephant is still there Jose! Stop ignoring it.

            Look if all these discussions started out with this: At our present stage of ignorance,…… then there would be a lot less antagonism. But no one ever does that, they say things like “well understood”.

            As it is I’ve managed to befuddle you with two or three posts. You have NOT argued towards proving what I said wrong you simply waived it off.

            Ha Ha. That won’t work

            Christoph, Yes I principally agree with what seems obvious; that colder objects (above absolute zero) radiate. That radiant energy has no free will to avoid being absorbed by another even warmer object.

            The jumper analogy is dumb you shouldn’t use it.

            00

          • #
            Jose_X

            >> Argument from Authority

            Ya, so if we are going to talk about friction, unless I sit here and explain the series of experiments that have led to discarding various theories on friction in order to stay with some other, then I am AfA simply because I would appeal to material spread widely in texts and presumably accepted by most scientists doing modern research? And for every physical effect, I have to go through this ordeal or else I am AfA?

            I don’t mind discussing particulars, but the fact you shun an entire branch (and some of its roots from other disciplines) is not the point because I am not and can’t prove this stuff to you. I am trying to be realistic about what you should expect out there (and also from myself if you want to present something that is detailed and which I can follow). Many have created the theories over many years. Simply thinking things are too complex is not going to sway many. Finding specific flaws will.

            >> “At our present stage of ignorance”

            Most papers have error ranges, some very widely.

            I know that quoted language may bother you (ie, a person), but if you spent time with the papers, you would be thinking towards constructive criticism and would understand that publish papers are a lot more conservative than are blogs and the media. Scientists disagree on their backing or rejecting of specific points, but that won’t stop many of them from agreeing generally to a poll about basic views in the field, and it won’t stop lots of non-experts from saying many things that are not backed by evidence and muddying the issue. This is why I mention that looking at various text books or at a range of papers may help and is ultimately what one has to do to have a credible rebuttal that doesn’t make obvious mistakes and assumptions from ignorance.

            The onus is on those rejecting an established science. The scientists already have their standards, but whose fault is it that most who reject the main ideas are the people who know least about what has actually been written in those papers and in those text books?

            >> You have NOT argued towards proving what I said wrong you simply waived it off.

            I think I “waived off” that if one has gone through the trouble to detail the physics of ghg and to perform many calculations (at different levels of detail.. from back of envelope to months-running computer programs) to support various conclusions, then we can accept those conclusions for the time being (until someone does a better job arguing against them) and try to find analogies to communicate the effect in order to clarify some aspect of those conclusions.

            Look, Anthony Watts finally got himself dirty by submitting a paper. I think he found out that things aren’t as obviously wrong as he assumed in that paper. That is what a skeptic should do or else keep studying and trying to understand. It’s one thing to quote widely accepted views (the so-called AfA). It’s another to waive off the collaborative work of the majority of people in a field but offer no alternative that does a better job.

            I understand the policy issue, but in the end what are we talking about? Maybe a tax? Wow. We have passed many taxes with much less scientific support. The world won’t end if we, eg, ask people to pay for carbon costs and maybe even redistribute those collections so that those using renewables get a little gift in the mail for not consuming a limited resource that generations into the future might find a real need for and real lack of. And if they are right, well thank goodness we did something.

            BTW, who would argue skepticism is bad, generally? I know people can’t understand something just because others say it’s true. And when there is a policy issue that bothers you, then the natural skepticism turns more active and more skeptical. But facts are that if you don’t know the details of what thousands are doing, you aren’t going to convince others or yourself for very long (“why, just because you said so?”). Being skeptic doesn’t mean they are wrong. It means you have not acquired confidence in their views and perhaps never will. If you make a good argument (which usually requires you know the field more than with passing glance), you will sway people.

            00

      • #

        >>> Is it too hard for you to use the b-quote function?

        >>> Someone that claims to know so much about the complex climate really ought to be able to click on one fricking button yes?

        Chill out, dude. Relax. It’ll be OK. And anyway —

        Svbstance Over stYle.

        But to keep you happy:

        Here is one idea for you: water vapor, convection, condensation, repeat.

        Are you under the impression that heat wouldn’t be even more rapidly lost directly to space if there was no atmosphere?

        Because you are correct, of course, that convection is part of how heat from the Earth reaches space in the presence of an atmosphere. However, it isn’t like the lack of an atmosphere would stop heat from reaching space.

        00

    • #
      Greg House

      Christoph Dollis says (#159): “I have no idea how anyone can claim that insulation (through absorption and re-emitting in the case of CO2) doesn’t cause a heated body (in both senses of the word) to be warmer than it otherwise would.
      Throw on a few jumpers (i.e., sweaters) and get back to me with the results of your science experiment.”

      =======================================================

      A sweater does provide some insulation but it works both ways, as “warning” and as “cooling”: 1) if the air outside your body is colder than your body, it prevents this colder air to cool your body by conduction and convection, but 2) if it was like 70C outside your body, the sweater would prevent your body from overheating for a while.

      People sometimes ignore the second option and mistakenly think that a sweater or a blanket really warm.

      Back to CO2, it does not work both ways. CO2 certainly cools the surface by preventing a portion of solar IR from reaching the surface. As for the other direction, CO2 does block some portion of outgoing IR as well, but the back radiation won’t affect the temperature of the surface, anyway not to any significant degree, see the R.W.Wood experiment: (http://www.wmconnolley.org.uk/sci/wood_rw.1909.html).

      00

      • #
        Jose_X

        >> but the back radiation won’t affect the temperature of the surface, anyway not to any significant degree, see the R.W.Wood experiment

        Woods didn’t test for the atmosphere case because the earth does not lose heat into outer space via convection, yet both boxes can exchange heat with the outside via convection. Thus, the experiment is not testing radiation only and perhaps not even predominantly. Woods’ experiment is very short on experimental detail (setup and measurements) and offers no math (almost no analysis of the physics), so there is very little you could prove or disprove one way or the other.

        Let’s give two scenarios, one where the way heat is transferred will result in the glass box being hotter than the rock salt box and one where the glass box would be colder, but, first, let’s consider some details.

        In both boxes, we have the bottom surface expected to gain energy from the sun. Also, it would lose energy to the rest of the box and surroundings via conduction as well as to the air inside the box via convection.

        In both boxes, the transparent side (glass or rock salt) might be expected to lose energy by convection to the air outside.

        The glass might gain or lose by conduction to where it meets the rest of the box (the glass might be hotter or colder than the rest of the box). The glass will absorb “back” (infrared) radiation from above. The glass will absorb infrared radiation from the heated bottom surface. The glass will emit radiation based on its temperature (and spectra). The glass may (a) gain or (b) give by convection to the inside air depending on whether the radiation it absorbs minus what it losses to the outside and to the box itself is would otherwise raise its average temp (a) below or (b) above the temp of the air layer on its inside surface. The glass will reflect some energy as well, both at the interior and at the exterior.

        The rock salt will reflect some energy. It will pass through most (but not all) of the back radiation from above and also the infrared radiation from below. The rock salt may gain or lose (probably gain) some conduction from the rest of the box. The rock salt will radiate a little based on its temperature (but it absorbs/emits in the IR range much less than does the glass). We can expect the rock salt to gain by convection from the inside air.

        Glass box noticeably hotter:
        Not doing any math (just like Woods), we might assume most of the energy transfer values between the box components described above are about the same for each of the two boxes (with a few exceptions here described). We’ll assume conduction is negligible (a good assumption towards outer space but horrible assumption for these boxes). We’ll assume that convection between the outside and the box surfaces are negligible (this is likely untrue). We’ll assume the sides are small in area (very low height/other-side ratio) with most of the area being top and bottom (so we can basically ignore the sides). Also, as we approach equilibrium, not looking at the materials involved, we might assume that the glass would absorb an amount of energy from radiation that would be greater than what is lost by the bottom surface to the inside air via convection. This means the glass would be hotter than the air just inside. With convection being weak everywhere as assumed, this means the glass would absorb about 340 W/m^2 from above (assume a generic value as found in nature near the tropics) and about 600 W/m^2 from the inside as might be the case for around 50 C as measured by Woods in his second case. The glass might radiate about 475 W/m^2 down to the bottom layer. In contrast, for the rock salt, it might pass through most of the 300 W/m^2 from above. So the result is that the bottom layer will clearly get more radiation at its equilibrium in the glass case (475) than in the rock salt case, (300) again, clearly with convection playing a minor role and the sides negligible.

        Glass box a little colder:
        Same as above, except that convection is very significant. This might mean that the majority of the glass and rock salt surfaces are each about the same temp as the outside. The air inside is obviously hotter than each of these and gives them heat which is then transferred into the outside. Anyway, the glass is at a temp like the outside and emits at this temp (we’ll assume) with an emissivity near 1. This radiation will probably be near what the rock salt allows to pass through, but certainly can be less in some cases — as we assume would be true in this second scenario — (eg, a higher altitude with little humidity may have a colder temp and say around 300 W/m^2 from the glass but a lower back radiation level).

        Conclusion:
        And there are many points in between these two scenarios, usually probably with the glass similar to the rock salt since outside downward back radiation levels tend to be comparable to the Stefan Boltzmann levels for the corresponding temperatures convection tends to produce for each box (ie, near “room temp” regardless of the box). But the glass can still be warmer with similar results. Qualitatively, I’m guessing (and Woods apparently was guessing/assuming a great bunch judging from his short paper with no math and not covering these points) that what rock salt allows to leave the box is partly covered by DLR, and we also might have a lower loss via convection/conduction from the inside air, through the rock salt, into the outside than when we compare to the glass, which does block the DLR coming in but then generates a not too different level of “DLR” itself back to the inside, with the difference between the glass and rock salt in-directed radiation mostly made up by less convection+conduction loss through the glass towards the outside. [And remember that the planet loses nothing to outer space via convection, so the only case that might make some sense is closer to scenario one above, the case where the role of convection is diminished to around 0.] In short, without doing math and without taking careful measurements, we had to make a load of assumptions above, and the wide range of assumptions allow a wide range of different observations. Thus, we can’t say much about a theory (eg, greenhouse or radiation) if we aren’t measuring and applying the math, in this case, because the range of possibility easily covers what Woods saw. In other words, what woods saw is consistent with radiation theory and greenhouse effect if the various numbers were to jive, but Woods didn’t measure these values (ie, nail down the heat exchanges at all surfaces and across all heat types) nor perform calculations based on them in order to say something meaningful about a given theory, so Woods experiment achieved little for/against these theories.

        Two main points of the conclusion (and sorry I am so wordy): Woods’ conclusion does not follow since we see above that we get very different results depending on the details, which Woods’ skipped. And, we can’t compare these boxes to the planet because convection plays a significant role helping heat dissipate the boxes but plays essentially no role in removing heat from the planet into outer space: to compare to the atmosphere, we might want to have a box within a box with a near vacuum in between these two to try and neutralize the losses of the inside box by convection.

        00

        • #
          Mark D.

          Jose says:

          Woods didn’t test for the atmosphere case because the earth does not lose heat into outer space via convection,

          No but you cannot ignore the tremendous movement of heat vertically in the atmosphere via convection.

          And, we can’t compare these boxes to the planet because convection plays a significant role helping heat dissipate the boxes but plays essentially no role in removing heat from the planet into outer space

          Oops you have ignored it! How dumb. So you believe that a process (convection) that raises tremendous amounts of heat to high elevations and thereby bypassing all of your frigging “jumpers” worth of insulation, plays NO ROLE?!?!?!?!?

          You are really rather thick I think.

          Try again please.

          00

          • #
            Jose_X

            >> No but you cannot ignore the tremendous movement of heat vertically in the atmosphere via convection.

            Yea, and you can’t ignore the movement of air inside your jumper either or inside an oven.

            >> So you believe that a process (convection) that raises tremendous amounts of heat to high elevations and thereby bypassing all of your frigging “jumpers” worth of insulation, plays NO ROLE?!?!?!?!?

            It does play a role. I was referring (without being clear, sorry) to convection outside the box. The examples of convection within our atmosphere would be analogous to convection within the boxes. The outside the box convection case does not exist for the planet because space does not cool the earth via convection.

            00

          • #

            Mark,

            The non photon transport of energy up through the atmosphere (convection, latent heat etc.) only matters to the surface equilibrium when there is not an equal and opposite amount of non radiant energy returning to the surface. Trenberth and others like to claim that more goes up than comes down and the difference is returned to the surface as ‘back radiation’. The problem with this assumption is that what he calls ‘back radiation’ includes significant amounts of non radiant energy. For example, water lifted against gravity, the heat energy in rain, much of which arose as latent heat was given back to the water as it condensed into rain and of course, weather and the energy supporting atmospheric and oceanic circulation currents.

            My hypothesis is that while the atmosphere/space boundary has only a photon energy flux to balance, the surface/atmosphere boundary has energy transported by photons and energy transported by matter (air and water), whose instantaneous sum must offset the energy being stored and release by the atmosphere, moreover; averaged over time, the net photon flux, the net matter flux and the net flux between the atmosphere and the surface are all zero.

            The proof of this is here: half-of-the-energy-is-flung-out-to-space-along-with-the-model-projections/ which calculates the net balance when accounting for only the measured radiative components and the result is that the net measured photon balance is zero, therefore; the net matter balance must also be zero. Note that while the above analysis derives a 50/50 split from the data assuming a net zero photon flux, the 50/50 split is a physical requirement of the atmosphere, where energy leaves (top and bottom) over twice the area from which it arrives (bottom), making a net zero photon balance the more significant result.

            George

            00

          • #
            Mark D.

            Hi George, I don’t think I’m in disagreement with what you have said.

            The balance of energy in/out is going to find equilibrium based on the composition as a percentage of the atmosphere. Water in all forms being the largest player and nearly impossible to quantify on a global scale. The maths and incantations of the warmists, the argument ad-infinitum over back radiation and cold objects passing photons to warm objects are interesting material for endless papers. None of them will matter much for me as more than bad guesses until we have the ability to measure at that scale.

            It’s going to be a while. Maybe never if we die off as a species due to something that we SHOULD have been spending research money on.

            Good to hear from you.

            00

          • #
            Jose_X

            >> The maths and incantations of the warmists, the argument ad-infinitum over back radiation and cold objects passing photons to warm objects are interesting material for endless papers. None of them will matter much for me as more than bad guesses until we have the ability to measure at that scale.

            Science is not for everyone, true.

            If you decide to critique a work or produce one of your own, stop by and drop a link. Then we can look at it and judge for ourselves.

            When you study, the symbols may start to make sense. When you don’t, it will be like hearing Greek (for someone who doesn’t understand it). It will all seem like gibberish. Really. Until you learn the logic and components of Greek, it will seem gibberish. Really.

            Anyway, the oceans continue to warm. We just had another yearly world temp average in the top 10. In the US, we just had the hottest year by a full degree F! You may not want to learn Greek, but so far the predictions are in the ballpark, which is more than can be said for the “skeptics” (who tend to be way off, or who make no predictions because (to paraphrase) it’s just impossible to know enough about the planet).

            01

          • #
            Jose_X

            >> The maths and incantations of the warmists, the argument ad-infinitum over back radiation and cold objects passing photons to warm objects are interesting material for endless papers. None of them will matter much for me as more than bad guesses until we have the ability to measure at that scale.

            I understand that we don’t know how much we don’t know, etc, and that to an outsider maybe the “Greeks” are talking gobblygook. That is true about everything. What will guide this conversation is that as time passes more or fewer people will support mainstream (and mainstream will adjust itself as well) based on how nature reacts, across the decades especially.

            Also, if someone is serious about the subject, they will probably give some of the text a real shot, maybe take a few classes if necessary, or at least hang around certain forums and try to write their own papers. It’s easy to criticize, but not to participate. It’s doubtful that there is no method to the madness. If it is garbage, it should not be that hard to pose in the right forums some of these crucial issues or find flaws.

            00

          • #
            Mark D.

            Jose, don’t misunderstand my comment. Science is all about gaining understanding. Obviously, maths are used and understood well enough by scientists and I’m not suggesting we stop looking for answers because “it’s too complex”.

            Just stop bragging about how much “we know”. What we “know” about the earth and her climate systems is just in infancy. If you and warmist friends left it at that I’d be doing something else right now. But that isn’t how it’s going down and you know it. You cannot attribute what we are observing in climate to human activity yet politicians are doing just that. They are abusing us with your infant science.

            Presently, there is no maths that can resolve the complexity of the atmosphere and no model that will yield what is needed. We only have reasonably accurate measurements in the last 30 years or less yet here you are claiming:

            the oceans continue to warm. We just had another yearly world temp average in the top 10. In the US, we just had the hottest year by a full degree F! ………….

            So you’ve been outed as just another alarmist, another fear monger. You are in denial of how little we know even when in your own words:

            I understand that we don’t know how much we don’t know,

            And if you said that much earlier……

            00

          • #
            BenAW

            Hi Mark

            You may be interested in the graph in this article
            The deep oceans have lost some 17K in the last ~85 million years.
            So much for still warming oceans 😉
            With the temperature of the deep oceans explained by warming from magma eruptions, Earths surface temperatures are easily explainable. No greenhouse effect required.
            The atmosphere simply acts as an insulation blanket, just slowing the cooling rate.

            00

          • #
            Jose_X

            >> climate systems is just in infancy. … You cannot attribute what we are observing in climate to human activity yet politicians are doing just that.

            Let me suggest that you are speaking out of a great deal of ignorance and not because of a serious effort to understand the backing behind those views held by scientists. Can you offer something so that I would feel more comfortable thinking that you have indeed done the research and then have found the science lacking? Do you have a summary page of your views that I can read or a paper posted on the Internet that perhaps was rejected by an important journal?

            I am aware that some people are working on alternative theories, some of which are rather complex and are tangential to climate science and have not gained traction from scientists studying those areas (eg, the one mentioned in comments earlier on statistical mechanics). Realistically, until they make headway among scientists, few people will probably take the time to read the ins and outs of those theories (but one can link to the information and hope for the best, right?).

            FWIW, yes, I am a skeptic on various fields of knowledge, but I have found out consistently that, as I learned more about what the experts actually thought (rather than what the headlines said), I gained appreciation and some degree of conversion into those ideas. If I don’t care too much about the subject, like with say Chinese, I may simply accept that I will live in ignorance and will have limited sway.

            >> They are abusing us with your infant science.

            To clarify (and to assume for a second that “your infant science” refers to me somehow), I am not a climate scientist.

            As just stated, I think your comment comes from a lack of familiarity with details of what has been written on the subject, where significant strides having been made at least since the ’60s.

            >> We only have reasonably accurate measurements in the last 30 years or less

            It’s unreasonable to believe that all the temperature gathering prior to that is useless. If you look at say the BEST study, it shows that before present day measurements there are huge error bars, but the data is still useful, just like very imperfect proxy data is also useful. Statistical analysis considers these issues. It is useful to know that temps at a given site using a given technology and conducted by given people using a given methodology has produced certain results. As people and sites and methodologies change, patterns nevertheless are left behind, and these show up when you compare across thousands and millions such data points.

            A Watts’ paper has a problem of criticizing the results of the data because they did not use the raw data, but rebuttals quickly followed explaining that, when people switch measurements from 9 AM daily to 3 PM daily, the data has to be adjusted and can’t be used raw along with the other data. So, yes, scientists are more than aware of these issues and they fight amongst themselves on trying to achieve fair and widely accepted data analyses that hopefully will give more correct answers and keep the error bars as small as possible.

            Scientists make mistakes. That’s why I keep mentioning that you should read some and write a paper on mistakes you have found (hopefully not making the same arguments others have already made).

            >> yet here you are claiming

            It’s not my fault you reject the main conclusions of an entire field of science.. and without having done a decent review of the material.

            >> So you’ve been outed as just another alarmist

            For accepting the main points of a field that has had decades of growth and has had lots of the most modern technology accessible in order to home certain views, and after trying to some degree to look for answers skeptics have posted? Hardly.

            >> And if you said that much earlier

            I spoke only now of our general ignorance on what we don’t know (well, I think I had said some time ago as well, but I don’t remember where, nor does that matter) because of the direction you took the conversation, but that always applies to every field of knowledge and every bit of knowledge (even alleged “facts”). In practical conversations, we tend to leave out hints of this doubt, unfortunately, since it is very tedious, yet one can always clarify this point when necessary.

            00

          • #
            Jose_X

            >> You may be interested in the graph in this article
            The deep oceans have lost some 17K in the last ~85 million years.

            I should point out that to science (and to someone trying to resolve an issue personally) charts without references that skip on the important details are not too useful. [I didn’t read the whole page, so this point may not apply to that chart.]

            CO2 has been greater in the past. Temps have been greater.. But it’s also true that humans (or our near ancestors) were not around, or at least certainly not dominant. The worry to people largely is not that life won’t go on, but that the lives of many have a high risk of being seriously impacted negatively and of many species not adapting fast enough. What is the point of governments (frequently, a bond between individuals living in an area) and of science if not to care about these sorts of issues that might result in great upheaval and destruction of important parts of our future civilizations in addition to promoting hardship and death? It’s always a dice game of probabilities. Fools ignore high odds of negative outcomes that otherwise could have been improved. And it is much more about our great grand kids than it is about us.

            >> So much for still warming oceans

            Oh, but humans’ best data and analysis shows they are. That data and analysis you mentioned, potential as flawed as any other data/analysis, does not cover the very fast changes that have occurred in modern times.

            >> The atmosphere simply acts as an insulation blanket, just slowing the cooling rate.

            It may have been that left to natural processes (and barring major disruptions) that the Earth was headed back (or maybe cycling) to a cold period and maybe frozen oceans, but that is irrelevant to the 21st century humans and their descendants because we are now changing the planet significantly. *We* are a major disruption. Heck, with our nuke knowledge, we have the potential to change the long-term climate likely more than the vast majority of most meteors that have hit the earth ever could. We already have changed major bodies of “fresh” water significantly and will more so in the upcoming decades. We have changed forests and swamps tremendously in many cases. We have changed many mountains and hills. We have changed ecosystems vastly.

            Anyway, the sun keeps adding energy, and we are tinkering with the strength of that insulation blanket.

            .. at least I think this is what the most dedicated scientists in numerous relevant fields have concluded as they have been having their public debate with their public studies (open to all).

            00

        • #
          Greg House

          Jose_X says (159.2.1): “Woods didn’t test for the atmosphere case because the earth does not lose heat into outer space via convection, yet both boxes can exchange heat with the outside via convection.”
          ==========================================
          All the differences between the boxes in the R.W.Wood experiment and the whole atmosphere do not matter, because the experiment was not designed to deal with the whole atmosphere not was it designed to compare the boxes with the atmosphere.

          It was designed to compare 2 states: “trapped radiation” versus “no trapped radiation”, other conditions being (almost) equal. The experiment demonstrated that “trapped radiation” did not have any (significant) effect on the temperature of the source.

          Since the “trapped radiation effect” is the basis of the “greenhouse effect” (as it was presented by the IPCC http://www.ipcc.ch/publications_and_data/ar4/wg1/en/faq-1-3.html), the conclusion is that the “greenhouse effect” is zero or negligible.

          00

          • #
            Jose_X

            As I tried to explain, the earth case has no convection form of removal into space. The boxes do have convection at the outside boundary into the outside environment. This means that you don’t have to account for those effects on the earth but you do in the box experiment. In the box experiment, that convection effect can certainly neutralize the radiation “trapped” effect by throwing in another path for heat to move, a path that does not exist for the planetary case.

            If I enable a new path, then there may not be any “trapped” effect in play any longer (the degree of “entrapment” would depend on parameters that were not measured).

            00

          • #
            Greg House

            Jose_X says (#159.2.1.2.1): “As I tried to explain, the earth case has no convection form of removal into space. The boxes do have convection at the outside boundary into the outside environment. This means that you don’t have to account for those effects on the earth but you do in the box experiment. In the box experiment, that convection effect can certainly neutralize the radiation “trapped” effect by throwing in another path for heat to move, a path that does not exist for the planetary case.
            ===============================================

            Again, the conditions for both boxes, including convection at the outside boundary into the outside environment were equal. There was no “additional” convection, like provided by a fan or a wind or whatever for the box with glass lid. There was no “another path”. Equal conditions for both boxes, remember?

            So, if the “trapped radiation” had provided additional energy for warming, the air in the box with the glass lid would have become warmer, than the air in the box with the rock salt lid, but it did not. This is a very simple point, please, make an effort. Hence the experiment proves that the trapped radiation has practically no effect on the temperature of the source and the IPCC “greenhouse effect” has no basis in real physics.

            00

          • #
            Jose_X

            >> There was no “additional” convection

            This is your guess.. until you quantify it with math/physics and/or measurements.

            The measurements by Woods were crude and ill-defined. For example, the rate of convection perhaps might cool the glass any extra amount that the radiation would contribute depending on the relevant time constants (and other details). There is no measurement of the total amount of heat leaving the boxes for example. There is no effort to neutralize different convection/wind in the vicinity of each box (eg, box within a box). None of this is measured, and you would simply be guessing that there is no difference. There could have been a huge difference of the attributes of the air masses just outside the boxes (due to the box surfaces themselves), but it was not measured in any way. That experiment did not take the measurement necessary to pin down heat flows. All it measured was steady state at one location. Again, if the radiation effect is slower than convection, it can be neutralized. This overcompensation, if it existed, would work against a rise in temp inside the box. [See the second analogy .. last paragraph below.]

            This said, the other point I mentioned was that the rock salt case allows DLR and other radiation (were the boxes near a roof or building?) to pass through into the box to a greater degree than the glass (to first approx, rock salt allows all in and glass absorbs all of it). This pass through radiation is a source of heat not directly accessible to the inside of the glass box. This value certainly makes up for some, all, or beyond the amount of “back radiation” from the glass itself back into the box. In the planetary case, there is no large source of DLR coming from outer space. If there were, a non-ghg atmosphere might behave closer to a ghg atmosphere (note that the sun’s radiation has little DLR component). Each box, however, has that outer environment DLR hitting it (but directly passing through only for the rock salt) — another variable that cannot be ignored and makes the box experiments different than the planet case.

            >> the air in the box with the glass lid would have become warmer, than the air in the box with the rock salt lid

            To add to the convection discussion, let’s consider a possible model. Let’s say that the radiation is absorbed throughout the glass’ thickness. This means that some of the energy is captured close to the outside air (within the glass pane but near the outside surface). That energy will more quickly dissipate than if it came from the internal air and had to pass through the entire pane’s thickness. Thus, this is energy from the box’ bottom that disappears through outside convection much more easily than energy passing through the glass (or rock salt), so effectively it’s not caught. Maybe we need glass pane thickness of 5x in order to make sure most of the radiation is captured near the inside? An analogy would be our atmosphere if it had less CO2 (one optical thickness or a fraction of one rather than several). The greenhouse effect would be less if the absorption happens closer to space or isn’t even saturated (ie, consider a halving of CO2.. it would lower the global ave temp). The current atmosphere can capture most of the capturable photons within less than 1km I think, and then can recapture again most emissions at that level directed upwards… and then again, I think.

            What about the rate of heat flow across the glass pane vs across the rock salt? These have different values, yet they were ignored. Saying similar thickness panes were used doesn’t mean heat moves with the same ease through those barriers if the heat “impedance” is greater in one than the other. And this impedance has much to do with the final temp. Put differently, the rock salt might allow longwave through more easily, but it might impede conduction to a greater amount, nullifying the advantage it had. Again, none of this was considered or measured in the super simplified and almost without detail Woods experiment.

            Another question: how do we know that the final temp under direct sun (steady state value) wouldn’t be significantly different if observed after say 10 hours or 30 hours? The sun is only at its max value for a short time and that destroys that observation possibility (unless the experiment is in a plane flying around the world fast enough). This is one reason why many things are usually done in a laboratory — to control as many variables as possible. As temps rise, the parameters of the material change and effects that might be weak at a lower temp might dominate, leading to similar temp paths near the low temps but diverging later on.. if we could keep the sun’s intensity constant for say 20 hours. We don’t know if this issue is important. It was not tested or measured or analyzed.

            Let me give an analogy to demonstrate the importance of time constants and of final steady state values, by using water inside pits. Let’s say we have two open pits. One has cement for sides and bottom and one is natural dirt. If we raise the water level of each pool by adding at equal rates and fast enough and then stop observations, we might conclude that each pit would be filled the same. However, if we add water more slowly or if we watch for a longer period, we might observe that the natural pit would have the sides absorb enough water and ultimately top out at a lower value than the pit with a man-made cement-concrete interior. That is because eventually the time scale of ground absorption would have time to have its effect or maybe this effect would be more pronounced at depths above 20 meters or whatever.

            Let me modify this analogy to show what I meant about convection potentially working differently for each box (as mentioned in the first paragraph). Lets say that one pool is filled with a hose’s end directly inside the rising water. The only wet area is the water itself and the sun evaporates only at the top of the rising water. The other pool is filled by throwing buckets of water at the sides of the pool, which then drain down and pool at the bottom. The sun here does the job I was talking about convection doing for the boxes example. There is more evaporation in the case of throwing water at the sides of the pool because the total exposed surface area of the water in that pool is greater. The sun does its work silently, much as the air might cool one of the boxes more than the other even though the box (pool) boundaries and outside environment might look about the same. Thus, if the pool with water thrown at the sides was actually using more water in-flow, the end resulting water level (the temperature in the box) might be the same, simply because the sun (wind) was doing more work. Unless we carefully track the heat flows, it’s not obvious that we can in fact place more water into one pool (more re-radiating of photons in the glass box) but end up with the same level (same temp) because of greater evaporation loss (greater convection heat loss) to the environment since the water paths (photon paths) were different.

            00

          • #

            The energy trapped by atmospheric GHG’s constitutes an omni-directional flux of photons in the absorption bands of GHG gas molecules. This is evident by observing the radiation spectrum emitted by the Earth, whose attenuation in non overlapping bands of complete absorption of surface emissions is only 3db. This means that half of whats absorbed by atmospheric GHG’s must be returned to the surface if half is escaping to space (See: Figure 7-8 Terrestrial radiation spectrum measured from a satellite …). Note that the omni-directional nature of this flux may confuse near surface measuring devices, but not satellite measurements.

            The absorption band flux in the atmosphere arises because most GHG molecules are already in an energized state where the probability of stimulated emission upon absorption approaches 100% (See: Two-level Atom Radiative Processes). The path of a quanta of energy through the atmosphere is quite long as it bounces between GHG molecules until it eventually escapes the planet or returns to the surface. The 50/50 split of GHG related energy as it relates to the planets energy balance is the consequence of a uniform distribution of direction for any photon emitted by a GHG molecule. Virtually none of this flux is heating the N2 and O2 in the atmosphere and while atmospheric water can absorb this flux, water will emit an equal and opposite flux as broad band black body radiation consequential to its temperature.

            The 50/50 split of absorbed surface energy in the rest of the spectrum is a consequence of clouds which absorb surface emitted photons across half the area that BB photons leave. The clear sky transparent bands pass as much as 100% of surface emissions while clouds can block up to 1/2 (capture 100%, emit half up and down). The amount of clouds adapts by adjusting the ratio of cold clouds and warm surface until the average emissions are equal to the average, post albedo, solar input, which is independently a function of cloud cover and surface ice. Only one value of average cloud coverage fits each combination of average temperature, ice and solar input and the 50/50 split is the final constraint which dictates the steady state average surface temperature and its relationship to incremental atmospheric absorption.

            George

            00

          • #
            Greg House

            Jose_X says (159.2.1.2.3): “the other point I mentioned was that the rock salt case allows DLR and other radiation (were the boxes near a roof or building?) to pass through into the box to a greater degree than the glass (to first approx, rock salt allows all in and glass absorbs all of it). This pass through radiation is a source of heat not directly accessible to the inside of the glass box. This value certainly makes up for some, all, or beyond the amount of “back radiation” from the glass itself back into the box.”
            ============================================

            Professor Wood took care of exactly the problem you described. I can not believe you missed it, the article is a so easy reading. Here we go (http://www.wmconnolley.org.uk/sci/wood_rw.1909.html):

            “To test the matter I constructed two enclosures of dead black cardboard, one covered with a glass plate, the other with a plate of rock-salt of equal thickness. The bulb of a thermometer was inserted in each enclosure and the whole packed in cotton, with the exception of the transparent plates which were exposed. When exposed to sunlight the temperature rose gradually to 65C, the enclosure covered with the salt plate keeping a little ahead of the other, owing to the fact that it transmitted the longer waves from the sun, which were stopped by the glass. In order to eliminate this action the sunlight was first passed through a glass plate.

            There was now scarcely a difference of one degree between the temperatures of the two enclosures. The maximum temperature reached was about 55C.”

            So, the only difference between the boxes (apart from the disputed effect of the trapped radiation) was different thermal conductivity of the glass lid and the rock salt lid. However, this contributed to a higher temperature in the box with the glass lid. Other conditions being equal, that means that the effect of the trapped radiation was even more negligible.

            00

          • #
            Jose_X

            >> Professor Wood took care of exactly the problem you described.

            If you want me to believe that statement, then you explain the setup and the values (eg, in 20 minutes the temp was x or y). Detail those things that Woods didn’t (even make them up as you need to), and I will respond as if that were a new experiment like Woods [You can make up values since if I don’t like them I will complain, but do try to match what it seems Woods might have gotten.]

            Make sure to specify the glass thickness and rock salt thickness in each experiment. The second appears to use two pieces of glass for the glass box and 1 glass and 1 rock salt for the other box.

            Specify the environment (eg, indoors, outdoors). Give me an idea of the temperature and where on the globe you are (so I can guesstimate an IR ambient radiation level for each box for each of the 2 experiments).

            Bottom line is this. One: When you add glass filters, you block more shortwave (because of a small amount of reflection and absorption by glass since glass is not air even if it gets close as concerns shortwave), so the max temp will drop at least some. Two, if the inner sealed layer is glass, it will absorb a lot of the IR radiation coming up but will not radiate back half of it. It will radiate back down an amount that depends on its temperature, and its temperature is kept modest by the convection current just outside the box (unless you place the box in a vacuum, which we can assume Woods didn’t do). The rock salt allows IR from below to escape (most of it, not all) and IR from above to go into the box. In either case, rock salt or glass, and in either experiment filtered or unfiltered, the glass top box is getting energy into it from a shortwave source and from IR at near room temp, whether its IR from the glass cooled by the air outside or whether its IR from ambient environment passing through rock salt into the box. To know which box should heat faster and precisely how much, we need lots of details like topology, location of filter, material of items, ambient IR, etc, details Woods didn’t record for us, but we can see that no matter, it makes sense to have the rock salt lead because there is less impediment to IR from above (since rock salt is mostly transparent to it), and it makes sense the rock salt gets to about the same temp as the glass box because it may be allowing as much, less, or more IR from ambient into the box while the glass might be allowing similar IR levels due to its own temp being near room temp and near back radiation levels as have been measured in many parts of the world at various temperatures. Key point is that air convection can keep the glass cool, so it would radiate downward a modest level of IR (and similar to the rock salt IR coming in), that level definitely being less than if the IR from below were reflected back into the box entirely because the glass will definitely be cooler than the inside bottom of that box (unless we used a vacuum chamber).

            >> However, this contributed to a higher temperature in the box with the glass lid.

            It’s not clear that is the case for the 55C case, and it appears it was not the case in the 65C case at least not while Woods was watching and waiting. I don’t see what point you are trying to make, regardless.

            Again, each box in each experiment has IR going in from the environment. That amount is around what corresponds to the ambient temp. In the rock salt case, it passes through from outside. In the glass case, it is generated from the glass based on the temperature of the glass, which is near room temp as cooled by convection on the outside layer of the box. And each box in each case has coming out the IR from the heated bottom. In the rock salt case it just passes through. In the glass case, it is mostly absorbed. There is a little reflection, but we can’t tell how much from the info Woods gave us.

            At least one person who tried to repeat the experiment noticed differences depending on how well you insulated and if you double paned with air gap and tried to limit convection, etc. Without blocking convection significantly, eg, via a vacuum chamber, it’s difficult to allow the glass of the glass box to get sufficiently hot to make that box significantly hotter than the rock salt box.

            00

        • #

          Jose,

          When one spends time investigating climate science they quickly learn that the physical foundation for the massive positive feedback required to support the consensus 3C per 3.7 W/m^2 of forcing is somewhere between non existent and speculation, which leads to either intense skepticism or a faith based belief in the consensus. The catastrophic model of anthropogenic emissions is so tenuously supported by actual physics that whenever I ask a believer to back up their claims with hard physics, the appeals to a broken authority increase and the stonewalling accelerates. This tells me that no warmist knows enough about the physics to have an informed opinion about the actual effects of incremental CO2. Feel free to demonstrate otherwise …

          The foundation papers supporting the massive positive feedback cited by the first TAR are horribly flawed, moreover; climate science has built upon their flawed assumptions ever since. The insane idea that sensitivity is properly expressed as degrees per W/m^2 arose from these papers. Hansen was Bode compliant with dimensionless feedback and gain, but he mixed them up while assuming unit open loop gain. Schlesinger ‘fixed’ the Hansen formulation by switching feedback and gain, calling the Stefan-Boltzmann relationship open loop gain and introducing degrees per W/m^2 as the units of gain (sensitivity). The only apparent reason for this non Bode compliant formulation was to obscure the fact that the equivalent dimensionless closed loop gain required to support 3C is obviously too large. This obfuscation was double acting as it also obscures the assumption of unit open loop gain. FYI, it was Schlesinger who referred me to these papers when I asked him for the physical and mathematical foundation for CAGW, and as expected, he didn’t take my analysis of his work very well.

          Let me illuminate this flaw in simple, unambiguous terms. Each W/m^2 of incident solar power, which for all intents and purposes is equivalent to a W/m^2 of IPCC forcing, produces 1.61 W/m^2 of surface emissions for an average surface gain of 1.61. The gain decreases as the temperature increases owing to the T^4 relationship between surface temperature and the power emitted by the surface dictating that the incremental gain as temperatures rises must be less than the average gain of 1.61, which of course is confirmed by measurements.

          To raise the average temperature of the planet from its current average of 287K up to 290K requires 16 W/m^2 of incremental input to the surface, necessitating an incremental gain of 4.3 (16/3.7), which is obviously too large. You might be tempted to blindly claim that CO2 increases the gain and every input W/m^2 contributes more and in fact, you would be correct as GHG effects manifest as gain, not feedback, but doubling CO2 only increases the closed loop gain by less than 1% and not the greater than 4% required by the consensus. Ironically, Hansen had this conceptually correct, but computationally incorrect because he wanted the effects of CO2 to be feedback.

          George

          00

          • #
            Jose_X

            >> When one spends time investigating climate science they quickly learn that the physical foundation for the massive positive feedback required to support the consensus 3C per 3.7 W/m^2 of forcing is somewhere between non existent and speculation

            Please provide a reference (since I have not read the IPCC report except a few sections here and there).

            I suspect that you aren’t defining the values the same as the IPCC and maybe the confusion lies there.

            This might be relevant http://en.wikipedia.org/wiki/Climate_sensitivity#Sample_calculation_using_industrial-age_data . In it, they derive a value not of 3 C but of 2.1 C I think. And I think I read somewhere that the new IPCC estimate may be closer to 2.5 C.

            Regardless, of the value, that section may help clarify a few things. Also, the definition of climate sensitivity is the change in temp at the surface for a CO2 doubling after equilibrium is reached. Also, I think the 3.7 W/m^2 is the forcing contribution if only CO2 is changing, and it’s the value one would measure at TOA.

            So, I think you are looking at this as if the IPCC says that 3.7 at TOA would lead to 3C (or 16 W/m^s) at the surface. That is not the definition. Rather, if we ignore all other factors, natural and man-induced (eg, like increased water vapor from temp rises and like aerosols), the physically/mathematically derived TOA forcing for 2x CO2 is calculated to be 3.7. However, the actualy climate sensitivity would be what we see happen on Earth’s surface when 2x is actually reached in reality, assuming we ignore man-made effects like aerosols. [I am guessing the aerosols and other effects might be ignored because they would not be long lasting and/or because they would not be a natural response to CO2, but I am not sure of the definition.] Now, if it is believed that at 2x CO2 and after equilibrium, that there would be a 3C rise in surface temp (or some other smaller value perhaps). And there would be a corresponding rise at TOA that would be perhaps 1/1.6 (or some other number) as large. This value being say 3/1.6=1.88.

            So I think your mistake is that you claim that the IPCC believes that 3.7 W/m^2 at TOA will lead to 16 W/m^2 at surface rise. But the 3.7 is not what they expect to correspond to the 16. They expect a much larger value (eg, 16/1.6=10 if we use a linear approximation). The 3.7 is the theoretical change at TOA we would observe if all variables remained the same except 2x CO2. Obviously, in real life other things change, and the climate sensitivity is the change we expect to see after everything is taken into account in the real world (excluding other man-made changes).

            Does this sound right?

            >> The insane idea that sensitivity is properly expressed as degrees per W/m^2 arose from these papers.

            Climate sensivity is an arbitrary definition even if it doesn’t give with what you would expect to be convention. [You should know that engineers violate mathematics and physics conventions here and there, and that changing conventions is something that also happens within a given field periodically over time. What matters is that the application and formulas are consistent with whatever convention is established. A newly introduced convention in a different field of study might make one cringe, but that is another matter.]

            00

        • #

          Jose,

          The Wikipedia article you referred to relied on assumptions upon assumptions and didn’t even convey a weak theoretical case. The only factual reference was

          albeit with substantial uncertainty

          when referring to AR4. That there is so much uncertainty means the consensus doesn’t have a clue about how the climate system operates. The stated range of 3C +/- 1.5C represents a difference in net radiant power leaving the surface of between 8 W/m^2 and 25 W/m^2, which as a consequence of 3.7 W/m^2 of incremental forcing, becomes an incremental gain of 4.5 +/- 2.3 with an uncertainty > 50%. Compare this to the measured average absolute gain of 1.61 +/- .07, with < 5% uncertainty. A 50% uncertainty applied to the absolute gain would mean the surface temperature should be somewhere between 241K and 318K. If the nominal incremental gain of 4.4 claimed by the IPCC is applied to the 239 W/m^2 of post albedo incident power, the surface temperature would boil water. These inconsistencies are fatal flaws with consensus theory.

          Dimensionless gain (sensitivity) is what Schlesinger obscured by introducing gain units of degrees per W/m^2 and this obfuscation hides these fatal flaws. While Hansen's original paper did conform to Bode (except for a failure to include open loop gain), Schlesinger's reformulation did not, yet his analysis and conclusions were predicated on conformance with Bode. This serious error forms the foundation of the theoretical case presented in AR1 and built upon since. This is not a matter of some deviant convention, but of a failure to adhere to the requirements of the mathematics used to quantify the response of feedback control systems, per Bode.

          One further point is that the 1.1C 'theoretical' effect of doubling CO2 (based on a gain of 1.6) is is the post feedback increase in the steady state average temperature when forcing is increased by 3.7 W/m^2, where the IPCC defines forcing to be roughly equivalent to post albedo input power. Its important to understand that this is a post feedback result which accounts for the net effect of GHGs and clouds. The pre feedback result would be the Stefan-Boltzmann temperature increase arising from 3.7 W/m^2 of incremental output power, which is slightly less than 0.7C and represents an incremental gain of 1. The measured incremental gain is so close to 0.7C per 3.7 W/m^2 of incremental post albedo input power, that the SB based theoretical and the measured incremental gain are equivalent within measurement error. This link describes the previous plot in more detail.

          George

          00

          • #
            Jose_X

            1. The Wikipedia example I showed before was of calculations to explain a 2008 paper that tried to guesstimate the climate equilibrium sensitivity value from current measured values. That example showed a number of values I thought might be useful for explaining my point. OK, I’ll try something else. Note that they derived a 1.6, but that is not the same figure as your 385/240=1.6. It’s a coincidence.

            2. Wikipedia covers things that are supported with acceptable references. The articles are not text books and will frequently skip lots of information needed to understand a theory well. If you ever want to challenge something I quote from there, state your view and hopefully with some references or analysis.

            3. The United States National Academy of Sciences at least by 1979 had taken a position that CO2 being released by man is a problem because of the resulting extra warming produced from atmosphere effect (greenhouse effect). It obviously should mean something when such a select body of professionals across all scientific fields takes a position on any issue — and before Gore!

            4. Climate sensitivity, despite using the word “sensitivity” does not have to adhere to the conventions of a “sensitivity” variable in any other field.

            As used here, it is a value measured in C referring to the change in temp of air surface temp for a doubling of CO2.

            Instead of getting twisted because of that word, show (if you can) where they use or calculate that value incorrectly.

            Perhaps in the future, convention will change it if is deemed more useful to measure the climate in a different way and define sensitivity in a different more useful way. It’s trivial to change W/m^2 on surface to C on surface and vice-versa, but for the time being they have decided to use a unit that means something immediately to most people.

            I should add that control systems theory allows open-loop gain and closed-loop gain to be defined in arbitrary dimensional units. The inputs and outputs of interest need not be of the same nature. Sensor behavior is frequently described in just this way.

            Additionally, you are identifying climate sensitivity with gain, but that does not adhere to the standard definition of climate sensitivity. You seem to recognize that climate sensitivity refers to deg C change @ 2x and that @ 2x carbon dioxide has an equivalent forcing of 3.7 at TOA, but you think that they are saying that 3.7 is the entire additional forcing at TOA when 2x occurs relative to 1x. That is wrong, and that is probably why you think the climate scientists are looney.

            I’ll try to explain below.

            5. >> Dimensionless gain (sensitivity) is what Schlesinger obscured by introducing gain units of degrees per W/m^2

            Despite using the word “sensitivity” above, climate sensitivity is measured in C not in C/(W/m^2).

            What you call the gain (ratio of TOA inputs to surface outputs, with each measured in W/m^2) is not related to climate sensitivity.

            6. >> Compare [a 50% uncertainty in surface W/m^2 gain at 2xCO2] to the measured average absolute gain of 1.61 +/- .07, with > One further point is that the 1.1C ‘theoretical’ effect of doubling CO2 (based on a gain of 1.6) is is the post feedback increase … Its important to understand that this is a post feedback result which accounts for the net effect of GHGs and clouds.

            Wrong. I haven’t seen that. Please provide a reference from IPCC literature (since it’s their definition I think we are arguing about) or from a selection of important research papers.

            It might be important for you to drop that idea from your head if you want to stop being frustrated with climate science.

            The Wikipedia link quotes a paper: “CO2 climate sensitivity has a component directly due to radiative forcing by CO2, and a further contribution arising from feedbacks, positive and negative. Without any feedbacks, a doubling of CO2 (which amounts to a forcing of 3.7 W/m2) would result in 1 °C global warming…”

            Did you catch that? “Without any feedbacks” the “CO2 climate sensitivity” has a “component due to radiative forcing by CO2” of “1C”. This is obviously a calculated (theoretical) result since the planet always has feedbacks turned on.

            This says that of the 3 C that comprises the climate sensitivity value, 1.1 C of it (aka “1 C”) comes directly from CO2 radiative forcing at TOA (which we know is 3.7 W/m^2). 3.7 * 1.6 = 5.9. 5.9 represents what takes 287K to 288.1K.

            Again, that CO2 component radiative forcing (and the 1.1 C) is a calculated value.

            If we consider feedbacks, if we care to try and measure the earth’s “sensitivity” to 2x CO2, then we get 10.0 W/m^2 at TOA, where this is the sum of the “component due to radiative forcing by CO2” [3.7 W/m^2] added to “a further contribution arising from feedbacks, positive and negative” [6.3 W/m^2].

            If we take 3.7 and 6.3 we get 10.0 total. That would roughly match a 16 W/m^2 gain on the surface (using the 1.6 gain). And that 16 W is the 3C climate sensitivity value.

            Where did I get the 6.3? It’s implied if we need to get to about 10.0 at TOA in order to roughly achieve the 16 needed on the surface so as to take 287 K into 290 K (as is believed will happen at 2x CO2).

            math recap:
            3.7 * 1.6 = 5.9 [CO2 contribution at TOA and surface]
            6.3 * 1.6 = 10.1 [Feedbacks contribution at TOA and surface]
            3.7 + 6.3 = 10.0 [CO2 and Feedbacks sum at TOA]
            5.9 + 10.1 = 16.0 [CO2 and Feedbacks sum at surface]
            10.0 * 1.6 = 16.0 [CO2 and Feedbacks sum at TOA and surface]

            In other words, the forecast is for a doubling of CO2 to result in 3.7 W/m^2 + 6.3 W/m^2 of new positive forcing at TOA, and this will produce an increase on the surface of 16 K, which means the air temp will go from 287 K to 290 K (a 3 C increase, aka the climate sensitivity value).

            BTW, if 2x CO2 is supposed to add 10 at TOA, where does the 10 appear in discussions and why are we talking about 3.7 so much instead?

            Well, these are all analysis results. In doing our analysis, we (“we” meaning climate scientists) know how to calculate 3.7. However, we believe that what will happen won’t be 3.7 but will be some other value. As climate science progresses, they are closing in on that other value (which hypothetically could be negative). Currently, that net value is believe to be around 10, as would be implied for 3 C increase on the ground.

            I believe this explanation is consistent with what I have read about climate sensitivity and the rest, and I think your misunderstanding where the 3.7 applies has been giving you fits and lots of anger at climate scientists.

            8. >> The pre feedback result would be the Stefan-Boltzmann temperature increase arising from 3.7 W/m^2 of incremental output power, which is slightly less than 0.7C and represents an incremental gain of 1.

            You are not defining a standard quantity here.

            You are saying simply this: if we ignore all effects from the planet (water and air) except for the straight S-B effect, then 3.7 W/m^2 increase would result in 0.7 C increase.

            This is fine, but it has nothing to do with climate sensitivity and is simply an analysis exercise for an introductory course exercise in the subject.

            If climate science decided to analyze a set of variables you don’t like, that is does not mean their analysis is incorrect. You have to specifically show where what they did is wrong or useless. And if you are misunderstanding their definitions, you should gain peace of mind from resolving that misunderstanding.

            00

          • #

            Jose,

            With regard to 1) its no coincidence. The 1.6 usually arises from the Arrhenius calculation, which if you pay attention to the details is just calculating the ratio of surface radiant power output to the post albedo radiant input power. Most people get lost when the chain rule is applied. An example of a coincidence is the fact that the natural warming as we came out of the Little Ice Age was coincident with the Industrial Revolution and increasing CO2 emissions by man (unless the later was a consequence of the former).

            Wikipedia is a less than reliable resource for climate science, although it does articulate the IPCC party line and the views of the IPCC’s contrived consensus. This is largely the work of William Connolley who was a long time moderator of climate related articles and is a radical environmentalist with far left views which has heavily biased his moderation.

            Your points 2) and 3) are blatant appeals to authority, which requires assuming that they are right, moreover; the science of this ‘authority’ is a house of cards built upon a mathematical error, which when corrected, causes the illusion that incremental CO2 can be harmful to collapse. After dozens of cordial exchanges with Schlesinger, I discovered this error and asked him to explain it, after which he became defensive, irate and stopped talking to me.

            Your statement 4) is equivalent to saying that its OK to claim compliance and then ignore the science and math when its the only way to prove your point. This is either classic denial or an attempt to justify fraud.

            Here is the origin of the 3.7 W/m^2 of incremental forcing from doubling CO2.. If you want to see the flawed consensus view on this look up ‘climate sensitivity’ on Wikipedia. At the stated sensitivity of 0.8 K per W/m^2, the result from CO2 alone is 3C +/- 1.5 when accounting for uncertainties. This is absurdly high, has far more uncertainty than needed and is what tells me that consensus climate science is horribly broken.

            You are also wrong about climate scientists knowing how to calculate the 3.7 W/m^2, I have never seen anything but empirical derivations based on speculative interpretations of sparse data to support this. Line by line spectral analysis shows doubling CO2 with nominal water vapor and clouds decreases the power not absorbed by GHGs and clouds by about 2%, or about 3.6 W/m^2, however; this would only equate to 1.8 W/m^2 of equivalent IPCC forcing since only half of what the atmosphere and clouds absorb is returned to the surface while the remainder is directed to space.

            You will never see contradictory evidence in any IPCC report. The IPCC has become the de-facto arbitrator of what constitutes climate science and only accepts science which is aligned with their stated agenda of distributing western wealth to less developed countries under the guise of climate reparations. Any evidence that CO2 emissions are less than harmful or … gasp … beneficial degrades the biggest lever they have against the developed world.

            Regarding your math, the feedback component is absurd. You are claiming 171% positive feedback which is a physical impossibility since the absolute maximum is 100%. This is another Bode constraint that the Schlesinger error obfuscates in order to make the impossible seem plausible. In addition, you are multiplying your impossibly large feedback by the closed loop gain of 1.6 which already accounts for all feedbacks with time constants of decades or less which includes the dominant feedback from clouds.

            Regarding 8), I certainly am using a standard quantification. In fact, I’m using the one used by Bode, which Schlesinger claims to be the reference for the foundation math in his paper, moreover; I have measurements to back up everything I say. The measurements are from the averages of 30 years of surface observations covering nearly 100% of the surface, most of which have IR and VIS images from 2 different satellites with 10-30 km pixels at 4 hour intervals. Furthermore, these are measured net long term averages which by definition must account for all feedbacks, positive, negative, known and unknown. The 1.6 post feedback, average absolute gain and the post feedback incremental gain of 1.0 are both confirmed by this data, where gain is defined as W/m^2 of surface output radiant power divided by W/m^2 of surface input radiant power. Stefan-Boltzmann can be trivially applied to convert between dimensionless gain and sensitivity expressed as degrees per W/m^2.

            Finally, I have pointed out exactly how, when and where climate science broke. I’ve also explained the error in great detail and have shown data that falsifies the consensus point of view. In other scientific fields falsification is an important component of the scientific method, yet for some reason, climate scientists refuse to accept falsification tests. Feel free to try and falsify any of my claims. Keep in mind that arguments to authority do not constitute falsification.

            George

            00

          • #
            Jose_X

            1) >> The 1.6 usually arises from the Arrhenius calculation, which if you pay attention to the details is just calculating the ratio of surface radiant power output to the post albedo radiant input power.

            No. That calculation you are talking about is what you did, and I mentioned it (385/240).

            The 1.6 in that example is a cumulative value of the supposed change since 1x CO2. It is the total as shown by this graph http://www.ipcc.ch/publications_and_data/ar4/wg1/en/fig/figure-ts-5-l.png and you can read about it here http://www.ipcc.ch/publications_and_data/ar4/wg1/en/tssts-2-5.html

            >The understanding of anthropogenic warming and cooling influences on climate has improved since the TAR, leading to very high confidence that the effect of human activities since 1750 has been a net positive forcing of +1.6 [+0.6 to +2.4] W m–2.

            At 2x CO2, that value is expected to be over 2 (all off CO2 effects won’t have been realized), yet clearly we aren’t going to get that ratio represented on the planet by changing the TOA and surface fluxes by a few percentages.

            You are talking about the total fluxes at TOA and at surface currently being in a ratio of 1.6 (no dimensions). They are talking about net changes in W/m^2 at TOA. (W/m^2)/(W/m^2) is very different from W/m^2. The 1.6 is pure coincidence.

            2) 3) >> blatant appeals to authority, which requires assuming that they are right

            You seem to have a problem with the world’s traditional scientific institutions (including universities) and the positions they have almost universally taken on climate science through a slow process covering decades and voluminous amounts of research. I’m still willing to listen, but I don’t see you backing your theories with physics (math and experimental results) as they do on a continual basis, if imperfect.

            >> house of cards built upon a mathematical error .. I discovered this error

            So what is the error?

            4) >> Your statement 4) is equivalent to saying that its OK to claim compliance and then ignore the science and math when its the only way to prove your point.

            Please take me through the logic that makes you claim that my statement 4 means that. I went back and can’t find anything to support your point, so I think you are relying on other information to make that conclusion.

            >> At the stated sensitivity of 0.8 K per W/m^2

            Did you read what I wrote? You continue to make that mistake unless you have changed what I think is your position and are talking about something else. You have been implying (best I can tell) that you think the IPCC is saying that a total gain of 3.7 W/m^2 at TOA results in 3 C at ground. Back that. Quote where they make that claim.

            To get this straight, please confirm that you believe that the IPCC believes that a move from 240 to 244 W/m^2 post albedo forcing at TOA is expected to correspond with a change from 287K to 290K on the surface, more or less.

            That is not what they believe and you have not shown where they imply or state that. You are attacking a straw man.

            Their 3.7 is a subcomponent of the total TOA changes anticipated. It appears they support something like a move from 240 W/m^2 (or whatever it would have been at 1x CO2) to about 250 (ie, about an increase of 10 from 1x CO2).

            5) >> You are also wrong about climate scientists knowing how to calculate the 3.7 W/m^2, I have never seen anything but empirical derivations based on speculative interpretations of sparse data to support this.

            I’ll reply later.

            >> Line by line spectral analysis shows doubling CO2 with nominal water vapor and clouds decreases the power not absorbed by GHGs and clouds by about 2%, or about 3.6 W/m^2, however; this would only equate to 1.8 W/m^2 of equivalent IPCC forcing since only half of what the atmosphere and clouds absorb is returned to the surface while the remainder is directed to space.

            I think you are talking about that paper you wrote, right? I’ll take another look to refresh my mind, and maybe also look at your replies related to it, but I don’t expect to go into that again if you don’t answer questions I pose and as you keep invoking physical claims that are unproven (I’m not referring to the 50/50 but to something else but can’t remember right now).

            7) >> Regarding your math, the feedback component is absurd. You are claiming 171% positive feedback which is a physical impossibility since the absolute maximum is 100%.

            You have never heard of an electronic amplifier with a gain of 3?

            Describe the simple block diagram that you think shows a violation of something.

            8) >> I’m using the one used by Bode, which Schlesinger claims to be the reference for the foundation math in his paper

            Can you provide more clues as to what is this reference and who is this Bode (I thought you meant of Bode plots fame).

            >> Furthermore, these are measured net long term averages which by definition must account for all feedbacks, positive, negative, known and unknown.

            I don’t know where you are going with this yet until you link to a paper (a graph and data is not a paper), but I should point out that you can’t be certain of “all” feedbacks, etc, ever. For example, you have no idea that the steady state to a 1 ppm CO2 increase doesn’t take 50 years to settle, and you mentioned data going back only 30 years. You can try to quantify these unknowns, but this requires arguments and math at a minimum, unless you want to make it an assumption that stability has been reached (which means likely you are using a default argument). Anyway, this is a minor and side point, but you may want to be clear that you would be making an assumption.

            >> Finally, I have pointed out exactly how, when and where climate science broke. I’ve also explained the error in great detail and have shown data that falsifies the consensus point of view.

            I haven’t read anything that does what you claim. Can you tell me what you are referring to specifically?

            00

          • #
            Jose_X

            George, I am reading that 3.7 issue more closely on the web. I may be wrong about the IPCC’s interpretation which I got mostly from that Wikipedia page. [Remember, I don’t do this for a living and I am still trying to understand the issues whenever I spend some time on this, which has been less and less this last half year.]

            In any case, there is no problem in having the TOA not move too much but have surface increase by a lot (assuming the calculations show those results). Using an insulation analogy, if you get stronger insulation, the near outer layer may be about the same temperature because the outside is mostly the same temp, but the inside being warmed by a hearth fire will mean the house gets hotter and hotter at a much faster pace.

            00

          • #
            Jose_X

            I am jumping the gun since I should be looking at the 3.7 more closely, but let me add a few things before I go back and review closely what I wrote in the last 2 large comments.

            The 1.6 is then expected to keep increasing but would only go up a little anyway (385+16)/(240+4). This just reflects that the medium between TOA and surface has changed its properties a little during that time (because of the 2x CO2).

            Getting back to your original complaint about the 1.6 and the 4 ratio difference, I don’t see why that bothers you. We are looking at the function of surface flux in terms of TOA flux. Assuming it goes through the 0,0 value, the 1.6 is the average slope from (0,0) to (240,385). The 4 is the instantaneous slope at x=240. One can easily draw many curves that fulfill these boundary conditions. There is nothing radical about this possibility from a mathematical perspective.

            BTW, I would not be surprised if I am still misunderstanding the basic gist of “forcings” and “feedbacks”, mostly because I have not looked at it closely.

            So while I can’t explain this 3.7 at TOA yet, would like you to confirm if you still think the 4 tangent slope is a problem on a curve with a secant average of 1.6?

            I won’t comment on the 171% feedback that my comment suggested to you because it depends on the block diagram. Clearly, I developed in these last 2 comments a model that was rather different than what you were describing and what the IPCC apparently is describing as well.

            And yes there was an inconsistency with me talking about 10 W/m^2 total TOA and then apparently stating that 1.6 W/m^2 was the total TOA addition from the 1x point in time (though it wasn’t clear to me I had made that implication or that latter statement). The “insight” I had for that first of 2 long comments was apparently a dead end, and I didn’t realize it as I was writing it.

            Also, my current note about the 1.6 and 4 in this comment is what I was originally going to write. It’s also what I wrote about at Barry Bickmore’s website some time over a year ago to you presenting just this issue.

            00

          • #
            Jose_X

            Actually, it’s 3 recent and fairly long comments (not 2), where I apparently severely misinterpreted what the 3.7 W/m^2 was referring to, starting with these statements:

            > Please provide a reference (since I have not read the IPCC report except a few sections here and there).
            > I suspect that you aren’t defining the values the same as the IPCC and maybe the confusion lies there.”

            I’ve looked at this before, so I actually thought I had gained a new insight into the 3.7.

            00

          • #
            Jose_X

            I said >> mentions a paper from 1998 (behind a paywall) that I think calculated the 3.7 and some other numbers. I think it’s a computer program that uses the spectra data on CO2 (and other ghg), not sure how.

            a) A direct link is here http://folk.uio.no/gunnarmy/paper/myhre_grl98.pdf
            b) That paper simply curve fits logarithmic curves (and variants) to data that has been composed by the 3d models.

            The closest I can currently find to a derivation of 3.7 W/m^2 is that it comes from 3d computer models using fluid/heat/radiation physics and lots of data collected at many points around the world. Different models come to different values and that leads to an error margin around that 3.7. For all I know the open source program maintained by giss (or ?) called community something or other, has an immediate option to simply spit that result out after being fed data from various public databases.

            Ramanathan/Coakley considered a 1d model (so it just analyzes height variation approximations for radiation and convection), and I think their result from the late 70s came to around 4.

            00

          • #

            Jose,

            An incremental gain of 4 with an average gain of 1.6 is indeed absurd. The gain decreases as forcing increase which is a consequence of the T^4 relationship between temperature and power as each degree of warming requires more steady state input power than the one before, therefore; delta T / delta P decreases as P increases because P increases faster than T. Look again at the plot I linked to at the end of 159.2.1.4. Its a plot of measured post albedo input power along X and surface temperature along Y. Each little dot is the average of 1 month of data for each 2.5 degree (280 km) slice of latitude (> 20M individual measurements at the equator and > 10M at mid latitudes). The larger blue and green dots are the averages of 30 years of monthly averages for each 2.5 degree slice. The slope of the blue and green dots is the incremental gain. This is the same plot, except that the Y axis is radiant power emitted by the surface. In this case, the change in surface power is slightly less than the change in post albedo input! The use of yearly averages cancels out seasonal variability and the significance of 2.5 degree slices of latitude is that the dominant difference between adjacent slices is a difference in solar insolation and how this difference affects average temperatures is indicative of how the climate responds to incremental forcing.

            Theres a good physical reason why the radiant power emitted by the surface changes at the same rate as the post albedo input power. Its a simple case of understanding the origin of the power being emitted, which ultimately is the power arriving from the Sun. This does point out another flaw in the IPCC ‘science’, which asserts that incremental atmospheric absorption from GHG’s is equivalent to changes in solar power. While the magnitudes are similar, the effects are orthogonal. The influence of clouds and GHG’s is to add a bias to the linear gain curve, shifting it up and down as clouds adjust to meet the steady state requirements. The relationship between GHG effects and the steady state average surface temperature can be derived from first principles. If T is the net transparency of the atmosphere, it can be quantified as,

            T = t*(1 - c*p)

            where t is the size of the transparent window of the clear sky atmosphere with nominal water, c is the fraction of the planet covered by clouds and p is the average emissivity of those clouds. Plugging in real value, t=.52, c=.67 and p=.8, results in a net transparency of T=0.24. If T is the net transparency, (1 – T) is the net absorption. If F is the fraction of this absorption that is directed out in to space, we can solve for Po/Ps as,

            Po/Ps = T + F*(1 - T)

            which for F=1/2, E = Po/Ps = 0.62 = 239/385. Doubling CO2 decreases t by 2%, or down to 0.51 (.01*385 = 3.85 ~= 3.7). Now, T=.237, resulting in a new value of E of of .618, which for constant Po, Ps = 386.7 W/m^2 and delta T is about 0.7C. Note how this happens to be equal to the 0.7C per 3.7 W/m^2 of solar power demonstrated by the global satellite data and this is not coincidental.

            Regarding the IPCC, they claim that a decrease in Po from 239 W/m^2 to 235.3 W/m^2 results in an increase in surface power from 385 W/m^2 up to 401 W/m^2. They claim that nebulously defined positive feedback causes this, but where is all the extra power coming from? It can only be the consequence of ‘feedback’ decreasing Po by another 10 W/m^2, or about 4%, which way too high compared to its measured value of about 1.5%. At the same time, clouds decrease by about 1% increasing T and Po/Ps, offsetting the albedo decrease! The last link in 159.2.1.4 refers to the data which supports the various sensitivities, all of which are renormalized to 3.7 W/m^2 of forcing. In the end, doubling CO2 increases surface temperatures by about 0.7C +/- 0.2C which is well below the lowest speculative guesstimate of the IPCC, can be derived from first principles and is corroborated by the best data available.

            George

            00

          • #
            Jose_X

            So a few main points of my current understanding on climate sensitivity, 3.7, etc.

            A lot of values are calculated with computers and then can be used in simple equations and in simple linear (or quadratic, etc) relationships since every relationship between 2 variables has at least some range that behaves linearly (or in some other more precise way than linear if the range is not that small). We can use extensive physics of molecular radiation and fluid/heat flow across the planet, leveraging detailed data on gases and measurements throughout the planet, to run a computer for a long time to get some accurate results (by hand it would take forever, and we’d need to use numerical algorithms anyway because explicit solutions frequently don’t exist for some of these equations), and then we can re-use these results through shortcuts to ask and answer lots of questions and not have to worry about running a computer for days for each such variation of a question we might have.

            We can ask things like what if CO2 has increased by 30% and several other molecules by some other percent. We can include volcano effects, sun variations, and other things.

            The primary linear relationship we want to leverage is between TOA and surface. We can use temps or irradiance or a combination (a linear relationship will exist despite the fourth power from Stefan Boltzmann, see last paragraph).

            So dT = lambda * dF is a nice proportionality relationship.

            OK, let’s start by seeing what a dF of 1 for the sun’s irradiance does to the planet’s surface temp. The computer chugs along and we get our answer. The answer is believed to be approx .8 C, which implies a lambda of .8 C/(W/m^2), and we call this climate sensitivity. To calculate this, we used our best physics. We considered water cycle, albedo ice-dependent changes,.. everything. Obviously, there is a range of values because of the variation in computer runs using different data and even different algorithms and uncertainty in our knowledge. Point is that after putting in our best physics into our best computational tool, we get this .8 (more or less).

            Now, every other variable we want to play with cheaply just needs to have its dF value calculated. Because of the linear approx that is in play, this will work for any variable (eg, aerosol, ghg, etc). It will work so long as we stay in the linear range (and this range depends on our precision needs; we can use other models that keep higher order Taylor expansion terms). The other assumption is that the variable will have an effect on the climate determined just as if the sun was shining hotter by that dF. So we find (using the big fast computer running a long time) equivalent dF for many particles and perturbations we want to study. Then, within the linear range, we will know all combinations of these particles because the many dFs would add up and produce a linear dT as well.

            OK, let’s look at carbon dioxide. We run the computer and find that average temp goes up 3 K for a 2x growth. From this we get that dF = 3.7 since dF = dT/lambda = 3/.8 = 3.7.

            We can do this for all sorts of other items to perturb.

            Further, by running the computer a long time, we can get not just dT for CO2 at 2x, but at 1% increase, at 2%, etc. From that we can fit a curve that gives approximately, dT = lambda * dF where dF = 5.35 ln (C/Cx). We can similarly get a simple equation for many items being perturbed. This way we can generate dF (and dT) to use in a wide range of scenario studies.

            BTW, for the specific case of 2x CO2, we have dF = 3.7, but we know the dF for any other increase in CO2 using the ln shortcut formula above.

            Also, for the special case of 2x CO2, we call the dT value (3 K) the Equilibrium Climate Sensitivity. That value is an ideal value that means we hold CO2 at some reference 1x value for a long time and measure the steady state (equilibrium) temp. Then we do the same for 2x. The difference in these two temps is the ECS. Now, computers don’t usually try to calculate that ECS, but instead use tricks to calculate something that comes close. So even with super fast computers and lots of time on our hands, the computer still needs help and only gives an approximate value.

            Finally, we could have used dFsurface = k * dFTOA and used a constant of proportionality that had no dimensional unit. It doesn’t matter. It would simply mean we would use different values to communicate that would be less handy than what we use now. Note that going from 287 K to 288 K implies an increase of 1 K but of 390.08 – 384.69 = 5.39 W/m^2. However, going from 288 K to 289 K (also 1K), while having to obey a fourth power monomial relationship, still looks fairly linear since that value is 5.45 W/m^2. Thus, if we wanted a dimensionless k, it would be about .8 * 5.4 = 4.32. Let’s see. dFsurface = 4.32 * 3.7 = 15.98. No, prob. Pick any dF for any perturbation and we get the correct dFsurface that matches the dT we would get if we had used lambda instead. This is because Stefan Boltzmann (like most any continuous smooth function) behaves almost like a line if we stay within a narrow enough range.

            00

          • #

            Problem here is that the computer models have too many dials. GCM’s and such have many, many knobs which can be tuned to exhibit any behavior you want and all seem to suffer from a divergence problem, where the long term steady state is different, given constant insolation and different initial conditions. A proper model will always converge to the same steady state, independent of initial conditions. Running a divergent model N times and taking the average is not a solution and just a GIGO process.

            Again, you should look at the links in 159.2.1.4. These plots measure the sensitivity of various parameters to changes in forcing and these measured results are consistent with the simple formulation for E in the previous post and contradict the high sensitivities assumed by the IPCC. The data doesn’t lie, moreover; the ISCCP data came from Hansen’s organization!

            00

          • #
            Jose_X

            >> An incremental gain of 4 with an average gain of 1.6 is indeed absurd. The gain decreases as forcing increase which is a consequence of the T^4 relationship between temperature and power as each degree of warming requires more steady state input power than the one before

            True S-B is the ultimate feedback and throttle on temp rises, but the claims are that the current shock to the system clearly is at least say “4x” greater than the ordinary behavior of the recent past. We are talking about a few degrees reaction shock (based on a clearly measured CO2 shock) and not 100 degrees or anything comparable to the existing 287K. We are in a linear range. There is nothing unusual about a momentary spike. When we push someone on a swing, we hardly push on them and then we provide an aggressive shock for a small moment. This short-term effect happens in many actions of our daily lives and in nature. We have a low average for a prolonged period of time and then a significantly large instantaneous slope.

            Even if we continue adding CO2, the SB headwind will surely triumph in the end keeping us from getting too high of a temp, but that doesn’t mean plenty of harm can’t come to existing ecosystems that perhaps we currently take for granted to keep food and many other costs tame. Also, sea level rise looks to be a real threat because of how much a modest predicted rise (with historical precedent) can harm our current coastal cities in which we have so much invested.

            To bring in politics for a second, expect the many coastal blue states to stay solid blue if the Democratic party continues to be the only one treating these issues like if they might just maybe perhaps have some validity. And if droughts and floods later on pick up where they left off in recent years, expect the current Republican party to be rushed into retirement.

            >> therefore; delta T / delta P decreases as P increases because P increases faster than T.

            This statement only considers the T and P of a given body.

            (a) TOA and the (b) surface are different and have corresponding (if coupled) Ts and Ps. The claims by climate scientists aren’t a refute of calculus since they are comparing the T (or P) of one mass with the T (or P) of another. T_b and P_a are not each a part of the same S-B relation.

            >> Look again at the plot I linked to at the end of 159.2.1.4. Its a plot of measured post albedo input power along X and surface temperature along Y. Each little dot is the average of 1 month of data for each 2.5 degree (280 km) slice of latitude (> 20M individual measurements at the equator and > 10M at mid latitudes). The larger blue and green dots are the averages of 30 years of monthly averages for each 2.5 degree slice. The slope of the blue and green dots is the incremental gain.

            The 3.7 and similar discussion look at the whole earth’s surface as one sample point. Your observations with how P and T vary along the planet’s surface is irrelevant to the relationship that exists for the whole earth’s surface as it varies along a time domain. You will need to have each point be, not the average Ptoa and Psurf of a different chunk of the planet’s surface each averaged over a common time period, but the Ptoa and Psurf of a common surface (eg, the whole planet) averaged over each of different time periods. Ie, you are analyzing points in space domain (to see how P varies over the earth’s surface) while the 3.7 analysis deals with points in the time domain (to see how P varies over time).

            The ratios you measure (which do tend along a single line of a given slope.. say something close to 1.6) support the idea that the atmosphere may be near homogeneous in effect (of linking TOA P and surface P) around the world.

            The ratios I’m talking about (over time) have the atmosphere changing everywhere (eg, with more CO2 and ghg effect) if still remaining homogeneous in spacial behavior.

            Both effects are orthogonal, not contradictory. If you make the same analysis in 100 years, you may expect to get a similar looking graph, but with the numbers slightly adjusted so that the ratio value might be a little different everywhere. However, the degree the planet is expected to change in the next 100 years (a linear approximation range of where we are now) indicates that the 1.6 will hardly change at all.. but still enough to affect water level, weather events, and life in general for us very delicate creatures.

            >> This is the same plot, except that the Y axis is radiant power emitted by the surface.

            Actually, the last link I saw on that page and this new link were identical.

            >> This does point out another flaw in the IPCC ‘science’, which asserts that incremental atmospheric absorption from GHG’s is equivalent to changes in solar power. While the magnitudes are similar, the effects are orthogonal.

            I agree that we are changing the atmosphere in ways that adding more sun almost surely does not do (orthogonal).

            What my earlier comment pointed out is that this dT dF modeling approach enables shortcut efficient mathematics while we remain in the linear range. This is very useful to researchers of various types. However, the authoritative source for the effect of any particular combination of perturbations on the atmosphere is the results from computations using the sophisticated and much more fundamental math and physics enshrined in the computer models.

            [Sorry, for the following repetition of the same theme:]

            So, yes, what happens to the atmosphere when we add CO2 is not described by the dT and dF models. Instead this model gives the right results to certain types of useful prodding (eg, linear combinations of perturbations) yet is definitely not a model able to make claims about the new physics of the evolving climate (we use computers for that). We don’t derive climate sensitivity and many other predictions from that simple model. Rather, we use the model for some useful manipulations only after we have derived the key parameters (like climate sensitivity).

            The preceding is my (recent) understanding of what I at times have feared were divined formulas and values associated with climate sensitivity. I tentatively now think it might match the views of climate science, but I don’t know.

            >> The relationship between GHG effects and the steady state average surface temperature can be derived from first principles.

            As for this next part of your comment, it relates to that paper you put up, so I will comment later after I have refreshed my mind on it.

            >> Regarding the IPCC, they claim that a decrease in Po from 239 W/m^2 to 235.3 W/m^2 results in an increase in surface power from 385 W/m^2 up to 401 W/m^2. They claim that nebulously defined positive feedback causes this

            Can you provide a reference?

            I’ll skip the last part of your comment for now since I think it relates to these things that are pending.

            00

          • #

            Jose,

            Your understanding of the difference between the impulse response and the steady state response is incorrect. The impulse response can exceed the steady state average, but only if it is ‘ringing’ and subsequently bouncing below and above the steady state average until the bounce goes to zero, in which case, the steady state average has been achieved. By your understanding, what is the impulse response to the 240 W/m^2 of average incident power that is turned on and off every day for most points on the surface? You seem to be saying that if solar power was not a periodic impulse and present 24/7, the average emitted surface power would eventually reach over 4.3×240 = 1032 W/m^2, corresponding to an average temperature approaching the boiling point of water.

            There is no ‘shock’ from increasing CO2 as this has been increasing very slowly compared to the relevant time constant. For all intents and purposes, the equivalent of less than a year of CO2 emissions have not fully manifested their effect. So, of the 160 ppm man made increase during the last century, more than 158 ppm has fully manifested its effect and less than 2 ppm is yet to come. A shock would be a rising or setting Sun, a volcanic eruption or impact event, whose effects are manifested very quickly and indicative of time constants.

            The impulse response can be easily characterized. If E is the amount of energy stored by the planet, Psun is the power from the Sun, Pout is the power emitted by the planet and a is the albedo, we can write an equation which precisely quantifies the ebb and flow of energy stored by the planet.

            Psun*(1-a) = Pout + dE/dt

            Since energy must be instantaneously conserved, when the post albedo input power is greater than the power emitted by the planet, the planet stores the excess, increasing E and the surface temperature until Pout is again in balance and dE/dt == 0. When the planet is emitting more than it receives, the extra is being removed from the energy stored by the planet until the planet cools enough to again be in balance. Pout is approximately equal to E*Psurface, where E=0.62 thus Psurface varies as Tsurface^4. Sometimes people confuse the linearity between dE/dt and dT/dt as somehow overriding the T^4 dependency between temperature and emitted power. Of course, this linearity only affects the rate at which the steady state is achieved, not what the steady state will be.

            If we define an amount of time, tau, such that at the current rate of Pout, all of E would be consumed, we can rewrite this as,

            Psun*(1-a) = E/tau + dE/dt

            This is the exact differential form of the LTI system describing an RC circuit. The impulse response, steady state response and the response to any stimulus are well known by every EE, but apparently no climate scientist. The response to an impulse approaches the steady state value (when dE/dt == 0) at a rate of (1-e^-t/tau), where tau is the time constant, which is amount of time for Pout to reach 63% of its equilibrium value.

            Weather satellites measure Psun*(1-a) and Pout directly, thus dE/dt can be measured to high precision for each point on the surface and aggregated into slices, hemispheres and the whole planet. Each aggregation behaves exactly as predicted and are seasonal sinusoids centered around zero in response to seasonally sinusoidal Psun*(1-a) at each measured point on the surface, moreover; it demonstrates a net time constant on the order of months, not the many decades to centuries required by the consensus.

            Your objection to my plots is incorrect. I do not consider the Earth to be a single point. I model it as a gridded surface of points 280 km on a side and maintain the full time domain response with 4 hour samples. Each dot is averaged across a common time period, where only insolation varies. The end values I present are appropriately weighted yearly averages, whose change is what we mean by global warming (which is why its now referred to as ‘climate change’).

            A point you have right is that dT/dE is approximately linear over a small range, however, gain represented as a dimensionless dE/dE per Bode is linear across its entire range! Whats the point in removing linearity only to claim that the result approximates the characteristic that was purposely removed?

            The reference for my claim of the IPCC is the first TAR.

            George

            00

  • #

    […] Why greenhouse gas warming doesn’t break the second law of thermodynamics […]

    00

  • #

    Jo Nova, UN IPCC and GHGT are wrong. Jo Nova blog proves there is no consensus on the back-radiation issue.

    Electric blankets do warm you, nonelectric blankets don’t. Blankets, jackets and clothing reduce the rate of heat transfer between your body by reducing heat transfer coefficient. That may make you feel better or worse, depending. The rate of heat transfer by conduction is q = UA(Th– Tc). Wool blanket U < silk U. Confusion arises when GHGT defenders assert a blanket analogy that fails to specify whether q is constant or Th is constant. Human body releases energy of oxidation by IR radiation, thermal conduction/convection and perspiration. Normally Th = 37.0C, held constant by internal thermostat adjusting oxidation rate q, metabolism. So when you put on a blanket, U decreases and so does q. You may feel better if (Th– Tc) is large, but not if it is small or 0. For atmosphere U = 0 so q = 0 for any (Th– Tc).

    I explained this to Roy Spencer two years ago but he didn’t get it. GHGT promoters need to verify their wet blanket analogy applies to atmosphere. Tossing out simple, invalid analogies and mantras won’t cut it. Since there is no heat transfer by conduction through atmosphere to space, the blanket analogy is false. But the debate well is already poisoned and corrupted by false prophets ignorant of engineering.

    Where GHGT theory falls down is assuming the radiation intensity in all directions from cold atmospheric CO2, which does point down to surface, is thereby absorbed by surface, warming it. It is this last step which is not correct. They confuse S-B equation relating T to radiation intensity, emissive power or irradiance, to radiant heat transfer. Intensity is the upper limit of transfer rate to surroundings when they are at 0K. The words intensity and radiant heat transfer are not synonyms; that is why we invented both of them. For a mass body, intensity is a vector, T is a scalar. When people use S-B to assign T to radiation EMR vector field intensity vector, that T must be a vector. You can detect the difference between thermal scalar T = -10C and radiant vector T = +10C on a sunny ski slope, all by yourself.

    The rate of radiant energy transfer law between two bodies at T and t is Q/σ = ET4 – at4. When ET4 = at4, Q = 0 and there is no energy transfer, even if T and t are very high, (or in the rare case t > T, a < E) radiating with great intensity. Like neighboring stars. It is incorrect to say radiant energy transfers in both directions simultaneously, even if each radiator intercepts intensity from the other. (For one thing both ways cannot be detected.) Nothing gets absorbed or reemitted if Q = 0. This is the situation in my office most of the time. EMR is an energy field. It is like water behind a dam does not flow even if at high pressure. It takes a potential difference to get energy to flow. So Roy Spencer might detect down-welling radiation from cold CO2 with a pyrometer, like I can see ice cubes, but that does not prove radiant energy transfers from cold to hot. In Nov 2011 I proved if it did, it would be a perpetual motion machine; just what they need to drive global warming in perpetuity. http://www.slayingtheskydragon.com/en/blog/185-no-virginia-cooler-objects-cannot-make-warmer-objects-even-warmer-still

    For a step change in GHGT back radiation, the sequence converges to a new steady-state and a finite amount of energy is created (which is impossible). My proof the GHG back-radiation theory leads to creation of energy actually derived the rate of creation from the physical properties:

    Es = (K*F0 + f0)(1 + K)k/(1 – kK).

    K is the fraction of radiation from the first bar absorbed by the second colder bar, 0 < K < 1.
    k is the fraction of re-radiation from the second bar absorbed by the first hotter bar, 0 <= k 0 and f0 => 0.
    Since the denominator is 0 < (1 – kK) 0 and k > 0, the numerator term (1 + K)k > 0, and Es > 0.

    Es = 0 if and only if the fraction of back-radiation k = 0.

    Therefore GHGT is false because back-radiation = 0. This means you change the famous GHGT K-T Global Energy Flows diagram right side beige arrow down from 333 to zero and up arrow from 396 to 63 w/m2 of Earth surface. Just use correct absorptivity’s and emissivity’s and it all fits known physics.

    Roy Spencer’s famous 2010 hot plate analogy attempt to validate back radiation energy transfer notion so basic to GHGT does not apply to Earth’s atmosphere on two counts, 1) the atmosphere is not moving closer to the surface and 2) neither has a separate electric current energy source. In other words it is a false analogy or at least he failed to show it was true for Earth. http://www.drroyspencer.com/2010/07/yes-virginia-cooler-objects-can-make-warmer-objects-even-warmer-still/

    Global Climate Model says emissivity of Earth surface – atmosphere system is about 0.612. If so, corresponding global T for 238 w/m2 average emitted to space would be K = 100(P/5.67e)**0.25 = 100(238/5.67*0.612)**0.25 = 287.8 = 14.6C. That means correcting Hansen’s black body assumption, e = 1 and GHGE = 15 – (-18) = 33C, for colorful Earth emissivity 0.612 gives GHGE = 14.6 – 14.6 = 0, nothing to it. http://principia-scientific.org/supportnews/latest-news/118-correcting-ghg-theory-black-body-assumption-changes-ghe-from-33c-to-nothing.html

    There are three terms in S-B Law: I, T and e. We know I = 238 w/m2 from satellite measurements and solar incidence 1366*0.7/4 = 239 in – 1 to plants = 238 to space. We are stuck with notoriously poor ability to measure T or e, let alone deduce them from physics and chemistry first principles. Some pick T = 14.6 and deduce e = 0.612 from S-B. Others pick e = 0.612 and deduce T = 14.6C. GHGT picked e = 1 and deduced T = -18C, claiming disaster! It is a matter of style. None of which tell us the effect of CO2 on T.

    BTW, I see some overlap between CO2 absorption spectrum left side and solar incident spectrum right side. So first 100 ppm does absorb some incoming and emit it back to space. That would divert radiant energy to and from surface, cooling surface a bit. The Earth system emits to space at same rate with slightly higher emissivity due to introduction of CO2, so it must radiate at a lower T. That’s cool. The second 100 ppm would divert less due to Lambert-Beer Law, and cool Earth a lesser amount. The fourth 100 ppm would have a much smaller cooling effect, law of diminishing returns. Determining the effect of CO2 on average absorptivity / emissivity of 100 km thick atmosphere takes a some work to quantify those little bits. If I get in the mood to quantify it, I might, but I am reconciled with nature for now. I think the whole 400 ppm today causes about T4 – T0 = – 0.5C or so. The next 100 ppm, 400 to 500 would cause an additional -0.01C. Always < 0C. Not enough to get excited about. No tipping points. All is calm, all is bright.

    You want a simple explanation for the GHGT effect? There it is. One short paragraph, fits on a bar napkin.

    BTW2, Since systems engineers proved long ago one cannot prove or disprove cause and effect from correlated measurements, all the discussion about CO2 and T data is utterly irrelevant to the issue does CO2 affect T and if so why and how much? Only science and systems engineering can answer that. If I said all the data since 1492 says every rooster always crows 3.14159 minutes before sunrise appears, without explaining the underlying physics, you may accept correlation but must reject the conclusion roosters cause sunrises because you know they are not cause, for sure.

    Pierre R Latour

    00

    • #
      Jose_X

      >> Since there is no heat transfer by conduction through atmosphere to space, the blanket analogy is false.

      Electric blankets insulate you to keep the heat generated by your body from dissipating that quickly.

      The atmosphere insulates the ground + water to keep the heat generated by the sun from dissipating into space that quickly.

      This is an analogy. The blanket analogy is *not* false because both blankets and the atmosphere are insulators, working to help another body gain or dissipate heat more slowly.

      >> The rate of heat transfer by conduction is q = UA(Th– Tc).

      The nontrivial equations of heat at the atmosphere + space boundary are radiative equations because there is no significant conduction or convection exchange with empty space (“For atmosphere U = 0 so q = 0 for any (Th– Tc)”). The effects are described partly by quantum mechanics. Computer models (to handle the complexity) have estimated the greenhouse effect fairly accurate and along the lines of simpler models.

      >> Where GHGT theory falls down is asumming the radiation intensity in all directions from cold atmospheric CO2.. is thereby absorbed by surface, warming it… Intensity is the upper limit of transfer rate to surroundings when they are at 0K.

      At the microscopic level, when a CO2 molecule yields a photon, that photon can be absorbed by the ground and would “warm” the ground a little bit. The CO2 would cool. This happens in addition to (and much less frequently than) thermal contact of that CO2 molecule with other gas molecules. The CO2 both gains and loses photons on a continual basis. The main difference with gases like O2 and N2 is that these last don’t absorb well the photons that the ground tends to emit when at ordinary Earth temperatures.

      All along the sun is bombarding the earth with a huge amount of radiation. So to warm the planet all that has to happen is to slow down the rate of dissipation of that energy a little. Once the rate slows down, the temperature will rise enough to once again balance out the radiation in from the sun vs out into space.

      This is all perfectly consistent with S-B and every other part of physics.

      Please, appeal to a scientific paper or text on specifically what you think is inaccurate and then argue why that is inaccurate, but you are assuming here that the science argues one way while in reality the science presents a much more detailed argument involving much more complex math, “solved” more precisely through computers and loads of measurement data.

      BTW, have you calculated what kind of “potential difference” it would take to have the planet’s average surface temp warm by 3C over many decades? Surely, it doesn’t take much of a difference at all, right? ..since that average imbalance is to last so many years and the sun gives us such a strong flux continually.

      >> So Roy Spencer might detect down-welling radiation from cold CO2 with a pyrometer, like I can see ice cubes, but that does not prove radiant energy transfers from cold to hot.

      That simple observation by itself doesn’t prove very much at all, but are you denying the physical meaning behind “photons” and “energy conservation”? Can you appeal to an experiment done where convection and conduction effects have been factored out so that the experiment would suggest that a cooler object cannot give up radiation and have that radiation later be absorbed by a warmer object?

      Again, please don’t change the story to what you think the scientists are arguing. Appeal to something specific in a contemporary and accepted advanced textbook or paper on this subject.

      >> In Nov 2011 I proved if it did, it would be a perpetual motion machine; just what they need to drive global warming in perpetuity.

      I started to look at what you wrote, but I don’t understand the starting point that Fi = F0 + k*fi. This doesn’t make sense to me. So if you just have one “warm” body in empty space, then the energy coming from it will always be F0 because k and/or fi will be effectively 0? Isn’t that what that equation suggests? [and obviously, that would be a highly inaccurate depiction of reality since if would imply “infinite” energy source and surely doesn’t model the earth or air under any time duration comparable to say a decade or even much less time.]

      I glanced at the rest of the comment, but I think I have said enough for now and prefer to wait to see if I am misunderstanding the equation above.

      00

      • #
        Jose_X

        [I said]>> So to warm the planet all that has to happen is to slow down the rate of dissipation of that energy a little.

        This is one place where back radiation plays a role. At a given moment, the sun has sent some energy, but on average that energy will take longer to leave the planet + atmosphere system if it is “deflected” back more — if the path into space is longer and involves hanging around the planet + atmosphere for longer. There is more energy in the air, on average, because you don’t just have the energy that recently came in, but a larger fraction of energy that came in a while back and also a longer while back, etc. This translates to higher temperature.

        We can cook a hotdog with mirrors by deflecting sun’s energy into a particular area. Temperature is a measure of concentration of energy. At equilibrium you no longer have an increase in temp, true, but you can still have that higher temp because the average intensity was raised before equil was established.

        If you have an oven with a given current coming in to heat the coils, that is not enough to imply a temp inside. You also can’t tell by how hot it gets outside near the oven. You need to know how good is the insulation (the oven walls), as really good insulation can hide the fact the oven might be super super hot. From the outside, you have the same modest energy coming in and leaving, but a lot of energy accumulated inside before that balance was reached leads to a very hot interior. Conversely, weak insulation means the same incoming and outgoing measurements as before may correspond this time with merely a warm oven interior. So in each case we have the same measurements into and out of the system and the energy is in balance in each case, but in one case we have a very hot interior and in the other a lukewarm interior. The reason for the very different internal temperatures is that the other parameters, eg, the quality of insulation, are different across the two examples.

        Analogy of a tub. Even with the same intensity of water coming in, if you make the drain less effective (eg, clog it or make pipes more narrow), you will have water accumulate in the tub to a higher water level until the higher pressure pushes hard enough to again balance the recently diminished outgoing rate with the steady incoming rate. This clogging is a sort of “insulative” effect, where you get some extra of the “flowing” content to hang around, even under the same incoming conditions, simply because it is harder for it to leave. A higher temp, pressure, etc, must be reached to push hard enough against the higher pushback (resistance, insulation, etc) to reach the old flow rate out. [I am not using precise technical terms since I just want to help establish some intuition.]

        Some of these radiation conclusions are not obvious on Earth because we have to remember that we also have convection and conduction, which serve to average things out much more than would be the case if we only had radiative effects. Whereas during the course of a day different parts of a table might otherwise go towards noticeably different temperatures (eg, based on radiative items shining at it) if these pieces of the table were isolated from each other and from everything else by a hypothetical thin vacuum slice, in real life we have conduction and convection that would keep the temperature of all parts of the table much more even.

        GHG significance:

        If you add more sweaters to yourself on a cold day, you retain heat better and the skin temp will rise (even becoming very hot). If you add more atmosphere, you get the same effect for the planet. The main issue is that there is a significant difference between a modest amount of GHG vs none at all. [Because of a logarithmic relation, adding too much GHG stops being that much of a big deal relative to the earlier also large quantity.] With next to *no* GHG effect, the earth can radiate directly into space, and the atmosphere, even if somewhat “thick”, would be mostly transparent and not function as an insulation cover. Think of this no GHG atmosphere as clear glass. Making the clear glass thicker in width still lets almost the same quantity of light through (next to nil times 10 is still next to nil). In contrast, adding “some opacity” to the glass definitely will reach a point much earlier where the thick layer no longer allows light to pass. So some GHG is needed to turn the “uninsulatative” atmosphere, no matter how thick, into one that does provide insulation. Basically, there is a very large difference between having some CO2, which has a reasonable probability of absorbing in the earth temp IR range, vs. having no ghg gases, which has an extremely tiny (quantum mechanical) probability of absorbing in that range. The GHG turns the atmosphere into an layer of insulation. Then, the thicker that layer, the warmer it can get from the same steady sun input irradiance.

        00